You are on page 1of 348

- -- - -~-~.

---
}f ti>'PilEIL_rtAtlON
CP Publication KVPY chapterwise with www.puucho.com
,,.
-~--- -~ ~----~-----~-- - - · - - - - - _:-,. ___ . ---

(Kishore Vaigyanik Pratsahan


. ¥ojana)
STREAM - SX ·
Unit wise Old Examination Solved Paper

..
~

16VP'V .r . .. .,
.

www.puucho.com
CP Publication KVPY chapterwise with www.puucho.com

,· - - - - ----· ------- ---


\

{
.

\

Years KVPY [STREAM SX]


CHAPTERWISE SOLVED PAPERS
-
I
~-
'
--- ----- -- - .. - --- -- -· -· --- _., --- --- . ... .... ·- --- _.... ·- ·- -- - -- . --- ~ ---- -- --- --· ---

PHYSICS, CHEMISTRY,
MATHEMATICS 8t BIOLOGY

www.puucho.com
CP Publication KVPY chapterwise with www.puucho.com

Information contained in this work has been obtained by Career Point from sources believed to be reliable.
However, ndthcr Career Point nor its authors guarantee the accuracy or completeness of any information
published herein, and neither Career Point nor its authors shall be responsible for any errors, omissions, or
damages arising out of use of this information. This work is published with the understanding that Career Point
and its authors arc supplying information but are not attempting to render any professional services. If such
services are required, the assistance of an appropriate professional should be sought.

CAltEER POINT
CP Tower, Road No.-1, lPIA, Kofa (Raj.)
Email : publication@cpil.in

Copyright© 20 I 8, by Career Point Ltd.


No part of this publication may be reproduced or distributed in any form or by any means, electronic, mechanical.
photocopying, recording, or otherwise or stored in a database or retrieval system without the prior written pennission of the
Publishers. The program listings (if any) may be entered, stored and executed in a computer system, but they may not be
reproduced for publication.

This edition can be exported from India only by the publisher.

Price : t 520.00

Published by Career Point Ltd.


CP Tower, Road No.-1, IPIA, Kota (Raj.)
Email: publicalion@cpil.in

Book No.: CPP-131

www.puucho.com
CP Publication KVPY chapterwise with www.puucho.com

Preface

Whenever a student decides to prepare for any examination, her/his


first and foremost curiosity is about the type of questions that he/she
has to face.

We feel great pleasure to present before you this book. We have


made an attempt to provide unit wise collection of questions asked in
KVPY with answer and solutions to majority of questions. Solutions to
the questions are not just sketch rather have been written in such a
manner that the students wil! be able to understand the application of
concept and can answer some other related questions too.

We firmly believe that the book in this form will definitely help a
genuine, hardworking student.

We have tried our best to keep errors out of this book. Comment and
criticism from readers will be highly appreciated and incorporated in
the subsequent edition.

We wish to utilize the opportunity to place on record our special


thanks to all team members of Content Development for their efforts
to make this wonderful book.

Career Point Ltd.

www.puucho.com
CP Publication KVPY chapterwise with www.puucho.com

CONTENTS
PHYSICS
Chapter Page No.
1 ll/lechanics 03-33

• Answers & Solutions 17

2 Heat& Waves 34-50

• Answers & Solutions 42

3 Electrodynamics 51-73
• Answers & Solutions 62

4 Optics 74-84
-
• Answers & Solutions 79

~ Modern Physics
-·-
85-96
• Answers & Solutions 91

CHEMISTRY
Chapter Page
1. Physical Chemistry 99-120
• Answers & Solutions 111

2. Inorganic Chemistry 121-131


• Answers & Solutions 126

3, Organic Ch~mistry -
132-156
• Answers & Solutions 146

MATHEMATICS
Chapter Page
1,. Trigo~ometry 159-166
• Answers & Solunons 162

Algebr~_
'IL - .. - - - - . - ... --167-192
-- 177
---- --
• Answers & Solutions

3. 2D Coordinate
- _..... _.... 193-206
• Answers & Solutions 198
cont ....

www.puucho.com
CP Publication KVPY chapterwise with www.puucho.com

Chapter Page No.


4. Differential Calculus 207-218 7
• Answers & Solutions 212

CI Integral Calculm 219-231


• Answers & Solutions 224

~ Vector & 30 232-236


• Answers & Solutions 234

BIOLOGY
· Chapter · Page No.
.___1_.__D_iv_e_rs_ity in the Uving_~orld, Structural Orga_n_isa_ti_on_in_P_la_n_ts_&_A_n_i_ma_ls_ _ _ _ _ _ _23_9_-2_4_2_ ____,
• Answers & Solutions 242

r. . _2._ _Ce_l_l: ~tI:Ucture & functions _ _ _ _ _ _ _ __ 243-249


• Answers & Solutions 248

Q.--- Plant e~vsiology:_ _ __ 250-2S3 7


• Answers & Solutions 253

C4. Human physiology_ __ _ _ _ _ _ _ _ _ _ _ _ _ _ _ _ _254-260


_ _ _ _ -...J\
• Answers & Solutions 259

~_5_ _R_ep}'Otfilction,~tics & evolution _ - - - - - - - - - - - -------2-61--272 -~J


• Answers & Solutions 269

[:6.-- Biology in Human Welf~r:_~, Biotechnology_-


_ _ _ ---·-- _ _ _ _ _ _ _ _ _ _2_7_3_-2_80
• Answers & Solutions 278

L1. __ Ecology __-_-_------~---_-~-~~·_-____________ 281-284.==J


• Answers & Solutions 284

MODEL TEST PAPERS


Practice Test Page No.
L 1. __ Practice Test-1_ _
------- -~- ---···~ ·--........-~ ~~-~-
287-307
---~ - - - - - ----
• Answers & Solutions 300

L 2. Practice Test -2
-- ·-------------- --- - - - - --·-- - ---- ·--- - - - - -----
- - 308-330-· 7
- - - - - - -------- j
• Answers & Solutions 321

L3- ___ Practicelest-3 ..._________________ ~ ~ - - - ---~~- ______ ,_ -____ -- 331-354 ·-7


• Answers & Solutions 344

www.puucho.com
CP Publication KVPY chapterwise with www.puucho.com

Physics
[ToP1c-w1sE KVPY [STREAM-SX] Questions with Solutions]

www.puucho.com
CP Publication KVPY chapterwise with www.puucho.com

Mechanics
L
] I
1. A pen of mass 1m' is lying on a piece of paper
of mass M placed on a rough table. If the
(A) _l
21rV~
{TI ,) 2xv~
( ~-1(4T .,
coefficient of friction between the pen and
fn
paper, and, the paper and table are µ 1 and
respectively, then the minimum horizontal
µ2, l ~-T
(C)-
2n: m
(D)-1
2rc v-;;
force with which the paper has to be pulled for 4, A comet (assumed to be in an elliptical orbit
the pen to start slipping is given by - (20101 around the sun) is at a distance ofd.4 AU from
J,k) (m + M) (tt1 ~ µ2) g the sun at the perihelion. If the time period of
(B) (mµ, + Mµ2)g. the comet is 125 years, what is the aphelion
(C) {mµ1 + (m + M) µ2} g distance ? AU : Astronomical Unit. I2010]
(D) m(µ1 + µ2) g (A) 50 AU (B) 25 AU
(_9A9.6 AU (D) 24.6 AU
2. Two masses m1 and m2 connected by a spring
of spring constant k rest on a frictionless 5. A book is resting on shelf that is undergoing
vertical simple harmonic oscillatiot15 with an
surface. lf the masses are pulled apart and let
amplitude of 2.5 cm. What is the minimum
go, the time period ofoscillation is - [2010]
frequency of oscillation of the shell for which
the book will lose contact with the shelf ?
(Assume that g = 10 m/s 2) !2010}
(A) 20 Hz ..{D) 3.18 Hz
(C) 125.6 Hz (D) IO Hz

6. Two masses m1 and m2 are connected by a


massless spring of spring constant k and
1
(C) T=21t ~( : )
unstreched length£. The masses are placed on a
frictionless straight channel-which we consider
our x-axis. They are initially at rest at x = 0 and
(D) T = 2n ,/( : 2)
x =£,respectively. At t = 0, a velocity ofv0 is
suddenly imparted to the first particle. At a later
3. A bead of mass m is attached to the mid-point time to, the centre of mass of the two masses is
of a taut, weightless string of length f, and placed at- 120101
on a frictionless horizontal table. (A) x = m::if
m1+m2
T~-----1,----..,_T
,.,' X ' ....... (B) x = m1f + m2 v0 t
- ~~

-
4---------------~-------~
~,
m1 +m2 m1 +m2
f m2l' m2v0t
(C) X = -~-+------
Under a small transverse displacement x, as ml + m2 m1 + m2
shown. if the tension in the string is T, then the JP) x= m2£ + m 1v 0 t
frequency of oscillation is- [2010) m 1 +m 2 m 1 +m 2

www.puucho.com
CP Publication KVPY chapterwise with www.puucho.com

TOPIC• WISE KVPY [STREAM SX] Questions with Solutions

7. A ball is dropped vertically from a height of h 11. Two plates each of the mass m are connected
onto a hard surface. If the ball rebounds from by a massless spring ns shown.
the surface with a fraction r of the speed with
which it strikes the latter on each impact, what c:::=====;======:::J m
is the net distance traveled by the ball up to the
10th impact ? (2010]
.
(A) 2h!.:.:_
JO
1 20
(B) h___:_E.__
c::=========:::J m
1-r - l-r2 A weight W is put on the upper plate which
· 1- r 22 l 20 compresses the spring further. When W is
(C) 2h---h (m 2h ....=:_ - h removed, the entire assembly jumps up. The
l-r 2 '../ 1-r2
miwmum weight W needed for the assembly to
8. A certain planet completes one rotation about
jump up when the weight is removed is just
its axis in time T. The weight of an object
placed at the equator on the planet's surface is a more than - (2011}
fraction f (f is close to unity) of its weight (A) mg ~ mg (C) 3 mg (D) 4 mg
recorded at a latitude of 60u. The density of the
planet (assumed to be a unifonn perfect sphere 12. If the speed (v) of the bob in a simple pendulum
is given by- (20101 is plotted against the tangential acceleration (a),
:A 4 - f 3n: B 4 - f 3rt the correct graph will be represented by- [2011]
, / ) 1- f 4GT 2 ( ) l + f 4GT 2
a a
(C) 4-3(~3rt {D) 4-2f ~
1- f 4GT2 I - f 4GT 2

9. A particle of m~ss m undergoes oscillations


about x = 0 in a potential given by

V(x) = ½kx 2 - Vocos (~),where Vo, k, a are (]) (II)


a
constants. If the amplitude of oscillation is
much smaller than a, the time period is given
by- [2010}

·27t·ka~
(A:)
J
~
+V 0
(B) 21r.J¥

~ (Ill)
(C) 21e
.
(D)2rt
v~ a

10. A narrow but tall cabin is falling freely near the


earth's surface. Inside the cabin. two small
stones A and B are released from rest (relative
to the cabin). Initially A is much above the
centre of mass and B much below the centre of
mass of the cabin. A close observation of the (1V)
motion of A andB will reveal that - 12011] (B) Il (C) ill (D)IV
(A) both A and B continue to be exactly at rest
relative to the cabin
~ A moves slowly upward and B moves slowly
13. TI1e Quantum Hall Resistance R.-i is a fundame.ntal
downward re la ti vc to the cabin constant with dimensions of resistance. If h is
(C) both A and B fall to the bottom of the cabin Planck's constant and e the electron charge, then
with constant acceleration due to gravity the dimension ofRtt is the same as - [20111
(D) A and B move slightly towards each other
(A) e2/h ~,h/c2 (C) h2/e (D) e/h2
vertically

www.puucho.com
CP Publication KVPY chapterwise with www.puucho.com

• MECHANICS
14. Four students measure the heigbt of a tower. 15. Figure below shows a small mass connected to
Each student uses a different method and each a string, which is attached to a vertical post. If
measures the height many different times. The the ball is released when the string is horizontal
data for each are plotted below. The measurement as shown, the magnitude of the total acceleration
(including radial and tangential) of the mass as a
witb highest precision is (2011]
function of the angle 0 is~ [2011]
~~~)&~
"'
'a30 :Known
·i::
lHeight
f-o '•
I
'o20 I

....CJ Initial ''
''
510 Position
;:I
z
10 20 30 40 (A) g sine (B) g~3cos 2 0 + 1

(C)gcosO (el,gJ3sin·2 0 + I

16. A particle moves in a plane along an elliptic path


:Known
: Height
x2 y2
given by - + - = I. At point (O, b), the
a2 b2
x-component of velocity is u. The y-componcnt
of acceleration at this point is- [2011]
JAJ -bu2 / a2 (B).- u2 / b
10 20 30 40 (C)-au2 /b2 (D)-u2/a
Height (m)
(ll) 17. A liquid drop placed on a horizontal plane has a
near spherical shape (slightly flattened due to
-;30 :Known gravity). Let R be the radius of its largest
·c :Height horizontal section. A small disturbance causes
E-<
'o20 the drop to vibrate with frequency v about its
...
~
equilibrium shape. By dimensional analysis the
eIO
;:l . V
z ratio ~ can be (Here a is surface ·
vcr /pR3
10 20 30 40
Height (m) tension. p is density, g is acceleration due to
(ill) gravity, and k is arbitrary dimensionless
constant}- [2O12J
"'
-;:;30
·c;
I
1Known J,A) kpgR 2 /a (B) kpR 2 /ga-
E-< :Height
(C) kpR 3 /gcr (D) kp/gcr
'o2O
....
V

'510 18. Seven identical coins are rigidly arranged on a


;:)
z flat 1able in the pattern shown below so that each
coin touches its neighbours. Each coin is a thin
10 20 30 40
disc of mass m and radius r. Note that the
Height (m)
(IV) moment of inertia of an incli vidual coin about an
axis passing through centte and perpendicular to
(A) I (B) II (C) Ill (D)lV the plane oftbe coi~ is mr 2 / 2 (2012}

www.puucho.com
CP Publication KVPY chapterwise with www.puucho.com

TOPIC• WISE KVPY (STREAM SX] Questions with Solutions

21. A particle is acted upon by a force given by


F =- cxx.3 - px4 where a and 13 are positive
constants. At the point x = 0, the particle is -
(2012)
(A) In stable equilibrium
The moment of inertia of the system of seven __..-(B) In unstable equilibrium
coins about an axis that passes through the (C) In neutral equilibrium
point P (the centre of the coin positioned (D) Not in equilibrium
directly to the right of the central coin) and
perpendicular to the plane oflhc coins is- 22. The potential energy of a point particle is given

(A) 55 mr2 (B) 127 mr2 by the expression V (x) = -ax + p sin ('YJy).
2 2 A dimensionless combination of the constant
(C) lil 2
(D) 55 mr2 a, 13 and r is- (2DJ2]
~mr
{A) a/j3y (B) cl!py
l9. A planet orbits in an elliptical path of eccentricity (C) y/a'3 -tr') o.y/!3
e around a massive slar considered fixed at one of
the foci. "Ibe point in space where it is closest to 23. A ball of mass m suspended from a rigid
the star is denoted by P and the point where it is support by an inextensible massless string is
farthest is denoted by A. Let v., and Y• be the released from a height h above its lowest point.
respective speeds at P and A. Then - I20121 At its lowest point it collides elastically with a
Vp block of mass 2m at rest on a frictionless
surface. Neglect the dimensions of the ball and
the block. Atler the collision the ball rises to a
maximum height of- 120121

I
m I
I
Vp l+c 2 I
(B) - = - - rs
VA 1-c
2m

(A) b/3 (B) h/2


(C) h/8 ve) h/9
20. A body is executing simple harmonic motion of
amplitude a and period T about the equilibrium 24. A panicle released from rest is fulling through
position x = 0. large numbers of snapshots are a thick fluid under gravity. The fluid exerts a
taken at random of this body in motion. The resistive force on the particle proportional to
probability of the body being found in a very the square of its speed. Which one of the
small interval x to x + ldxl is highest at - [2012] following graphs best depicts the variation of
;.71 x= ± a (B) x=±a/2 its speed v with time t - f2012J

(C) x=O (D} x=± ~

www.puucho.com
CP Publication KVPY chapterwise with www.puucho.com

t MECHANICS .
I'

(A)
(

Jnitially, both springs are unstretched. The


(B) block I is slowly pulled down a distance x and
t
released. Just after the release the possible
values of the magnitude of the accckration of
the blocks a1 and a2 can be- (2012]
V
~ilher (a = a = (ki ; ; )x J
1 2 2

r
(kl+ k2)XJ only
(B) ( a = a ---'---=.c.-
L 2 2m

V
(C) (a,= a = (ki ~~ )x Jouly
2
2

(D) Either ( a 1 = a 2 = (ki ; ; 2 )x)


r or [a1 =a~= (k1k2)x
- (k 1 + k 2 )m
g)
25. A small asteroid is orbiting around the sun in a
circular orbit of radius r 0 with speed V 0• 27. A simple pendulum ls released from rest at the
,0
A rocket is launched from the asteroid with horizontally stretched position. When the string
speeq V = aVo, where V is the speed relative makes an angle 0 with the vertical, the angle ~

to the sun. The highest value of a for which which the acceleration vector of the bob makes
the rocket will remain bound to the solar with the string is given by- (2012]
system is (ignoring gravity due to the asteroid
and effects of other planets)- f2012J
_sy'Jz (B) 2 (C)./J (D) I

26. Two blocks (1 and 2) of equal mass m are


connected by an ideal string (see figure shown)
over a frictionless pulley. The blocks are (A) $= O ~$= tan_tta;e)
attached to the ground by springs having spring
constants k 1 and k 2 such that k 1 > k2 (C) ljl=tan- 1 (2tan0) (D)· (jJ =1t/2

www.puucho.com
CP Publication KVPY chapterwise with www.puucho.com
)

TOPIC- WISE KVPY [STREAM SX] Questions with Solutions


28. A wheel of radius R with an axle of radius R/2 31. A point particle of mass 0.5 kg is moving along
is shown in the figure and is free to rotate about the x-axis under a force described by the
a frictionless axis through its centre and potential energy V shown below. It is projected
peI]Jendicular to the page. Three forces (F, F, 2F) towards the right from the origin with a speed v.
are exerted tangentially to the respective rims.
V (in J)
as shown in the figure. [2013)
-4- - - - -
3
2

--4 -3 -2 -1 1 2 3 4 5
2f
X (inm)
The magnitude of the net torque acting on the What is the minimum value of v for which the
particle \'rill escape infinitely far away from the
system is nearly
origin? [2013)
____..+A) 3.5 FR (B) 3.2 FR
(C) 2.5 FR (D) 1.5 FR
(A) 2.Ji. ms- 1
Jff} 2 !TIS-I
(C) 4 ms- 1
29. Young-Laplace law states that the excess
(D) Tbe particle will never escape
pressure inside a soap bubble of radius R is
given by tlP = 4cr/R where cr is the coefficient 32. A bullet of mass m is fired horizontally into a
of surface tension of the soap. The Eotvos large sphere of mass M and radius R resting on
a smooth horizontal table
number Eo is a dimensionless number that is
used to describe the shape of bubbles rising
through a surrounding fluid. It is a combination
1~
h
~ OM J -R
of g, the acceleration due to gravity, p, the ~ I
density of the surrounding fluid, o and a Tbe bullet hits the sphere at a height h from the
table and sticks to its surface. If the sphere
characteristic length scale L which could be the
starts rolling without slipping immediately on
radius of the bubble. A possible expression for impact, then - [2013 J
Eo is (2013]
(A)~= 4m+3M (B) ~= m +M
? L~ L1 R 2(m+M) R m+2M
(B) pL- (C). pg - (D) _g-
og a ap (C' ,h =10m+7M (D) _11_= 4m+3M
., R 5(m+M) R m+M
30. A plank is resting on a horizontal ground in the
33. A small boy is tlrrov.~ng a ball towards a wall 6 m
northern hemisphere of the Earth at a 45° latitude.
in front of him. He releases the ball at a height of
Let the angular speed of the Earth be © and its 1.4 rn from the ground. The ball bounces from th~
radius re. The magnitude of the frictional force on wall at a height of 3m, rebounds from the ground
the plank will be: (20131 and reaches the boy's hand exactly at the point of
2
release. Assuming the two bounces (one from the
(B) mrero wall and the other from the ground) to be
-.fi perfectly elastic, how far abead of the boy did the
mr©2 ball bounce from ground? (2013]
(C; _e_
2
(D) zero (A) 1.5 m (B) 2.5 m
(C) 3.5 m (D) 4.5 m

www.puucho.com
CP Publication KVPY chapterwise with www.puucho.com

t MECHANICS
.. it'-;··
34, A ball is rolling without slipping ·in a spherical 37. A uniform thin rod of length 2L and mass m
shallow bowl (radius R) as shown in the figure lies on a horizontal table. A horizontal impulse
and is executing simple harmonic motion. If the J is given to the rod at one red. There is no
radius of the ball is doubled, the period of friction. The total kinetic energy of the rod just
- oscillation - [2013] after the impulse will be (2014]

(A) I:_ (B~ £_ (C) 212 (D) 6J2


2m m m m
(A) increases slightly 38. A solid cylinder ·p rolls without slipping from
(8) is reduced by a factor of l/2 rest down an inclined plane attaining a speed Vp
(C) is increased by a factor of2 at the bottom. Another smooth solid cylinder Q
lL>) decreases slightly of same mass and dimensions slides without
friction from rest down the inclined plane
35. A solid sphere rolls without slipping, first
attaining a speed VQ at the bottom The ratio of
horizontally and then up to a point X at height h on
an inclined plane before rolling down, as shown -
the speeds ( :; ) is - (2014]

(A) ,/J/4 ./(B),/3/2


(C) J213 (D) .J4/3

39. A body moves in a circular orbit of radius R


lb.e initial horizontal speed of the sphere is -
under the action of a central force. Potential
(20131
due to the central force is given by V(r) = lcr (k
~J ~IOgh/7 (B) .J7gh/5 is a positive constant). Period of revolution of
(C) .J5pp/1 (D) .J2g11 the bo<;i.y-isproportional to- 12014)
Jp-~ 12 (B) R- 112 (C) R312 (0) K 512

36. A block of mass m slides from rest at a height


40. A simple pendulum is attached to the block
H on a frictionless inclined plane as shown in
which slides without friction down an inclined
the figure. It travels a distance d across a rough
plane (ABC) having an angle of inclination
horizontal surface with coefficient of kinetic
rx. as shown. [20141
friction µ, and compresses a spring of spring
C
constant k by a distance x before coming to rest
momentarily. Then the spring extends and the
block travels back attainting a final height 9f h.
Then - r2013J

While the block is sliding down the pendulum


oscillates in such a way that its mean position
the direction oftbe string is-
(A) at angle a to the perpendicular to lheinclined
~) h = H - 2µ ( d + x) planeAC.
(B) h= H + 2µ(d-x) (B) parallel to the. inclined plane AC.
(C) h= 1-1 -2µd + ior.2 / mg (C) vertically downwards
(D) h= H:..2µ (d + x:) + kx.212mg JD) perpendicular to the inc_lined plane AC.

www.puucho.com
CP Publication KVPY chapterwise with www.puucho.com

TOPIC• WISE KVPY [STREAM SX] Questions with Solutions

41. Water containing air. bubbles flows without 45. The dimensions of Stefan-Boltzmann constant
turbulence through a horizontal pipe which has cr can be written in terms of Planck's constant
a region of narrow cross-section. In this region h, Boltzmann constant ka and the speed oflight
c as cr = b11 kall c1. Here (2014)
the bubbles 12014]
(A) a.= 3, p = 4 and y = -3
(A) move with greater speed and arc smaller (B) a= 3, p =-4 andy = 2
than in the rest of the pipe je(a_ = -3, p= 4 and y = -2
tm move with greater speed and are larger in (D) a.=2, ~ =-3 andy=-1
size than in the rest of the pipe
46. A small box resting on one edge of the table is
(C) move with lesser speed and arc smaller struck in such a way that it slides off the other
than in the rest of the pipe edge, l m away, after 2 seconds. The coefficient
(D) move with lesser speed and are of the same of kinetic friction between the box and the table-
(2015]
size as in the rest of the pipe
~ u s t be less than 0.05
(B) must be exactly zero
42. A solid sphere spinning about a horizontal axis
(C) must be more than 0.05
with an angular vclOC:ity CiJ is placed on a horizontal (D) must be exactly 0.05
surface. Subsequently it rolls without slipping
47. Two uniform plates of the same thickness and
with an angular velocity of - (2014] area but of different materials, one shaped like
(A) 2ro/5 (B) 7ro/5 SS:,.)"2.ro/7 (D) ffi an isosceles triangle and the other shaped like a
rectangle are joined together to form a
43. Consider the system shown below. composite body as shown in the figure. If the
centre of mass of the composite body is located
at the midpoint of their common side, the ratio
F between masses of the triangle to that of the
X
rectangle is - [2015)

A horizontal force F is applied to a block X of


mass 8 kg such that the block Y of mass 2 kg
adjacent to it does not slip downwards under
gravity. There is no friction between the 48.
(A) 1 : I
8
(B) 4 : 3 (Cj)-,4 (D)2 : t
wo spherical objects each of radii Rand masses
horizontal plane and the base of the block X. m1 and m2 are suspended using two strings of
equal length L as shown in the figure (R << L).
The coefficient of friction between the surfaces
The angle, 0 which mass m2 makes with the
of blocks X and Y is 0.5. Take acceleration due
vertical is approximately
, - [2015]
to gravity to be 10 ms-2 . The minimum value of
Fi§. r2014]
:A) 200 N (B) 160N
(C) 40 N (D) 240 N

44. The maximum value attained by the tension in


the string of a swinging pendulum is four times m1
the minimum value it attains. There is no slack (A) m1R
in the string. The angular amplitude of the (m 1 +m 2 )L

pendulum is [2014] (C) 2m 2R


r (m 1 +m 2 )L
(A) 90° (B),60° (C) 45° (D) 30°

www.puucho.com
CP Publication KVPY chapterwise with www.puucho.com

•49.MECHANICS
. A horizontal disk of moment of inertia 4.25 kg-m 2 · (A) a.= -2, p = -2, and r = 4
is
with respect to its axis of symmetry spinning · ..,(BJ a = 2, P= 2, and y = --4
counter clockwise at 15 revolutions per second (C) a= 3,P= 3, and y = -2
about its axis, as viewed from above. A second
(D) a= -3, p = -3, and y = 2
disk nf moment of inertia 1.80 kg-m2 with respect
to its axis of symmetry is spinning clockwise at 53, One end of a slack wire (Young's modulus Y,
25 revolutioru per second as viewed from above length L and cross-section area A) is clamped
about the same axis and is dropped on top of the to rigid wall and the other end to a block (mass
first disk. The two disks stick together and rotate m) which rests on a smooth horizontal plane.
as one about their axis of symmetry. The new The block is set in motion with a speed v. What
anbrul.ar velocity of the system as viewed from is the maximum distance the block will travel
above is close to - [2015] after the wire becomes taut? [2015)
(A) 18 rcvolutionslsccond and clockwise
(B) v~2mL
(B) 18 revolutions/second and counter clockwise AY
(C) 3 revolutions/second and clockwise
(D) 3-revolutions/sccond and counter clockwise -
(C)v~
-
2AY
(D) L ,~v.
\AY
50. A boy is standing on top of a tower of height
54. The moments of inertia of a· non-uniform
85 m and throws a ball in the vertically upward circular disc (of mass M and radius R) about
direction with a certain speed. If 5.25 seconds four mutually perpendicular tangents AB, BC,
later he hears the ball hitting the ground, then CD, DA arc L, h, I) and Le, respectively (the
the speed with which the boy threw the ball is square ABCD circumscribes the circle). The
(take g == 10 mll, speed of sound in air= 340 mis) distance of the center of mass· of the disc from
12015) itc; geometrical center is given by., (2015}
(A) 6 mis (B) 8-m/s (C) 10 mis (D) 12 mis I I 2 2
~ -v(l1 -I3) + CI2 -~4)
51. A simple pendulwn of len&rth e is made to
1 I 2 .2
oscillate with an amplitude of 45 degrees. The (B) - - -v01 -13) +(12 -14)
12MR
acceleration due to gravity is g. l..ct To= 2rc ~.e I g . I- I 2 ~
(C) - - "\1(11 - l2) + (13 - l4f
The time period of oscillation of this pendulum 3MR
will be - (20151 l J 2 2- ·
(D) - - -v01 +13) +(12 +14)
(A) T0 irrespective of the amplitude 2MR
(B) slightly less than To
55. An object is propelled vertically _to a maximum
~lightly more than T 0 height of 4R from the surface of a planet of
(D) dependent on whether it swings in a plane radius R and mass M. The speed of object
aligned with the north-south or east~wcst when it returns to the surface of the planet is -
directions [2015]

52. The dimensions of the area A of a black hole can


be written in tenm; of the universal gravitational
fl2 ~2~~M (B)~GM
2R
constant G, its mass M and the speed of light c
(C) J3GM (D) ~GM
as A= G0 M~cY_ Here - (2015) 2R SR

www.puucho.com
CP Publication KVPY chapterwise with www.puucho.com

TOPIC· WISE KVPY [STREAM SX] Questions with Solutions


56. Uniform ring of radius R is moving on a 60, Consider two masses with m 1 > m2 connected
horizontal surface with speed v and then climbs by a light inextensible string that passes over a
up a ramp ofi'nclination 30° to a height h There is pulley of radius R and moment of inertia I
no slipping in the entire motion. Then his (2016]
about its axis of rotation. The string does not
(A) ,j-f2g (3//g (q 3v2/2g (D) 2-?/g
slip on the pulley and the pulley turns without
57. A particle with t?tal mechanical energy, which friction. The two masses are released from rest
is small and negative, is under the influence of separated by a vertical distance 2h. When the
a one dimensional potential U(x) = x4/4 - x 2/2 J
two masses pass each other, the speed of the
Where x is.jn meters. At time t = Os, it is at
masses is proportional to [2016]
x =- 0.5 rn Then at a later time it can be found
(2016]
(A)
(A) Anywhere on the x axis
(B) Between x = :---1.0 m to x = 1.0 m
i.,,-(C) Between x =- LO m to x = 0.0 m
(m 1 +m 2 )(m 1 -m 2 )
(D) Between x = 0.0 m to x = 1.0 m (B)
l
m1 +m, +-2
58. A particle at a distance of l m from the origin - R
starts moving such that dr/dO = r, where (r, 0)
are polar coordinates. Then the angle between
resultant velocity and tangential velocity component
is 12016]
(A) 30 degrees
..JE) 45 degre_es
(D)
(C) 60 degrees
(D) Dependent on where the particle is
61. A smaller cube v,ith side b (depicted by dashed
59. A light balloori filled with helium of density
lines) is excised from a bigger uniform cube with
PHc is tie<l to a long tight string oflength eand the
string is attached to the ground. If the balloon is
side a as shown below such that both cubes have
displaced slightly in the horizontal direction from a common vertex P. Let X = alb. If the centre of
the equilibriu~ and released then. [2016] mass of the remaining solid is at the vertex. 0 of
(A) 1he ballon tmdergoes simple hannonic motion smaller cube then X satisfies. [2016]

with period 2n. ( Pair )!..


. Pair - Pttc g (----- ,}~ 1b
(B) lhe ballon w,dergoes simple hannonic motion I .. {.
I ·1,i,
I I ,

with period 2n ( Pair - p lie !:_


Pair
Jg a
(jI ______ ..,I ,

~ ) The ballon un?ergoes simple harmonic motion

with periog 2n ( PHe )!:_


Pair -PHe g
(D) The ballon undergoes conical oscillations with
.)!;''f<3 X 2 -X - 1 = 0
0
-

(13) X • -X - 1 = 0
· d2 (Pair +Ptte)'
peno 1t - (C) X 3 + X 2 -X- l =0
Pair - PHe g (D) X 3 - X 2 -X + I = 0

www.puucho.com
CP Publication KVPY chapterwise with www.puucho.com

t MECHANICS
62. Consider a bowl filled with water on which
some black pepper powder have been sprinkled
uniformly. Now a drop of liquid soa·p is added
at the centre of the surface of water. The
picture of the surface immediately after this
will look like 15 Nov, 20171

(B)
@ .
(A) o----M- M

(D)C:)
63. Which of the following plots represents
schematically the dependence of the time .5Y 0----M-(D) 01--L----M-

period of a pendulum if measured and plotted


as a function of its oscillations? (Note :
· amplitude need not be small) 15 Nov, 2017]
65. Two satellites S 1 and S2 are revolving around a
planet in the opposite sense in coplanar circular
I- I- concentric orbits. At time t = 0, the satellites
"I:! ',;I
0
0
are farthest apart. The periods of revolution of
~8 (B) ·5
0
E
S 1 and S2 are 3 h and 24 h respectively. The
b i= raditL-. of the orbit of S I is 3 x 104 km. Then tbe
AmplitudeO Amplitude8 orbital speed of S2 as observed from
(5Nov,2017J
I-
I-
',;I '1::1 (A) tbe planet is 41t x 104 km h-1 when S2 is
0 .9
cq·s (D) t
closest from S 1•
0
E
i= b
s (B) the planet is 2x x l 04 km h- 1 when S2 is
closest from S1.
Amplitude 0 Amplitude 0
(C) S1 is ;r x 104 km h- 1 when S2 is closest
from S1
64, On a pulley of mass M bangs a rope with two
~ 1 is 31t x 104 km h- 1 when S2 is closest
masses mi and m2 (m1 > m2) tied at the ends as
from S1
shown in the figure. The pulley rotates without
any friction, whereas tbe friction between the
66. Force F applied on a body is written as
rope and the pulley is large enough to prevent
any slipping. Which of the following plots best F = {n.F) n+ G, where n is a unit vector .
represents the difference between the tensions The vector G is equal to [5 Nov, 2017]
in the rope on the two sides of the pulley as a (A) a X F (B) rlx(nxF)
function of the mass of the pulley?
(C) (nxF)xF/JFI ~nxF)xn
IS Nov, 20171

www.puucho.com
CP Publication KVPY chapterwise with www.puucho.com

TOPIC•WISE KVPY [STREAM SX] Questions with Solutions

67. One end of a rod of length L'·"l m is fixed to a 69. The· figure of a centimeter scale below shows a
point on the circumference of a wheel of radius particular position of the vernier calipers. In
this position the value· of x shown in the figure
R == 11.[:i m. The other end is sliding freely
is ( figure is not to scale) (5 Nov, 20171
along a straight channel passing through the
center O of the wheel as shown in the figure
below. The wheel is rotating with a constant
angular velocity ro about 0. I5 Nov, 2017]

(A) 0.02 cm (B) 3.65 cm


(C) 4.15 cm ,. . . J.J?) 0.03 cm
70. A star of mass M (equal to the solar mass) with
a ·planet (much smaller than the star) revolves
The speed of tl1e sliding end P when 0 =60° is around the star in a circular orbit. The velocity
(B) (J)
of the star with respect to the center of mass of
3 the star.planet system is shown below :
[5 Nov,2017)
(D)~
..fi 80 I I
--- 60 I \ (' \
]_ 40
68. A solid cube of wood of side 2a and mass Mis 20
I \ I \
0
'g 0 \ I \
resting on a horizontal surface as shown in the \ I I
j -20
\ \
figure. The cube is free to rotate about a fixed ...."'
---40
\ J I
5 --60
a:tis AB. A bullet of mass m (<<M) and speed CFJ /
-80
v is shot horizontally at the face opposite to 0 2 3 4 5
days
ABCD at a height 4a/3 from the surface to
The railius of the planet's orbit is closest to
impart the cube an angular speed (J.)_ It strike the
(1 A. U. = Earth-Sun distance)
face and embeds in the cube. 1l1en ro~ is close (A) 0.004A. U. (B) 0.008 AU.
to (note : the moment of inertia of the cube JGT0.004 A.U. (D) 0.12 A.U.
about an axis perpendicular to the face and
71. The distance s travelled by a particle in time tis
passing through the center of mass is 2Ma2/3)
1 2
[5 Nov, 2017] s=ut- -gt
2
~---~C
The initial velocity of the particle was measured
D
to be u = 1 . l I ± 0.0 I mis and the time interval of
the experiment was t = 1.01 ± 0.1 s. The
acceleration was taken to be g""' 9.8 ± 0.1 mltl.
With these measurements, the student estimates
the total 'distance travelled, How should the
student report the result? [19 Nov, 2017)
(A) Mv/ma (B)Mv/2ma
(A) 1.121 ± 0.1 m ...,-(B) 1.1 ± 0.1 m
(C) mv/Ma J(6) mv/2Ma (C) 1.12 ± 0.07 m (D) 1.1 ± 0.07 m

www.puucho.com
CP Publication KVPY chapterwise with www.puucho.com

• MECHANICS
72. A massive black hole of mass m and radius R is 75. A unifonn rope of total length l is at rest on a
spinning with angular velocity w. The power P table with fraction f of its length hanging (see
radiated by it as gravitational waves is given by figure). lf the coefficient of friction between the
P = Gc-~mxRyro'\ where c and G are speed of table and the chain isµ then {19 Nov; 20171
light in free space, and the universal·
gravitational constant, respectively. Then

(A)x=-1,y=2;z=4
(B)x=l,y=l,z=4
(C) X =-1, y = 4, Z = 4
._)B) X = 2, y = 4, Z = 6
[19 Nov, 20171

,~
I=µ
(A) (B) J= 11(1 + µ)
73. One end of a rod of length L is fixed to a point yr1 = 11 o + 11µ) (D)/ = 1/ (µ + 1/µ)
on the circumference of a wheel of radius R.
The other end is sliding freely along a straight 76. A star of mass M and radius R is made up of
channel passing through the centre O of the gases. The average gravitational pressure
wheel as shown in the figure below. The wheel compressing the star due to gravitational pull of
is rotating with a constant angular velocity co the gases making up the star depends on Ras
21t 119 Nov, 20171
about 0. Taking T = the motion of the rod

lS
0)

(19 Nov, 2017] 0)+. (B) _!_


R
(C)-1 1
R2 (D) R6

77. A small ring is rolling without slipping on the


circumference of a large bowl as shown in the
figure. The ring is moving down at P1, comes
(A) simple harmonic with a period ofT down to the lower most point P2 and is
(B) simple harmonic with a period ofT/2
climbing up at P3. Let vCM denote the velocity
~Ynot simple harmonic but periodic with a
period ofT of the centre of mass of the ring. Choose the
(D) not simple harmonic but periodic with a correct statement regarding the frictional force
period ofT/2 on the ring. 119 Nov, 2017)

74. A rope of mass 5 kg is hanging between two


supports as shown. The tension at the lowest
point of the rope is close to (take g = 10 mls2)
(19 Nov, :Z017] ; -'
''' '
''

30° (A) It is opposite to vCM at the points Pi, P2 and


P3.
(.~--~~s opposite to VcM at P1 and in the same
direction as vcM at P3_
(C) It is in the same direction as vCM at P 1 and
(A) 22 N (B) 44 N opposite to vo,,.1 at Pl.
(C) 28 N .(P)-14N (D) It is zero at the points P 1, P2 and P 3•

www.puucho.com
CP Publication KVPY chapterwise with www.puucho.com

TOPIC• WISE KVPY [STREAM SX] Questions with Solutions


78. A solid cube of wood of side 2a and mass M is These beakers (without stand) are placed on
resting on a horizontal surface as shown in the weighing pans and register reading Wa, Wi,, W c
fib'llfe. The cube is free to rotate about the fixed and Wd for a, b, c and d, respectively. (Effects
axis AB. A bullet of mass m (<<M) and speed of the mass and volume of the stand and string
v is shot horizontally at the face opposite to are to be neglected) [19 Nov, 2017]
ABCD at a height 'h'. above the surface to
impart the cube an angular speed mc so that the
cube just topples over. Then roe is (note : the
moment of inertia of the cube about an axis
perpendicular to the face and passing through
the center of mass is 2Ma3/3) I19 Nov, 2017)
a b
--~·C

A
C d
(A) ~3gM/ 2ma (B)-.J3g/4h
(A) Wa =Wb=Wc= Wd
(C) ~3g(Ji-1)/2a _JJf ~3g(./2-l)/4a ~B)Wb=Wc>Wd>Wa
(C) Wb =We> W 0 > Wd

79. A unifonn thin wooden plank AB of length L (D) Wb > We> Wd> Wn
and mass M is kept on a table with its B end
slightly outside the edge of the table. When an
impulse J is given to the end B, the plank
moves up with centre of mass rising a distance
'h' from the surface of the table. Then-
119 Nov, 2017]
(A) h > 9J 2/8M2g
(B) h = J2/2M 2g
~ J2/2M 2g < h < 9J2/8M2 g
(D) h < J 2/2M 2g

·so. Four identical beakers contain same amount of


water as shown below. Beaker 'a' contains only
water. A lead ball is held submerged in the
beaker 'b' by string from above. A same sized
plastic ball, say a table tennis (TT) ball, is held
submerged in beaker 'c' by a string attached to a
stand from outside. Beaker 'd' contains same
sized IT ball which is held submerged from a
string attached to the bottom of the beaker.
"' \'
\

www.puucho.com
CP Publication KVPY chapterwise with www.puucho.com

• MECHANICS

ANSWERS
L (A) 2. (A) 3.(8) 4. (C) 5, (B) 6. (D) 7. (D)

8. (A) 9. (A) 10. (B) 11. (B) 12. (A) 13. (B) 14. (A)
15. (D) 16. (A) 17. (A) 18. (C) 19. (A) .!O. (A) 21. (B)
22. (D) 23. (D) 24. (C) 25. (A) 26. (A) 27. (B) 28. (A)
29. (C) 30. (C) 3 l. (B) 32. (C) 33. (A) 34. (D) 35. (A)
36. (A) 37. (C) 38. (B) 39. (A) 40. (D) 41. (B) 42. (C)
43. (A) 44. (8) 45. (C) 46. (A) 47. (C) 48. (B) 49. (D)
50. (B) 51. (C) 52. (B) 53. (A) 54. (A) 55. (A) 56. (B)
57. (C) 58. (B) 59. (C) 60. (C) 61. (A) 62. (C) 63. (A)
64. (C) 65. (D) 66. (D) 67. (A) 68. (D) 69. (D) 70. (C)
71. (B) 72. (D) 73. (C) 74. (D) 75. (C) 76. (A) 77. (B)
/
78. (D) 79. (C) 80. (B)
'

:=- .. __,, - .. _ - .--~ -


~ ~~~
SOLUTIONS

:. f = f1 + f2 + Ma
l. f == µ1mg + µ2(m + M)g + M(µ1g)
:. f = (rn + M)(µi + µ2) gj
For pen to start slipping maximum horizontal
I I I I
force on it is f= µ 1mg '+--x
I
,<1---X,-+1I k ' 2----flI
:. a= µ1g is the maximum common ~~I I

2. ~OOOCiOOOD4W0 :
acceleration for both pen and paper I
I

F.B.D. for both pen and paper


Let the masses be slightly displaced by x 1 and x2
from this equilibrium position in opposite direction
so net stretch in spring is x = x1 f- x 2• Because of
this a restoring force kx will act on each mass and
therefore equation for m 1 & mz will be
cfx,
and m, - - - =-k:x
- dt 2
butasx=x 1 +x2
d 2x '
d-x, '
d-x,
. -=--+---
.. dt 2 dt 2 dt 2

d 2x 1 d2 x 1
F-f1 -f2 =Ma replacing values of - 2- and - - from
dt dt 2
also f2 = µ2N2 = µz(m + M)g acceleration equations we get

www.puucho.com
CP Publication KVPY chapterwise with www.puucho.com

TOPIC•WISE KVPY [STREAM SX] Questions with Solutions

d2:2 = -(-1-+_l_J
dt m1 m2
kx

4.
also if - 1- + - 1- = ..!.. then mis the effective
m1 m2 m
mass in this case therefore
d 2x 2 -kx 2 k 2n: 0.4+y
- =-cox= - orffi = - andT= - r=--
dt 2 m rn ro 2

also ( ;: J J = ( ;: by Kepler's law of time-

periods {T1, r 1 are taken for earth)

3.
.-. (,JJ =[(~AUJ
Let the angle ofT with the vertical he 8 then
solving we get&= 49.6 Aij
F.B.D.
s. Book will loose contact with the shelf when a= g

-c-o~
Tsin8 Tsin8 Now l a I= ro2x :. g = cl A
(A~ Amplitude)

!Tcos8
lrcm8 ri = 1t
A
also f;.. ~
2n:

:. f=-1 18
:. 2T cosO =ma 2n:VA
X replacing g = 10 rrJs2 a~d A= 2.5 x 10-2 m
also cos 8 = ~x' +(¼)' We get [= 3.18 Hzj

~Vo
£2 2 i}. 6. [iiiil-,oooooooo4mil
given .t>>x -+x ::: -
4 4
-2Tx ( . . t1 • ·· · _ m1v0 +O _ m1(0)+m2 (l)
a= m( ¾)
negative s1~ or restori~~ .·. . , VcoM - ---"--=-- ' XiCO~i -
m1 +m2
---=...c...c..._-=--c...;..
m1 + m2
also XcoM = XiCOM + VcoMt
force)
2i
. XcoM -
•• - (- m
~ -) m 1v 0 t
+-~~
4T) x a lso thi s 1s
or a = - ( m.e . .-1ar to the
. s1nn m 1 + m2 m 1 +m 2

equation of SHM i.e. a= - olx 7. Total distance=

w= ~4T & f= ~ ( v; + r2 v~ + r4 v; + ...upto 101h terms)- h


mi 2n: 8 g g
v2
f= _I ~4T = - 0 (1 + r 2 + r 4 + .... + 10d' term)-h
21e mi g
also v 0 = ~2'/)i

www.puucho.com
CP Publication KVPY chapterwise with www.puucho.com

• MECHANICS

.·. Total distance =~h [l-(r


1- r-
)-h
·
2
t
10. •
2h(l-r 20 )
or otal distance = ~ - CM
(1- r-)

aa > ac> aA
3
8. v= (GM also T = lrrr or T = 2rr,/ r aa=g
-y---;- v GM
aAICM = aA -aCM (t)
when v is replaced by \ ~ aBICM = all -aCM (J-)

now g.,lf= g- ai R,, cos 2 ~


11. For lower block +ve lift, kx 2 mg
g-olR cos0°
now f= e :::;,x2 mg
g - olRe cos 60° k
. 4g(f-l)
solvmg we get Re= -~- -
co-(f - 4)
GM - olRe
or - ( f - l ) = - - ( f - 4 )
RC 2 4
· 3_ 4GM (f -1)
Ro; - ~ (f-4)

nowp= 4
M ____
,.. 3 ol (f-4)
16 nG (f-1)
-nR
3 e
3

. 2n 3n(f-4)
also T = -; :. p = 4T2G(f -1) W/E theorem

-mg (h + x) + (½kh -½kx


2 2) = 0 -0
9. V(x) = ½lot 2 - V0 cos(~J m2g2 1 ' I m2g2
:::;,-mgh- - - + -kb·- - - - =O
k 2 2 k
E = - dV =-(kx + V0 sin(~) x .!_) kh2 3m2g2
dx. a a --mgh---=O
sincex<<a 2 2k
mg ±~m2g2_ +3m 2g2
. (XJ ~ ;
:, SlD ; X h ... -...::...__.!....---=..
k
_..;;;.._

mg± 2mg _ 3mg -mg


or E = -(k + ~~ ) x = k __k_'_k_

This resembles F =- kx :. h= 3mg


k
W+mg=kh
W+mg=3mg
I 2 W=2mg
[' = 2n m.a
1 ka2 +vo

www.puucho.com
r'
CP Publication KVPY chapterwise with www.puucho.com

TOPIC• WISE KVPY [STREAM-SX] Questions with Solutlohs


12. InSHM x2 y2
16. -+-=I
a=-cix a2 b2
v = co ..fA 2 -x 2 llx = uat (O. b)
if'= ol (A2 -x2) uy=O
a2 2x clx + 2y dy =0
J= ul A2 -co2 x -
CO2 a2 dt b2 dt
2
Again cliff. w.r.t. to time
J+ ~ =ci A2 2x d2x+ 2 (clx)2 2y d2 y+ (dy)-o
ol 2
a 2 dt 2 a 2 dt + b 2 dt 2 b2 dt
v2 a2
--+--=l acceleration at (0, b) is
oiA 2 ro 4A2
ay= 2 u
-b 2
i.e. ellipse
a

V
Vxi P
13. R=-=-=- 17, .,......v........~ is·dimensionless
ii2 i2 ~a/pR~
h
energy= hv = -- kpgR 2 I a is also dimensionless
t
Power= .energy
t 18, By using parallel axis theorem, I= 1 1mr1 !
h
P=-
t2 19. Using conservation of angular momentum
. e v,.r, = v,..r,.
1= -
I Vp _ rA a+ae
p h
---=--
v,. r., a-ae
i2 - ~
8U 3 4
21. F=--=-ax -~x
14. Precession mean ox
every time reading is coming nearly same.
(8:1)
OX 11=0
=O & (
02
~) =O
OX it=O

.ts~ . .
1' ."-
22. [a.] =MLr2
[~]= MI!r2
[y]= L

g 23. Let velocity of each blocks are v, and v2


respectively then
Radical acceleration mv1 + 2mv2 = mv
= V2 = 2gisin8 2 gsinO
wherev= J2gh & v2-v1 = v
R i hence from above equations,
tangential acceleration = g cos a V
we have v, = --
J
total acceleration = 4g2 sin 2 9 + g 2 cos 2 e 3

~ g../1 + 3sin 2 8 bcx:v2 or h'= h


' 9

www.puucho.com
CP Publication KVPY chapterwise with www.puucho.com

• MECHANICS .,

24. mg-
, kv2 =mdv
--
•I + •2 + 't3 = 'tnct
dt R
2FR + FR + - F = •net
2
25. Rocket will remain bound to the solar system
till its mechanical energy becomes zero . J.5FR= Tuel

. GMm 1 2 ·
1.e. - - - -mv =0 29. Based on dimensional analysis.
r0 2 p = [IvlL- 3]
1 2 GMm 2 2GM g=Lr2
-mv =- - :::> v = - -
2 r0 r0 CJ"" [M12]
2 2 2GM L::::L
a vo = - -
ro pgI.2 = ML-3 xLT-2 xL2
2 GM
Ci X - - - = - -
2GM CJ Mr 2
To ro = (MoLor°)
a=Ji
f
N + mro2 R cos 45°. sin 45°
4 +0 mro R cos 45°
;------'~.-- 2

26. mro2 R cos 45° cos 45°


30. ---

T+ k 1x-mg = ma .... (i)


k2x + mg-T = ma .... (ii) ..
'. Plank is in rest
From equation (i) & (ii)
1 1
T= (k~ ;k1 )x + mg and
f= mID
2
R. ..fi. . .Ji

a -- ( k,2+mk2 ) X ifT is non-zero.

If T is zero, then
dU
k 1x-mg k 1x 31. F=--
a1 = ~-- =- -g dx
m m
(KE)mm = 1J
k 2 x+mg k2 x
a2 = ---=-----=- = -- + g I " 1
m m -mv-= =) .!. x 0.5v2 =I
2 2
27. By energy conservation v2=4 :::> V = 2m/s

Ian ~=ah~ with such speed it able to cross x =-2 and then
it will go to co.

32. By principal of conservation of momentum


28. mv0 = (M + m)V ... (1)
by principal of -conservation of angular
momentum

www.puucho.com
CP Publication KVPY chapterwise with www.puucho.com

TOPIC• WISE KVPY [STREAM SX] Questions with Solutions

ro ( 2 NIR. +mR
5 2 2) == n;iv 0 (b-R) ... (2)
35. Using energy conservation
2
J
- x Mv2 + -1 x -2 .MR2 x ( -V ) = Mgh
Vo"" Rw 2 2 5 R
by (I) and (2) v2 v2

h 7M+ 10m 2 + s c= gb

R 5(M+m) 7v 2
10 ::gh

~ v==Por
34.
36. Using work energy theorem conservation at A & B
A

H
t
Total energy remain constant l
I 2 I 2 , 2
I ~
TE= - Mv + - x - Mr" x <O + Mg [R- r] mgH-µmg(d+x)--kx~=Af<..E =O ... (I)
2 2 5 2
using work energy theorem at B & C
dTE
[1-cos8] = - 1 2
dt -kx -µmg(d+x)-mgh=O ... (2)
2
dv :Mr 2 1 dv
M xv - + - - x - x 2 v - +Mg[R-r} from (I) & (2)
dt 5 r 2 dt mgH- 2 µmg (d + x) =mgh
(+ sin 9) dO = 0 h =H-2µ (d + x)
dt
dv 2 dv . v
~ v-+-v- +g[R-rJsmO x - - =O
dt 5 <lt R-r

'!.... a + g sin8 = 0
5
5 J= mv .... (1)
a=- - g9
7 where vis the velocity of centre of mass.
5 After impulse rod get angular velocity ro
a(R-r) = --g8
7 Angular impulse = lro

:. a.=- 5g0 JxL= m(2L)2 xro .... (2)


7(R-r) 12

1 = mLco
co-~
v?(R-rj 3

T = 2 11: ~7(R -r)


5g
lro= ! I
from equation (1); v :=:.I_
As r increase T decrease slightly. m

www.puucho.com
CP Publication KVPY chapterwise with www.puucho.com

t MECHANICS

Kinetic energy= KE= -mv +-Iro

1 J2 1 mx4L
2
2 9J2
2
1 2 1 2

F--qk<-[!t~k]
=> -m-+-x x--
m2I1.
2 2
Jl 36J 2
m 2 12
rnciR=-qk => m(~J R=-qk
:::) -+--
2m 24m m(4rc 2 )R =- ·k :
4812 21 2 T2 .
=> --=>-
24m m => "r2 ex: R => T a:: R 1f2

38. If perfect rolHng (solid cylinder P) 40. Block slides downward.along the inclined
According to energy conservation law plane with acceleration g sin a.

mgh= -1 mvp2 + -1
2
1 -
2
(v• )R
2

Herc,
I -+ moment of inertia, R -+ Radius
mR"
I=--
2
V
(!) = ....l.
R mg s~·in2a. mg sina.
l 2 1 mR 2 v; •
mgh= - mvp+ - - - - - mg sina.cosa.
2 2 2 R2
, ,"
mgh = _!_ mvi [1
2
+.!.]2 =.!_2 mv~ x!2 ., ' '
,
mg At equilibrium
,') a T =FNcl
3 2
mgh =-mvp
4
2 4
Yp = -gh .... (1)
3
If sliding without friction
(solid cylinder Q)
According to energy conservation law
1 2
mgh= 2 mvQ mg[l -sin2 a.J~ mgcos2 a.
=> v~ = 2gb .... (2) mgcos 2 a
tan 0 = - - - - -
from equation (I) and (2) mg sin a. cos a.

VQ _ 2gb
-2 - - - - - => - - -
_ 3 VQ _
/½ 3 tan 0 = cot a.

V~ (tgh) 2 Vp 2
tan 8 = tan (90 - a)
0 =(90-a.)
string is peipendicular to inclined plane .
39. q -+ constant

www.puucho.com
CP Publication KVPY chapterwise with www.puucho.com

TOPIC- WISE KVPY [STREAM SX] Questions with Solutions


41. According to Bernoulli theorem F = N + 8a = 10 a [from equation (l)]
In the region of narrow cross section of pipe, F= 10 x20=2OOncwton
KE of fluid will be greater and pressure energy
will be lesser.
=:> less pressure results into larger in size of air
bubble and greater KE results its greater speed. , ·,-T2
44.
42. Initial sphere is slipping and finally it start
@Qv=O
rolling. During its motion t about point of @ V r~gcos 0
contact is zero. mg
.-. Angular momentum of sphere about point of Centripetal force at point A :
contact remain conserved. mv 2
T1....;.mg= - - •... (1)
l
AtpointB:
T2=mgcos8 .... (2)
According to question
Slipping Rolling T1 =4T2 .... (3)
mv 2
=:> mg+--= 4 mg cos e
l
Im= (I+ MR.2) ro'
[from equation (I) & (2)]
¾MR 2 @= (¾MR +MR }al'
2 2
:::> mg (4 cos 0-1) = -
rnv 2
....(4)
f.
I 2© According to conservation of energy between
ro = -
7 pointAandB
1 2
43. According to free body diagram Also -mv + 0 = 0 + mgf. (1-cos 0)
2
---?a mv2 =2 mgf (I -cos 0)
N'
rnv 2
- = 2 mg(l -cos 0) ....(5)
l
F X From equation (4) & (5)
N
mg (4 cos 0-1) =2 mg(l -cos 0)
4 cos 8 - I = 2 - 2 cos fl
8g 6cos8=3
F-N=8a .... (1) I
cos 0= -
N=2a .... (2) 2
f=2g=20 .... (3) 8=60"
=:> µN =20
20 20 45. CT = ho: kl c1
:::;, N = - = - = 40 .... (4)
µ 0.5 cr --), Steffan boltzmann constant
::::;-,N=2a h--), Planck's constant

N 40 , k0 --), Boltzmann constant


:::;, a= - =- = 20 mfs- .... (5)
c--), speed of sight
2 2

www.puucho.com
CP Publication KVPY chapterwise with www.puucho.com

t MECHANICS
According to stcfan's law Displacement in time 't'
I
_g_ =a"t ~X=vot- -µbrt
2

At 2
Q I I ~
0= - X - 1 = 2Vo- - µg(2t
At T4 2
2vo = (I + 2µg)
[cr] = [M1L2r2] = [MIT"3K-4]
[L2 ][T][K4 ]
vo= (½+ µg) .... (ii)
E [M 1l:r2 ]
& E=hv => h= - => [h] = - - - l
v [T-1] - + µg> µgt (t= 2 sec)
2
[h] = [M1L211]
1
[c] = [LT 1] - + µg>2µg
2
- 3k T
E- [k ]- [M1L2T-2]
- ll => B - - - - - 1
2 [K] µg<-
2
fknl = [M1L212K-1]
1
According to homogeneity principle of => µ< -
2g
dimension
=> µ<0.05
[Mlr3K-4] = [MILTlf [M1L2rK-1i [L111f
on comparing powers00OO000O ofM, L, T, K
on both sides
n+Jl=l .... ( 1)
2o.+2p +y=O .... (2)
-a-2P-r=-3 .... (3)
47.
-P=-4 .... (4)
~=4
o.=-3
Putting values is equation (2)
2 (-3)+ 2 (4) + y= 0 Equal are
y=-2
..!..ah=ab
Ans: 2
a=-3 h=2b
P=4
M 1 .!::.
= Mi~ [centre of mass of combination
y=-2 3 2
at the mid-point of their common edge]
+-- =I~
46.
µ:g CH: Ml
M2 2 h

a= µg ~ d = Im -----..i
:: = ¾[½]
Velocity after time t M1 =i
M2 4
v= Yo-µgt
v::2:0::::) vo;.:: µgt .... (i)

www.puucho.com
CP Publication KVPY chapterwise with www.puucho.com

TOPIC-WISE KVPY [STREAM SX} Questions with Solutions


48. 50.

e
V)
00
II
:!:: Sound

V,
CM

Using concept of COM Time taken to reach sound after hit t, = _!!_
v.
m1r1 = m2r2
r1 + r2 = 2R t~ = ( 3~ 0 ) sec ; ts = 0.25 sec

For ball time of flight T1


( : : +1)r2 =2R
Ti+ t, = 5.25 sec
2m 1R T1 = 5.25 - 0.25
r2 =
m1 + m2 T1 = 5 sec

L sin ih = r2 IR<< L] 1 ~
For baU V0t - -gt- =-II
2
e~= 2m 1R
~ (m 1 +m 2 )L =!> Vo (5) - }_ 10(5)2 = - 85
2
5Vo = 40
49.
V 0 = 8 mis

51. T = 2rc Jf [ 1+ ~:) (11ris is valid when e ls

not small)

To= 2rc If (for small 0)

2
T=T0 [ 1+ 9 16)
T>T0

Their axis of rotation is common. 52. A=G"M 13 C1


AnguJar momentum conservation [M0L 2T°] = ~ r 2f' [M)1l[LT 1)7
1L 3

l1ro1 - w,.h = (11 + 12) w -a.+p=o :::;,. o.=P


2rc (4.25) N 1 -2rc (1.8) N 2 =(4.25 + 1.80) N (2rc) 3a. +y=2
{4.25 x 15 - 1.8 x 25) = (6.05) N -2a-y= 0
63.75-45 = 6.05 N on solving
N = 3 rev/s. a = 2, 13 = 2, r =--4

www.puucho.com
CP Publication KVPY chapterwise with www.puucho.com

• MECHANICS

53. Initially wire is slack so it do not have any 55. Using energy conservation
deformation energy. When block is given some _ GMM + ..!_ mV 2 = _ GtvL\1
velocity it move due to kinetic energy, one wire R 2 SR
get taut. Internal force get develop in wire and
V= 2 ~2GM
KE start decreases and deformation energy of 5R
wire increase. Till block come at rest using V is the velocit:- · by which obj cct is projected.
energy conservation When object return to earth its speed will be V.
1 ~ I . 2
- mv- = - Y x (stram) x A x L
2 2
56. I
-mV 2 + -mR 2 -
2
I (V)
2 R
2
=mgh

{Using conservation of energy)

[2y2 + 2y2) = mgh


\

y2
h=-
g

54. --- - -- f=
57. at t = 0, X = 0.5
x4 x2
u=---
4 2

D C :::) ..!.. X _!__ ..!.. X ..!.. :::) I_!. I


4 16 4 2 4
du 4x 3 2x
Icm+m(R.+y)2=l3 ...(1) -=---==x3 -X
dx 4 2
I=+ m (R-y)2 = I1 ...(2) du 2
- =x (x -1)
from (I) & (2) dx
11 -I3 = m [(R-y)2-(R +y)2]
-du = 0 at pomt
.
o f maxima
. & mJmma
..
11 - I3 = m (2R) (- 2y) ... (3) dx
1cm' + m (R + x)2 = 4 ... (4) d~:)Q ;
[-dx· X =~ l , f ,
1cm' + m (R-x)2 =12 ... (5) , = - I pomt o maxima
X ,.Q
from (4) & (5)
(h - 14) = m [(R- x)2- (R + x/J d2u)
[ dx . o f 1ruruma
= 2.pomt ..
2
Ii - 14 = m f(2R) (-2x)] ... (6) x-±l

(3f + (6) 2
:::) (11 -13)2 + (Ii- 14)2 = (m2 x 4R2 x 4(x2 + /)
distance of CM from -1 0 +

4mR

particle will found between (-l, 0)

www.puucho.com
CP Publication KVPY chapterwise with www.puucho.com

TOPIC· WISE KVPY [STREAM SX] Questions with Solutions

60.
(r, 9)

58.

_ i_ _ - -
2h
m1 ___

t
PE=O
h ---- h-------·
i
V V

The total mechanical energy of system = conserved


Hence , KEi + PEi =KEr + PEr
I 2 1 2 I 2
0-01;2gx2h= -m2v-+-m1v + -Ico
2 2 2
AB = Direction of resultant velocity
-
AD =Direction of tangential velocity
-m1gh-m2gh

Also co= -
V

dr r R
't;/tana= - =-
rd9 r
tan a= I
a.= 45°

59.

V OC

61. Centre of mass of remaining cube x coordinate = b

xrn1~--,a - ', .7

0 y
3 a b
t"O= V (pAit- Pue) g l sin 8 pa x--pb 3 x-
For small angular displacement (8) xCM-- 2
3 3
2
pa -pb
to = V (PAir - PHe) g l e
Ia.= V [PAir- PHc] g C 9 We will consider removed mass as a negative mass
PIie V t2 a= V [PAir- PH,] f. 8 g pa4 - pb4

a= [P/\ir-PHe] ! 9 b= 2 2
Pne I. pal -pb3
3 4
ab-b = - - -
a4 b4
2 2
2a3b-2b4 = a4 -b4
= 21t l Pue put a= bx=> 2b4 x3 b- 2b4 = b 4x4 - b4
B (p Air -P11e) 2x3 - 1 = = x4

www.puucho.com
CP Publication KVPY chapterwise with www.puucho.com

• MECHANICS

a.3-2+ 1 =x4 65.


= (x2 - 1) (x2 + 1)
2[x3 - l]
2[x- l][x2 + l + x] =[x- l][x + l][x2 + l]
2x2 + 2 + 2x = xJ + x + x2 + l
x3 - x2 - x - 1 "" 0

62. due to soap bubble surface tension is reduced


4 3
· therefore in that area. Black paper powder will 9 -(3xl0 )
242 - -r3-
sink.

63. Time period will increase as the amplitude is -3x3


- -_ (3xl0
24x24
--
T
4 3
J
increases.
3xl0 4
64. 4 r
r= 12 xJ04
orbital speed of S2 seen from planet = roi r

= -2:rc 4
X }2 X 10
24
= 1t X 104 km b-1
mg-T, =m1a ... (I) 21t 4
V1 = ro 1r1 = - x 3 x 10
T2-m2g= m2a ... (2) 3
::::,. 21t X 104 km h-l
(T1 - T2) x R • Ia [·.· a= r a]
Ia
R
(1) + (2)
(m1 - m2)g + (Tz -Ti)= (m, + m2)a

(T1 -T2) = -¼[(mi -mz)g+(T2 -Ti)]


R m 1 +m 2 (2t + ~=) t=1t

=>(Ti-Ti) [1+ 2 I ] ~=_!_


k (m 1 + m 2 ) 24 2

= I (m 1 - m 2 ) g
12
t= - hr
k2 m1 +m2 9
Angle rotate by both satellite
_ [
:::::, T1- T2-
I(m1 -m2)g ] 2n: 12 8n:
I+ (m1 + m 2 )R 2 01=- X-=>-
3 9 9
1 2 21t 12 :re
= 211R (m1 -m2)g 02= - )( - => -
24 9 9
(½MR 2 +(m1 +m 2 )R 2) velocity of S2 seen from S1 = V1 + V2
= 31t x' 104 km h- 1

www.puucho.com
CP Publication KVPY chapterwise with www.puucho.com ,r

-
66. (nxF)xn =-[nx(nxF)J
TOPIC• WISE KVPY [STREAM SX] Questions with Solutions

mvr=lro
=-[n(n.F)- F(n.n)l
= F-n(n.F)J => (i) =
mv(¥)
!Ma 2
=G 3
mv
67. 2Ma

~
---x----- 69. x: = 0.3 cm- 3 x scale division of vernier calipers
9
""0.3 cm-3 x -
100
tan 60° = ~ = ~ 1 - = Ji :a tan e
R 11,./J 30-27
100
x= ~1+½ = 1 3
100
R2 +x2 -L2
cos8=---- =0.03 cm
2Rx
::c::> R2 + x2 -L2 = 2Rx cos 9
2
=>2xdx =2R[x(-sin9)+cos0dx] 70. T2 = 41t a2
dt dt GM

dx [x- R cos0] = - Rx sin0 dS


dt dt
dx d0
--""'V& - = (i) T= in year
dt dt
a= radius in AU.
Rx sin Bro
=>v=
x-Rcos9 3
:. T= 3 days= - year
365

. - ( 3
.. a- -
)2/3
365

a=O.04A.U.
2ro
v=~
3
71. This question is related to significant numbers
68. as in the question it is_ asked how student
REPORT the result.
On analysis the values of u, t and g the
reported result must have three significant
number.

I= 1cm + M(a.J2Y Hence correct answer is [BJ


= 8 2
-Ma
3

www.puucho.com
CP Publication KVPY chapterwise with www.puucho.com

MECHANICS ·
• - 1
72. p= ML2T"~, C ;= Lr T = T 1 sin30°
,R=L, m=M
w=T" 1
50 1 25 25../3
G=~1L3r2 - 2-fj . 2 = ../3 = -3-
[ML21 3] = [~ 1 L 3 1 2] [LT 1T5 M~ U' rz 2
solve we get
T=l4.4IN
X = 2, y = 4, Z = 6
75.
73. N

f,
M"

f;.= µN = µM"g = µ (M- Mr)g


at equilibrium.
R z + x2 -L2 fr= M'g
cosO= - - - -
2Rx ~tM( 1 - f)g =MFg
:::> x2 = 2Rx cosec 0 + L 2 -R2 µ(1-f)=f
displacement of S.H. M. is in the form of
µ=(µ+l)f
x = A sin wt + c
Therefore it is not S. H. M.
It is S. H. M. only which.
It L=R
::::) x2 = 2Rx cos 0 76. Consider a spherical shell of radius r and radial
:::> x = 2R cos 0 thickness dr. P & P + dP are pressure at its
x = 2R cos wt [ S.H. M.] inner and outer surface.
But period of this motion is T
Let gr = gravitational acceleration at distance
r(< R)
74,

:. 2T1 cos 30°= mg= 5 x 10 = 50.


2T1 cos 30° = 50 For equilibrium of this shell -
(P + dP) (4nr2) + (4n:r2 dr) p gr= (P) (4n:r2)

{p =~=Density of sphere}
4n:R 3

----.......T

www.puucho.com r,
'l
CP Publication KVPY chapterwise with www.puucho.com

TOPIC-WISE KVPY [STREAM SX] Questions with Solutions

4 2 79.
::::;, dP = - - n:Gp r dr
3
{(-) ve sign indicates that pressure is
decreasing with radius}

Case-I
Considering angular momentum w.r.t. end A
ro = Angular velocity just after impulse then

2
(ro) [ -ML
3-
) =J(L)

3J
ffi"" -
ML
::::;, velocity of CM
1 V = roL =-1:!._
HencePoc - 4 ... (1)
R cmi 2 2M'
::::;, Avcrage pressure will also be proportional
Case-II
l
to- Apply conservation o moment, if
R4
::::>correctansweris[A] V = 2 = Velocity of CM just after impulse
77. fri will increase m. then M V=i = J
fr3 will decrease w.
V = ..:!_ ... (2)
=2 M
comparing (I) & (2)

::::;, velocity of CM just after impulse would be


78. between above two extreme values

with (1), Mghm"-". < I (3J)


2(M) lM
2

h
i ...(3)

conservation of energy
2
I 2 r::; with (2), Mghmax > 2 J)
J M( M
-IAroc =Mg(av2 -a).
2
2 2 2 r;:;;2 J2
IA=Icm+Ma = -Ma +M(av2) h >-- ... (4)
3 max 2M2g
8 2
IA= -Ma Use (3) & (4)
9
. J2 9J2
..!.. ~ Ma 2 rn 2 = Mg a(.J2 -1) Hence - -2- < hm"" < - -2 -
2 3 C
2M g 8M g
ffic = ~3g(,.fi. -1) / 4a

www.puucho.com
CP Publication KVPY chapterwise with www.puucho.com

• MECHANICS
so_

(A)

T No

(R}

CNa = (M + ML)g-T
No=Mg+B

B=T+mTg
(C)
T = B-(mrg)

'
(M+mr)g
Nc=T+(M+mT)g
Therefore N1. = B - mrg + {M + ITlr)g
NL=B+Mg

(D)

(M+mr)g
No={M + mr) g.

www.puucho.com
CP Publication KVPY chapterwise with www.puucho.com

218
. :.· ... ... ··
'./.
'
Heat& Wave
------====-· I
1. A van dcr Waal's gas obeys the equation of state (A) 31 seconds

( P + ~: }v-
n~)- nRT . Its internal energy is
(B) 29 seconds
(C) 30 seconds
(D) the interval will depend on the distance of
2 the bus from the passenger
given by U = CT - n a. The equation of a
V 4. Velocity of sound measured at a given temperature
quasistatic adiabat for this gas is given by- (2010] in oxygen and hydrogen is in the ratio - (2010}
(A) 1 : 4 (B) 4 : 1 (C) 1 : 1 (D) 32 : I
(A) 'f°'nRV .. constant
(B) ·_r<c+nRJ'llRy = constant s. A piece of hot copper at l00°C is plunged into
(C) T°nR (V-nb) = constant a pond at 30°C. The copper cools down lo 30°C,
(D) p(C+nRYnR(V- nb) = constant while the pond, being huge, stays at its initial
temperature. Then- [2010]
2. An ideal gas is made to undergo a cycle depicted (A) copper loses some entropy, the pond stays
by the PY diagram alongside. The curved line at the same entropy
(B) copper loses some entropy, and the pond
from A to B is an adiabat (2010)
gains exactly tile same amount of entropy
p
(C) copper loses entropy, and the pond gains
more than this amount of entropy
A~ (D) both copper and the pond gain in entropy
B 6. An ideal gas with heat capacity at constant
volume Cv undergoes a quasistatic process
described by py« in a P-V diagram, where a is
V a constant. The heal capacity of t~c gas during
Thcn- this process is given by- 120101
(A) The efficiency of this cycle is given by (A) Cv (B) Cv.7'nR
unity as no heat is released during the cycle
(C) Cv + nR (D) Cy+ nR 2
(B) Heat is absorbed in the upper part of the 1-n / 1-a
straight lin~ path and released in the lower part 7. An ideal gas with constant heat capacity
(C) IfT1 and Tz are the maximum and minimum
temperatures reached during the cycle, then Cv = I2 nR is made to carry' out a cycle that is
depicted by a triangle in the figure given below.
the efficiency is given by l - T2 [2010]
T1
p
(D) The cycle can only be carried out in the
reverse of the direction shown in figure
I
3. A bus driving along at 39.6 kmph is approaching
a person who is standing at the bus stop; while
honking repeatedly at an interval of 30 seconds.
If the speed of the sound is 330 mis, at what
V1 V2 V
interval will the person hear the horn? (2010] The following statement is true about the cycle-

www.puucho.com
CP Publication KVPY chapterwise with www.puucho.com

1 HEAT&WAVE
11. Jet aircrafts fly at altitudes above 30,000 ft
(A) The efficiency is given I - Pi Vi where the air is very cold at - 40°C and the
P2V2
pressure is 0.28 atm. The cabin is maintained at
.
(B) Thee ffi c1ency . given
1s . by I - -
1-P1Vt
- 1 aim pressure by means of a compressor which
2 P2 V 2 exchanges air from outside adiabatically. In
(C) Net heat absorbed in the cycle 1s order to have a comfortable cabin temperature of
(P2-Pi)(V2-V1) 25°C, we will require in addition- [2011]
(D) Heat absorbed in part AC is given by - (A) a heater to, mn the air injected into the cabin
1 (B) an air-conditioner to cool the air injected
2(P2V2-P1V1) + 2 (P1V2-P2V1) into the cabin
(C) neither a heater nor an air-conditioner; the
8. A container with rigid walls is covered with compressor is sufficient
perfectly insulating material. The container is (D) alternatively heating and cooling in the two
divided into two parts by a partition. One part halves of the compressor cycle
contains a gas while the other is fully evacuated 12. A speaker emits a sound wave of frequency fo.
(vacuum). The partition is suddenly removed When it moves towards a stationary observer
The gas rushes to fill the entire volume and with speed u, the observer measures a frequency
comes to equilibrium afier a little time. If the Ji. If the speaker is stationary, and the observer
gas is not ideal, [20111 moves towards it with speed u, the measured
(A) the initial internal energy of the gas equals frequency isfi. Then - [2011]
its final internal energy (A)Ji =Ji <fo (B)fi > Ji
(B) the initial temperature of the gas equals iL~ (C)jj <fz (D)/i =h > fo
final temperature 13. At 23°C, a pipe open at both ends resonates at a
(C) the initial pressure of the gas equals its final freqnency of 450 hertz. At what frequency does
pressure the same pipe resonate on a hot day when the
(D) the initial entropy of the gas equals its final speed of sound is 4 percent higher than it would
entropy beat 23°C '? [20111
(A) 446 Hz (B) 454 Hz
9, Two bulbs of identical volumes connected by a (C) 468 Hz (D) 459 Hz
small capillary are initially filled with an ideal
gas at temperature T. Bulb 2 is heated to 14. One mole of an ideal gas at initial temperature
maintain a temperature 2T while bulb 1 remains T, undergoes a quasi-static process during which
at temperature T. Asswne throughout that the the volume V is doubled_ During the process the
heat conduction by the capillary is negligible. internal enerb'Y U obeys the equation U = aV3,
where a is a constant. The work done during this
Then the ratio of final mass of the gas in bulb 2
process IS- [2011]
to the initial mass of the gas in the same ·bulb is
·(A) 3RT / 2 (B) 5RT / 2
close to - (2011] (C)SRT/3 (D)?RT/3
(A) l/2 (B) 2/3 (C) 1/3 (D) 1
15. A constant amount of an ideal gas undergoes
10. Two rods, one made of copper and the other the cyclic process ABCA in the PV diagram
steel of the same length and cross sectional area shown below
are joined together. (The thermal conductivity
C
of copper is 385 J.s- 1 .rn-1. K-1 and steel is , . . . . 500 --------1\___
50 J.s- 1.m- 1.K-1.) If the copper end is held at
100°C and the steel end is held at 0°C, what is
the junction temperature (assuming no other
f 200 --------1-~
A' I B
heat losses)? (2011)
(A) 12°C (B) 50"C (C) 73°C (D) 88°C

www.puucho.com
\
CP Publication KVPY chapterwise with www.puucho.com

TOPJC•WJSE KVPY [STREAM SX] Questions with Solutions

The path BC is an isothermal. The work done (A) The temperature of the water would increase
by the gas during one complete cycle, beginning from IO .0° C to a final tempera hire of 12° C
and ending at A, is nearly- (20111 (B) The temperature of the water would increase
(A) 600 kJ (B) 300 kJ from 10.0" C to a final temperature of 18° C
(C) -300 kJ (D) ----000 kJ (C) The temperature of the water would increase
from 10.0° C to a final temperature of 14°C
16. An ideal monatomic gas expands to twice its (D) The temperature of the water would increases
volume. lfthe process is isothennal, the magnitude from 10.0° C to a final temperature of 11~ C
of work done by the gas is W 1• If the process is 21. A standing wave in a pipe with a Ienb>th L = 1.2 m
adiabatic, the magnitude of work done by the gas is described by
is Wa. Which of the following is true? [2012] y (x,t)= Yo sin [(21t/L)x] sin I(21t/L)x + rr/4]
(A) Wi=W.>O (B) Wi>Wa=O Based on above information, which one of the
(C) W,>Wa>O (D) W;>Wa>O following statements is incorrect. [2012]
(Speed of sound in air is 300 m s- 1}-
17. The ratio of the speed of sound to the average (A) The pipe is closed at both ends
spee.d ofan air molecule at 300 K and 1 atmospheric (B) The wavelength of the wave could be 1.2 m
pressure is close to- (2012] (C) There could be a node at x = 0 and antinode
(A) I (B) ./11300 at x=LJ2
(D) The frequency of the fundamental mode of
(C).J300 (D) 300 vibrations is 137.5 Hz

18. Two identical bodies are made ofa material for 22 The equation of state of n moles of a non-ideal
which the heat capacity increases with temperature. gas can be approximated by the equations
One of these is held at a temperature of 10 D°C
( P + ~:) (V - nb) = nRT
while the other one is kept 0° C. lf the two are
brought into contact, then, assuming no heat loss to where a and b are constants characteristic of the
the environment, the final temperature that they gas. Which of the following can represent the
will reach is- (20121 equation of a quasistatic adiabat for this gas
(A) 50° C (B) Less than 50" C · (Assume that Cv, the molar heat capacity at
(C) More than 50° C (D) O" C constant volume, is independent of temperature)?
(2013J
19. A cylindrical steel rod of length 0.10 m and
(A) T(V - nbl'cv = constant
thermal conductivity 50 W .m-1 K- 1 is welded end
to end to copper rod of thermal conductivity (B) T(V-nbfvlR = constant
400 W.m- 1• K- 1 and of the same area of cross
section but 0.20 m long. The :free end of the steel
(C) (T +~
VR
)
2
(V - nol/Cv = constant
rod is maintained at 100° C and that of the copper
,.and at 0° C. Assuming that the rods are perfectly ( nab) (V - nbfv
2
· (D) T + V 2 R IR = constant
insulated from the surrounding, the temperature at
the junction of tl1c two rods- (2012] 23. An engine moving away from a vertical cliff
(A) 20° C (B) 30" C (C) 40° C (D) 50° C blows a horn at a frequency f Its speed is 0.5% of
the speed of sound in air. The frequency of the
20. lne total energy of a black body radiation source
reflected sound received at the engine is [2013)
is collected for five minutes and used to heat
(A) 0.990 f (B) 0.995 f
water. The temperature of !he water increases (C) 1.005 f (D) l.010 f
from 10.0° C to l l.0°C. The absolute temperature
of the black body is doubled and its surface area 24. The bulk modulus of a gas is defined as
halved and the experiment repeated for the same B == -VdP/dV. For an adiabatic process the
variation of Bis proportional lo pn_ For an ideal
time. Which of the following statements would be
gas, n is [2013]
most nearly correct? (2012)
(A) zero (B) 1 (C) 5/3 (D) 2

www.puucho.com
CP Publication KVPY chapterwise with www.puucho.com

t HEAT&WAVE
25. The average distance between molecules of an 28. The three processes in a thcnnodyrumtlc cycle
ideal gas at STP is approximately of the order shown in the figure are : Process 1-+2 is
of [2013) isothermal ; Process 2---+3 is isochoric (volume
(A) l run (B) l 00 nm remains constant); Process 3-+I is adiabatic.
(C) 100 cm (D) 1 µm TI1e total work done by the ideal gas in this cycle
is 10 J. The internal energy de.creases by 20 J in the
26. The figure below shows pressure variation in isochoric process. The work done by the gas in the
two different sound waves in air with time at a adiabatic proce~'i is -20 J. TI1e heat added to the
given position. Both the figures are drawn to the system in the isolhcnnal process is- (2013]
same scale. [2013] p
I

p WJW\AAWave 1
•t
~
'-------+V
3

(A) 0 J (B) 10 J (C) 20 J (D) 30 J


p
29. J\ metal Li c prong consists of 4 rods made of the
same material, cross-sections and same lengths
as shown. The three forked ends are kept at 100°C
and the handle end is at 0°C. The temperature of
the junction is - (2013]
T = l00°C
Wave2
Which of the following statements is true? T = 0 ° C - - - - - - - T = l00°C
(A) Wave I has lower frequency and smaller
amplitude compared to wave 2 · T= 100°C
(B) Wave I has higher frequency and greater (A) 25°C (B) 50°C (C) 60°C (D) 75°C
amplitude compared to wave 2 30. A solid expands upon heating because -
(C) Wave 1 has shorter wavelen1:,rth and greater (2014]
. amplitude compared to wave 2 (A) the potential energy of interaction between
(D) Wave I has shorter wavelength and smaller atoms in the solid is asymmetric about the
amplitude compared lo wave 2 equilibrium positions of atoms.
(B) the frequency of vibration of the atoms
27. In the P-V diagram below the da~hcd curved increases.
line is an adiabat. (C) the beating generates a thermal gradient
between opposite sides.
(D) a fluid called the caloric flows into the
interatomic spacing of the solid during heating
thereby expanding it
.__~
_ _y
_,.y
31. Consider two thermometers T1 and T2 of equal
For a process that is described by a straight line length which can be used to measure temperature
joining two points X and Y on the adiabat · over the range 01 to 02. T1 contains mercury as the
(solid line in the diagram) heat is : (hint : thcnnometric liqwd while T2 contains bromine. The
Consider the variations in temperature from X volumes of the tv,ro liquids are the same at the
to Y along the straight line) [20131 temperature 81. The volumetric coefficients of
(A) absorbed tlu·oughout from X to Y expansion of mercUIV and bromine are 18 x 10-s K 1
(B) released throughout from X to Y and 108 x 10-5 K" 1: respectively. The increase in
(C) absorbed from X up to an intermediate length of each liquid is the same for the same
point Z (not shown in the figure) and then increase in temperature. If the diameters of the
released from Z to Y capillary tubes of the two thermometers are
(D) released from X up to an intermediate point d 1 and d2 respectively, then the ratio d1 : d2
Z (not shown in the figure) and then absorbed would be closest to (2014]
fromZ to Y (A) 6.0 (B) 2.5 (C) 0.6 (D) 0.4

www.puucho.com
CP Publication KVPY chapterwise with www.puucho.com

TOPIC• WISE KVPY [STREAM SX] Questions with Sol1;1t1ons


32. An ideal gas follows a process described by 38. A horizontal steel railroad track has a length of
PV 2 = C from (Pi, V1, T1) to (P2, V2, T2) (C is a 100 m when the temperature is 25"C. The track
constant). Then [2014] is con.strained from expanding or bending.
(A) if P 1 > P2 then T2 > T1 The stress on the struck on a hot summer day,
(B) ifV2 > V1 then T2 < T1 when the temperature is 40°C, is (Note : the
(C) ifV2 >VI then T2 > T1 linear coefficient of tbennal expansion for steel
is l.l x 10-5rc and the Young's modulus of
(D) ifP1 > P2 then V1 > V2
steel is 2 x 10 11 Pa) [2015]
33. A whistle emitting a loud sound of frequency (A) 6.6 x 10 7 Pa (B) 8.8 x 10 7 Pa
540 Hz is whirled in a horizontal circle of radius (C) 3.3 x 10 7 Pa (D) 5.5 x 107 Pa
2 m and at a constant angular speed of 15 rad/s. An ideal gas undergoes a circular cycle centered
39.
The speed of sound is 330 mis. The ratio of the at 4 atm, 4 lit as shown in the diagram. The
highest to the lowest frequency heard by a maximum temperature attained in this process is
listener standing at rest at a large distance from closed to - 120151
the center of the circle is [2014] P(atrn)
(A) 1.0 (B) 1.1 (C) 1.2 (D) 1.4

34. One mole of a monoatomic ideal gas is


expanded by a process described by PV3 = C
where C is a constant. The heat capacity of the
gas during the process is given by (R is the gas
constant). {2014] ....._......
2___._4___.6_ _ _ y (lit)
(A) 2R (B) ~R (C) IR (D)R (A) 30/R (B) 36/R (C) 24/R (D) 16/R
2 2
,40. A gas at initial temperature T undergoes sudden
35. A closed bottle containing water at 30°C is open
expansion from volume V to 2V. Then - (20161
on the surface of the moon. Then- [2015)
(A) The process is adiabatic
(A) the water will boil (B) The process is isothermal
(D) the water will come as a spherical ball
(C) The work done in this process is nRT tne(2)
(C) the water will freeze where n is the number of moles of the gas.
(D) the water will decompose into hydrogen and (D) The entropy in the process does not chaoge
oxygen
41. A container is divided into two equal part I and
36. The state of an ideal gas was changed isobaric.ally. II by a partition with a small hole of diameter d.
The graph depicts three such isobaric lines. Which The two partitions are filled with same ideal gas,
of the following is true about the pressures of the but held at temperature T1 = 1:50 K and T11 = 300
gas ? ....-,.__,...-.-,- [2015) K by connecting to heat reservoirs. Let ).1 and ;1.11
V be the mean free paths of the gas particles in the
two parts such that d >> "- 1 and d >> Au. Then
).1/). 11 is close to - {20161
(A) 0,25 (R) 0.5 (C) 0.7 (D) 1.0
42. A nurse measures the blood pressure of a seated
T patient to be 190 mm of Hg - [2016]
(A) P1 = P2 = P3 (B)P1 >P2>P3 (A) The blood pressure at the patient's feet is
(C) P1 < P2 < P3 (D) P1 / P2 = P3 / P1 less than 190 mm ofHg
(B) The actual pressure is about 0.25 times the
37. The intensity of sound during the festival season atmospheric pressure
increased by l 00 times. This could imply a decibel (C) The blood pressure at the patient's neck is
level rise from - [2015] more than 190 mm of Hg
(A) 20 to 120 dB (B) 70 to 72 dB (D) The actual pressure is about 1.25 times the
(C) 100 to 10000 dB (D) 80 to 100 dB atmospheric pressure

www.puucho.com
CP Publication KVPY chapterwise with www.puucho.com

t HEAT&WAVE
43. Thermal expansion of a solid is due to the u
120161
(A) symmetric characteristic of the inter atomic
potentia I energy curve of tlie solid
(A) ~----~-
(B) asymmetric characteristic of the inter a d
atomic potential energy curve of the solid
'---------•V
(C) double well nature of the inter-atomic
potential energy curve of the solid u

~
(D) Rotational motion of the atoms of the solid

44. A nuclear fuel rod generates energy at a rate of (B) L___J


5 x 108 Watt/n{ It is in the shape of a cylinder a d
of radius 4.0 mm and length 0.20 m . A coolant --------•V
of specific heat 4 x 10 3 J/(kg-K) flows past it at
a rate of 0.2 kg/s. The temperature rise in this u
coolant is approximately - f2016] C

(A) 2°C (8) 6 °C


(C) 12 °C (D) 30 °C
(C)
45. A hearing test is conducted on an aged person. It
d
is found that her threshold of hearing is 20
a
decibels at 1 kHz and it rises linearly with
'---------• V
frequency to 60 decibels at 9 kHz. The minimum
intensity of sound that the person can hear at
u
5 kHz is- [2016J
C
(A) IO times than that at l kHz b
(B) 100 times than that at l kHz
(C) 0.5 times than that at 9 kHz (D)
(D) 0.05 times than that at 9 kHz

46. A.a ideal gas is tak~n reversibly around the


cycle a-b-c-d-a as sho'>'m on the T (temperahrre) '----------• V
- S (entrophy) diagram [2016]
T 47. The heat capacity of one mole an ideal is found
to be CV= 3R (l + aRT)/2 where a is a constant.
The equation obeye.d by this gas during a reversible
adiabatic expansion is - [20161
(A) TV 312 eaRT =constant
(B) TV 312 e 3~RT/ 2 =constant
(C) TV 312 = constant

.___-------s
The most appropriate representation of above
(D) TV 312 e 2aRT/J =constant

cycle on a U (internal energy) - V (volume)


diagrame is

www.puucho.com
CP Publication KVPY chapterwise with www.puucho.com

TOPIC• WISE KVPY [STREAM SX] Questions with Solutions


48. A transverse wave of frequency 500 Hz and 51. The number of gas molecules striking per
speed I 00 mis is traveling in the positive x second per square meter of the top surface of a
direction on a long string. At time t = 0 s the table placed in a · room at 20°C and
displacements at x = 0.0 m and at x = 0.25 m 1 atmospheric pressure is of the order of
are 0.0 m and 0.02 m, respectively. The
(kii"" 1.4 x 10-23 J/K., and the average mass of
an air molecules is 5 x 10-27 kg) f5 Nov, 2017)
displacement at x = 0.2 mat t = 5 x 10-4 s is
(A) I 027 (B) I023
[5 Nov, 2017] (C) 1025 (D) 1029
(A) ---0.04 m (B)-0.02 m
(C) 0.04 m (D)0.02m 52. One mole of an ideal monatornic gas undergoes
the following four reversible processes :
49. A thin piece of thermal conductor of constant Stepl : It is first compressed adiabatically from
thermal conductivity insulated on the lateral volume V 1 to 1m3 •
sides connects two reservoirs which are Step 2 : then expanded isothermally to volume
maintained at temperatures T1 and T2 as shown. 10 m3•
Assuming that the system is in steady state, Step 3 : then expanded adiabatically to volume
which of the following plots best represents the V3.
dependence of the rate of change of entropy of Step 4 : then compressed isothermally to
volume V 1•
the ratio of temperatures Ti/T2 (5 Nov, 20171
· Tf the efficiency of the above cycle is 3/4 then
V 1 is, [5 Nov, 2017J
(A) 2m3 (B) 4m3
(C) 6 m 3 (D) 8 m3

53. A gas obeying the equation of state PV = RT


undergoes a hypothetical reversible process
(A)~ (B) .lo
-. -· :;
"0
described by the equation, PV5/3 exp (- : : J= c 1

where c 1 and Eo are dimensioned constants.


Then, for this process, the thermal
compressibility at high temperature
(5 Nov, 2017]
(A) approaches a constant value.
(B) is proportional-to T.
(C) is proportional to T 112
(D) is proportional lo 1'2.
. '
50. Two bottles A and B have radii R;;: ·and R 11 and 54. Consider the following statements for air
heights hA and ho respectively with Ra = 2RA molecules in an air tight container.
and hs = 2hA. These are filled with hot water at (I) the average speed of molecules is larger
than root mean square speed
60°C. Consider that heat loss for the bottles
(II) mean free path of molecules is larger than
takes place only from side surfaces. If the time the mean distance between molecules
the water to cool down to 50°C is (III) mean free path of molecules increases with
tA and ts for the bottles A and B, respectively, temperature _
then tA and ts arc best related as rs
Nov, 20171 (IV) the rms speed of nitrogen molecule is
(A) tA = tH smaller than oxygen molecule
(B) t!J ""2tA The true statements are: 119 Nov, 2017)
(C) 1s ""4tA (A) only II (B) H & III
(D) tA = tA/2 (C) II & IV (D) I, II & rv

www.puucho.com
CP Publication KVPY chapterwise with www.puucho.com

t HEAT&WAVE
55. A light beam ,travelling along the x axis with 58. A planet of radius Rp is revolving around a star
planar wavefront is incident on a medium of of radius R·, which is at temperature T·. The
thickness t. In the region, where light is falling distance between the star and the planet is d. If
the refractive index can be taken to be varying the planet's temperature is / T", then f is
proportional to 119 Nov, 2017)
such that dn > 0. The light beam on the other
dy . (A) ~R./d (B)R"/d
side oft~e medium will emerge [19 Nov, 2017} (C) R Rp/d2
0
(D) (R /d)"
0

(A) parallel to the x-axis


(B) bending downward 59. One mole ofan ideal rnonatomic gas undergoes
(C) bending upward· the following four reversible processes :
(D) split into tw'? or more beams Step 1 - it is first compressed adiabatically
from volume 8.0 m3 to 1.0 m3•
Step 2- then expanded isothermally at
56. A bomb explodes at time t =0 in a uniform,
temperature T 1 to volume 10.0 m3•
isotropic medium of density p and releases
Step 3 - then expanded adiabatically to
energy E, gencraring a spherical blast wave.
The radius R of this blast wave varies with time volume 80.0 m 3•
t as : [19 Nov, 2017) Step 4- then compressed isothermally at
(A) t (B) {115 temperature T2 to volume 8.0 m3•
(C) t 114 (D) 1312 Then T1ff2 is 119 Nov, 20171
(A)2 (B)4
57. A closecl pipe of length 300 cm contains some (C) 6 (D) 8
sand. A speaker is. connected at one of ilc; ends.
The frequency of the speaker at which the sand_
will arrange itself in 20 equidistant piles is
close to (velocity of sound is 300 mis)
[19 Nov, 2017)

(A) lOkHz (B) 5 kHz


(C) 1 kHz (D) 100 kHz

www.puucho.com
CP Publication KVPY chapterwise with www.puucho.com

TOPIC• WISE KVPY [STREAM SXJ Questions with Solutions

ANSWERS
1. (C) 2.(B) 3. (B) 4. (A) s. (C) 6. (C) 7.(B)
8. (A) 9. (B) 10. (D) 11. (B) 12. (B) 13. (C) 14. (D)
15. (D) 16.(C) 17. (A) 18. (B) 19.(A) 20.(B) 21. (B,D)
22. (A) 23. (A) 24, (B) 25. (A) 26. (D) 27. (C) 28. (D)
29. (D) 30. (B) 31. (D) 32. (B) 33.(C) 34. (D) 35. (A)
36. (B) 37. (D) 38. (C) 39. (B) 40. (A) 41.(C) 42. (D)
43. (B) 44. (B) 45. (B) 46. (A) 47. (A) 48. (D) 49. (B)
50. (B) 51. (A) 52. (D) 53. (A) 54. (A) 55. (C) 56. (B)

57. (C) 58. (A) 59. (B)

1. For adiabatic process 2. From the analysis of P-V diagram we can


dQ = 0 and -d.U = dW easily say that Bis the correct option.
::::) -nCv AT= PAV or -nCvdT = PdV

~,IE
when change is very small V
n-a
~
---+
DOW given U= CT-- ,..-iiiia,,
v 3. ~
2 d-~"""'*=~.,.,._,.
:. dU = CdT + n : dV
V m/s,

~~-r:;5:r::~=dW. .
V = 39.6 km/hr = 11
d d-l lx30
t1= - andt2= - - -
(l) 330 330
Now at= t1 - t2 = 1
now t1 =30 sec :. t2 = 29 sec.
2
also P = ( nRT )- n : replace it in ( l)
V-nb v-
4.
-(CdT + nV adV) = ((~)-
2
2 V-nb
nV a) dV 2
2

:. -CdT= ( nRT )dv


V-nb
C dT dV
---=--
nR T V-nb
Integrating we get 5. Using theory of entropy it is evident that
-.en J°'nR = .en (V-nb) + k answer is (C).
(k---+ constant of integration) 6. Direct fonnula is to be used
ln (T"-'nR)(V - nh) = - k
~r ('f°nR)(V - ob)= constanij f=Cv+~I
www.puucho.com
CP Publication KVPY chapterwise with www.puucho.com

• HEAT&WAVE

7. Cv=

f=3
3
2 R,
.
Cr=Cv+R=
5R
2
PV
n1=-
. RT
' PV . 'n 1
n2=·---:::) - =. -
:2

1 - . R2T n2 · ·I
W = -(V2-V1)(P2-P1)
2 . - n1 + n2 = n

ForBA Q=nCrAT= ~(¾R}n n1 = 2; ;ni=:


mass of gas oc n,
s . . n
= 2(P1V1 -P2V2)
. M2 =i =!
. 1 . M1 n ·3
For AC QAc= -(P1 +P2)(V2-V1)+nCvAT 2 - .
. 2 . .
3
now nCvAT= .:...(P2V2-P1V1)· '100-T · T-0
~ 10. --":"'---
R1 , R2·
now
100-T R{
1 . . --=·-·
W -(V2 -V,)(P2 -Pi) T R;
,,- - 1 2 3 ·R=~
Q 2 x(P1 +P2)(Y2 -V,)+ 2 (P2V2 -P1'(1l KA
using formula for heat we can calculate· heat .&. = k2·
R2 k1
.absorbed in AC.
100-T ·SO' IO

lau 1-1
T .: 385 =77.
8. 7700 - 77 T = 10 T
7700=87T>
..
expansion is against T = 7700 = s·soc.
vacuum :.AW= 0 87 .
Insulated container:. AQ =0 100 .___ T ·_
Cu_ _ Steel
_....,. 0
. First law of thermodynamics
AQ=AW+.AU
O=O+AU 11. PVl=C .
0=0+AU p'--YJ'I =C
AU=O co.2s) 1-r x (23J)l = 11-r x r
7
y=-
9. Mole conservation · 5
n1 +n2=n (0.28)1-715 X (233)1/S = 11-7/S X T7/S
T715 = 233 71~
(0.28f2i'S
X

'T = 233 (0.2sr211


233, •,
T=---
V V (0.28)21~·
Tis coming·.
Initial no. of moles =n1 =n2 = 0
. 2 more than 298 Kor 25°C
finally when temp of I vessel is T, &: another IS 2T :. Tis more than 25°C
so to cool it an extra ac is required.

www.puucho.com
CP Publication KVPY chapterwise with www.puucho.com

TOPIC~ WISE KVPY [STREAM SXJ Questions with Solutions


p
12. s.-+ u .o 500 ---~
f1 = fo[v] IS.

S.
v-u
V +-- ,Q
200 --1~8
A•
-"'----........:....'---v
r - - r [v+u] 2
12-10~-
v Wc..... A=O
f2 - f1 = fo ( u + v - ~v-) Process BC T=Constant
V v-u
PcVc=Pe Ve
-uz f.
f,,-f1 = 0 =-vc 500 x 2 = 200 x V8
- (v)(v-u)
Ve= 5m3
:. f1 > f2
WA-+>D = 200 [Vs-VA]= 200 [5 -2] = 600
13. D.=v We-+c > W A.....e
focv :. Net work done is -ve
_&=_!!_ ·.- Wo..... c<l200KJ
f2 V2
:. Total W <-600 JU
450 =~
f2 I.04v 1
fi = 1.04 x 450 = 468Hz. p

16. , W·
14. U=aV 3 '
~ . 1

fnRT =aV1 :. :Wa V


2 V 2V
PV=RT
PV =aVJ
R
P=CV2 ykT mn
-x-
17.
2V 2V m SkT
W= jPdV =
V
ICV dV= ~ (8V -V
V
2 3 3)

18. We have, AO= mSAT= CT


7V 3C
---
3
Where C ->- heat capacity
Let final temperature becomes T. then
fRT ""aV3 Heat lost = Heat gain
2 C ( 100 -T) = C' {T - 0)
PV=RT
100
f PV =aV3 orT= ( C') <50 (·: C' > C)
2 l+-
c
P= 2a y2
f steel Cu
C= 2a 19. I00°C .__I_ _ _ _ __ i . . . ._ _ _ ___.l O"C
f T
W = '!_ X V3 x ~ = !__ fn RT"" ?RT 50 x Ax (100-T) 400x AX (T-0}
3 f 3f 3 0.1 0.2

www.puucho.com
CP Publication KVPY chapterwise with www.puucho.com

• HEAT&WAVE
20. Energy radiated, U oc AT~ t 25. Average distance between molecules of an ideal
gas is 3.3 nm.
= U2 = A/2(2T)\t =B
U1 AT\t 26. lt is clear by graph
that wave I have high frequency and
= U~ =8U1
smaller amplitude than wave 2
= mSAt 2 = 8mSAt1 v"" )..f= constant.
= 811 =86tl l
foc -
i.
21. If v =330 the "D" will also be correct option.
If we consider option "8'' that seems to be p
incorrect as per given condition. So question is l
about incorrect statement.
So answer should be 11 B 11 and "D 11

(i) Z1t = ( 411:J 211 =k =Propagation constant


28.
\:j
.________
3
v
A l 11.

or 'J,. = lll or A. = g2 = 0•6 Qtot = W12 + W2.1 + W11 = \Vw,t


1n isochoric process. AU"" -20
(ii) V= 0. :::::> f= V/A.
:. Qn=AU=-20
Q31=0

23.
V

'
f
app
= V-Vo
v+v
s

s
f
I •
0
W23 = 0 (Isochoric process)
W31 = -20 joule (given)

Q12 + Q23 + Q31 =Wm1


Q12-20 +O = 10
Q,2 = 20 + 10 = 30 joule

V 100
v--
soo f= 499 f= 0.990 f
v+....!... 501 13
500 29. 0 ___.f---wA~-111-100
----'IMM,.....

24. PVl" = constant


dVY dP 100
P.-+V'-=0
dV dV
ii +h + i)-1-i4=0
py. vr--• + yr ~ =0 0-T 100-T 100-T 100-T
dV --+---+---+--- 0
R R R R
Py+V dP =0 300-4T= 0
dV
dV T =300 => 75cc
-V dV =yP 4

B=yP
30. The frequency of vibration of the atom increases
B cc P 1 :. n=I on heating a solid body resulting into increase in
intermolecular separation.

www.puucho.com
CP Publication KVPY chapterwise with www.puucho.com

TOPIC~ WIS~ KVPY [STREAM SX] Questions with Solutions

31. Increase in length of each liquid is same .


M onoatonnc 5
34. gas ::) r=-
!J.f, = l'l.f. 3
tt.VHg _ /'l.VBrominc n=l
ndf - mli PV3 = Con comparing with PV 0 =C
Here a= 3
(V)yHr, !t.8 _ V 'YHrumine LiO Heat capacity
ndf nd;
C "' ~ ...: ___!_
18x 10-5 y-1 ·a..:.1

'YBrnmine 108x10-5 C= ..!::_ _ __!_


(¾) (2)

PV2=C
C=R [%~½]
32.
C=R
::) (n~T)v2=C
35. Because on earth there is no atmosphere. So
::)TV=C water will hoil.
::) T1V1 =T2V2 (At-Boiling point vapour pressure= Atmospheric
::) lftcmperaturc incrcas.cs, volume decreases pressure, in open vessel)
and vice versa
36. Ideal gas equation PV = nRT
::)V2>V1thcnT2-<T1 ..
For isobaric process
V=( n;-) T (Va: T (straight line))
Slope ofline = ( n:)
33. I
slope oc -
p
(person) slope 3 > slope 2 > slope 1
. P3 < P2 < Pi
v1 and v2 are speed of whistle
I v1 != ! v2 I= @ = 15 x 2 = 30 mis
Maximum :frequency h~rd ~
37. _Loudness of sound in decibel dB = 10 log 10 ( 11 J
Here, · when :intensity of sound become 100 I then new
f--?
v
original frequency (540 Hz)
speed of soW1d
--?
· _decibel level = dB' = 1 0 l_og 10 ( 1 ~~ I J
v, -)-Speed of whistle dB' - dB= 10 log 10 100
dB'-dB =20
fmu=f[-V ] :. decibel rise by 20 dB
v-v. ~nly one option i.e. 80 to 100 dB match with it.
Minimum frequency heard'-?
38. · When some body is constrained from expanding
f. _ f[ V ] . fmax _ V+ Vs• _ 330+30 _ 360 or bending then on heating thermal stress get
mm V+ V5 , f min V- Vs 330 - 30 300 develop in the body.
· Stress = Y a. /'l.T = 2 x l0 11 x 1.1 x 10~5 x
fma,,: 6
- =- =1.2 (40-25)=3.3 x 107 N/m2 =3.3 x 107 Pa
fmiD 5

www.puucho.com
CP Publication KVPY chapterwise with www.puucho.com

t HEAT&WAVE

39. T= PV 42. Blood pressure is gauge pressure= 190 mm Hg


R Atmospheric pressure= 760 mm Hg
T will be maximum when PV is maximum Actual pressure= 190 + 760 mm Hg= 950 mm
Hg= 1.25 x 760 mm Hg
T= PV = (4+2sin8)(4+2cos0)
R R 43. Thennal expansion of a solid is due to asynnnetric
As sin 0 and cos 0 both characteristic of inter atomic potential energy cwve
can not be equal to 1 for same value of 0 of the solid
36
:. Tcannot be -
R
Tmax should be less than -
R
36
44.
()---e:---
1'·= 24
1 dm X S.6.T=dO
R curve dt dt
p I
24 dB= 5 x J0 8 x volume of rod
r=-
dt
lsclh~l CIIIVC oflc:mp. = 24 = 5 X }08 X 1C X (4)2 X }0~ X O.Z
R
--+-------v IO
=5 X }0 X 11: X 16 X 2 = 1600 1C
0.2 x 4 x 103.6.T= 1600:rc
curve is above isothermal curve 8 X 102.6.T = 16 X 10 2,i:
. more than -24 on the given
:. temp. 1s . process ~T= 3.14 x 2 ~ 6.28°C
R
. 24 36 45.
So T IJl.')J[ he between - and -
R R
only one option is present ~=log10(f)
0 60 1----------,..e
40. In sudden expansion gas do not get enough 401-----,,,,,,.
time for exchange of beat. 20 --
:. Process is adiabatic.

:r
41. As dimension of hole is very small than mean 5 9
path, then at equilibrium effusion rate of gas in

bmh rtioo ~ual. ]


at 5 KHz Hearing capacity= 40 dB
Intensity at I KHz

~=lOlogL:) ::=, 1=1 10(&,)


0

(I)1 KHz= lo 10(?0/lO) = lo(10)2


For this °'
(I)s KRI: = Io 10<4 10>=lo( l 0)4
(l)urnz. l
15KHz. 100

--n:
T

46.

-~-----s

www.puucho.com
CP Publication KVPY chapterwise with www.puucho.com

- be:::;, Isothermal process so U remain constant


cd:::;, lsentropic process so S remain constant
TOPIC- WISE

49.
KVPY [STREAM SX] Questions with Solutions

...
he
µd
,___ _ _ _...,..y
be should be straight line parallel .to V & cd

graph should be ~ d

47. Adiabatic process 50.


TV'-1 =C
2
y= 1 +-
f hE =2ho
2
TVr =C
Cv = f R = 3R(I + aRT) - ~

2 2 m
tee -
fR 3ReaRT A
2 2 t=k. Vp =k. ph
2 2 A
f- 3eEIRT to:: h
t h h
2 Jell!.T
:. ~ =~ = - 0 => tu =2tA
TVle.itT =C ~ TV 2 = C t8 b8 2h 0
3
Ans. given is TV 2 eaF..T
So no option is matching may be due to printing 51. V = ~3RT = ~3kT
rms M Mo
mistake.
48. y ""A sin(kx - oot) P= N >c 2mV=~
at X = 0.025, y = 0.02
v=v11.
1.01 x 105 = N x 2 x 5 x 10-27 x Yrms
100 I N ... l.0 l x 105 ~ 5 x 10-21
A.=-=-m
500 5 2 x5 X 10-27 ~3 X l.4x 10- 23 X 293

y=0.02 = Asin( 2 t ·¾) 52.


= 6.43xI0 27

y = 0.002=A sin( 5;) p

=>A=0.02m
:. y = 0.02 sin (kx - rot)
= 0.02 sin(I0n: x 0.2 - l0001t x 5 x 10-4]
= 0.02 sin[21t - 0.5:n:J
= 0.02 sin ( 3;) =--0.02m

www.puucho.com
CP Publication KVPY chapterwise with www.puucho.com

t HEAT&WAVE
In y direction refractive index is increasing
11 = l - _IL = l -
T2
r)V2
, V1
2/J
therefore speed of light is decreasing

213 56. The energy \vill propagate in form of spherical


_ _!_ =- ( - 1- ) ::::;,. V 1 = 8m3 blast wave which is longitudinal in nature.
4 v1 Hence velocity of propagate of disturbance
PV
53. py513 = Ci eEo

5 PV
£n P + - £n V = £n c1 + -
3 E0
dP + ~ dV =O+ PdV+VdP
P 3 V E0

dP [_!_-~]
P Eo
=dV [P-2__]
3V v=f!-
l V Wherc P = Pressure
- -- • PV =nRT
dV P E0
dP P-~ v= ~yRT
3V Vp

_1
C- - -
dV _ [ ~o -
- -
P~]
""=--'----==-
4m3
WhereV= - -
3

V dP [P- 3~] & v = velocity of propagation= dr


dt
At very high temperature c = - 1- dr yRT
Eo
=>
dt
(inr31r
3 )
54. For ideal gas mean free path of molecules is
larger than mean distance between molecules dr k
:::;, -=-
dt r312
55.
k"" constant ... ~
=> rdr=kd1
Jr312dr = Jkdt
r512 = kt :::;, r = (kt)2'5
::::, r cc r'5
Hence correct answer is (B)
Alternate solutioo
y R OC Ea pb tc
Dimension of R = L
Lo: (ML2 r 2t (ML-3)b (TY
2a-3b= 1 ... (i)
X c-2a=O ...(ii)
a+b=O ... (iii)

www.puucho.com
CP Publication KVPY chapterwise with www.puucho.com

TOPIC• WISE KVPY [STREAM SX] Questions with Solutions


from (i), (ii) & (iii)
1
a=-
5
c=2a
2
c= -
5
Ra: t215
=> T=T vu =rr.
• [I(
57.
A

,~~ 300cm ~ Hence correct answer is (A)

).. 59.
20 - ... 300
2 p
1c = 30cm= 0.3 cm
v=VA

v= SOO = 1000 Hz
0.3
v= lkHz

58. The planet should be in Thermal equilibrium -+--~--'-8---'-10-8-0--v (m3)


with star.
Amount of heat energy emitted by start per
AtoB
second E 1 = ( crT 04
) ( 4n:R 01
) TA vr-l = TB vJ-1

.._. d = distance between star and planet T2= ( V1)r-l = (~)i-i


Hence, amount of energy reaching planet per T1 V2 8
2
~
unit area per second=
4n:d ;: = (tY 4

.. ( crT 04 ) ( 4n:R 02 )
4ttd 2
.-1 .1
oT R
d2
Hence energy received by planet per second

~~ (aT:r·r:
T = Temperature of planet then amount of
energy emitted by planet per second
E3 = (crT 4 ) (41tR ~)

www.puucho.com
CP Publication KVPY chapterwise with www.puucho.com

1. The circuit shown consists of a switch (S), a


--~===-=--_.
4.
Electrodynamics ·
A charged particle of charge q and mass m,
I
battery (B) of emf E, a resistance R, and an gets deflected through an angle 8 upon passing
inductor L. [2010) through a. square region of side 'a' which
contains a uniform magnetic field B normal to
R its plane. Assuming that the particle entered
L the square at right angles to one side, what is
B
the speed oftbe particle? [2010]

c,,i,y!! 8 a cot 8 (B) qB a tan 9


The current in the circuit at the instant the ,Ym m
switch is closed is- qB , qB ,
(C)-acot·0 (D)-atan·e
(A) E/R (B) E/R(l - e) m m
(C) 00 rnfo

2. Consider a uniform spherical volume charge


,
5. Three equal charges +q are placed at the drree
vertices of an equilateral triangle centered at
distribution of radius R. Which of the following the origin. They are held in equilibrium by a
graphs correctly represents the magnitude of restoring force of magnitude F(r) = kr directed
the electric field E as a distance r from the toward.11 the origin, where k is a constant. What
center of the sphere ? (20101 is the distance of the three charges from the

rdEb_ (B)E[L origin?

(A) [-l_q 2]112


6ni; 0 k
(Y.J.. .Ji
'lI2n:t 0
{20101

q 2]11J
k
R r R r
[ ']2/J 2

(C)Eb__ (D)EtL
f;; 2] 13
(C) _l_~ (D) [ _v_.l3 .9....
6m: 0 k 4n:i:0 k .

6. Consider the infinite ladder circuit shown


below.
R r R r L L L L

3. A charge +q is placed somewhere inside the


cavity of a thick conducting spherical shell of
inner radiw; Ri and outer radius R2• A charge
- Q is placed at a distance r > R2 from the For which angular frequency ro will the circuit
centre of the shell. Then the electric field in the behave like a pure inductance? [2010]
hollow cavity- [20101
(A) LC (B) _I_
(A) depends on both +q and- Q
Ji. ~
(B) is zero
(C) is only that due to - Q (C)~ (D) 2L
.,1/ ..frE Jc
~ is only that due to +q

www.puucho.com
CP Publication KVPY chapterwise with www.puucho.com

TOPIC- WISE KVPY [STREAM SX] Questions with Solutions

7. Two identical particles of mass 'm' and charge Voltage


q are shot at each other from a very great
10 V • -- - --
distance with an initial speed v. The distance
of closest approach of these charges is- {20101

(A) q2 (FJ? q2
0
8tts 0 mv 2 411:c 0 mv 2
...:/. (I) (II)
2
(C) q (D) 0
2x&0 mv 2

8. For a domestic AC supply of 220 V at


50 cycles per second, the potential difference
between the temtlnals of a two pin electric
outlet in a room is given by - (2011]
(A) l (Bµi
(A) V(t) = 220Ji. cos (lOOnt) (C)HI ~IV
~~V(t) = 220 cos (50 t)
11. An isolated sphere of radius R contains uniform
,Y,J V(t) = 220 cos (100 nt) volume distribution of positive charge. Which
(D) V(t) = 220.fi. cos (50 t) of the curves shown below correctly illustrates
the dependence of the magnitude of the electric
9. In the circuit shown below the resistance are field of the sphere as a function of the distance
given in ohms and the battery is assumed ideal r from its centre? [2011]
with emf equal to 3.0 volts. The resistor that
dissipates the most power is - [2011]
40Q
]
~ -~ j ~
'E "'E!
,... r
'
~
30!1 0 R r
(I) (II)

i.::]
i
:s &i ~
u --
U OJ
I
I
~ G l ~
J~u '§im I
,
(B) R2 ~
Q
E: I
I
~ E '
J.Ll
{D)Rt 0 R r 0 R r
(Ill) (IV)
In the circuit shown, the switch is closed at
time t ~ 0. (201 ll (A)l ~11
R (C) Ill (D) IV

12. The surface ofa planet is found to be uniformly


~

lOVL
charged. When a particle of mass m and no
charge is thrown at an angle from the surface of
L the planet, it bas a parabolic trajectory as in
projectile motion with horizontal range L. A
particle of mass m and charge q, with the same
s----~.......... initial conditions has a range Ul. The range of
Which of the graphs shown below best particle of mass m and charge 2q with the same
represents the voltage across the inductor, as initial conditions is- (2011]
seen on an oscilloscope ? (A) L (B) V2 (.g,£,3 (D) LJ4

www.puucho.com
CP Publication KVPY chapterwise with www.puucho.com

• ELECTRODYNAMICS .

13. At a distance£ from a uniformly charged long 16. The capacitor of capacitance C in the circuit
wire, a charged particle is thrown radially shown in fully charged initially. Resistance is
outward with a velocity u in the direction R.-
perpendicular to the wire. When the particle
reaches a ilistance 21!. from the wire its speed is
found to be ../2 u. The magnitude of the
velocity, wben it is a distance 4£ away from the
wire, is (ignore gravity) [2011} After the switch S ls closed, the time taken to
reduce the stored energy in the capacitor to half
-!:)ff3 u (B) 2u
its initial value is 12012]
(C) 2./z u (D) 4u (A) RC/2 (B) 2RC In 2

14. A rectangular loop of wire shown below 1s


(C) RC In 2 ~c;n2
coplanar with a long wire carrying current L
17. A loop carrying current l has the shape of a
regular polygon of n sides. If R is the distance

I - from the centre to any vertex, then the


magnitude of the magnetic induction vector B
at the centre of the loop is -
r A iAoI tan-': (B) µoI
(2012]

~~2nR n 2R
µ I ?n µ 0I n
(C) n -0-tan.:.... (D) -tan-
The loop is pulled to the right as indicated. 2nR n nR n
What are the directions of the induced current
in the loop and the magnetic forces on the left 18. A conducting rod of mass m and length l is free
to move without friction on two parallel long
and the right sides of the loop ? !2011]
conducting rails, as shown below. There is a
Induced current Force on left side resistance R across the rails. In the entire space
Force on right side around, there is a uniform magnetic field B
(A) 9'nmterclockwise To the left To the right nonnal to the plane of the rod and rails, The rod
[..!})'Clockwise To the left To the right is given an impulsive velocity v0 - {2012]
(C) Counterclockwise To the right To the left 0
(D) Clockwise To the right To the left H

15. Two batteries V 1 and V 2 are connected to three R

v,C 1:J ~
resistors as shown below.

. II y, t he trutia
Fma ~
... I energy 2mvo
l
v,
(1).,\%( be converted fully into heat energy in
the resistor
IfV 1 = 2V and V 2 = 0 V, the current I= 3 mA. (B) Will enable rod to continue to move with
IfV1 = 0 V and V 2 = 4V, the current I= 4mA. velocity v0 since the rails are frictionless.
Now, if V1 = 10 V and V 2 = IO V, the current I (C) Will be converted fully into magnetic
energy due to induced current
will be- f2011J
(D) Will be converted into the work done
(A) 7 mA _ (B) 15 mA
against the magnetic field
(C) 20 1nA ~ 25 mA

www.puucho.com
i
CP Publication KVPY chapterwise with www.puucho.com

TOPIC•WISE KVPY [STREAM SX] Questions with Solutions

19. A steady current I flows through a wire of 23. Consider an initially neutral hollo:w conducting
radius r, length L and resistivity p. The current spherical shell with inner radius r and outer
produces heat in the wire. The rate of heat loss radius 2r. A point charge +Q is now placed
in a wire is proportional to it~ surface area. The inside the shell at a distam;e r/2 from the centre.
steady temperature of the wire is independent The shell is then grounded by connecting the
of/L . [20121 outer surface to the eatib. P is an external point
..J/0 l (B) r at a distance 2r from the point charge +Q on the
(C) I (D) p line passing through the centre and the point
charge +Q as shown in the figure [2013}
20, In one model of the electron. the electron of
mass m., is thought to be a uniformly charged
shell of radius R and total charge e, whose
electrostatic energy E is equivalent to its mass
m., via Einstein's mass energy relation
E = mec 2 • ln this model, R is approximately
(ffic = 9.J 10-3 ] kg, C = 3 X 108 ms- 1, J/4 1t~
X

= 9 x 109 Farad m- 1 , magnitude of the electron


ch?e = l.6 x 10-• 9 C)- (2012] The magnitude of the force on a test charge +q
!Jf) l.4x10- 15 m (B) 5.3 x 10- 11 m placed at P will be
(C) 2 X 1o- 13m (D) 2.8 X I o-35m (A) _l_ qQ (B) _1_ 9qQ
4rrE 0 4r 2 41tE 0 100r2
21. Consider three concentric metallic spheres A, B (C) _1_4q~
and C of radii a, b, c respectively where a< b < c, 4nE 0 2Sr·
A and B are connected whereas C is grounded
The potential of the middle sphere B is ;aised 24. Consider the circuit shown in the figure below:
to V then the charge on the sphere C is- {20121
r A, .,d.'°oV~ (B) 4 ac C
~--rn... c-b - ru:oV c-a
be
(C) -t4nE 0V~- (D) zero
c-b ~ - - , I[.......---,,,.,,,........-
V
22. As shown in the figure below, a cube is fanned All the resistors are identical. The charge stored
with ten identical resistance R (thick lines) and in the capacitor, once it is fully charged, is
two shorteing wires (dotted lines) along the [2013]
arms AC and BD. (2012] (A) 0 (B) ~CV
l::,--
,c;;..--+---r""
....
.-----.--::
..~B
(C)
'
3.cv
3
JB)1cv
' 8
D

25. A blackbox: (BB) which may contain a


combination of electrical circuit elements
(resistor, capacitor or inductor) is conne~ted
A
with other external circuit elements as shown
ReyiCtance between P<?int A and B is- below in the figure (a). After the switch (S) is
,JA) R/2 (B) SR/6 closed at time t = 0, the current (I) as a function
(C) 3RJ4 (D) R of time (t) is shown in the figure (b). (2013]

www.puucho.com
CP Publication KVPY chapterwise with www.puucho.com

t ELECTRODYNAMICS
+ t • + t +
I
'
I
I
'I
I
I
I
'
I
I
•I I
I

'''
I I
'II I
'I
I
I
I
I

L____,
I I I
I I I I
I
I
I
I
'' '
1
'I
I
•I
'
I I '
Magnetic field
(a)
s (b)
During its 111oti n inside the chamber 12013J
From this we can infer that the blackbox (A) the force on the electron remains constant
contains (B) _lh~netic energy of the electron remains
(A) A resistor and a capacitor in series ~constant
(8) A resistor and a capacitor in parallel (C) the momentum of the electron remains
~resistor and an inductor in series constant
(D) A resistor and an inductor in parallel (D) the speed of the electron increases at a
uniform rate
An arrangement with a pair of quarter circular <&) The figure below shows a circuit and its input
coils of radii r and R with a common centre C
voltage Vi as function oftime t. (2013]
and carrying a current 1 is shown. 12013] lk!l

_I lV -=-
T
Vj

The pcnncability of free space is


magnetic field at C is:
(A) yol (1/r - l/R)/8 into rbe page
jJJ'rµol ( 1/r - l/R)/8 oul of the page
(C) µol (l/r+ 1/R)/8 out of the page
(D) µol (1/r + l/R)/8 into the page
Assuming the diodes io be ideal, which of the
27. The circuit shown has been connected for a following graphs depicts the output voltage v0
as function of time t?
long time. The voltage across the capacitor is : Vo
(2013]
l kn 4V ---~.::\
l' ·-.
.-· .........,
.,···'

2 kn
·-.,.._- - - -4 V
4µF

Vo

(A) 1.2 V
(C)2.4 V
~~~~v 4V - -- ~-~\\
-
~-UV ..'....... \
(B) ',--,====-,,-,t:.....----4
28. An electron cntel'li a chamber in which a \.. /Jo
/
uniform magnetic field is present as shown. \.
V~--- -4V
Ignore gravity.

www.puucho.com
CP Publication
,/
KVPY chapterwise with www.puucho.com

TOPIC-WISE KVPY [STREAM SXJ Questions with Solutions


./ Vo (A) (Q + q)2 /4rcf-Oa
/ (B) zero
4V -- - -_.~-~ ~;44rrf-Oa (D) Qq /41tf.oa

33. Two identical parallel plate capacitors of


(C)
capacitance C each arc connected in series with
,. ____ a battery of emf, E as shown. If one of the
-4V
capacitors is now filled with a dielectric of
Vo dielectric constant k, the amount of charge
4V - - - -......,.·., which will flow through the battery is (neglect

C
internal resistance of the battery) (2014]
c~-----lc
~
(D) \._
/
\....../___
I t--------------
-4V

E
30. Two identical metallic square loops L1 and Li
are placed next to each other with their sides (A) k+l CE (B) !$.::±: CE
paraUel on a smooth horizontal table. Loop Li 2(k - l) V"2(k+l)
is fixed and a current which increases as a
(C) k-2 CE (D) k+ 2 CE
function of time is passed through it. Then loop k+2 k-2
l..z (2014]
(A) rotates about its center of mass 34. A cer1ain p-n junction, having a depletion
(B) moves towards L1. region of width 20 µm, was found to have a
(C) remains stationary
breakdown voltage of 100 V. If the width of the
7oves away from Li
depletion region is reduced to 1 µm during its
31. An electron enters a parallel plate capacitor production, then it can be used as a Zener diode
,,,,,.... for voltage regulation of - f2014J
with horizontal speed u and is fond to deflect /
e
by angle on leaving the capacitor as shown. It __,(f,.)'5 V (B) 10 V
is found that tans = 0.4 and gravity is (C) 7.5 V (D) 2000 V
ricgligible (2014]
_.,~ 35. What are the charges stored in the 1 µF and
++++++
- ~ L " -- 2 µF capacitors in the circuit below, once the
~ - - J . ?_______ _ currents become steady? [2014]
e
4 kn lµF
M½-----1
iful horizontal speed is
r·u~~F
If th · doubled, then l k.Q 2 k.O:
1 be
(B) 0.2 (C) 0.8 (D) 1.6

32. Consider a spherical shell of radius R with a ~ii----'


total charge -tQ uniformly spread on its surface / 6V
(center of the she) I lies at the orib'lll x= 0). Two (A),§ µC and 4 µC respectively
point charge, +q and - q arc brought, one after .,...(:B) 4 µC and 8 µC respectively
the other, from far away and placed at x = -a/2 (C) 3 µC and 6 µC respectively
and x = + a/2 (a< R), respectively. Magnitude
(D) 6 µC and 3 µC respectively
of the work done in this process is [2014]

www.puucho.com
CP Publication KVPY chapterwise with www.puucho.com

ELECTRODYNAMICS
~,
~! For a diode connected in parallel with a 39. A metallic ring of radius a and resistance R is
/
resistor, which is the most likely held fixed with its axis along a spatially
current (l)-voltagc (V) characteristic? (2015] / uniform magnetic field whose magnitude is Bo
sin (oot). Neglect gravity. Then, (2015]
(A) the current in the ring oscillates with a
frequency of2ro.
(B) the joule oearting loss in the ring is
proportional to a2,
V (C).,ffie force per unit length on the ring will be
V proportional to B~ .
(D) the net force on the ring is non-zero

V 40. A wire bent in the shape of a regular


n-polygonal loop carries a steady current I.
Let e be the perpendicular distance of a given
segment and R be the distance of vertex both
from the centre of the loop. The magnitude of
the magnetic field at the centre of the loop is
given by - (2015]

37. A point charge Q(= 3 x 10- 12q rotates (A-~Pol sin (n/n) (B) nµol 'sin (n/n)
..;,· 21tC 21tR
uniformly in a vertical circle of radius R =l mm.
The ax.is of the circle is aligned along the (C) nµol cos (n/n) (D) nµoI cos (n/n)
magnetic axis of the earth. At what value of the 2.tf 21tR
angular speed oo, the effective magnetic field at 41. Two small metal balls of different mass m 1 and
the center of the circle will be reduced to zero?
m2 are connected by strings of equal length to a
(Horizontal component of Earth's magnetic
fixed point. When the balls are given equal
fiel<!Js:30 micro Tesla) [2015]
(b-Jl0 1I rad/S (D) 109 rad/g charges, the angles that the two strings make
(C) l O11 rad/s (D) 107 rad/s with the vertical are 30Q and 60°, respectively.
The ratio m1/mz is close to - (2015]
,
(~)1.7
.
(B) 3.0 ,
I
38. An ac voltmeter connected bet\veen points A
and B in the circuit below reads 36 V. If it is ~q o.ss en) 2.0 I
/
connected between A and C, the reading is I
39 V. The reading when it is connected 42. Consider the regular array of vertical identical
between B and D is 25 V. What will the current carrying wires (with direction of current
voltmeter read when it is connected between A flow as indicated in the figure below)
and D ? (Assume that the voltmeter reads true
protruding through a horizontal table. If we
rms voltage values and that the source
scatter some diamagnetic particles on the table,
generates a pure ac) J2015J
they are likely to accumulate - (2015]

,i}~V
C
(C) 61 V
(B)JIV
(D) .J3351 V
(!-)

@
® ,:!) ® ;~:, ®
(!) @B (~)

(•,_:;' ..o.
'O'

® . (!} ® (~) ®
,.
II

r.•.·.·,\ .o $..c
'OII;:;;! .o..
v;;,

i~)
T"I' view

www.puucho.com
CP Publication KVPY chapterwise with www.puucho.com

'
' TOPIC• WISE KVPY [STREAM SXJ Questions with Solutions

around regions such as A 46. A conducting bar of mass m and length .f,
· (B) around regions such as B 4 moves on two frictionless parallel rails in the
(C) in circular regions around individual wires presence of a constant uniform magnetic field
such as C of magnitude B directed into the page as shown
(D) uniformly evef}"\Vhere in the figure . The bar is given an initial
velocity v0 towards the right at t.,. 0. Then the
The distance between the vertex and the center
(20161
of mass of a uniform solid planar circular
X X X X ~ X X
segment of angular size 8 and radius R is given
by- 120151 X X X X X X

X X.,
.,:
:RX l ...x+
X X X

X X X X X X

X X X X X X X
(A)~ R sin(8/2) (B) R sin(B/2)
v 3 8 8 (A) Induced current in the circuit is in the

(C) f (1)
R cos
2
(D) -Reos (0)
3
clockwise direction
(D) Velocity of the bar decreases linearly with
time
44. In the circuit shown below, all the inductors
JS). .
D"istance the bar travels before it comes to

(assumed ideal) and resistors arc identical. The


a complete stop is proportional to R
cUITent through the resistance on the right is I
(D) Power generated across the resistance is
after the key K has been switched on for along proportional to l
time. The currents through the three resistors
(in order, from left to. right) immediately after 47. A metallic sphere is kept in between two
the key is switched off are - (20151 oppositely charged plates. The most appropriate
representation of the field lines is - [2016}

(rn
~upwards, I downwards and I downwards
(A)

(B) 21 downwards, l downwards and I downwards


(C) I downwards, I downwards and I downwards
(D) 0, I downwards and I downwards

45. Physical processes arc sometimes described


visually by lines. Only the following can cross -
(C) (D)
../ . (2016)
~)treamlines in fluid flow
'(B)J-incs of forces in electrostatics
~J Rays in geometrical optics
(D) Lines of force in magnetism

www.puucho.com
CP Publication KVPY chapterwise with www.puucho.com

• ELECTRODYNAMlCS

48. Two infinitely long parallel wires carry currents 51. 1f the input voltage V; to the circuit below is
of magnitude l 1 and Ii and are at a distance 4 given by V;(t) = A cos(2nft), the output voltage
cm apart. The magnitude of the net magnetic is given by V0 (t) = B cos(21tft + qi) -
field is found to reach a non-zero minimum
values behveen the two wires and l cm away
from the first wire. The ratio of the two currents
and their mutual direction is (2016)

~ 9 , antiparallel

(B) .2."" 9, parallel Which one of the following four graphs best
I1 depict the variation of$ vs f? [2016]

(C) 12 = 3, antiparal!el
I1
I-,
(D) -----"--- = 3, parallel
]l

49. Consider a cube of uniform charge density p.


The ratio of electrostatic potential at the centre
of the cube to that at one of the corners of the
cub~ [2016]

JA:12 (B).fi/2
52. If some charge is given to u solid metallic
(C) .,fi (D) 1 sphere, the field inside remains zero and by
Gauss's law all the charge resides on the
50. Two infinitely long wires each carrying current surface. Suppose now that Colomb's force
1 along the same direction are made into the between two charges varies as l/r3• Then, for a
geometry as shown in the figure. The magnetic charged solid metallic sphere (5 Nov, 2017)
field at the point Pis [2016) (A) field inside will be zero and charge density
inside \Vill be zero.
(B) field inside wi[l not be zero and charge
density inside will not be zero.
(C) field inside will not be zero and charge
density inside will be zero.
/
(D)tficld inside will be zero and charge density
V inside will not be zero.

53. A bird sitting on a single high tension wire does


not get electrocuted because [5 Nov, 2017}
(A) the circuit is not complete.
(B) the bird feet has an insulating covering.
(A) µol (B) µol (.!_ + .!_ l. (G),;;apacitancc of the bird is too small and the
m r n 4)
../ line frequency is too small.
(C) Zero m/~oI (D) resistance of the bird is too high
7' 2m

www.puucho.com
CP Publication KVPY chapterwise with www.puucho.com

TOPIC· WISE KVPY [STREAM SXJ Questions with Solutions


54. A positive charge q is placed at the center of a (C) All ions will hit the screen.
neutral hollow cylindrical conducting shell with
its cross section as shown in the figure below.
(D) Only ions with q = 2 Iv Im will hit the
B2w2
(5 Nov, 20171 screen.

Thin shell
56. A square-shaped conducting wire loop of
dimension a moving parallel to the x-axis
approaches a square region of size b (a < b)
where a uniform magnetic field D exists
Which one of the following figures correctly
pointing into the plane of the paper (see figure).
indicates the induced charge distribution on the
conductor (ignore edge effects). As the loop passes through this region, the plot
correctly depicting its speed (v) as a function of
xis {5 Nov, 2017]

atb-l·-------1 lb
' : B®:
t ~---------
'
x=O X
V V

(A)

0 X 0 X

V V
55. A rectangular region of dimensions w x l (w << l)
bas a constant magnetic field into the plane of
(C) (D)
the paper as shown. On one side the region is
bounded by a screen. On the other side positive
ions of mass m and charge q are accelerated
a X X

from rest and towards the screen by a parallel


plate capacitor at constant potential difference 57. Let the electrostatic field E at distance r from a
V < 0, and come out through a small hole in the point charge q not be an inverse square but,
upper plate. Which one of the following instead an inverse cubic, e.g. E =k ..2.. r
statements is correct regarding the charge on rJ

the ions that hit the screen? [5 Nov, 2017) Here k is a constant. Consider the following
Screen two statements 119 Nov, 2017]
(i) Flux through a spherical surface enclosing
the Charge is Q"" Qendosed fEo
(ii) A charge placed inside unifonnly charged
shell will experience a force.
V····+~·· Choose the correct option.
O····J.: .... (A},.('.)nly (i) is valid
(A) Ions with q > -2...;.Iv--'-Im_ will hit the screen. ~ Only (ii) is valid
82w2
(C) Both (i) and {ii) are invalid
(B with q < 21 v Im will hit the screen. (D) Doth (1) and (ii) are valid
B2w2

www.puucho.com
CP Publication KVPY chapterwise with www.puucho.com

• ELECTRODYNAMICS

58. The black shapes in the figure below are closed 60. A square-shaped wire loop of mass m,
surfaces. The electric field lines are in red. For resistance R and side 'a' moving with speed v0,
which case the net flux through the surfaces is parallel to the x-axis, enters a region of uniform
non-zero? [19 NOY, 2017J magnetic field B, which is perpendicular to the
plane of the loop. The speed of the loop
changes with distance x (x < a) in the filed, as

•• (a) (b)

(D) Vo
[19Nov,2017)
B2a2
(B)vo- - - x
2Rm

61. Consider the R-L-C circuit given below. The


(c) (d) circuit is driven by a 50 Hz AC source with
(A) In all cases net flux is non-zero peak voltage 220 V. If R = 400 .0., C = 200 µF
(B) OnJy (c) and (d) and L = 6 H, the maximum current in the
_JyYOniy (a) and (b) circuit is closest to (19 Nov, 20171
(D) Only (b), (c) and (d)

59. A particle of charge q and mass m enters a


region of a transverse electric field of E0 }

with initial velocity ~0 i. The time taken for the


{B) 0.55 A
change in the de Broglie wavelength of the
(D) 5.5 A
charge from the initial value of ),0 to 'A,of3 is
proportional to [19 Nov, 2017]
(A) __s.. (B~
m q

(C) ff (D)l

www.puucho.com
CP Publication KVPY chapterwise with www.puucho.com

TOPIC- WISE KVPY [STREAM SX] Questions with ~olutions

ANSWERS
1. (D) 2. (A) 3. (D) 4. (A) 5. (B) 6. (C) 7. (B)
8. (C) 9. (A) 10. (D) 11.(B) 12. (C) 13. (A) 14.(B)
15. (D) 16. (D) 17. (A) 18. (A) 19. (A) 20. (A) 21. (A)
22. (A) 23. (D) 24. (D) 25. (C) 26. (B) 27. (D) 28. (B)
29. (A) 30. (D) 31. (A) 32. (C) 33.(B) 34. (A) 35. (B)
36. (A) 37. (A) 38. (A) 39. (C) 40. (A) 41. (A) 42. (A)
43. (A) 44. (A) 45. (C) 46. (C) 47. (B) 48. (A) 49. (A)
50. (D) 51. (C) 52. (D) 53.(C) 54. (A) 55. (B) 56. (B)
57. (B) 58.(C) 59. (B) 60. (A) 61. (A)

1. Using the equation I = Io (I - e 1L ) 4.


Put t = 0 we get I = 0
Just when the battery is closed inductor
provides infinite resistance to the current flow
:. current is zero initially.

2. V

Pr kQ Now sin0= ..!. R= mv


E,-= ~ for O~r<R; E,-= -~- for r:C:R
3E0 r· R' qB
1 :. [_=_q_B_am-co-t9-1
:. E cc. r for r < R and E oc 2 for r :C: R
r
:. curve is
5.
,'
'+q
I
I

3.
•-Q
+q .,.,_ _ _........__ _ _ _ _'"T"'""3-+0o Fq
----a
Fnc1 =2Fqcos30"

For a conductor electric field inside its cavity is F(r) = kr


now F 001 on a particle is 2Fq cos 30° due to the
only due to inside charge and not due to outside
other two charges
charge.

www.puucho.com
CP Publication KVPY chapterwise with www.puucho.com

• ELECTRODYNAMICS

Fnel~ lk; X-'!


a
2

-
40Q

R1
i/2
i/2 R3 60Q Ri 30Q
9. 3.0V i/6~..--'
R2 40n
alsor= ½[~a) Rs 20D

:. a= ..fJ r replacing it in Fnct we get Power dissipate in R 1 is maximum as its current


is maximum an.:. its
F
ne
r= (Jir)2
2kq2 x[-./3) = kq2
2 .fir2 resistance is also 400 which is higher than
R5R.,.
this is balanced by F(r)
lxq'.'_ V __l\. . r
:. F(r) = Fnd ~ kr = f;, ? lei= ~e
4rrt: 0x, 3 r- 10.
R
L

6. Let the equivalent impedance of the circuit be Z


So Z = roL + Z' 11. Whenr<R
ZX,.
DOW Z' = ~
Z+Xc Whcnr> R

Z=wL+
r Zx-1
we
z+-1-
-J
\. cuC
. mLC±~(roLC) 2 -4LC
on solvmg we «et Z = - - - - - - - ~
0
2C
for Z to be purely inductive 12. For uncharged particle

,/LC I
2 L= u 2 sin20
cu2L 2C2 -4LC=O or~= ... (i)
g
Range for particle of mass 111 and charge q.
7. Using law of conservation of mechanical
L u sin20
2
energy .... (ii)
2 qE
Lnitial K.E. = Final P.E. g+-
m
I 2 l 2 kq2 From (i) and (ii)
-mv +-mv = ~
2 2 r u 2 sin20 u 2 sin20
- q1 2g
.. r- 2 g+ qE
4n:& 0 mv 111
~mg=qE
8. R.M.S. value = 220 V Range of particle of mass m & charge 2q.
Peak value = 220 .fi R= u 2 sin28 u 2 sin28 L
w=21tn
= 21t X 50 g+ 2!E = g(l+ ~:) = 3
= 100 TC
V(t) = 220../2 cos (lOOitt)

www.puucho.com
CP Publication KVPY chapterwise with www.puucho.com

TOPIC• WISE KVPY [STREAM SX] Questions \Yith Solutions

13. 15.
+
+ A B C

;:a~--~--~
! 4
+
+
+

/ energy conservation at A & B


1 2 I .,
I qVA + - mu =qVe+ -mx 2u·
2 2
q[VA-Ve]= !2 mu2

A. 1 2
qx --ln2= - mu
21t Eo 2
energy conservation at A & C In each case Rcq & R is same only V1 & V2 is
I 2 1 2 changing:. V«i is changing
qVA + - mu =qVc+ - mv
2 2
Case -1 V = 2xR 2 + OxR 1
q [VA-Ve]+.!. mu2 = .!.m.; cq
R 1 +R 2
2 2
~ ln4+ _!_ mu2 = _!_ mv2
[V1 =2, V2= O]
21t Eo 2 2
V = 2R2
2 qi.. In 2 + _!_ mu2 = _!_ mv2 •11 R1 +R2
21tEo 2 . 2
2 1 2 1 2 Case-2 V = 4R1
mu+-mu=-mv- eq R1 +R2
2 2
.!u2 = ..!...; :::> v= Jiu [Yi =O, V2=4]
2 2
.!i= I= 2R2
14. Flux is inward arid it is decreasing as 12 4 4R1
loop is going away from wire _ 10R 1 + 10R 2
Case-3 V ~q-
R1 +R2

I
~ = 2R 2 ::::) ~ = 2xl.SR 1
I' IO(R 1 +R 2 ) I' 10(2.5R1)

or I'= 25mA

: . direction of induced current is clockwise


16. Discharging -

-O-
Q "" Qo e·IIRC '

U= .2:. = Q ~ e·211Rc or U = U 0c• 21/Rc


F ® F 2C 2C

Uo- o
- -Ue-211RC :::::>---
t- RCtn2
2 2
Force on left side is in left and force on right
side is in right.

www.puucho.com
CP Publication KVPY chapterwise with www.puucho.com

e ELECTRODYNAMICS

17. 24.
·-. R R

Buet = n x B1 = n. µu. 1 2sin2: R


47t R cos-
1t n
n

18. Due to energy conservation

19. Concept of fuse wire

cl 2
20. --=me Kirchhoff voltage law in loop ABCD
8:rcc 0 R •
solving for R l'R SR.
+- t- V =0
3
21.
8R i=V
3
. 3V
1=-
8R
:!. =ix SR
C 3
. SR
q= Cp C -
3
V8 = kq + k(-q) = V (Given)
b C = C x SR x 3V = 5CV
4rre 0.bc V 3 SR 8
q c-b ·
Charge on C = -q 25.

22. The given circuit can be simplified into two


wheatstone bridge in parallel
23.

.
1= E [ 1-e
R -!1]
L

Potential of sphere = 0
:. X""O
Electric field of +Q &- Q get cancel out at
point outside of sphere.
:. Force onP = 0

www.puucho.com
CP Publication KVPY chapterwise with www.puucho.com

TOPIC• WISE KVPY [STREAM SX] Questions with Solutions

26. 30.
Fixed

Ba1c= - x -
4n-r 2
µ01 1t µoi 1t
--x-
4n:R 2
DD L1 Lz
When current through 11 increases then flux
linked through L2 will increase.
® ®
:. According to lenz law L2 will move away.
=> µ;1 [f-~] ®
31.
Out of page

27.
l k..0.

2kQ E
4µF 4------ f.

Horizontal displacement= C
At steady state no current flow through R,
capacitor t= -
, • lkQ behave as conducting wire. u
Vy= Uy+ at
i Jill A
eE C
=O+ - X -
m u
2kQ eE f.
v=-><-
Y m U
v~ remain same and it is equal to u
D C tan9='!.:!_=eE !._ x..!..= eEi
i= 6 =2 x 10-3 Amp. vx m u u mu 2
(2 + 1) X 10 3 1
tan9 cc 2
Potential drop across c = potential difference u
betweenA&D When speed u is doubled then tan 9 will
:::;., ix2xI03 . I
=> 4 volt become -th.
4
28. F = qVB sin 90 .-. tan 9 = 0.4 = 0.1
4
F remain constaot in magnitude but direction
change 32.
->
F is not constant.
Fis perpendicular to electron velocity
KE remain constant
as F is acting
Momentum will change.
as KE remain constant
speed remain same + +
C •

www.puucho.com
CP Publication KVPY chapterwise with www.puucho.com

• ELECTRODYNAMICS' • • •. ' : . (. j '


34. Break down voltage is proportional to width of
PE; Initial energy of system depletion region.
(self energy of shell) :. When width reduce to l µm thus become
PEf= Final energy of system
- 1- times then break down voltage also
Q2 q X (-q) kQ X q kQ(-q) 20
--- + ' +--+---
8rci::oR 4ni::0a · R R I . . . I
Q2 q2 become -tirr. s thus it become 5 vo t.
=>------- 20
8nEoR 4ns 0 a So Zener diode can used for voltage regulation
(self energy of shell) (Interaction energy of5 volt.
bet\veen various charges)
Work done= PEf- PE;
35.
= -q2
4ns0a
. 2
Magnitude of \vork done = _q__-
4nE 0a

33.

C E
C C

~
i--------------11----l At steady state current does not flow in the
branch of capacitor.
:. we can replace all resistor connected in
.. CE branch of capacitor with wire new circuit is
lmhal charge on both C =-
2
lµF
C kC ~~Q___, -Q
lkO 2kQ

2µF
q --q
------1------1q I , .
New charge on each C = ( kC )E 6V 1

k+l
. 6
!=---~
Change in charge on C is supplied by battery (2+J)xl0 3

Ch:uge supply by battery = ( kC )E - CE => 2mA


k+l 2 Potential drop across 2kf.! is same as potential

=> CE[~ - l
k+l
.!_.
2J
drop across 1 µF & 2 µF.
Potential drop across 2 kQ = i x 2 x 103
=2x 10-1 x2x 103 =4volt.
==> CE[ 2~~\) J Charge on
lµF=Q=l x4x 10---6=4µC
,r
Charge passes through battery is change supply
/ by battery Charge on
2µF = q = 2 x 4 x I o---6 = s µC
:. Ans. CE[ 2~~\)]

www.puucho.com
CP Publication KVPY chapterwise with www.puucho.com

-
54. Option 'A' is correct option. According charge
conservation & Gauss's law.
TOPIC-WISE KVPY [STREAM SX] Questions with Solutions

[El= kq
r3

55. :. d4'= E. ct§

i ,,
, 4t = d$ =f fE.ds = ~; . 4m2

t ,,'
I 4' = kq47C ~ qen

l v'

G
r eo
t +
<E--r~
Ions will hit ifr > w. ++ +
(
w=qh..V +
+
1 2 + +
-mv =qV
2 force on q' is zero

v= ~2!V
58. 4' = qen
to
mv 2
- - =:aqvB q~0 is not zero for a and b. therefore flux ( 4') is
r
not zero.
=> r = mv = !!!_~2qV
qB qB m 59.
y
r= _2_~2mV
B q
Yyt ,,lf
_IL,.....,.vo
-1 ~2mv:
-->w ,, '
B q , .
,,

2mV
-->w
2D2 --+-=-+----~x
q Vo
2mV h I
=>q<-- ').= - =>A.OC -
w2B2· mv v
v= ~v~ + v~
56. Vy= Uy+ ayl

X )( )( )( v =0 + qEo t
Y m

Lr
)( )( )(

(3vo) = ~v; + v~
v= const. => v 2y = 8v02

inside B speed will be constant therefore D => qEo t= 2.Ji v0


m
option is correct, representation of speed
2..fim
=>t= --Vo
57. qEo
m

G
toc -
q

www.puucho.com
CP Publication KVPY chapterwise with www.puucho.com

• ELECTRODYNAMICS

...60. 61, XL = roL = 21t x 50 x 6 = 600 nO.


y
Xe = ...!... = 1 _ 100 50
roC 2x80 x 200 x 10-G - 2n: = -;- Q
'X X X X
lmax = V = 220
X X X X
z JR2 + (Xr, - Xc)2
x x 1 x 15:
xx~ V
X X X X = ,===2=2~0===

V~mr= v
.._____,JL_____.__ _ ___,.

Bo.
• X

400 2 +( :J
6001t- 5

0
lmox = 0.120 A

-vBa .r--7
L:3
i = vBa/R
p = iiB = vBa · Ba
R
vB 2a 2
Jlill= - -
R
-mdv vB2a2
--=
dt R
vdv B2a2
J =>-m-=v--
dx R
B2 2
f Jdx
. V X

-m dv=+
Vo 0
B2 ~
- m (v - v0) =~
R
- B2a2
=,.v-vo- - - x
mR

www.puucho.com
I
1
CP Publication KVPY chapterwise with www.puucho.com

Optics
I
1. The folloY.ing travelling electromagnetic wave 5. A narrow parallel beam of light falls on a glass
~ = 0, Ey = Eosin(kx + ffil:), ~ = -2Eosin(kx - rot) sphere of radius R and refractive index µ at
is- 120101 normal incidence. The distance of the image
(A) elliptically polarized from the outer edge is given by- [2010]
j.l3'rcircularly polarized ~ (B) R(2+µ)
(C) linearly polarized 2(µ-1) 2(µ-1)
(D) unpolarized
(C) R(2-µ) (D) R(2+µ)
2(µ + 1) 2(µ + I)
2. A point source of- light is placed at the bottom
{ of a vessel which is filled with water of
6. A plane polarized fight passed through
refractive index µ to a height h. lf a floating
successive polarizers which are rotated by 30°
opaque disc has to be placed exactly above it so
with respect to each other in the clocl-...vise
that the source is invisible from above, the direction. Neglecting absorption by the
radius of the disc should be- f2010J polarizers and given that the first polarizer's
(A)-h- rY!f h axis· is parallel to the plane of polarization of
~ ~f12 -1 the incident light, the intensity of light at the
exit of the fifth polarizer is closest to - 12011]
cq---f!----
µ~-1
(D) µh (A) same as that of the incident light
(B) }l.5% of the incident light
~µ2-1
'1,C1'1D% of the incident light
3. Three transparent media of refractive indices (D)zero
µ 1, µ 2, µ 3 respectively, are stacked as shown. A
ray of light follows the path shown. Nd light
7. In a Young's double slit set-up, light from a
laser source falls on a pair of very narrow slits
enters the third medium. (2010]
separated by 1.0 micrometer and bright fringes
· separated by LO millimeter are observed on a
distant screen. If the frequency of the laser light
is doubled, what will be the separation of the
bright fringes? ~ ',c (2011J
(A) 0.25 mm ( '\(B)'0.5 mm
(C) 1.0 mm (D) 2.0 mm
Then-
(A) µ1 < µ2 < µ3 (-§,) µ2 < µ1 < µ3 8. A material is embedded between two glass
(C) µ1 < µ3 < µ2 _.-(I)) µ3 < µ1 < µ:;: plates. Refractive index n of the material varies
with thickness as shown below. The maximum
4. In Young's double slit experiment, the distance incident angle (in degrees) on the material for
between the two slits is OJ mm, ·the distance which beam will pass through the material is-
between the slits and the screen is 1 m and the [2011]
wavelength of the light used is 600 nm. The n

.
intensity at a point on the screen is 75% of the 1.5 Glass Material Glass

Z
maximum intensity. What is the smallest
d. ta e of this point from the central fringe? 1.2
{2010]
0 mm - (B) 2.0 mm · X
(C) 0.5 mm (D) 1.5 mm (A) 60.0 (C) 43.5 (D) 32.3

b
,1, www.puucho.com
f
,t-i, '-
CP Publication KVPY chapterwise with www.puucho.com

t OPTICS
9. In a Young's double slit experiment the 13. Monochromatic light passes through a prism.
intensity of light at each slit is 10• Interference Compared to that in air, inside the prism tbe
pattern in observed along a direction parallel to light's (2014]
the line S1 S:i, on screen S.- r20121 (A~pEcd and wavelength are different but
/ frequency remains same.
(B) speed and frequency are different but
wavclengtl remains same.
(C) wavelength and frequency are different,
but speed remains same.
(D) speed, wavelength and frequency are all
s difTcrcnL

The rmrumum, maximum, and the intensity 14. The flat face of a plano-.convex Jens of focal
averaged over the entire screen arc respectively length IO cm is silvered. A point source placed
!}'(o, 4Io, 2lo (B) 0, 410, lo 30 cm in front of the curved surface will
(C) Io, 2Io, 3lo/2 (D) 0, 2o, lo produce a (2014J
(A)? I image 15 cm away from the lens
10. An unpolarized beam of light of intensity lo .2..,'f
real image 6 cm away from the lens
passes through two linear polarizers making an (C) virtual image 15 cm away from the lens
angle of 30° with respect to each other. The (D) virtual image 6 cm away from the lens
emergent beam will have an intensity- (20121

(A) 3Io (B) Jir0 15. A concave mirror of radius of curvature R has a
4 4 circular outline of radius r. A circular disc is to
(C) 31.J>""" (D) .!_q_ be placed normal to the axis at the focus so that
{..?g 8 it collects all the light that is reflected from the
mirror from a beam parallel to the axis. For
11. On a bright sunny day a diver of height h r << R, the area of this disc has to be at least
stands at the bottom of a lake of depth H. · [2014]
Looking upward, he can sec objects outside the
lake in a circular region of radius R. Beyond
this circle be sees the image of objects lying on
the floor of the Jake. If refractive index of water
is 4/3, then the value ofR is- (20121
~-h)l.fi (B) (H-h)/M 16. The angles of incidence and refraction of a
(C) 3h.fi (D)(H - h) t ../573 monochromatic ray of light of wavelength ). at
an air-glass interface are i and r, respectively. A
1 ~A ray of light incident on a glass sphere parallel beam of light with a small spread &l in

;( (refractive index ./3) suffers total internal wavelength about a mean wavelength )., is
refracted at the same air-glass interface. The
reflection before emerging out exactly parallel
refractive index µ of glass depends on the
to the incident ray. The angle ofincidence was
wavelength J.. as µ(J..) = a+ b/J..2 where a and b
(2013]
(A) 75° are constants. Then the angular spread in the
(C) 45° angle of refraction of the beam is f2014J

www.puucho.com
CP Publication KVPY chapterwise with www.puucho.com
II
I

TOPIC~ WISE KVPY (STREAM SX] Questions with Solutions

(A) if the material has a refractive index very


(A) sini oi..l nearly equal to zero.
'),.3 cosr
(B) only with gamma rays with a wavelength
smaller than the atomic nuclei of the
material
~ ; the material has a refractive index less
than zero.
17. A beam of monocnergetic electrons, which (D) only if the wave travels in M with a speed
have been accelerated from rest by a potential faster than the speed of light in vacuum.
U, is used to form an interference pattern in a
Young's Double slit experiment. The electrons 20. Electrons accelerated from rest by an
are now accelerated by potential 4U. The fringe electrostatic potential arc collimated and sent
through a Young's double slit setup. The fringe
width - (2015]
width is w. If the accelerating potential is
(A) remains the same
A is ha[fthe original fringe width
doubled then the width is n~osc to - f201(?J
(A)0.5w r-ttlJ0.7w
(C) is twice the original fringe width
(C) 1.0 w (D) 2.0 w
(D) is one-fourth the original fringe width
21. A monochromatic light source S of wavelength
18. A cubical vessel has opaque walls. An observer 440 nm is placed slightly above a plane mirror
(dark circle in figure below) is located such that Mas shown. Image ofS in M can be used as a
she can see only the wall CD but not the virtual source to produce interference fringes
bottom. Nearly to what height should water be on the screen. TI1e distance of source S from 0
poured so that she can see an object placed at is 20.0 cm, and the distance of screen from O is
the bottom at a distance of 10 cm from the 100.0 cm (figure is not to scale). If the angle
corner C? Refractive index ofwatcr is 1.33. e = 0.50 x w-J radian-., the width of the
12015] interference fringes observed on the screen is -

ALJD

B C
s ·--
(2016]

(A)J9cm (B) 16cm


.§lf27 cm (D) 45 cm (A) 2.20mm
(C) l.10 mm
19. Electromagnetic waves emanating from a point
A (in air) are incident on a rectangular block of A glass prism has a right -triangular cross
material M and emerge from the other side as section ABC, with LA = 90°. A ray of light
shown. The angles i and r are angles of parallel lo the hypotenuse BC and incident on
incidence and refraction when the wave travels the side AB emerges grazing the side AC.
from air to the medium. Such paths for the rays Another ray, again parallel to the hypotenuse
BC, incident on the side AC suffers total
arc possible ..------. (2015) internal reflection at the side AB. Which one of
the following must be true about the refractive
indexµ of the material of the prism? jZ016J
A
( ~ < µ < .J2 (B) µ > ,.{j

ULr .tvl au (C) ~l < H (D) .J2 < µ < ,.{j

www.puucho.com
CP Publication KVPY chapterwise with www.puucho.com

• OPTICS
23. A carrot looks orange in colour because of the z
~ carotene molecule. in it. This means that the
~ carotene molecule absorbs light of t8 ffi

~
~
wavelengths (5 Nov, 20171 ,,..u u
r:f)
(A) longer than 550 nm. 0
'-0
,.9,l-}1hor1er than 550 nm.
(C) longer than 700 run.
!
(D) shorter than 700 run. I I

24. It was found that the refractive index of material ---

~
of a certain prism varied as LS + 0.0041?.2, (B)
where A is the wavelength of light used to
measure the refractive index. The same z z
material was then used to construct a thin prism
of apex angle 10°. Angles of minimum
I I
deviation C<'>m) of the prism were recorded for
the sources with wavelengths A1 and A2 (D)
(C)
respectively. Then [5 Nov, 2017)
(A) Om (A1) < Om (12) iD~1 < J.2.
(B)pn, (11.1) > Dm (:l2) if11.1 > 11.2. z z
J_C?J Om 0-1) > Dm (]'-2) iD.1 < A2.
(D) 8.., is the same in both the cases. 27. Three circularly shaped linear polarisers are
placed coaxially. The transmission axis of the
25. Two circularly shaped linear polarisers are
first pol ariser is at 30°, the second one is at 60"
placed coaxially. The transmission axis of the
first polarizer is· at 30° from the vertical while and the third at 90° to the vertical all in the
the second one is at 60°, both in the clocl-.'Wise clockwise sense. Each polariser additionally
sense. If an unpolarised beam of light of absorbs l 0% of the light. If a vertically
intensity I= 20 W/m2 is incident on this pair of polarised beam of light of intensity I = l 00
polarisers, then the intensities 11 and Ii \V/m2 is incident on this assembly of polarisers,
transmitted by the first and the second

Z laris s, respectively, will be close to

C 11 = 10.0 W/m2 and Ii= 7.5 W/m2


[5 Nov, 2017)
then the final intensity of the transmitted light
will be close to
(A) 10 W/m
5),1-0W/m2
2
119 Nov, 20171
(B) 20 W/m2
(D) 50 W/m2
(B) 11 = 20.0 W/m2 and h = 15 W/m2
2
(C) 11 = 10.0 W/m- and 12 = 8.6 W/m
?

2
(D) 11 = 15.0 W/m and h = 0.0 W/m-
?

28. Consider a plane parallel beam of light incident


on a plano-cylindrical lens as shown below.
26. A parallel beam of light is incident on a tank Which of the following will you observe on a
filled with water up to a height of 61.5 mm as screen placed at the focal plane of the lens?
shown in the figure below. Ultrasonic waves of
[19 Nov, 2017)
frequency 0.5 MHz are sent along the length of y
the water column using a transducer placed at·
the top, and they form longitudinal standing X

waves in the water. Which of the schematic


plots below best describes the intensity
distribution of the light as seen on the screen? SCREEN
Take the speed of sound in water to be
1,500 mis. (S Nov, 20171

www.puucho.com
CP Publication KVPY chapterwise with www.puucho.com

TOPIC-WISE KVPY [STREAM SX] Questions with Solutions

(A) The screen will be uniformly illuminated.


(B) There will be a single bright spot on the
screen.
(C) There will be a single bright line on the
screen parallel to the x-axis
~JTI1ere will be a single bright line on the
screen parallel to they-a.xis

Back surface of a glass (refractive index n and


thickness t) is polished to work as a mirror as
shown below. A laser beam falls on it and is
partially reflected and refracted at the air-glass
interface and fully reflected at the mirror
surface respectively. A pattern of discrete spots
of light is observed on the screen.
[19 Nov, 2017)

Glass I
Mirror f I

The spacing between the spots on the screen


will be

(A4cos8 2tsin 0
'/· ,J ~ -Sm~e
.~ (B)-===
_,,TI~ ~n 2 -sin 2 8
2ttan0 (D) 2tsin0
(C)-====
,.Jn 2 -sin 0 2
~
f-~

.
www.puucho.com
CP Publication KVPY chapterwise with www.puucho.com

t OPTICS

I · ANSWERS
1. (B) 2. (B) 3. (D) 4. (A) 5. (A) 6.(C) 7. (B)
8. (B) 9. (A) 10. (C) 11. (A) 12. (D) 13. (A) 14. (B)
15. (B) 16. (C) 17. (B) 18. (C) 19. (C) 20. {B) 21.(B)
22. (A) 23~ (B) 24. (C) 25. (A) 26. (A) i7.(C) 28. (D)
29. (A)

• SOLUTIONS
--:rn I 119

1. From the equation of Ey & E.: it is evident that 4. d= 0.1 mm. D = l m.). = 600 nm
wave is circularly polarized. Ir= 75 % of maximum or Ip= 310
Where 10 is the intensity of a single wave
2.
now

Ip= 31 0 = (,J!;)2 + (jl;)2 + 2 ~ coslj,


1t yd
:. cos~= cos-, also ax= -
3 D
A x 1
now ax= - x - = -
source 2n 3 6
r should be such that rays beyond it got totally
J.D 600xl0-9 xl C""':""7:1
internally reflected
For this O> C or sin 9 > sin C
y= 6d = 6x0.lxl0- 3 orty=lmj
5.
1 r
also µ = - - -;==== > -
sine ~h2 +r2 µ
r I
In limiting case -:=== - -
~h2 +r2 µ

solvfog we get ~ - ~µ2h -1 I ~1 I µ-.l


now - - - = - - :::>Vi= - -
µR
V1 co R µ-1

3. At first incidence light is deviated towards the now 1 µ = I-µ


normal therefore µ 2 > µ 1• Also at second Vr -(2R-V1) -R
incidence TIR takes place therefore µ2 > µ3,
replace VI by µR and solving for V f
also µ, > µ 3 because for the same angle in µ-1 .
medium µ2, angle in µ1 medium is less.
we get V = R(µ- 2)
f 2(µ-1)
First image is real and second is virtual. 85

6. I= Ia {cos 2~)4

=Iox(¾r =30%ofio

www.puucho.com
CP Publication KVPY chapterwise with www.puucho.com

TOPIC- WISE KVPY [STREAM SX] Questions with Solutions


7. Separation Onel = 180"
. bt fri nge= -).D
B ng 61 + 62 + 03 = 180°
d i-r+ 180°-2r+i-r= 180"
f). =C 2i=4r
lf f is doubled
i = 2r ...(1)
}, become halved
: . ~ become half
Snell law
I
I}= - = 0.5 mm
1 x sin i = ../3 sin r ... (2)
2
from ( 1) & (2) sin i = ,,/3 sin i_
2
8. 1.5 x sini = 1.2 sinr
. 1.5 .. . i
2SID - COS -
i = v-'
r::;3 •
Sin -
i
smr"" - suu 2 2 . 2
1.2
T/R should not take place
::::.cos
.J3
.·. sinr < I 2 2
. . < l ; s1m
-1.5 srru . . < -12 ; s101
. . < OB
.
i
- =30
1.2 15 2
sin 45 = Jz = 0.707
::::. i = 60°

imax > 45 13. On refraction of light, frequency remain


unchanged. However speed and wavelength get
9. Im;n = 0 ; Im""= 4Io
Iav = 2Io
change.

14.
10.
R

h'
11. R=--
. Jµ2
-1

12.
f=IOcm
After silvering of flat face lens behave as
mirror of focal lec.gth feq·

~ n(l)
,=-(2)
~ (3)
~

1 I I I
- =-+ +-
feq r1 r2 f3
1 2 1
::::, - ... -+ -
feq fl f2
l 2 1
-=-+-
feq 10 o:i

feq = 5

www.puucho.com
CP Publication KVPY chapterwise with www.puucho.com

1 OPTICS

+ 0
O=~os.rdr ( aJ..2 +-,r-b) +sinr(-2b)
T d1
d). 2b sinr cosr dr (v..2 + b)
A
dr.,. 2bdl. tanr
30cm
l. (al2 + b)

mirror i.ormu la -l
.c.
= -1 + -1 Br = (2b tan r)8A.
f U V (al?+ bA)
l 1 l
=> - = - - + -
-5 -30 V 17. Debroglic wavelengt~ of electron
v=-6cm
Image is real and 6 cm away from silvered lens. i. = .J h
2mK.
Kinetic energy K=qV I I
15. 1
A,CC-
rv
Potential becomes four time so wavelength
becomes half.
C
Fringe width (P) = i.D
d
I~rd. I
Fringe width becomes half.
I
18.

Area of disc
= 'ltX2 = 'It j2 tan2 8 h

=rr. {~[1-sece]}\an2 8
R2
""'It -[1-sec 8] 2 tan2 8 <E-+
4 (h- lOcm)
=> = 1t R 2(2:_) (.!:_)
4 R 2
R 2
From diagram r = 45°
using snell law
4 . . .
- s1ru =sin r
3
. • 3
stru=--
16. Snell law ..fi.x4
sin i =µsin r
. . ( b) .
sm1= a+ J...2 smr 3

Differentiating with respect to 1

O=cosrdr (a+ ~)+sin{; (-2)}11. .fjj


. 3 . h-10
tan1= - - => tan1= - -
.Jn h

www.puucho.com
CP Publication KVPY chapterwise with www.puucho.com

TOPIC• WISE KVPY [STREAM SX] Questions with. Solutions


htani=h-10 ::::> 2x2oxe
lO=h [1-tani] => 40 x 0.5 x 10-3 cm
h,;= 10 2 x 10-2 cm
1-tani p =AD= 440x 10--6 x 120xl0 2
=> 27 approx. = 27 cm d 2 X 10-z X 1Q2
264 X w-2 =} 2.64 mm
19. 22.
B

Meta mater:ials are the material for which


refractive index is negative for them. Refra~tion
diagram is shown,
here. In question same type of diagram is given.

20. l3 = AD
d
h . h
A=---.===
mV ~2mqtJ.V
13 o: A
. 1
l3oc .JiN
as .6. V is double

13 is .Jz times ofP01d


1311ew=0.7P r+ 0c= 90° ....•(1)
=0.7w 1 x sin (90° - a) = µsin r
cos a= µsinr ..... (2)
21. 90°-e> Be ..... (3)
µsin e = I x sin a: ..... (4)
Screen
(3) &(4)
90" 8c > e
cos 8c >sine
sino.
cos Be> - -
µ
. 2 rJ
. 2 8c> - 1 [1 -µ 2 sm
l -sm
µ2 .
---D,---+I
1 - - 1 > - 1 [1 - µ 2 sin 2 (90° - 8c)J
µ2 µ2
Sand SI are source ofYDSE
1 l 2
8 = 0.5 x 10-3 radian (very small) 1-->--cos 8c
µ2 µ2
D = SO cos 8 + 100
= 20 x I+ 100 1]
1-_3:_>-[t--
µ2 µ2
= 120cm
d = 2 x SO sin 0

www.puucho.com
CP Publication KVPY chapterwise with www.puucho.com

• OPTICS

3
2>- It= .!,g_ = lO = 10 W/m2
µ2 2 2

µ_..
~
(1) & (2)
1J'/32
cos a= µ sin (90° - 0c)
= Io[
= 7.5 w/m2
~r
12 = I, cos 2 30'

= 10. !
cos a= ~r cos 0c
cos a< l 26. v=v).
µcos 0c < I 1500
=> l 3000 x 10---6

R<t
= --- =
0.5x10 6
1=3mrn
I I
I - -2< - 2-
µ µ

1 < -3___
~12
SCREEN
µ< .fi intensity vary according to graph 'A'

:. J¾ <µ< ,Ji 27.

23. VIBGIYOR
400 nm I 700 nm
Absorbed Reflected
therefore seen

· (om +A)
sm
24. n= sini = 2
sinr
sin(~) I1 = Io X 0.9
vertical
cos½o 0 = Io X 0.9 X
3
4
sin(8m2+A)
1.5 + 0.004 = ---'---:::.... 12 = 11 X 0.9 cos 230° = 11 X 0.9 X 3
12
sin(~) h = I2 x 0.9 cos230° = Ii x 0.9 x 3
4

l 4
OmOC -
A
:. 0m (11.1) > 0,,,(11.2) if 1. 1 < 11.2
=;,13 = lo (0.9)3 (¾J
I3= 30.75 w/m1 .
25. 28.
X

no deflection oflight beam


vertical
Io= 20 W/m2

www.puucho.com
CP Publication KVPY chapterwise with www.puucho.com

TOPIC- WISE KVPY [STREAM SX] Questions with Solutions

29.

-1---+
---d---t!

Therefore Ax = h1- h2

tan0=~ & tan0= d-L


h1 h2

6.x=-d- _ d-L
tan9 tanO

Ax=_!:_
tan9
Snell's Law
. O . l . sine
1 · sm =n·s10 r =:> sm r"" - -
n
·L
tan r = - =:> L"" 2 t tan r
2.l

sin 8

L= 2t-sin0
,Jn 2 -sin 2 8
:. Ax= 2tsin e
Jn 2 -sin 2 e tan 0
tan 8 = 2tcos 0
~n 2 -sin 2 0

www.puucho.com
CP Publication KVPY chapterwise with www.puucho.com

Modern Physics
I
I. A nucleus has a half-life of 30 minutes. At 4. Given below are three schematic graphs of
3 PM its decay rate was measured as 120,000 potential energy V(r) versus distance r for three
counts/sec. What 'Will be the decay rate at atomic particles : electron (el, proton (p J and
5 PM? (2010] neutron (n), in the prc.~ence of a nucleus at the
(A) 120,000 counts/sec
origin O. The radius of the nucleus is ro. The
(B) 60,000 counts/sec
scale on the Y-axis may not be the same for all
(C) 30,000 counts/sec
figures. The correct pairing of each graph with
(D) 7,500 counts/sec
the corresponding atomic particle is - 12011]
2. At time t = 0, a container has N 0 radioactive V V
atoms with a decay constant le. In addition, c
numbers of atoms of the same type are being Qi,--.---
ro r
added to the container per unit time. I low many
atoms of this type are there at t = T?
120101 (1) (2)
C
(A) -exp(-'..\.:I)-N 0 exp(-lT) V
).

C
(D) - exp(-tT) + No exp(-).T)
A r
C
(C) - {1 - exp(-AT)} + N0 exp(-AT)
A
(3)
,.
C
(D) - {1 + exp(-J. T)} + No exp(-Ji.T)
(A) (1, n)j (2, p \ (3, el
(B) {I, PJ, (2, ej, (3, n)
3. An electron collides with a free molecules
(C) (1, el, (2, PJ, (3, n)
initially in its ground state. The collision leaves
the molecules in an excited state that is (D) (1, p J, (2, n), (3, CJ

metastable and docs not decay to the ground


5. Due to transitions among lt'> first three cm..-rgy
state by radiation. Let K be the sum of the
initial kinetic cnergic.~ of the electron and the levels, hydrogenic atom emits radiation at three

molecule, and P the sum of their initial discrete wavelengths )ci, A.2 , and A3 (A. 1 < Az < ,.3).
Then - [2011]
momenta. Let K' and P'rcprescnt the same
physical quantities aft.er the collision. Then - (A) t.1 = ).2 + A)
[2011] (B) A1 + 7..2 = 11.3
(A)K=K:, P-P' (B) K'< K, P-= P' (C) lt?.1 + IIA2 = 1/}.3

(C)K=K', P-1:-P' (D)K'<K, PctcP' (D) l/i,1 = l!A2 + 1/.11,3

www.puucho.com
CP Publication KVPY chapterwise with www.puucho.com

TOPIC• WISE KVPY [STREAM SX] Questions with Solutions


6. A parent nucleus X is decaying into rumghtcr 10. In a photocell circuit the stopping potential, V0,
nucleus Y which in tum decays to Z. The half is a measure of the maximum kinetic energy of
lives of X and Y arc 40000 years and 20 years the photoelectrons. The following graph shown
respectively. 1n a certain sample, it is found
experimentally measured values of stopping
that the number of Y nuclei hardly changes
with time. If the number of X nuclei in the potential versus frequency v of incident light.
sample is 4 x I 0 20, the number of Y nuclei Vo(V)
present in its is- [20l2J 5
(A) 2 X 10 17 (B) 2 X 1020 ,/
4
(C) 4 X }0 23 (D) 4 X 1020
3
/
/
/
7. A radioactive nucleus A has a single decay 2 /

mode w·ith half life 't"A- Another radioactive /


nucleus B bas two decay modes 1 and 2. If 0
V ----,,,
dec:1y mode 2 were absent, the half life of B ,
p.4' 0.8 1.2 1.6 2
-1 ,
would have been r.A/2. If decay mode I were ,'
,'
-2
absent, the half life of B would have been 3tA.,

then the ratio i.:s/r.A is- [20121


The values of Planck's constant and the work
(A) 3/7 (B) 7/2
(C) 7/3 (D) 1 function as determined from the graph are
(taking the magnitude of electronic charge to be
8. A stream of photons having energy 3 eV each
e = 1.6 x w· 19 C) (20131
impinges on a potassium surface. The work
(A) 6.4 x 10-34 Js, 2.0 eV
function of potassium is 2.3 eV. The emerging
photo-electrons are slowed down by a copper (D) 6,0 X 10-34 Js, 2.0 eV
plate placed 5 mm away. If the potential (C) 6.4 x 10-34 Js, 3.2 eV
difference between the two metal plates is I V, (D) 6.0 x 10- Js, 3.2 eV 34

the maximum distance the electrons can move


away from the potassium surface before being ll. Two species of radioactive atoms are mixed in
turned back is- (2012] equal number. The disintegration constant of
(A) 3.5 mm (B) 1.5 mm
(C) 2.5 mm (D) 5.0 mm the first species is I\. and of the second is "JJ3.
After a long time the mixture will behave as a
9. A nuclear decay is possible if the mass of the
species with mean life of approximately (2013]
parent nucleus exceeds the total mass of the
(A) 0.70/). (B) 2.10/).
decay particles. If M(A,Z) denotes the mass of
a single neutral atom of an element with mass (C) 1.00/)., (D) 0.52/A.
number A and atomlc number Z, then the
Il1lnimal condition that the fl decay 12. Photons of energy 7 e V are incident on two

X~ ----t Yf.,.1 +p- +vc


metals A and B with work functions 6 e V and
will occur is (ll1c denotes the mass of the 3 eV respectively. The Il1lnimum de Broglie
p particle and the neutrino mass my can be wavelength of the emitted photoelectrons with
neglected): (2013] maximum encrgie.c; arc /\.A and ;>.. 8 , respectively
(A) M(A,Z) > M(A,Z+l) + me where/\.;,/ 11. 8 is nearly (2013]
(B) M{A,Z) > M{A,Z + 1)
(A) 0.5 (B) 1.4
(C) M(A,Z) > M(A,Z+ l) + Zm:
(D) M(A,Z) > M(A,Z+ l) - ~ (C) 4.0 (D) 2.0

www.puucho.com
CP Publication KVPY chapterwise with www.puucho.com

e MODERN PHYSICS
13. A singly ionized helium atom in an excited 18, A 1.5 kW (kilo-watt) laser beam of wavelength
state (n = 4) emits a photon of energy 2.6 eV. 6400 A. is used to levitate a thin aluminium disc
Given that the ground slate energy of hydrogen of same area as the cross section of the beam.
atom is - 13.6 cV, the energy (Er) and The laser light is reflected by the aluminium
quantum number (n) of the resulting state are disc without any absorption. The mass of the
respectively- 120.13} foil is close to (20141
(A) Er=-13.6 eV, n=l (A) I0-9 kg (B) J0-3 kg
(B) Er=-6.0 eV, n= 3 (C) 10-4 kg (D) 10--6 kg
(C) Er= - 6.0 cV, n = 2
{D) Er= - 13.6 cV, n = 2 19. When ultraviolet radiation of a certain
frequency falls on a potassium target, the
14, The half life of a particle of mass 1.6 x 10-26 kg photoelectrons released can be stopped
is 6.9 s and a stream of such particles is completely by a retarding potential of 0.6 V. ff
travelling with the kinetic energy of a particle the frequency of the radiation is increased by
being 0.05 cV. The fraction of particles which 10%, this stopping potential rises to 0.9 V. The
will decay when they travel a distance of I m work function of potassium is (20141
is - (2014) (A) 2.0 eV (B) 2.4 eV
(A) 0.1 (B) 0.01 (C) 3.0 eV (D) 2.8 eV
(C) 0.001 (D) 0.0001
20. Carbon-ll decays to boron-II according to the
15. A 160 watt light source is radiating light of following fonnula.
wavelength 6200 A uniformly in all directions. ~1C ~ ; 1B +e++v 0 +0.96MeV
The photon flux at a distance of 1.8 mis of the
Assume that positrons (el
produced in the
order of (Planck's constant 6.63 x I 0-34 J-s)
decay combine with free electrons in tbe
[2014)
atmosphere and annihilate each other almost
(A) l 02 m- 2 s- 1 (B) 1012 m-2 s-1
immediately. Also assume tbat the neutrinos
(C) 10 1? m-2 s-1 (D) 1025 nf 2 s- 1
(ve) are massless and do not interact with the
16. The wavelength of the first Balmer line caused environment. At t = 0 we have I µg of ~2 C . If
by a transition from then= 3 level to then= 2 the half-life of the decay process is 4.l, the net
level in hydrogen is~-+ The wavelength of the energy produced between time t = 0 and t = 2to
line caused by an electronic transition from will be nearly - I20151
n = 5 to n = 3 is - [2014] (A) 8 x 10 18 MeV (B) 8 x 10 16 MeV
(C) 4 x 10 15 McV (D) 4 x 10 16 McV
(A) 375 '}.. (B) 125 '}..
128 1 64 I
21. Two unifonn plates of the same thickness and
(C)~i. (D) 128;.
area but of different materials, one shaped like
125 1 375 I
an isosceles triangle and the other shaped like a
17. The binding energy per nucleon of 5B 10 is rectangle arc joined together to form a
8.0 MeV and that of jB 11 is 7.5 MeV. The composite body as shown in the figure. If the
enerp required to remove a neutron from centre of mass of the composite body is located
sB 1 is ~mass of electron and proton are at the midpoint of their common side, the ratio
9 .11 x 103 kg and 1.67 x I 0 27 kg, respectively) -
between ma."ses of the triangle to that of the
[2014]
rectangle is - (20151
(A) 2.5 MeV (D) 8.0MeV
(C)0.5 MeV (D) 7.5 MeV

www.puucho.com
CP Publication KVPY chapterwise with www.puucho.com

TOPIC-WISE KVPY [STREAM SX] Questions with Solutions

26. An electron and a photon have same


wavelength of 10-9 m. If E is the energy of the
photon and p is the momentum of the electron,
the magnitude ofE/p in SI units is 12016]
(A) 1.00 X 10-9 (B) 1.50 X 1011
(A) I : 1 (B) 4: 3
(C) 3: 4 (D) 2: 1 (C) 3.00 X 108 (D) 1.20 X }07

2-2. A 160 watt infrared source is radiating light of 27. If one takes into account finite mass of the
wavelength SO000A uniformly in all directions. proton, the correction to the binding energy of
The photon flux at a distance of 1.18 m is of the the hydrogen atom is approximately (mass of
order of- (2015] proton = 1.60 x IO ·27 kg, mass of electron
(A) 10 m-2s-1 (B) 10 10m-2 s- 1 = 9.10 X 10-Jl kg)- (2016]
(C) 1o's m-2s-1 (D) 102om-2s-1 (A) 0.06 % (B) 0.0006 %
(C) 0.02 % (D) 0.00 %
23. Photons of wavelength i. arc incident on a
metal. The most energetic electrons ejected
28. A photon of wavelength}. is absorbed by an
from the metal arc bent into a circular arc of
radius R by a perpendicular magnetic field electron confined to a box of length
having a magnitude B. The work function of ..J35hA./8mc . As a result, the electron makes a
the metal is (Where symbols have their usual transition from state k = 1 to the state n.
meanings)~ 120161 Subsequently the electron transits from the state
he e 2 B 2R 2 n to the state m by emitting a photon of
(A) - - m e + - - -
A· 2me wavelength}.'= 1.85 A. Then [2016)
(A) n=4; m=2
(B) he + 2me ( eBR
A. 2m 0
)2 (B)n=5; m= 3
(C)n= 6; m=4
(C) he -'ineC2 - e2B2R z (D)n=3; m= 1
A 2me
29. The magnitude of acceleration of the electron
(D) he - 2me ( eBR
A. 2me
)z in the n1h orbit of hydrogen atom is aH and that
of singly ionized helium atom is a He· The ratio
is
aH : aHc [5 Nov, 2017]
24. An electron with kinetic energy E collides with
(A) 1: 8
a hydrogen atom in the ground state. The
(B) 1: 4
collision wmbe elastic [2016]
(A) For all values ofE · (C) l: 2
(B) For E < 10.2 eV (D) dependent on n
(C) For 10.2 eV < E < 13.6 cV only
(D) For O< E < 3.4 eVonly 30. Using dimensional analysis the resistivity in
terms of fundamental constants h, Ille, c, e, so
25. The continuous part of X-ray spectrum is a can be ex pressed as [S Nov, 10171
result of the [20161
(A) Photoelectric effect
(B) Raman effect
(C) Compton effect (D) me£o
(D) Inverse photoelectric effect ce 2

www.puucho.com
CP Publication KVPY chapterwise with www.puucho.com

t MODERN PHYSICS
31. An electron in an electron rrucroscope with 35. Consider the following nuclear reactions:
initial velocity Vo i enters a region of a stray I. ~4 N +~ He --).~7 0 +X
transverse elcct.ric field E 0 j . The time taken for
II. : Be +i 11 --).~2 He+ Y
the change in its de-Broglie wavelength from
Then 119 Nov, 20171
the initial value on. to "A./3 is proportional to
(A) X and Y arc both protons.
(5 Nov, 20171
(B) X and Y arc both neutrons.
(A) Eo (B)-1 (C) ~ (D) ,/Bo (C) X is a proton and Y is a neutron.
Ea -..,Ea
(D) Xis a neutron and Y is a proton.
32. A particle of mass m moves around the origin
. . I l 2 2 •
ma potcntia - mw r , where r 1s the distance 36. The n-side of the depletion layer of a p-n
2 junction: [19 Nov, 20171
from the origin. Applying the Bohr model in (A) always has same width as of the p-side.
this case, the radius of the particle in its n1h
(B) has no bound charges.
orbit in terms of a= ,Jh /(2mnw) is
(C) is negatively charged.
15 Nov, 20171 (D) is positively charged.
(A) a..fn (B)an
(C) an 2 (D) an,Jn 37. Some of the wavelength observed in the
emission spectrum of neutral hydrogen gas arc
33. A neutron star with magnetic moment of
magnitude m is spinning with angular velocity
912, 1026, 1216, 3646, 6563 A. If broad band

w about its magnetic axis. The electromagnetic light is passing through neutral hydrogen gas at·
room temperature, the wavelength that will not
power P radiated by it is given by µ~mY oh11
be absorbed strongly is [19 Nov, 2017]
where µo and c are the permeability and speed
of light in free space, respectively. Then (A) 1026 A (D) 1216 A
(5 Nov, 20171 (C) 912 A (D) 3646 A
(A) x = 1, y=2, z= 4 and u =-3
(B) x = l, y = 2, z ""4 and u = 3 38. The emission series of hydrogen atom is given
(C) x =-1, y= 2, z=4 and u=-3 by
= 4 and U"" 3

34.
(D) x:==-l,y"" 2, z

To calculate the size of a hydrogen anion using


~ ~ R(nl( _ :J
the Bohr model, we assume that its two where R is the Rydberg constant. For a
electrons move in an orbit such that they are transition from n2 to Di, the relative change
always on diametrically opposite sides of the
t.J,,,f"A. in the emission wavelength if hydrogen is
nucleus. With each electron having the angular
replaced by deuterium (assume that the mass of
momentum ti = h/2x, and taking electron
interaction into. account the radius of the orbit proton and neutron are the same and
in tenns of the Bohr radius of hydrogen atom approximately 2000 times larger than that of
electrons) is (19 Nov, 2017}
(S Nov, 2017]
(A) 0.025 % (B) 0.005 %
(C) 0.0025 % (D) 0.05 %
(A) aB

www.puucho.com
CP Publication KVPY chapterwise with www.puucho.com

TOPIC• WISE KVPY [STREAM SX] Quest.ions with Solutions

39. When light shines on a p-n junction diode, the 40. Consider the following statements regarding
current (I) vs, voltage (V) is observed as in the the photoelectric effect experiment:
figure below: J19 Nov, 2017) (I) Photoelectrons are emitted as soon as the
metal is exposed to light
I Quad I
Quad2 (II) There is a minimum frequency below
which no photo-current is observed
(III)The stopping potential is proportional to
the frequency of light
V
(IV)The photo-current varies linearly with the
intensity of the light
Quad 3 Quad4
Which of the above statements indicate that
light consists of quanta (pholons) with energy
In which quadrant(s) does the diode generate
power, so that it can be used as a solar cell ?
proportional to frequency? (19 Nov, 2017]
(A) I and III only
(A) Quad 1 only
(B) Quad 1 and 3 only
(B) 11 and III only
(C) U, Ill and IV only
(C) Quad 4 only
(D) 1, ll and III only
(D) Quad I and 4 only

www.puucho.com
CP Publication KVPY chapterwise with www.puucho.com

• MODERN PHYSICS

ANSWERS
l. (D) 2. (C) 3. (B) 4.(A) 5. (C) 6.(A) 7.(A)
8. (A) 9. (A) 10. (B) 11.(B) 12. (0) 13. (B) 14. (D)
15. (C) 16. (B) 17. (A) 18. (D) 19. (B) 20. (B) 21. (D)
22. (D) 23. (D) 24. (B) 25. (D) 26. (C) '7. (A) 28. (C}
29. (A) JO. (C) 31. (B) 32. (A) 33. (A) 34. (B) 35. (C)
36. (D) 37. (D) 38. (A) 39. (C) 40. (D)
.··· I
- -~-~----r.--·~·--- ~•-·......,~---------~ -"""!'-- - - - - - - - - -
• SOLUTIONS

_..,-.---..n••-o--a.•-- ---------:·-:'?:·.·_
. ·· ...

.
G1ven 1_= 30 nunutcs.
. dN 120 K -
cow1ts
3. Collision of e- lead to excitation of molecules
1. - = -
dt sec so Collision is inelastic
After each half life, aclivity is reduced to half :. K' < K and loss of kinetic energy go for
therefore after n half lives activity reduces to excitation
of molecules. Momentum remain conserved
(½J· during collision.
-+ -+
dN P ""'P'
Also - a::N
dt
dN
- at 5 P.M. will be equal to activity
dt
4.

i.e. er (.16th)
remaining after four half Ii ve.<;.

= of the initial activity


outside the nucleus electric potential decreases
e- is negativity charged
:. its PE is negative even outside the nucleus
(di'dt l) a15P.M.
= ( 1- th) of the initial activity
16 . where
nuclear attractive force is negligible
(~)SP.M. = (17;)(~1P.M. (3) ---t c-
outside the nucleus
neutron will not
( dN) = 7500 counts/sec experience electric force
dt 5P.M. as it is neutral. So no potential energy
associated with
2. No - initial nucleon it outside nucleus.
att = 0, No I ---+ neutron
Addition is at a constant rate
--.----.----n ,-,.3
(},.N-C)=-:
---"T---0""2
5. J..2';
"' N' dN
J0 dt - NJ "-N-C
0
_ ___.,_.....__
n=..)
'
Integrating we get hc_hc+hc
C e-1..1 ---.- --
'-, ).;2 i.3
N = ,::-+,:-(i.No -C)
l I I
~= i (l -e-.\ 1) + Noe-4 j
-=-+-
'-1 A.z '- 3

www.puucho.com
CP Publication KVPY chapterwise with www.puucho.com

TOPIC• WISE KVPY [STREAM SX] Questions with Solutions


6. In radioactive equilibrium 4]._
rote of decay ofX c,c rate of decay ofY Aeu::::> -
6
N N
"N =:= "N ~"=-Y
J\,• X /1,'j )''
T" TY.
Meanh"f1e= - 6
4i.
3
'All.3'tA
7. 2A
'tM2 +}t:A
l.5
'to 3
-=- i.
TA 7
approx. 2.0 ;I.
s. K ""3-2.3= 0.7eV
h
S =__!S_ and E = V / d 12. De-Broglie wavelength l = ~
eE v2mKE
When KE is max. the A is minimum
9. J3 decay occur only when rest mass energy of
KE]Jlll](=hf-qi
reactant > rest mass energy of product.
In case of A, K 1 ::::> 7- 6 ::::> leV
as rest mass energy E = me2
In case of B, K2 ::::> 7 - 3 ::::> 4eV
:. M (A, Z) > M (A, Z+ l)+ m.,
10. "-A
AB
= JKEB = ~K 1
KEA K1
::::,.
Vt
II =2: l
4
I
13.
I

:3 ,
I
I

l '~
12.75
'
,, 1.8 1.6 12.l
~
-3.4
KEmax =eVo 10.2
hf= ij)+l<Eu,.:u,: 13.6
hf
Vo=--qi
En =-13.6
z2
e X -2
n
h
Vo"" -v-4> t: =- I3.6x22
e
'-'-4 42
when v=O
Vo""-$
Vo=-Zgiven
1
E,i-E,,=2.6-4 x 13.6 [ -
42
_...!_] =:2.6
02
qi= 2eV n=3
h 13.6x 22
~ =slope of graph Er=----
e 32
~- h 3 ::::>-6.0eV
e 0.8xl015
1.6x 10-19 x3 l 2
h"" 14. KE= -mv
0.8x 1015 2
b =6 X 10-)4
0.05 X 1.6 X 10-t9 = ½ 1.6
X X 10-26 X Vl

,:_ A1N +11.2N 0.05 X 2 X 107 = V 2


11. Aeu-
N+N l06=v2
A1+A2 v = l 000 m!sec
::::>
2

www.puucho.com
CP Publication KVPY chapterwise with www.puucho.com

• MODERN PHYSICS
neutron
time taken to travel a distance of 1 mis _!_
V Bn:akirlg of ,B11
17. sB 11 ,:,•.
../ /

~ - 1 -= 0.001 sec
energy given ~E, {::··.;;:'---._
. • .•• "'-- 5810
1000
Half life of radioactive material= 6.9 sec formation of 5 B 10 will release energy=> E2
T _ 0.693 E 1 = Binding energy of 5Bll
112- -A
=> 7.5 x 11 M V = 82.5 MeV
A.= 0.693 => 0.1 E2 = Binding energy of 5B 10 = 8.0 x 10
6.9 = 80 MeV
fraction ofparttcle decay in 0.001 sec Energy given= E 1 - E2
= 82.5- 80
1
or --sec= l -e-Al
1000 = 2.5 MeV
-O.lx-1- - -1-
=> 1 - e woo => 1- e 10000 18. Power of light= P
=> 0.0001 A
Laser beam

15. Intensity of light at 1.8 m = p 2


411(1.8)
I => 160
>
4x TC X (1.8)"
Photon flux = Number of photon per unit arcn.
I
=>-
be mg
'A.
. on A1 d'1,;c = -2P
Force actmg
D,.
=>- C
he
2x 1.5xl0 3 = w-5
] 60 X 6200 )(} Q-IO
=> - - - - - - - - - - - ~ 3.0xl08
4 X 'RX (1.8)2 X 6.63 X IQ-J 4 X 3 X 10 8 force acting on Al disc~ mg
=> 1.22 X 10 19 10-S = lil X lQ
m= 10- 6 kg

16. __!_=R
A.1
[-I __1]
22 32 19. KErru_.= e X V retarding
= e X 0.6
: 1 =R l¾-i] =0.6 eV
Photon energy = hf= E
_!_=R
A.1
[2-j36
.... (1)
When frequency increase by I 0% energy of
photon also increases by 10%

r¼- l
New energy= E' = 1.1 E
1 New KE.nu.= c x Vnotacdin,:= ex 0.9 = 0.9 cV
A\ =R 25
Einstein photo electric equation
_1 = R [ ~ ] .... (2) hf= KE 01 a.~+ $
).. 2 9x25 E=0.6 + ~ ... (I)
From (1) & (2) l.lE=0.9+~ ... (2)
A2 _ 5 . 16 1.1 = 0.9+¢
~ - 36..,. 9x25 0.6+¢
5 9x25 125 1.1 qi+ 0.66 = 0.9 + qi
Az=- x - - = - A 1
36 16 64 0.1 ¢ = 0.24
¢=2.4Ev

www.puucho.com
CP Publication KVPY chapterwise with www.puucho.com

., TOPIC- WISE KVPY [STREAM SX] Questions with Solutions

20. 1~C-> 1 ;e +p$+v+0.96 MeV ~=I[.!.]


M2 2 2
M= Mo ::::,.M= I µg
20 22 M, =I
M2 4
[Halflife = to ; n = t/1@ =:, n = lto =:, n = 2J
t
M = 0.25 µg (remained) 22. P = 160 watt, 11. = SOOOOA
Carbon us·ed::::,. M0 -M ::::,. 0.75 µg r
. 6 p I
Number of moles= (0.75xl0"
12
)

I=~
Number of reaction 47tr2

0.75xl0--<i p
= X 6.023 X 1023 nhv= - -
12 4:nr2
= 0.37 x I 0 17 reaction p
::::;,. n::::::---
Energy from reaction 4nr1(hv)
= 0.376 x 10 17 x 0.96 MeV
(n = no. of photons per sec per m2)
""0_36 X 10 17
= 3.6 x 10 16 MeV n=---
PX
:::::4 x 10 16 MeV 41rr2hc
Energy from annihilation n = 10 211 m-2 s- 1
= 2 moc2 (0.376 x 10 17)
=::- 1.02 (0.376 x 1017) MeV
23. R=mv
:::::4 x 10 16 MeV qB
Total energy = Ereaelioo. + Eanhillation
V= qBR =eBR
ET~8 x 1016 McV
m me
21.
he_,=~
A.
(Einstein photo electric equation)

,= he -K.Ema.i ==> = he _ _!_m


A A 2 e
(eBR)
me
2

2
=he_ 2m (eBR)
i,, C 2me

Equal are 24. When e- collide with atom which is massive in


1 comparison toe-. Max possible loss of KE= KE of
-ah=ab e- (initial KE)= E
2
h=2b if this Eis less than min excitation energy then
collision is elastic
M1 E. = M 2-
2 [centre of mass of combimilion
:. E < 10.2 eV (Minimum excitation energy)
3 2
at the mid-point of their common edge]
M, =I~ 25. Continuous X Ray is inverse of photoelectric
effect
M2 2 h

www.puucho.com
CP Publication KVPY chapterwise with www.puucho.com

t MODERN PHYSICS
he 31.
26. Energy of photon = E = -
y
"' h
Momentum of photon= P =- Vyt )?' tEoj
E=PC "' ,~VO

,
:. E = C = 3 x 108 m/s ,,
p ,,
-..+.a-----~x
Vo
27. 0.06%
h I
],.=-=>AOC-
28. mv v

v= Vfv2y + v2o

~ he
K.Eofe = -
_
Vy= Uy+ ayt

v = 0 + qEo t
Y m
A (3vo) = ~v~. + v~
h h
= ~ -------
"-dc-broglic
v2mKE ~ v 2y = 8v02

=> qEo t = 2.,J2 v0


mo Adebroglic m
----=!
2 "
2..fim l
m0 ../hi .)35hJ... m0 .[ii · t= - - v0 => tee -
- X - - = --~ => --=-- qEo Eo
2 .J2mc .JBmc 2.fi. 2.fi
=/35 = 5.8"" 6
Illfl h
Le. e- get excite to state 6. 32. IIl\T=n-
21t
:. n=6
v=nu
1 2 h
29. rcc-&vocz mr w=n-
z 2n:

:. according ( v
T
2
) oc z2 x _l_
(1/z)
=23
r= \ 2mnw
g
1
8

E
30. p=-
J
E= [ML 1 3 r 1] 33. P = [M L2T3]
I= [I L- 2] µo =[ML y-2 :r2]
h=ML2 T 1
m= [I L2]
Ille= M
c= LT 1 ro = [11]
£o=M- 1 L- 3 T4i2 C = [L T" 1]
h2 :. [M L2 T"3] =[ML r2 :r2r [I L2]Y [Tit [L11t
:. p=--2 X = l, y = 2, Z = 4, ll = -3
mecc

www.puucho.com
CP Publication KVPY chapterwise with www.puucho.com

TOPIC• WISE KVPY ISTREAM SX] Questions with Solutions

34. mem
µ=
me+m
where m = mass of nucleus
1n case of hydrogen atom

h _ (m.,)(2000m 0 )
mvr=n- ... (1) µ,-
2'lt me + (2000)me
mv 2 ke 2 ke2 Tn case of deuterium
&-=----
r r2 (2r)2 4000
--m
mv 2 3 ke 2 4001 e
r 4 r2 Hence,
1
=:,.mv2r= Ike2 ... (2) - =µz
-2=R2 -
4
Solving (I) t (2)
"'1RI µ,

4n&0 h 2 4 ""(4000me) (2001)


r=-..,;;,_- X - 4001 (2000mc)
me 2 3
4 4002
r= -a.1:1
3 4001
l2 -'-1
35. 1N14 + 2.1"'Ie4 -+ 110•1 + 1H I
=> -=--·=--
"'1
1
4001
+
J3c9 2H4-+ Jic 12 + on 1
A).. 1
- X 100= --X 100=0.025%
36 11. 1 4001
0 0 +
tt-+
. _o_
0_0
0 0 +++ 39.
0 0 ++-+
.p 0
- -n
-1ve11ncovm:d ions
-ve Ullcovered ions d..,.ep-lc-lio_n.,.region

37.

both i and V are of same sign

- 1 - - - - - - + - - - -0.54 eV 40. Statement I, II & III are correct


n =4 - 1 - - - - - - + - - ~ -
0.85 eV
n= 3 1- 1.51 eV
n= 2 -3.4 eV
I
n = l-]..---'-=-
91-2A-=---'------,----' -13.6 eV

Since hydrogen is in it's neutral state. Therefore


A.= 3646 A will not strongly absorbed
4
38, R= mce
8e~ch 3
where Ille = mass of electron
When we consider mass of nucleus also then
we replace m with reduced mass

www.puucho.com
CP Publication KVPY chapterwise with www.puucho.com

Chemistry
[ToP1c-w1sE KVPY [STREAM-SX] Questions with Solutions]

www.puucho.com
CP Publication KVPY chapterwise with www.puucho.com

Physical Chemistry
- - - ~ ·==--==--- - I
1. The energy of a photon of wavelength k = l s. Increasing the temperature increases the rate of
meter is (Planck's constant = 6.625 x 10-34 Js, reaction but docs not increase the - [20101
(A) number of collisions
speed oflight = 3 x 10 8 m/s) [2010]
{B) activation energy
(A) 1.988 X 10-23 J (B) 1.988 X 10-2 g J
(CJ average energy of collisions
(C) 1.988 X 10-3o J (D) 1.988 X 10-25 J (D) average velocity of the reactant molecules

2. The concentration of a substance undergoing a 6. In metallic solids, the number of atoms for the
chemical re.action becomes one-half of its face-centered and the body-centered cubic unit
cells, are, respectively~ 120101
original value after time t regardless of the initial
(A) 2, 4 (B) 2, 2 (C) 4, 2 (D) 4, 4
concentration. The reaction is an example of a --
12010} 7. From equations I and 2,
(A) zero order reaction CO2 ~ CO + l/2 02
(B) first order reaction [K.:1 = 9.1 X 10-n at 1000°C] (eq. I)
(C) second order reaction
H20~ H2+ 1/2 02
[K,_,1 =' 7.1 X 10-ll at 1000°CJ (eq. 2)
(D) third order reaction
the equilibrium constant for the reaction
CO2 + H2 ~ CO + H2O at the same
3. In the nuclear reaction
temperature, is- [2010)
'34 23•1 (A) 0. 78 (B) 2.0 (C) 16.2 (D) 1.28
90 Th -)~ 1 Pa+ X. X IS- [2010]

(A) -~ e (B) ~e 8. .For a first order reaction R --)o P, the rate

constant is k. If the initial concentration of R is


(C) H (D) H f [Ro], the concentration of R at any time 't' is
given by the expression- [2010]
4. A concentrated solution of copper sulphate, (A) [RtiJ ekl (B) [Ro]( l - e-ki]
which is dark blue in colow-, a mixed at room (C) [R.i] e-kl (D)[Rtl](l -i1)
temperature with a dilute solution of copper
sulphate, which is light blue. For this process - 9. Consider the reaction:
2 N02(g) --t 2 NO(g) + 02 (g). In the figure
(2010]
below, identify the curves X, Y and Z associated
(A) Entropy change is positive, but enthalpy with the three species in the reaction- [2010}
change is negative
(B) Entropy and enthalpy changes are both
positive
(C) Entropy change is positive and enthalpy
docs not change
(D) Entropy change is negative and enthalpy
change is positive
Time -

www.puucho.com
CP Publication KVPY chapterwise with www.puucho.com

TOPIC• WISE KVPY [STREAM SX] Questions with Solutions


(A) X = NO, Y = 02. Z = NO2 16. At 298 K, the ratio of osmotic pressures of two
(B) X = 02, Y =NO, Z=N02 solutions of a substance with concentrations of
(C)X=N03, Y=NO,Z=02 0.01 Mand 0.001 M, respectively, is [2011]
(D) X = 01, Y = N02, Z = NO (A) I (B) 100 (C) 10 (D) 1000

10. 2.52 g ·of oxalic acid dehydrate was dissolved 17. The rate of gas phase chemical reactions generally
in 100 mI of water, IO mL of this solution was increases rapidly with rise in temperature. This
diluted to 500 mL. The normality of the final is maillly because [2011]
solution and the amount of oxalic acid (mg/mL) (A) the collision frequency increases with
jn tbe solution are respectively- (2010] temperature
(A) 0.16 N, 5.04 (B) 0.08 N, 3.60 (B) the fraction of molecules having energy in
(C) 0.04 N, 3.60 (D) 0.02 N, 10.08 excess of the activation energy increases with
temperature
11, A solution containing 8.0 g of nicotine in 92 g (C) the activation energy decreases · with
of water freezes 0.925 degrees below the nonnal temperature
freezing point of water. If the molal freezing (D) the average kinetic energy of molecules
point dcprc.~sion constant Kr = - l .85"C mol- 1 increases with temperature
then the molar mass of nicotine is - (2010]
(A) 16 (B) 80 (C) 320 (D) 160 18. For a process to occur spontaneously (2011]
(A) only the entropy of the system must increase
12. For the reaction A-> B, L\H = 7.5 mor 1 and
11 (B) only the entropy of the suroundings must
AS" = 2.5 J mor 1• The value of ~G and the 0 increase
temperature at which the reaction reaches (C) either the entropy of the system or that of
equilibrium are, respectively, (20101 the surroundings must increase
(A) 0 kJ mor 1 and 400 K (D) the total entropy of the system and the
(D) -2.5 kJ mor 1 and 400 K surroundings must increase
(C) 2.5 k.J mor1 and200 K
19. When the size of a spherical nanoparticle
(D) 0 kJ mor 1 and 300 K
decreases from 30 nm to l 0nm, the ratio surface
area/volume becomes f2011)
13. The solubility product of Mg(OH)2 is 1.0 x I 0-12•
(A) 1/3 of the original
Concentrated aqueous NaOH solution is added to
(B) 3 times the original
a 0.ot M aqueous solution of MgCb. The pH at
(C) 1/9 of the original
which precipitation occur is- [2010]
(D) 9 times the original
(A) 7.2 (B) 7.8 (C) 8.0 (D) 9.0

20. . _ The value of the limiting molar conductivity (A°)


14. A metal wi1h an atomic radius of 141.4 pm
crystallizes in the face centred cubic structure. for NaCl, HCI and NaOAc are 126.4, 425.9 and
The volume of the urut cell in pm is- [2010] 91.0 S cm2 mor 1, respectively. For HOAc, A" in
(A) 2.74 x 107 (B) 2.19 )( 107 S cm2 mor 1 is I2011]
(C) 6.40 x 107 (D) 9.20 x 10 7 (A) 390.5 (B) 299.5 (C) 208.5 (D) 217.4

21. To obtain a diffraction peak, for a crystalline


15. Assuming ideal behaviour, the enthalpy and
solid with interplane distance equal to the
volume of mixing of two liquids, respectively,
wavelength of incident X-ray radiation, the
are [20111
angle ofincidcnce should be [20111
(A) zero and zero (B) + ve and zero
(A) 90° (D) 0° (C) 30° (D) 60°
(C) -ve and zero (D) -ve and - ve

www.puucho.com
CP Publication KVPY chapterwise with www.puucho.com

• PHYSICAL CHEMISTRY

22. The standard Gibbs free energy change 28. At 300 K the vapour pressure of two pure
(6G0 in kJ mor 1), in a Daniel cell ( E~11 ""1.1 V ), liquids, A and B are 100 and 500 mm Hg,
when 2 moles of Zn(s) is oxidized at 298 K, is respectively. If in a mixture of A and B, the
closest to 120111 vapour pressure is 300 mm Hg, the mole
(A)- 212.3 (B)-106.2 fractions of A in the liquid and in the vapour
(C)_-424.6 (D)-53.l phase, respectively, are - 12011]
(A) 1/2 and 1/, l (8) 1/4 and 1/6
23. In a zero-order reaction, if the initial concentration (C) 1/4 and 1/10 (D) 1/2 and 1/6
of the reactant is doubled, the time required for
half the reactant to be consumed 120111 29. Emulsification of IO ml of oil in water
(A) increases two-fold produces 2.4 x 10 18 droplets. If the surface
(B) increases four-fold tension at the oil-water interface is 0.03 Jm- 2
(C) decreases by half and the area of each droplet is 12.5 x 10- 16 m2,
(D) does not change the energy spent in the formation of oil droplets
is- 12011]
24. The adsorption isotherm for a gas is given by (A) 90 J (B) 30 J
the relation x = ap/(l + bp) where xis moles of (C) 900 J (D) 10 J
gas adsorbed per gram of the adsorbent, p is the
pressure of the gas, and a and b are constants.
30. For a one-electron atom, the set of allowed
Then x r20111
quantum numbers is - [2012]
(A) increases with p
(A) n = I, I = 0, m1 = 0, 111s = +½
(8) remains unchanged with p
(B) n = 1, I= 1, mi= 0, ms =+1/i
(C) decreases with p
(C) n=l,l=0,m1=-l,m,=-½
(D) increases with p at low pressures and then
remains the same at high pressure (D) n = 1, I = 1, m 1 = I, m,, =-½

25. Mn04- oxidizes (i) oxalate ion in acidic medium 31. Typical electronic energy gaps in molecules are
at 333 Kand and (ii) HCI. For balanced chemical about l.0 eV. In tenm of temperature, the gap
equations, the ratios [MnO-1- : C20/l in (i) and is closed to - [2012]
[Mn04-: HCI] in (ii), respectively, are- (2011] (A) 102 K (B) 104 K
(A) 1 : 5 and 2 : 5 (B) 2 : 5 and 1 : 8 (C) 103 K (D) !Os K
(C) 2 : 5 and 1 : 5 (D) 5 : 2 and 1 : 8
32. A zero-order reaction, A -? Product, with an
26. lf E" Fe1; /Fe= --0.440 V and E~ Fel> !Fe'· = 0.770 V, initial concentration [A]o has a half-life of
0.2 s. ff one starts with the concentration 2[AJ 0,
then E" Fe" 1Fe is- 12011]
then the half-life is - [20121
(A) 0.330 V (B)-0.037 V
(A) 0.1 s (B) 0.4 s
(C)-0.330 V (D)-1.210 V
(C) 0.2 s (D) 0.8 s

27. The electron in hydrogen atom is in the first


33. The isoelectronic pair of ions is -
Bohr orbit (n = 1). The ratio of transition energies,
{2012]
E(n = 1 --? n = 3) to E(n = I --? n = 2), is -
(A) Sc2+ and Vl+ (B) Mn3+ and Fe2+
(2011)
(C) Mn2+ and Fe3+ (D) Ni3+ and Fe 2+
(A) 32/27 (B) 16/27
(C) 32/9 (D) 8/9

www.puucho.com
CP Publication KVPY chapterwise with www.puucho.com

TOPIC-WISE KVPY [STREAM SX] Questions with Solutions

34. The oxidation state of cobalt in the foUowing


37. The C--0 bond length in CO, CO2 and co/-
molecule is- [20121
follows the order - [20121
0
(A) co < CO2 < co/·
II (B) CO2 < CO3 2- < CO
oc / c"-- /co

ocj
'c~-~--"'--c{--co
"-----.. . __c/ ~
{C) co> CO2> co/-
CD) CO3 2- < CO2 < CO

oc co 38. The equilibrium constant for the following

II
0
reactions are K 1 and K 2, respectively.
2P(g) + 3Clz(g) ..... 2PCh(g)
(2012]

(A) 3 (B) I (C) 2 (D) 0 PCl3(g) + Cli(g) - PCl5(g)


Then the equilibrium constant for the reaction
35. The pl<a of a weak acid is 5.85. The concentrations 2P(g) + 5Clz(g) .... 2PCls(g) is -
of the acid and its conjugate base are equal at a (A) K1K2 (B) K1K/ (C) K1 2K/ (D) K/K2
pH of- [20121
(A) 6.85 (B) 5.85 (C) 4.85 (D) 7.85 39. Doping silicon with boron produces a - [2012)
(A) n-type semiconductor
36. Among the following graphs showing variation (B) Metallic conductor
of rate (k) ,vith temperature (T) for a reaction, (C) p-type semiconductor
the one that exhibits arrhenius behavior over (D) Insulator
the entire temperature range is - (2012]
40. In the DNA ofE. Coli the mole ratio of adenine
to cytosine is 0.7. If the number of moles of
adenine in the DNA is 350000, the number of
(A) ~ moles of guanine is equal to - {2012]
(A)350000 (D)500000
1/r (C) 225000 (D) 700000

41. A metal is irradiated with light of wavelength

..::&
660 nm Given that the work that the work function
(B) ~ of the metal is 1.0 eV, the de Broglie wavelength
of the ejected electron is close to- [2012}
(A) 6.6 x 10-7 m (B) 8.9 x 10-11 m
1/r (C) 1.3 x 10--9 m (D) 6.6 x 10-u m

42. The inter-planar spacing between the(2 2 I) planes


ofa cubic lattice oflength 450 pm is - (2012]
(A) 50 pm (B) 150 pm
(C) 300 pm (D) 450 pm
1/r
43. The till for vaporinition of a liquid is 20 kJ/mol.
Assuming ideal behaviour, the change in internal
..:.:: energy for the vaporization of 1 mol of the liquid
(D) .E!
at 60°C and 1 bar is close to - [2012]
(A) 13.2 kJ/rnol (B) 17.2 kJ/mol
1/r (C) 19.5 kJ/mol (D) 20.0 kJ/mol

www.puucho.com
CP Publication KVPY chapterwise with www.puucho.com
,..

t PHYSICAL CHEMISTRY
44. Three moles of an ideal gas expands reversibly 49. The entropy change in the isothermal reversible
under isothermal condition form 2 L to 20 L at expansion of 2 moles of an ideal gas from 10 to
300 K. The amount of heat-change (in kJ/mol) 100 Lat 300 K is - l2013J
in the process is - (20121 (A) 42.3 JK- 1 (B) 35.8 JK- 1

(A) 0 (B) 7.2 (C) 10.2 (D) 17.2 (C) 38.3 JK-1 (D) 32.3 JK-1

45 The following data arc obtained for a reaction, 50. For tl1e isotbenr· 1 reversible expamion of an ideal
X + Y-+ Products. (20121 gas - [2013]
Expt. [X-0]/mol [Y0]/mol ratc/mol L-•s-• (A) L\H > 0 and AU = 0
1 0.25 0.25 1.0 X 10-6 (B) AH> 0 and AU< 0
2 0.50 0.25 4.0 X l 0-6 (C) All= 0 and AU= 0
J 0.25 0.50 8,0 X 10-6 (D) .6.H "" 0 and AU > 0
The overall order of the reaction is
(A) 2 (B) 4 (C) 3 (D} 5 51. If the angle of incidence of X-ray of wavelength
3A which produces a second order diffracted
46. For a zero-order reaction with rate constant k, beam from the ( I 00) planes in a simple cubic
the slope of the plot of reactant concentration lattice with interlayer spacing a = 6A is 30°, the
againsttime is - (2013] angle of incidence that produces a first-order
(A) k/2.303 (B) k diffracted beam from the (200) planes is -
(C) -k/2.303 (D) - k r201JJ
(A) 15° (B) 45° (C) 30° (D) 60°
· 47. The variation of solubility of four different
gases (GI, G2, etc.) in a given solvent with 52. The number of ions produced in water by
pressure at a constant temperature is shown in dissolution of the complex having the empirical
the plot. - [2013] formula, CoCb.4NH3, is - [2013]
(A) 1 (B) 2 (C) 4 (D) 3
G4
53. The spin-only magnetic moments of[Fe(NH3)1i+
and [FeF6]3- in BM are, respectively, [2013]
(A) 1.73 and 1.73 (B) 5.92 and 1.73
(C) 1. 73 and 5.92 (D) 5.92 and 5.92

Pressure 54. An ionic compound is formed between a metal


M and a non-metal Y. If M occupies half the
The gas with the highest value of Henry's law octahedral voilJ.c:i in the cubic close•packed
constant is - 12013] arrangement formed by Y, the chemical fonnula
(A)G4 (B) G2 of the ionic compound is - f2013]
(C)G3 (D)Gl (A) MY (B). MY2 (C) M2Y (D) MY3

48. For the reaction, A ~ nB the concentration 55. In 108 g of water, 18 g of a non-voltile
of A decreases from 0.06 to 0.03 mo! L- 1 and that compound is dissolved. At IOO"C the vapour
of B rises from O to 0.06 mol L- 1 at equilibrium. pressure of the': solution is 750 mm Hg.
The values of n and the equilibrium constant for Assuming that the compound does not undergo
the reaction, respectively, are- 120131 association or dissociation, the molar mass of
(A) 2 and 0.12 (B) 2 and 1.2 the compounding mor' is - [2013]
(C) 3 and 0.12 (D) 3 and 1.2 (A) 128 (B) !s2 (C) 152 (D) 228

www.puucho.com
CP Publication KVPY chapterwise with www.puucho.com

TOPIC- WISE KVPY [STREAM SX] Questions with Solutions


56. The standard electrode potential of Zn2+/Zn is 62. The Arrhenius plots of two reactions, I and II are
0.76 V and that ofCu1+/Cu is 0.34 V. The emf(V) shown graphically - [2014]
and the free energy change (k:J rnor\ respectively,
for a Daniel cell will be - . [2013)
(A) - 0.42 and 81 (B) 1.1 and- 213
Ink
(C)- J .1 and 213 (D) 0.42 and- 81

57. Consider the equilibria (1) and (2) the with


equilibrium constants K 1and K 2, respectively. UT
[2013] The graph suggests that -
l . (A) E1 > En and A1 > Arr
SO2(g) + -Oi(g) ~ SOJ(g) ... (1)
2 (B) En> E1 and Au> A1
..2SO3 (g) ~ 2SO 2(g) + O2(g) . _. (2) (C) E1 > Err and Arr > A1
K 1 and K 2 are related as - (D) Eu> E1and Ai> Arr

(B) Kf = ~2 63. In a cubic close packed structure, fractional


contributions of an atom at the corner and at the
2
(C) K2 = _1 (D) K2 = K 2 face in the unit cell are, respectively - [2014)
- 2 KI
. I (A) 1/8 and 1/2 (B) 1/2 and 1/4
(C) 1/4 and 1/2 (D) 1/4 and 1/8
58. The density and eq. wt. of a metal are 10.5 g
cm-3 and 100, respectively. The time required 64. The equilibrium constant K, of the reaction,
for a current of 3 amp to deposit a 0.005 mm 2A ~ B+C is 0.5 at 25°C and I atm.
thick layer of the same metal on an area of The reaction will proceed in the backward
80 cm2 is closest to- [2013] direction when concentrations [A], [BJ and [CJ
(A) 120 s (B) 135 s arc, respectively - [2014]
(C) 67.5 s (D) 270 s (A) 10-3, 10-2 and 10-2 M
(B) 10- 1, 10-2 and 10-2 M
59. The amount of Na 2S 20 3 .5H2 0 required to (C) 10-2, 10-2 and 10-3 M
completely reduce 100 rnL of 0.25 N iodine (D) 10-2, 10-3 and 10-3 M
solution, is - [2013]
(A) 6.20 g (B) 9.30 g 65. If the molar conductivities (in S cm2 mol- 1) of
(C) 3.10 g (D) 7.75 g NaCl, KCI and NaOH at infinite dilution are 126,
150 and 250 respectively, the molar conductivity
60. When 1.88 g of AgBr(s) is added to a 10-3 M of KOH (in S cm2 mor 1) is - [2014]
aqueous solution of K.Br, the concentration of Ag (A) 526 (B) 226 (C) 26 (D) 274
is Sx 10-10 M. [f the same amount of AgBr( s) is
added to a IO 2 M aqueous solution of AgN03, 66. Two elements, X and Y, have atomic numbers
the concentration of Br- is [2014] 33 and 17, respectively. The molecular formula
(A) 9.4 x 10- M
9 (B) 5 x 10- M
10
of a stable compound formed between them is -
(C) I x 10-]l M (D) 5 x 10-Jl M [2014]
(A) XY (B) XY2 (C) XY3 (D) XY4
61. The metal with the highest oxidation state present
in K2Cr04, NbCl5 and MnO2 is - [2014) 67. The number of moles of fu\1nO4 required to
(A)Nb (B) Mn oxidize one equivalent of KI in the presence of
(C) K (D) Cr sulfuric acid is - [2014]
(A) 5 (B) 2 (C) 1/2 (D) 1/5

www.puucho.com
CP Publication KVPY chapterwise with www.puucho.com

• PHYSICAL CHEMISTRY

68. ·. Three successive measurements in an experiment 75. The reaction, K2Cr2O1 + m FeSO4 + n H2S04
gave the values l0.9, 11.4042 and 11.42. TI1e correct -> Cr2(SO4)3 + p Fc2(SO,.)3 + K2SO4 + q H2O
way of reporting the average value is - I2014 f when balanced, m, n, p and q are, respectively :
(A) I 1.2080 {B) 11.21 [2014}
(C) l l.2 (D) 11 (A) 6, 14, 3, 14 (B) 6, 7, 3, 7
(C) 3, 7, 2, 7 (D) 4, 14, 2, 14
69. The latent heat of melting of ice at 0 °C is
6 kJ mol-1• The entropy change during the 76. The standard free energy change (in J) for the
melting in J K- 1 mo1- 1 is closest to - (2014( reaction 3Fe2+ (aq) + 2Cr(s) = 2Cr1"'{aq) + 3Fc(s)
(A) 22 (B) 11 (C)-11 (D)-22 given E~0 ,. IFe =--0.44 V and E~,"" /Cr = ---0. 74 V
is (F = 96500 C) (2014)
70. The density of acetic acid vapour at 300 K.. and (A) 57,900 (B) -57,900
I atm is 5 mg cm_;3 • The num bcr of acetic acid (C) 173,700 (D) 173. 700
molecules in the cluster that is formed in the
gas phase is closest to 12014) 77. The mte constant of a chemical reaction at a
(A) 5 (B) 2 (<::) 3 (D) 4 very high temperature will approach [20151
(A) Arrhenius .frequency factor divided by the
71. The molar enthalpy change fur HA'./) ~ I{iO(g) ideal gas constant
at 373 K and 1 atm is 41 kJ/mof. Assuming ideal (B) activation energy
behaviour, the internal energy change for (C) Arrhenius frequency factor
vaporization of I mol of water at 373 K and (D) activation energy clivided by the ideal gas
l atm in kJ mor 1 is : {2014) constant
(A) 30.2 (B) 41.0 (C) 48.1 (0)37 .9
78. The standard reduction potentials (in V) of a
72. The equilibrium constant (K,,) of two reactions few metal ion/metal electrodes are given below.
H2 + Ii ~ 2lll and N2 + 3H2 ~ 2NH3 are cr3+/Cr=--0.74; eu2+/Cu =·1034; Pb2+/Pb ""--0.13;
50 and 1()00, respectively. Ihc equilibrium constant Ag+/Ag = +0.8. The reducing strength of the
metals follows the order (2015J
of the reaction N2 + 681 ~ 2NH3 + 312 is closest
(A) Ag> Cu> Pb> Cr
to: [2014]
(B) Cr> Pb> Cu> Ag
(A) 50000 (B)20
(C) Pb> Cr> Ag> Cu
(C) 0.008 (D) 0.005
(D) Cr> Ag> Cu> Pb

73. Given that the bond energies of: NEN is 946 kJ


79. The molar conductivities of HO, NaCl, CH3COOH,
mor 1, H-H is435 kJ mor 1, N-N is 159 kJ mor 1, and CH3COONa at infinite dilution follow the order
and N-H is 389 kJ mor 1, the heat of formation of [2015)
hydrazine in the gas phase in kJ mor I is : (2014I (A) HCI > CH3 COOH >NaCl> CH3COONa
(A) 833 (B) 101 (B) CH3COONa > HCI > NaCl > CII3COOH
(C)334 (D) 1268 (C) HCI > NaCl> CH3COOH > CH3COONa
(D) CH3COOH > CH 3COONa > HCl > NaCl
74. The radius ofK+ is 133 pm and that of Cl is 181 pm
The volume oftbe unit cell ofKCl expressed in 80. The spin only magnetic moment of [ZCl 4) 2- is
10· 22 cm3 is : [2014] 3.87 BM where Z is [2015)
(A) 0.31 (B) 1.21 (A) Mn (B)Ni
(C) 2.48 (D) 6.28 (C) Co {D) Cu

www.puucho.com
CP Publication KVPY chapterwise with www.puucho.com

-~~:;;-:;;:;-------------:---:--7:::---=:-:-:T:-O:-:P~IC:....·..:.:W~IS=E:..:.KV::..:.P...:Y~[:.:S:..:.T.:.:R:E:.:AM:::..:S:X::l~Q=u:e:s~ti~o~ns~w~it~h_.::S:o~lu~t~lo~n::s.....
81. The pH of l N aqueous solutions of HC~ CH3COOH 87. The volume vs. temperature graph of 1 mole of
and HCOOH follows the order [20151 an ideal gas is given below
(A) HCI > HCOOH > CH;COOH X y
(B) HCl = HCOOH > CH3COOH --------.----+--------
(C) CH1COOH > HCOOH > HCI
(D) CH3COOH = HCOOH > HCl

82. Schottky defect in a crystal arises due to (2015]


(A) creation of equal number of cation and anion
vacancic~
(B) creation of unequal number of cation and
anion vacancies
(C) migration of cations to interstitial voids
(D) migration of anion'> to interstitial voids
100 200 300 400 500
83. For the reaction N2 + 3X2 -t 2NX3 where Temperature (K)
X = F, Cl (the average bond energies arc
The pressure of the gas (in atm) at X, Y and z,
F-F = 155 kJ mor 1 N-F = 272 kJ mor 1
respectively, are (2015]
Cl-Cl= 242 kJ mor 1, N-CI = 200 kJ mo1· 1 and
(A) 0.328, 0.820, 0.820
N N = 941 kJ mor1), the heats of formation
(B) 3.28, 8.20, 3.28
of NFJ and NCh in kJ mo1· 1, respectively, are (C) 0.238, 0.280, 0.280
closest to f2015J (D) 32.8, 0.280, 82.0
(A) - 226 and +467
(B) + 226 and - 467
88. The pKa of oxoacids of chlorine in water follows
(C)-151 and+3l1 the order (2016]
(D)+ 151 and-311
(A) HCIO < HClQ3 < HC!O2 < HC1O 4
(B) HCIO4 < HC103 < HCIO 2 < HCIO
84. The equilibrium constants for the reactions X = 2Y
(C) HCJO4 < HCIO2 < HC1O 3 < HCIO
and Z = P + Q are K1 and K2, respectively. lf the
(D) HCIO2 < HCIO < HClO3 < HC1O4
initial cor,entrations Md the degree of dissociation
ofX and Zare the same, the ratio K 1/K2 is [2015]
89. The packing efficiency of the face centered cubic
(A) 4 (B) l
(fee), body centered cubic (bee) and simple/ primitive
(C) 0.5 (D) 2
cubic (pc) lattices follows the order (2016]
(A) fee> bee> pc (B) bee> fee> pc
85. The geometry and the number of unpaired
electron(s) of[MnBr4] 2-, respectively, are (C) pc> bee> fee (D) bee> pc> fee
12015]
(A) tetrahedral and 1
90. The ratio of root mean square velocity of
(B) square planar and 1 hydrogen at 50 K to that of nitrogen al 500 K is
(C) tetrahedral and 5 closest to (2016]
(D) square planar and 5 (A) 1.18 (B) 0.85
(C) 0.59 (D) 1.40
86. The standard cell potential for Zn1Zn2+ l[Cu2+jCu is
1.10 V. When the cell is completely discharged, 91. 1t takes lh for a first order reaction to go to 50%
log [Zn2l/[Cu 2J is closest to [2015]
completion. The total time required for the same
(A) 37.3 (B) 0.026 reaction to reach 87 .5% completion will be [2016]
(C) 18.7 (D) 0.052 (A) 1.75 h (B) 6.00 h
(C) 3.50 h {D) 3.00 h

www.puucho.com
CP Publication KVPY chapterwise with www.puucho.com

• PHYSICAL CHEMISTRY

92. A unit cell of calcium fluoride has four calcium 96. Copper (atomic mass = 63. 5) crystallizers in a
ions. The number of fluoride ions in the unit FCC lattice and has density 8.93 g.cm-3• [2016]
cell is [20161 The radius of copper atom is closest to
(A) 2 (B) 4 (C) 6 (D) 8 (A) 361.6 pm (B) 511.4 pm
(C) 127.8 pm (D) 102..8 pm
93. The equilibrium constant of a 2 electron redox
reaction at 298 K is 3.8 x 10-3_ The cell 97. Given the standard potentials E 0 (Cu2 t-/Cu) and
Eu(Cu''/Cu) as 0.J40 V and 0.522 V respectively,
potential E° (in V) and the free energy change
the value ofE"'(Cu2+/Cu+-) is [20161
AG (in kJ mor 1) for this equilibrium
0
(A) 0.364 V (B) 0.158 V
respectively, are [2016] (C)-0.l82V (D)-0.316V
(A)- 0.071, -13.8 (B)- 0.071, 13.8
(C) 0.71,-13.8 (D) 0.071, -13.8 98. For electroplating, 1.5 amp current is passed for
250 s through 250 mL of 0 .15 M solution of
94. In the radioactive disintegration senes MSO 4 • Only 85% of the current was utilized for
electrolysis. The molarity of MSO4 solution
~62 Th-----+ f~ 8Pb , involving a and j3 decay, after electrolysis is closest to
the total number of a and J3 particles emitted [Assume that the volwne of the solution remained
are [2016] constant] [2016]
(A) 6 a and 6 P (B) 6 a and 4 p (A) 0. 14 (B) 0.014
(C) 0.07 (D) 0.035
(C) 6 a and5 p (D) 5 a and 6 p
99. 2.33 g of comp(Jund X (empirical fommla
95. In the following compressibility factor (Z) vs
CoH 12 N4Ch) upon treatment with c.::cess AgNO 3
pressure graph at 300 K, the compressibility of
solution produces 1.435 g of a white precipitate.
CH.; at pressure < 200 bar deviates from ideal
The primary and secondary valences of cobalt in
behaviour because (2016]
compound X, respectively, are [2016)
[Given : Atomic mass: Co= 59, Cl= 35.5,
Ag= 108]
(A) 3, 6 (B) 3, 4 (C) 2, 4 (D) 4, 3
1.8
100. The specific conductance (t-:) of0.02 M aqueous
acetic acid solution at 298 K is 1.65 x l o-4 S cm-1•
The degree of dissociation of acetic acid is
1.0 --------------
Ideal gas [2016)
[Given : equivalent conductance at infinite
dilution of H+ = 349.1 S cm2 mol- 1 and
CH 3COO- = 40.9 S cm 2 mor 1]
(A) 0.021 (B) 0.21
200 600 1000
(C) 0.012 lD) 0.12
Pressure (bar)
, 101. Among the following metals, the strongest
(A) The molar volume of CI-Li is less than its reducing agent is [S Nov, 2017]
molar volume in the ideal state
(A) Ni (B) Cu (C) Zn (D) Fe
(B) The molar volume of CH 4 is same as that in
its ideal state 102. Chlorine has two naturally occurring isotopes,
(C) lntennolecular interactions between CI~ 35Cl and 37CLJfthe atomic mass of Cl is 35.45,
molecules dccresascs the ratio of ruikal abundance or15CI and 37Cl
is closest to (5 Nov, 20171
(D) The molar volume of CI-Li is more than its
(A) 3.5: I (B) 3 :I
molar volume in the ideal state
(C) 2.5 : I (D) 4: I

www.puucho.com
CP Publication KVPY chapterwise with www.puucho.com

TOPIC•WISE KVPY [STREAM SX] Questions with Solutions


103. The reaction C2H6 (g)~ C2I4 (g) + H2 (g) is at (A) MAg: 2 Mcu: 3 MAI
equilibrium in a closed vessel at 1000 K. The (B) MAs : Mcu : MA1
enthalpy chanfe (.ru I) for the reaction is (C) 6 MAg: 3 Meu : 2 :MA1
137.0 kJ mor. Which one of the following (D) 3 MAg : 2 Mcu : MA1
actions would shift the equilibrium to the right?
[5 Nov, 2017] 107. A reaction has an activation energy of 209 kJ
(A) Decreasing the volume of the closed mor 1• The rate increases 10-fold when the
reaction vessel temperature is increased from 27°C to X °C.
(B) Decreasing the temperature at which the The temperature Xis closest to
reaction is performed [Gas constant, R = 8.314 J mol- 1 K- 1]
(C) Adding an inert gas to the closed reaction [5 Nov, 2017]
vessel (A) 35 (B) 40 (C) 30 (D) 45
(D) Increasing the volume of the closed
reaction vessel 108. A mineral consists of a cubic dose-packed
stmcture formed by 0 2- ions where half the
104. The enthalpy (H) of an elementary exothermic octahedral voids are occupied by AJ3+ and one-
reaction A ~ B is schematically plotted against eighth of the tetrahedral voids are occupied by
the reaction coordinate. The plots in the Mn2+. The chemical formula ofthc mineral is
presence and absence of a catalyst arc shown in [5 Nov, 2017j
dashed and solid lines, respectively. Identify {A) Mn3AhO6 (B)MnAliO4
the correct plot for the reaction. (5 Nov, 2017] (C) Ivlru¼01 (D) Mn2AhO3

109. For a 4p orbital, the number of radial and


H angular nodes, respectively, are [S Nov, 2017]
'l
(A) 3,2 (B) 1,2
(A)
(C)2,4 (D)2,l

110. 217Ac bas a half-life of 22 years with respect to


Reaction coordinate Reaction coordinate
radioactive decay . The dec.1y follows hvo parallel
.... - ... ...., patl1~ : mAc --+ 221Th an<l 221Ac --+ 223-,-,
rr. If the
,' '
HA' H A pcrccnta.gc of tbe two daughter nuclides are
2.0 and 98.0, respectively, the decay constant
(C) '' (D)
(in year-I) for 227Ac --+ 227 Tb path is closest to
\ft-
[5 Nov, 20171
Reaction coordinate Reaction coordinate (A) 6.3 X 10· 2 (B) 6.3 X 10·3
(C) 6.3 X 10·] (D) 6.3 x 10--4
105. Mg(OH) 2 is precipitated when NaOH is added
111. A system consisting of 1 mol of an ideal gas
to a solution of Mg1+. If the final concentration
undergoes a reversible process, A-+ B--+ C -> A
of Mg2 + is 10- 10 M, the concentration of (schematically indicated in the figure below). If
O~(M) in the solution is the temperature at the starting point A is
[Solubility product for Mg (OH)2 = 5.6 x w- 12] 300 K and the work done in the process B --+ C
(5 Nov, 20171 is 1 L attn, the heat exchanged in the entire
process in L atrn-is {5 Nov, 2017J
(A) 0.056 (B) 0.12
V

I:: _ _ :::~C
(C) 0.24 (D) 0.025
B

106. A constant current (0.5 amp) is passed for


hour through (i) aqueous AgN0 3, (ii) aqueous
CuSO4 and (iii) molten AIF3, separately. The A,I '
I

ratio of the mass of the metals deposited on the '


cathode is ~ - - ~ - -........-•P
l atm 2 atm
[MAg, Mu., MAI are molar masses of the (A) 1.0 (B) 0.0
respective metals] (5 Nov, 2017] (C) 1.5 (D) 0.5

www.puucho.com
CP Publication KVPY chapterwise with www.puucho.com

• PHYSICAL CHEMISTRY

112. A mixture of toluene and benzene boils at 117. The consecutive reaction X --J, Y -> Z takes
l00°C. Assuming ideal behaviour, the mole place in a closed container. Initially, the
fraction of toluene in the mixture is closest to container has Ao moles of X (and no Y and Z).
[Vapour pressures of pure toluene and pure The plot of total moles of the constituents in the
benzene at l00°C are 0. 742 and 1.800 bar container as a function of time will be
respectively. l atm = 1.013 bar] [5 Nov, 20171 119 Nov, 20171
(A) 0.824 (13) 0.744
(C) 0.544 (D) 0.624

113. A two-dimensional solid pattern formed by two


different atoms X and Y is shown below. The (A)
black and white squares represent atoms X and
Y, respectively. The simplest formula for the
compound based on the unit cell from the
pattern is (5 Nov, 2017]

time

'- Ao
0
"' ,.,,
.:::!E
0 <IJ

(A) XYs (B) X-1Y9


(C) 8 _g
(C) XY2 (D)XY4
"3~
~ g _______..

time
114, The oxidising ability of the given anions
follows the order [19 Nov, 2017]
(A) Tio:- < VO~- < Cro!- < MnO;
(B) vo~- < cro!- < Mn0 4 < Tioj-
(C) Cro!- < Mn0 4 < VO!- < TiO_:-
(D) VO!- < Tio:- < Cro~- < Mn0 4

115. An excess of A&Cr04 (s) is added to a 5 x 10-3 M 11!t The particle emitted during the sequential
K2Cr0 4 solution. The concentration of Ag+ in
the solution is closest to radioactive decay of 238 U92 to 206 Pb82 are
[Solubility product for Ag 2Cr04 = 1.1 x l 0- 121 f19 Nov, 2017]
(19 Nov, 2017] (A) 5 a and 6 I} (B) 6 a and 8 J3
(A) 2.2 X 10- 10 M (B) 1.5 X 10-5 M (C) 8uand413 (D) 8 a. and 6 f3
(C) 1.0 X 10 6 M (D) 5.0 x 10-3 M
119. The allowed set of quantum numbers for an
electron in a hydroge:1 atom is (19 Nov, 20171
116. The packing efficiency in a body-centred cubic
(A) n = 4, 1 = 2, m1 = 0, ms= 0
(bee) structure is closet to (19 Nov, 20171 (B) n = 3, 1 = 1, m; = -3, m. =-1/2
(A) 74 % (D) 63% (C) n = 3, 1 = 3, lnJ =-1, 111s = 1/2
(C) 68 % (D) 52% (D) n=2, 1 = 1, m;=-1, n1s= l/2

www.puucho.com
CP Publication KVPY chapterwise with www.puucho.com

TOPIC•WISE KVPY [STREAM SXJ Questions with Solutions

120, The plot that best represents the relationship 123. The maximum work (in kJ mor 1) that can be
between the extent of adsorption (x/m) and derived from complete combustion of 1 mo! of
pressure (P) is [19 Nov, 20171 CO at 298 K and I atm is
[Standard enthalpy of combustion of
CO = -283.0. kJ mor 1 ; standard molar
entropies at 298 K ; S02 = 205. I J mor 1,
Seo= 197.7 J mor 1, Sc02 = 213.7 J mor 1]
[19 Nov, 20171
(A) 257 (B) 227
{C) 57 (D) 127

124. 18 g of glucose (CJ{ 12 06) dissolved in 1 kg of


water is heated to boiling. The boiling point
(in K) measured at I atm pressure is closest
x/m
(B) to [Ebulioscopic constant, Kb for water is
0.52 K kg mor 1• Consider absolute zero to be
p -273.15°C] [19 Nov, 2017)
(A) 373.15 (D) 373.lO
(C) 373.20 (D) 373.25

125. Polonium (atomic mass= 209) crystallizes in a


log(x/m) simple cubic structure with a density of
(C)
9.32 g cm-3• Its lattice parameter (in pm) is
closest to (19 Nov, 20171
log (P) (A)421 (8)334
(C) 481 (D) 193

126. The following reaction takes place at 298 Kin


log(x/m) an electrochemical cell involving two metals A
(D)
and D, A2+(aq.) + B(s) ~ B2+ (aq.) + A(s)
With [A~= 4 x 10-~ Mand [B1 J = 2 x IO-3M
log (P) in the respective half-cells, the cell EMF is
1.091 V. The equilibrium constant of the
121. The pH of0.l M acetic acid solution is closest to reaction is closest to [19 Nov, 2017)
[Dissociation constant of acid Ka= 1.8 x 10-s] (A) 4 X 1036 (B) 2 X 10 37
[19 Nov, 2017] (C) 2 X 1034 (D) 4 X 1037
(A) 2.87 (B) LOO
(C) 2.07 .(D) 4.76

122, The limiting molar conductivities of the given


electrolytes at 298 K fo\1ow the order
p..o (K+) = 73.5, ;i.,o(Cr) = 76.3,
J =119.0, ;i.,0 (SQ~-) = 160.0 S cm2 mor1]
;i.,0 (Ca2
· [19 Nov, 2017)
(A) KCl < CaCh < K2SO4
(B) KC! < K.iSO4 < CaCl2
(C) K2SO4 < CaCb< KC!
(D) Ca Ch< K2SO4 < KC!

www.puucho.com
CP Publication KVPY chapterwise with www.puucho.com

t PHYSICAL CHEMISTRY

l. (A) 2. (D) 3. (A)


ANSWERS
4. (C) 5. (8) 6. (B) 7. (D)
"
8. (B) 9. (A) 10, (C) 11. (D) 12, (D) 13. (D) 14. (C)

15. (A) 16. (C) 17. (B) 18. (D) 19. (D) -o. (A) 21. (C)

22. (C) 23. (A) 24. (D) 25. (B) 26. (B) 27. (A) 28. (D)

29. (A) 30. (A) 31. (B) 32. (8) 33. (C) 34. (D) 35. (B)

36. (D) 37. (A) 38. (B) 39. (C) 40, (B) 41. (C) 42. (B)

43. (B) 44. (D) 45. (D) 46. (D) 47. (D) 48. (A) 49. (C)

50. (C) 51. (C) 52. (B) 53. (C) 54. (8) 55. (D) 56. (B)

57. (B) 58. (B) 59. (A) 60, (D) 61. (D) 62. (A) 63. (A)

64. (A) 65. (D) 66. (C) 67. (D) 68. (C) 69. (A) 70. (B)

71. (D) 72. (C) 73, (B) 74. (C) 75. (B) 76. (C) 77. (C)

78. (B) 79. (A) 80. (C) 81. (C) 82. (A) 83. (A) 84. (A)

85. (C) 86. (A) 87. (A) 88. (8) 89. (A) 90, (A) 91. (D)

92. (D) 93. (B) 94. (B) 95. (A) 96. (C) 97. (B) 98. (A)

99. (A) 100. (A) 101. (C) 102. (B) 103. (D) 104. (A) 105.{C)

106. (C) 107. (A) 108. (!3) 109, (D) 110. (D) 111. (C) 112. (B)

113. (A) 114. (A) 115. (B) 116. (C) 117. (B) 118. (D) 119. (D)

120. (C) 121. (A) 122. (A) 123. (A) 124. (C) 125. (B) 126. (B)

1 1
1.
E = he= 6.02x 10-34 x 3 x 10 8 6. Fee= 8 x - +6 x - =4 ·
A l 8 2
E = 1.988 X 10-2) J 1
Bee = 8 x - + I x l =2
8
2. Informative
7. (i) CO2 F co+ ½02 K1 = 9.1 X 10- 13
234 234
3. Th----+ Pa+ 1 e
0
(ii) H20 F' H2 + l_02 K2 = 7.1 X 10- 12
90 91 - 2
Object CO2+ H2 p CO + H20
equation (i) - (ii)
4. Infonnative
K1= ~ = 1.28
K2
5. Informative Ct = CoC-kl
R = R,,i e-kt

www.puucho.com
CP Publication KVPY chapterwise with www.puucho.com

TOPIC• WISE KVPY [STREAM SXJ Questions with Solutions

1 d[NO 2 ] 1 d[NOJ d[0 2] Surface area nd 2 6


9. r=- -"----== +---=+-- 19.
2 dt 2dt dt Volume - 1rd 3 -d
:. N0 2 is reactant so (Z) rate of dis appearance of 6
NO2= rate of formation of of NO d 1 = 30 nm d2 = IO nm
So NO is (X)
( Surface area) 6
10, Initial Normality Volume 2 = iL= .:!i_= 30 ,,,, 3
N = 2.52 x 1000 ,,,, 0.4 ( Surface area) ~ d2 10
Volume 1 d1
63x100
COOH
I · 2 H20 20. A (HOAc) = A"(NaOAc) + A°(HCl)-A°(NaCI)
COOH = 91 + 425.9- 126.4 = 390.5
:. N1V1 =N2V2
0.4 x IO= N2 x 500 21. From Bragg's equation
0.4 n>. = 2d sin8
N2= - =0.08 N
50 d= ).., n = I
Then final weight 1 x A. = 2 x ;i., sin8
N= wxl0O0 . 8 = 1-
Sill
ExVmi 2
8=300
Q,QS =. W 1000
X
63x500
22. Zn+ Cu+z ~ Zn +z + Cu
2Zn + 2C1/2 4 2Zn-2 + 2 Cu
12. At equation -6.G 0 =0
For 2 moles of Zn, n = 4
:. T= &-I_ 7.5xl000 = 300 K LiG0 =-nFE0 een =-4 x 96500 x l.1 =-424.6 kJ
ti.S . 25

13. MgCh---4- Mg+1 + 2cr 23, K"" Ao-A1


t
?
0.01 M .. K = ~
Ksp = Q = [Mg4'2] [Off] 2 2t112
10-12 = [O.Ol] [Oir]2 Ao
[mr]2= 10- 10 t112 = - Zero order
2K
[OH'"]= 10-5 t112 a: initial concentration
pOH =5 :. pH =9 so double times

a 24. According to Lagmuir curve


]4. r= - - = 141.4pm
2..fi. x= ~
a =2 x .Jz xl41.4 l+bp
:. V = a = (2 x .Jz xl41.4 )3
3
p400 x= -
a
b
15. For Ideal Solution xa:p
MI Mix =O LlV Mix= 0
25. I 6H~ + 2MnO4- + 5C2ol-
16. 1t=CRT 4 2Mn+2 + 10CO2 + 8H2O
=> 1t cc C at consl T Mn04- : C.20/- = 2 : 5
~"" ~= O.ot = 10 2K.Mn04 + 16 HCI
n2 C2 0.001 4 2KCI + 2MnC)i + 5C)i + 8H2O
MnO4- : HCI = 2 : 16 = l : 8
18. (6.S)systa:m + (~S)=ding> 0 (irreversible process)

www.puucho.com
CP Publication KVPY chapterwise with www.puucho.com

• PHYSICAL CHEMISTRY
26. Fc+3 /Fc 1
32. t !12 a; ----;;-:f
Fe+2 + 2e- ~ Fe E0 =-0.44 a
Fe+3 + e- ~ Fe..2 E0 = 0.770 for zero order reaction n =0
so t 112 oc a
Eo=?
(ti,2)1 -81
--=-
(t112h • a?.
8 0 = n 1E? + n2 E~ = 2(-0.44) + Ix (0.770)
D3 3 so - .2- = LAol--
(t112h 2[AJo
-_ -0.88 +
3
0.770 -
_ - O. 037 l
VO t
t112 = .4 sec

33. Mn+2 =23 e-


27. AE= 13.6 [~-~)eV/atom ; Fe+l = 23 e-
n1 n2

H = Ka +lo [Conjugate base]


JS. p p g [Acid]
·:[Conjugate base]= [Acid]
pH"" pKa ""5.85

u;o.l
28. = _ ____,2=---- K -A
- e- -Ea
36.
RT
.!.
100 . + 500.~
2 2 lnK=lnA-!L
so
=---
RT
50+250
so 300 = (-400) XA + 500 37. Bond Length cr,: - - - -
l
300 Bond Order
l CO,B.O. =3
= 1/6 XA=-
2 0 = C = 0, B.0. = 2
l /0"
:. Xe= - O=C B.0.= 1.33
2
'o-
29. Total droplets= 2.4 x 10 18 2P(g) + 3C'2(g) ..... 2PC11(g)
38
tqtal area = total droplcl~ x area of one drop
= 2.4 >< 10 18 >< 12.5 >< 10-IIS
2PCl3(g) + 2C! 2(g) - 2PCl 5(g)
"' 12.5 >< 2.4 X 102 m2 Net reaction : 2P(g) + 5Ch(g) ..... 2PCl5(g)
Energy consumption = 0.03 x 12.5 x 2.4 x I 02 K=K1K~
' = 90 Joule
40.. A=0.7
C
31. ~KT= I.6xio- 19 3Sxl04 =0.?
2
C
% X 1.38 X 10-23 T = 1.6 X 10- 19 C= 35xl04 =5x10s
0.7
T= 104 K
Moles of cytosine = moles of guanine

www.puucho.com
CP Publication KVPY chapterwise with www.puucho.com

TOPJC•WISE KVPY [STREAM SX] Questions with Solutions

41. E=~+K.E. dC ·
46. -- = k (for zero order)
·: E = bC = 6.6xl0-34 x3><10 8 dt
A. ,660x!0-9 C t

=3x10- 19 J JdC = - Jkdt


C0 O
~ = lev = l.6x10- 19 J
Ct-Co=-kt
K.E.=3xl0- 19 -l.6xI0- 19 =1.4xl0- 19 J Ct =-kt+Co
for wave length of emitted electron slope~- k
h 6.6xI0-34
ho= .J2oiKE = -J2 x9.1 X} 0-JI X 1.4 X l Q-Jg 47. According to Henry's law
P !!llS = Ktt Xgas (constant temperature)
6.6xI0-34 _ :. for same pressure
= · 25 = l.32x JO 9 meter
sx10- K11 Xgas = constant
:. greater the value of Ktt, smaller is solubility
and vice-versa.
42.
48. A :.;::=: nB
t"" 0 0.06 0
cqm 0.06-x nx
.'. 0.06 - X = 0.03
43. 6.H = L\E + An&RT x=0.03
20 =t.E + 8.314+ 10-3 x 333 &nx = 0.06
=> i.\E = 17 .2 kJ/mol
0"" 0.06 = 2
0.03
V
44. W :cc -2.303nRT Jog--1.. [BJ 2 m ( 2
Vj &Kc:=: _ _ eq_= 0.06) -0.12
[A] eq"' (0.03) 1
=-2303x3x 8.314xI0-3 x300log 20
2
=-17.2kJ/mol
49. t\S = nR ln ( ~: )
45. r = K[X]"[Y]Y
Total order= n = x + y
Byexp.(1) & (2)
t\S =~ x 8.314 x .en (\00°)
r1 K[.2S]"'[.2S]Y 1.0xlO-li = 2 X 8.3 }4 X 2.303 J/k
-;:;- = K.[.S0]"'[.2S]Y = 4.0x 10--f.i =38.3 J/K
l l
- - - x-" 50. M = 2d sin 8
(2? - 4' - -
By exp .(I) & (3) 2 x 3 =2 x Ji x sin 30"
!i... = K[.zsr [.25]Y Ix!O~
6 -
r3 K[.25]1'"[.50]Y 8xl0-<'i 1X 3=2 X ..J4 X Sin e
I 1
(2)Y = g'Y =
3

So Total order= 2 + 3 = 5
,', 6 X Sin JQ 0
=2X 1 X Sill ·e

0=30°

./
.- ;,,
www.puucho.com
CP Publication KVPY chapterwise with www.puucho.com

f PHYSICAL CHEMISTRY
60. K,µ{AgB,) = [AgJ [Brl
2 Ions = (5 x 10- 1°) (I o-3)
co-ordination no. 6 =5 X 10-13

Now
53. [Fe(NH3)6]:i- so unpaired e- l
5 x 10-13 = (l0- 2) [Brl
Inner sphere complex
[Brl=Sx 10-11 M
(Low spin complex) :.µ =../1(1 +2) = 1.73 BM
[FeF6] 3 - so unpaired e- 5 61. K 2Cr0 4 :::;, Cr' 6 (highest oxidation state)
Outer sphere complex NbCJ 5 .::::,.Nb+5
(High spin complex) MnO2:::;, Mn+4
:. µ = ~5(5+ 2) = 5.92 BM
62. For plot between Ink vis 1/T y-interccpt is !n A
54. M y -E
& slope is __ a
1 R
4X-=2 4
2 therefore; En < E1 and A1 > An
:. Formulae= MY 2
63. Comer .::::,. .!. .: : ,. Face :::;,
8 2
760- 750 _ ( ~ )
55.
760 - (\088) 64. Q = IB][C] & Kc= 0.5
[A2]
MM=228 For option (A)
(I 0-2 ) X (10-2 )
Q -'------'---'-----=- cc: JOO
56. Ecen = {0.34 - (- 0.76)} volt [t o-3 ]a
= LI volt & Q > Kc i.e. reaction proceed in backward
& 6-G = - nF E~e11 direction.
6-G = - 2 x 96500 x 1.1 x 10-3 k.J
=-213 k.J 65. I 26 = i";,a, + A~1-
150 = l~. + J-.~1-
57. or K 2
1
=-1-
K2
250 = i';ia· + ,._~ir
(150 + 250-126) == A:. + "'7,w
58. W= eq-wtxixt
or A~on = 274
96500
( 0.005) x 80 x 10 _5 = I 00x 3x t(sec) 66. Atomic no. 33 and 17 belongs to 151h & I 7th
10 96500 group respectively therefore co-valcnt bond

59.
t = 135 s

I2 + Na2S2O3.5H2O _,. Na2S4O6 + 1


VII lance factor ~ 2 valency-fuc!or = I
form between both elements
x•l
X
XY3
y-1
Atomic no. 33 = As
Atomic no. 17 = Cl
AsC'3
r
100
:. cq. ofh =0.25 x --= 0.025
1000 67. KMnO4 +Kl+ H2SO4 -------) MnSO4 + I2 +
:. cq ofNa2S2O,.5H2O = 0.025 K2SO4+JhO
v.f= 5 v.f= l
or mo! ofNa2S2O3.SH2O = ( -0.025)
1- :. (eq)KMno. =(eq)KI =1
~ ... 4.

:. mass ofNa2S2O3.5H 2O = (0.025 x 248) Eq. =V.F. x mole


g= 6.2 g 1 = 5 x mole
Mole= 1/S

www.puucho.com
CP Publication KVPY chapterwise with www.puucho.com

TOPIC- WISE KVPY [STREAM SX] Questions with Solutions


68. The correct way of reporting the average value 78. SRPJ- Reducing poweri
should have exactly the same number of digit
after decimal which has least digit after decimal 79. Since conductance of H+ is highest so molar
among the data given. conductivity of HCI will be highest and after
that conductance of CH3COOH will come
69.
~s = AH Melting = 6 x l 000 J . order
Tr-.P 273 K HCl > Cl-13COOH >NaCl> CH3COONn
= 21.978 :::: 221/k
80. [ZCLS2 is 3.87
70. PM=pRT x + (-I) 4 = 2
M =( 5x0.08:l x300)=: 123.15 u = [n(n +2)
x=+4-2
Number of acetic acid molecule
x=+2
=123.15:::!2
60
Mn+2 ->4s 0 3d 5 1111111111 I
NC 2 ~4s0 3dg 11~11~11~11 I 1 I
71. W=-nRT=-(1 x8.314x x373)kJl0- 3
=- 3.10 kJ Co+2 ~ 4 s 0 3d 7 [1~ [1~11 ! 1 I 1 [
q=i:lli=41kJ
& L\E= q + w= (41-3.l)~ 37.9 kJ
CuT2 ~4s 0 3d 9 11~!1~11~11~11 I
81. pH=-log [Hl
72. H2 + Ii ~ 2 HI ; ~=50 1
N2 + 3H2 ~ 2 NH] ; K,= 1000 pHoc - -
[H+]
K, = 1000 Order of pH
CH,COOH > HCOOH > HCl
(50) 3
Acidic strength order
Kc= 0.008 CH,-C--OH < H--C--OH < HCI
ll II
73. N2 + 2H2--, 1 N2!L i:llir 0 0
Al-Ir= I x EN,.,N + 2 En-H -4 EN-H - 1 EN-N Acidic strength oc stability of Anion
+J
= [(1 X 946) + (2 X 435)-4 X (389)-} X (159)] k.J 0 ~r 0
= 101 kJ/mol Cl 8 > H--C--0 >~--0
Size II II
large 0 0
a
74. rK. +rc1- =2
82. By definition
a
133 + 181 = -
2 83. N2 + 3X2~2NX3
a=2(133+181) N 2 +3F2~2NF3
a=628 pm L\H NFJ = 94] + 3( 155) - 6(272) = -226
or a= 628 x 10-to cm
L\HNCli = 941 + 3(242) - 6(200) = +467
and
volume = a3 = (6.28 x 10-8)3 cm3
84. X ~ 2Y k 1 Z ~ P + Q k2
= 2.4767 x 10-22 cm3 1 0 1 0 0
:::! 2.48 x 10- 22 crn3 1-a 2a 1-a. a a.
= - (6 X 96500 X 0.3) J (2a) 2
=- 173,700 J -- 2
1_ _ (1- a) = 4a =4
K= Ae-Ea/RT
:1,k - 0.2 a2
77. .• 2

T--; oo k'":' A (1-a)

www.puucho.com
CP Publication KVPY chapterwise with www.puucho.com

1 P~YSICAL CHEMISTRY
85. [Mn Br4T2 91. tin= 1 hr
x+(-1)4=-2 2.303 a
x=+2 ts1.s = - - l o g - -
k a-x
Mn--------> 4s23d5
Mn+2 --------)-4S 3d5
0 = 2.303 log a
0.693 a-.&75
ltltltltltl
We know that Br is weak field ligand so 1-Iund's 1
Rule is applicable = 2.303 lo 8 = 2.303 x3x.30l0
4s 4p 0.693 g 0.693
then It It It It It I D I I =3
Br Br Br Br
Hybridization is -----+ sp3 92. No. of F will be equal to eight since for one
and No. of unpaired electron.= 5 Ca+z there should be two F ion.

a= 1.1 - 0.0591 log zn+2 93. AG 0 = -2.303 x 8.314 x 298 log (3.8 x 10-3)J
86.
2 cu+z = l3809.3876J = 13.809 Kg
-1.l = 0.0591 10g Zn ,z AG0 =-nFE0
2 cu+2 13809.387 = -2 X 96500 X E0
2n+2 -E0 ce11=.071
log--=37.3
cu+2
. 232-208
94. no. of o.-partrcle = - - - = 6
87. At V=50L T=200K
X 4
n=l no. of ~-particle = 4
Px = nRT = 1 x 0.0821-x 200
V 50
= 0.328 95.. Z= (VM)r <l atp<200bar
(VM),
At z p. _ lx0.082lx200 0 _821
2 - 20 :. (VM)r < (VM)i
At Y p _ Ix0.082lx500 0_821
y- 50 d= NxM
96.
NAxa 3
88. Acidic strength of acid is
HClQ4 > HCI01 > HCI02 > HCIO 8_93 4x63.5
[Hi t ka t p1m -1- 6.023x 10 23 x a 3
.-. pk• order: HCl04 < HClO, < HCI02 < HClO a3 =4.72 X 10-Zl
a= (47.2 x10-24) 113
89. = 3.61 x 10-g cm
IBCC
68%
= 3.61 x10- 10 m
=361 pm
order FCC> BCC > SC force a= 2..fi. r
a
90. r=--=127.8
2./2
~ 97. Cu ' 2 + ze-- --:,. Cu .1.G~ = -ntE;F
(Vrms) tt, =_,J=
~=== = 1.1 8 Cu+ 2 + e- -----+Cu LiG;. = -n2E;F
(Vrm.s)o 2 3 x Rx 500
28

www.puucho.com
CP Publication KVPY chapterwise with www.puucho.com

TOPIC~WJSE KVPY [STREAM SX] Questions with Solutions

102. Mav = M,n1 + M2n2


n 1 +n 2
-n 3 E;F = -n 1E;F +n 2 E;F
JS.45 = 35n1 +37n 2
E" n,E: -n 2E; n 1 +n 2
3
03
:. ll1 : D2 =3 : I
2 X 0.34-.522
= 1 103. According to Lechatclicr principal on
= .158 V increasing volume of closed vessel equilibrium
ixt will shift towards right.
98. No.ofmole= - - - - ,
96500xv.f
104,· For exothermic reaction
ixt l.Sx250
nxv.f =--xn= - - - x 0.85 dli= Er-ER< 0
96500 96500x2 Eu-EA<O
n= .00165 (deposited) Eu<EA
250><.15 .. Catalyst does not changes the initial and final
ni = 1000 =.0375 (uutial mole)
position of the reaction so correct answer is (A)
n1e11. = .03585 (mole left)
Mteft= ·03585 =0.143
105.· Ksp Mg(OH)2 = [Mg-2] [OHl2
.25 5.6 X 10-12= [10-l~[Olr]2

M.wt = 59 +12 + 14 x 4 + 35.5 x 3 = 233.5


[Dir]= Js.6x10-2 =0.24M
99.
CoH12N,Cl3 AgN0 1 ) 1.435g AgCl 106. Ag' Cu 12 :Al+3
2.33 no. of eq. deposit X X :x
233 _5 g = 0.01 mole (0.01 mole)
X X X
no. ofmol deposit
(i) 0.01 mole molecule produce 0.01 mole 1 3 2
AgCl 6x : 2x Jx
no. of mole deposit 6 . 2 3
.·. one replaceable Clf! ion
mass deposit 6MAg: 3Meu: 2MAI
so formula of complex is
[CO(NH3)4Ch]CI
3. [ 1
(ii) Oxidation no. of Co is +3 so primary 107. Iog10 = -209xl0
--- ---1]
2.303 x 8.314 300 T
valency is 3.
•. 9.16 x 10-~ =3.33 x 10-3 _ _.!._
(iii) Coordination no. is. 6 so sec. valency is 6 T
so ans. is 3,6 T=308.4K
or 35°C=X
,_ l000xK
100. M M
1000 X 1.65 X 10--4
108. No of 0-2 per unit cell= .8 )( .!. + 6 x ..!.. = 4
8 2
= = 8.25
0.2
No of Al+3 per unit cell= 4 x .!_ =2
).~ =='-~(H+)+ii.':t(CH 3COO-) 2
=349.l+ 40.9 No ofMn+:i. per unit cell= 8 x .!. = 1
=390 . 8
8.25
a.=-~.0211 MoAl:z()4
390
lt,9. no. of radial node= n--t'-1 = 4-1- I= 2
101. According to their SRP value. no. of angular node= .e = I

www.puucho.com
CP Publication KVPY chapterwise with www.puucho.com

t PHYSICAL CHEMISTRY
110• . 117. X--tY--tZ
Th t=O Ao O D
( t=t Ao-x y z
Ac So total moles of constituents will be more at
C any time 't' as compared to t = 0

%Th=~= .2_ . 238-206


RT lOO 118. no of a. particle= - - - = 8
4
%Ac=~= 98 no of fJ-particlc = 6
Rr 100

.._ -
R 2
1 _
-- 119. n=2 l=O m=O 111s=±1/2
R2 98
RT=R1+R2 1 m=-1,0,+l 1Jls=±l/2
0.693 -R 98 R
- - - 1+- I
22 2 120. ~ ocPl/n
.-. R1 = 6.3 X 10--4 m

~ =kP11n
111. For process A-)- C, W = 0, AE = 0, q = 0
m
For process A ----)- B
W = -l (1.5 - 1) = --0.5 lit. attn X 1
log - = log k + - log P
AE=q+W m n
O=q+W y=c+ mx
q =-W =-(-0.5) = 0.5 lit. atm
For process B----)- C, W = 1 atm
AE=q+W 121. [HJ= JK.,.c = ~l.8x10-5 x0.l = ../Ii x 10 3

O=q-W pH= 3-log {1.34) = 2.87


--q=W=-latm
:. Total heat exchanged= l.5
122. A KCI = 73.5 + 76.3 = 149.8
O

112. 1.013 = 0.742 Xt+ 1.8 Xb A0 C;,Cli = 119 +2 X 76.3 = 271.6


1.013 = 0.742(1-Xb) + 1.8 Xb
Xb=0.256 A°KiS04 = 2 X 73.5 + 160 = 307
:. Xi= 1-Xb = 0.744

113. The unit cell of the above pattern will consist of 123. Co(g) + .!_ 02 (g) ~ CO2(g) AN = -283.5
2
8 white square and I black square i.e. it will
form centre unit cell. AS= 213.7 -197.7 - ZOSOI =-86.5
:. No. of white square Y = 8 2
No. of black square X = I
AG =-283- 29 Sx(-86·5) =-2570 kJ
:. Formula XY8 1000
Wnw:=-AG =-(-257 kJ) =257 kJ
114. It is decided by SRP value

115. l.lx 10-12 = [Ag+]2 (5x 10-13] 124. ATb=Kbxm


.-. [AgJ = 1.5 x 10-~ = 0.52 X J8/180
l
2x4/31tR 3 = 0.052
116. l] = ---- X }QO := 68%
:. B-P=373 ,15+0.052

(1J = 373.2 K

www.puucho.com
CP Publication KVPY chapterwise with www.puucho.com

TOPIC~WISE KVPY [STREAM SX] Questions with Solutions

125. d= NxM E _ Eo .0591 l 2xl0-3


126. cell- ,;,:It- ~2- og 4x10-l
NA x'a 3

932 = lx209 1.091 = E"c.11 - .OS9 l log (.5)


- _6.023xl0 23 xa 3 2
:. a =' 37.2 X 10-24 S
3 E"c,11 = 1.099
a= 3:33 x 10-8 cm .0591 I k
Eoc.11=- -2- og
;::;: 334 pm
. log k = - L099 x 2 =- 37.22
.0591
k= 2 X JQ'7

·,

~-

www.puucho.com
CP Publication KVPY chapterwise with www.puucho.com

Inorganic Chemistry
I
I. The number of isomers of Co (diethylene 7. Diborane is formed the elements as shown in
triamine) Ch is - [2010] equation ( 1) 120101
(A) 2 (B) 3 (C)4 (D) 5 2B(s) + 3Hi(g) --t B2H6 (g) --- (1)
Given that
2. Among the following, the re-acid ligand is - H2O(/) -). H 2 0 (g)
{20101 AHi°=44kJ
(A)F (B) NH3 (C) CN' (D) r 2B(s) + 3/2O;.{g)-). B2O3(s)
AH 2°=-1273 kJ
3. The bond order in o/· is- f2010J
(A)2 (B)3 (C) 1.5 (D) I B2H6{g) + JO2{g) --t B2O3(s) + 3H2O (g)
llli3° = -2035 kJ
4. The shape of the molecule CIF3 is- [2010]
H2(g) + 1/2 0 2 (g)--,. H 20(/)
(A) trigonal planar (B) pyramidal
Afl,/' = - 286 kJ
(C) T-shaped (D) Y-shaped
the AH 0 for the reaction ( 1) is-
5. The correct structure of PC1 3F 2 is- [2010J (A) 36 kJ (B) 509 kJ
F F (C) 520 kl (D)-3550 kJ
I --=- Cl I
-= Cl
8. The Crysta[ Field Stabilization Energy (CPSE)
(A)Cl-P" (B)F-P"
I Cl ICl
and the spin-only magnetic moment in Bohr
F Cl Magneton (BM) for the complex K3[Fe(CN) 6}
Cl are, respectively- (2010)
I --=-p
F
(A) 0.0 As and Jii BM
(C)CI-P~ (D) Cl'- I ....::cp
I
Cl
F
Cl.,,
-p--

'F (8) ~2.0 As and J3 BM

(C)---0.4 A5 and ,124- BM


(D) -2.4 Ci. 5 and 0 BM
6. Identify the cyclic silicate ion given in the
figure below [2010] 9. The hybridizations of Ni(CO) 4 and Cr(I I 2 O)/+,
respectively, are 120111
where (A) sp3 and d 1sp2 (B) dsl
and d 2sp3

A=~ 10.
(C) sp3 and d2sp 3 (D) dsp2 and sp3d2

Extraction of silver is achieved by initial


complcxation of the ore (Argentitc) with X
• =Si followed by reduction with Y. X and Y
O=O . ry:specti vely are (20111
(A) [Si40isJ24-- (B) [Sic,O 1s] tlf- (A) CN"" and Zn (B) CN- and Cu
(C) [S4012] 12 (D) [Si6O24)1 2• er
(C) and Zn (D) Br- and Zn

www.puucho.com
CP Publication KVPY chapterwise with www.puucho.com

TOPIC· WISE KVPY [STREAM SX] Questions with Solutions

11. All the products formed in the oxidation of 20. Hydrolysis of BCI; gives X which on treatment
NaBH.i by 12, are [2011) with sodium carbonate produces Y. X and Y,
(A) B2~ and NaI (B) B2H6, H2 and NaI respectively, are - (2013)
(C) Bl3 -and NaH JD) NaBL and HI (A) H3BO3and NaBO2
(~) H3B03and Na2B401
12. The spin-onJymagnetic moments of
(C) B2O:s and NaBO2
[Mn(CN)6]4- and [MnBr4]2- in Bohr
(D) B203 and Na2B40J
Magnetons, respectively, are [2011]
(A) 5.?.2 and 5.92 (B) 4.89 and 1.73
21. The numbers of lone pair(s) on Xe in XeF2 and
.
(C) 1.13 and.5.92 (D) 1.73 and 1.73
XeF4 are, respectively, - (2013) .
13. XeF6 hy_drolyses to give an oxide. The (A) 2 and 3 (B) 4 and 1
structure. of XeF5 and the oxide, respectively, (C) 3 and 2 (D) 4 and2
are- (20111
(A) oct.altedral and tetrahedral 22. In the structure of borax, the nu~bers of boron
(B) distorted octahedral and pyramidal ato~ and B-O-B units, respectively are- (20131
(C) octahedral and pyramidal (A) 4 and 5 (B) 4 and 3
(D) distorted octahedral and tetrahedral (C) 5 and4 (D) 5 and3

14. Among. the following, the species with the


23. Aqueous solution of a metallic nitrate X reacts
highest-bond order is - (2012)
(A) 02·
• .
(B) F2
. +
(C) 02 (D) F.2 with Nl-4OH to form Y which dissolves
in excess NH4OH. The resulting complex
15. The mo~culc with non-zero dipole moment is is reduced by actaldehyde to deposit the metal. X
~ 120121 and Y, respectively, are - 120131
(A) BCl3 (B)BeCh (A) Cs (N0 3) and CsOH
(C)CC4·· (D)NCh (B) Zn(N0 3)2 and ZnO
,.
(C) AgN03and Ag20
16. For a t~trahedta.l complex [MCI.i] 2-, the
. (D) Mg(NO3)2 and Mg(OI-1)2
spin-only _magnetic moment is 3.83 B.M.
The· element M is - (2012]
(A) _Co, (B) Cu (C) Mn (D) Fe 24. In aqueo1L11 solution, [Co(H2O)6]2+ (X) reacts with
molecular oxygen in the presence of excess
17. The C--0 bond length in CO, CO2 and CO32- liquor NH3 to give a new complex Y. The
follows the order- 12012] number of unpaired electrons in X and Y are,
(A) co,< CO2< co/- respectively- [2013]
(B) CO2< co/-< co
(C) co> CO2> col- (A) 3, l (B) 3, 0
(D) C032- <CO2< CO (C)3,3 (D)7,0

1S. Among the following, the species that is both 25. The number of geometrical isomers of
. tetrahedral and diamagnetic is- [2012] [CrCh(en)(NH3)2], where en = ethylenediamine,
(A) [NiC4] 2- (B) [Ni(CN)4]2-
h- PH~
(C) Ni(CQ)4 (D) [Ni(H2O)ti]2+
(A) 2 (B) 3 (C) 4 (D) 1
19. Among the following, the set of isoelectronic
ions.is - 12013] 26- The element that combines with oxygen to give
(A) Na+, M(+. r, er an amphoteric oxide is - [2014]
(B) Na\ Ca \ Y-, o- (A) N (B)P
(C) Na+, Mg2+, F", 0 2-
(C) Al (D)Na
(D) Na+, K+, s2-, er

www.puucho.com
;
CP Publication KVPY chapterwise with www.puucho.com

• INORGANIC CHEMISTRY

27. Ni(CO)4 is (2014] 33. In alkaline medium, the reaction of hydrogen


(A) tetrahedral and paramagnetic peroxide with potassium permanganate produces a
(B) square planar and diamagnetic compound in which the oxidation state of Mn is
{C) tetrahedral and diamagnetic 120151
(D) square planar and paramagnetic (A) 0 (B) +2 (C) +3 (D) +4

28. The energies of d,.y and d; orbitals in 34. When H2S gas is passed through a hot acidic
. contammg
aqueous soIutmn . . Al3._·, C u21-, Pb2-,- and
octahedral and tetrahedral transition metal
complexes are such that- (2014] Ni2\ a precipitate is formed which consists of
12015]
(A) E (d,.y) > E ( d;) in both tetrahedral and
(A) CuS and AliS3 (B) PbS and NiS
octahedral complexes (C) CuS and NiS (D) PbS and CuS
(B) E ( d,y) < E ( d;) in both tetrahedral and
octahedral complexes 35. The electronic configuration of an element with
(C) E (dxy) > E ( d;) in tetrahedral but E (d;y)
the largest difference between the rt
and 2nd
ionization energies is (2015)
<E( d; )in octahedral complexes 2 2
(A) ls 2s 2p 6 2 2
(B) ls 2s 2p6 3s 1

(D) E (d,.y) -< E ( d;) in tetrahedral but E (d"y) (C) ls2 2s2 2p6 3s2 (D) ls2 2s2 2p1

> E ( d;) in octahedral complexes 36. The order of electronegativity of carbon in sp, sp2
and sp3 hybridized states follows (20151
29. XeF6 on complete hydrolysis yields 'X'. The (A) sp > sp2 > sp3 (B) sp3 > sp2 > sp
molecular formula of X and its geometry, (C) sp > sp3 > sp2 (D) sp2 > sp > sp3
respectively, are: (2014]
(A) XeO2 and linear 37. The most abundant transition metal in human
(B) XeO3 and trigonal planar body is (2015)
(C) XcO3 and pyramidal (A) copper (B) iron
{D) XeO4 and tetrahedral (C) zinc (D) manganese

30. The Lewis acid strength of BBr3, BCl3 and 38. If!:~0 when fused with KOH and oxidized. in air
2
BF3 is in the order (20151 gives a dark green compound X. 1n acidic
(A) BBr3 < BCl3 < BF3 s9lution, X undergoes disproportionation to give
(B) BCIJ < BF3 < BBr1 an intcn,;c purple compound Y and MnO2• Tlte
(C) BF, <BC13 < BBr3 compounds X and Y, respectively, are 12015]
(D) BBr3 < BF3 < BCh (A) K2Mn04 and .KMn04
(B) Mn2O1 and KMn04
31. 0 2 - is isoclectronic with (20151 (C) K2MnO4 and Mn2O1
(A) Zn2.;. (B) Mg2+ (0) KMnO,i and K2Mn0,1
(C) K+ (D) Ni2t
39. A _metal (X) ctissolves both in dilute HCI and
32. The H-C-H, H-N-H, and H-0-H bond angles dilute NaOH to liberate H 2• Addition of NH 4Cl
(in degrees) in methane, ammonia and water and· excess NILOH to an HCl solution of X
are respectively, closest to (2015] produces Y as a precipitate. Y is also produced
(A) 109.5, 104.S, 107.1 by adding NILCI to the NaOH solution of X. The
(B) 109.5, 107.1, 104.5 species X and Y, respectively, are (2015]
(C} 104.5, 107.1, 109.5 (A) Zn and Zn(OH) 2 (B) Al and Al(OH),
(D) 107.1, 104.5, 109.5 {C) Zn and Na2ZnO2 (D) Al and NaAIO2

www.puucho.com
CP Publication KVPY chapterwise with www.puucho.com

TOPIC-WISE KVPY [STREAM SX] Questions with Solutions

40. The shape of SC Li is best described as a (2016] 45. The hybridization of the central atom and the
(A) square (B) tetrahedron shape of [10 2F5]2- ion respectively, are - 12016]
(C) square pyramid (D) see-saw sp3d3

41. Among the following atomic orbital overlap,

(A)4
the non-bonding overlap is (2016]

\(\;%0
,,\17\, 7
(B)

~
2
-

(B) F<;!Vo
, I~

F .

fpy;J
.,\(/. 72-
o<!VF
(D)~

(C)
42. Among the following complexes, the one that F
can exhibit optical activity is [2016]
(A) [CoCl6] 3-
(B) [Co(en)CLir
(C) cis-[Co(en)2Ch] 3+
(D) trans-[Co(en)iCbt

43. The molecule with the highest dipole moment


among the following is (2016]
(A) NH3 ~. (B) NF3
(C) CO .,-.' (D) HF
.·,<~. 46 . Which of the following is NOT an oxidation-
reduction reaction ? [5 Nov, 2017]
44. The most stable Lewis acid-base adduct (A) H2 + Br2 ~ 2HBr
among the following is (2016] (B) NaCl+ AgN03 -4 NaNO 3 + AgCI
(A) H20 ~ BCh (B) H 2S ~ BCl 3 (C) 2 Na2S2O3 + 1i-+ Na2S4O6 + 2Nal
(D) Ch + H7_0 -+ HCl + HOCI
(C) H3N -+_ BCh (D) H3P ~ BCl3

www.puucho.com
CP Publication KVPY chapterwise with www.puucho.com

• INORGANIC CHEMISTRY
47. The thennal stability of alkaline earth metal 55. The complete hydrolysis of XeF 6 results in 1he
carbonates-MgC03, CaC03, SrC03 and formation of (19 No"·, 2017)
BaC03, follows the order [5 Nov, 20171 (A) Xe02F2 (B) XeQF4
(A) BaCO.3 > SrCO3 > CaC03 > MgCO3 (C) Xe03 (D) Xe02
(B) CaCOJ > SrC03 > BaC03 > MgC03
(C) MgC0 3 > CaC03 > SrC03 > BaC03 56. The reactivity of the following compounds
(D) SrC03 > CaC03 > MgCO3 > BaCO3 towards water is in the order (19 Nov, 2017]
(A) Cl20~< P20s < B203
48. When a mixture of diborane and ammonia is (B) 8203 < P20 5 < Clz01
heated. the final product is [5 Nov, 20171 (C) P205 < B103 < Cl:,.01
(A) BH3 (B) NH<tBl-4 (D) B203 < Cl201 < Pz0.:1
(C) NH2NH2 (0) B3N3H6
57. Among the following complexes, the one that
49. The molecule which is NOT hydrolysed by can exist as facial (Jae) and meridional (mer)
water at 25°C is [5 Nov, 20171 isomers is [19 Nov, 2017]
(A) AICl3 (B) SiCLi (A) [Co(NOz)J(NH3)3]
(C) BF3 (D) SF6 (B) K3[Fe(CN)6]
(C) [Co(H2O)i(NH3)4]Ch
50. Among the following compounds, the one (D) [C0Cl(NH3)s]Cl2
which docs NOT produce nitrogen gas upon
heating is [5 Nov, 20171 58. The fusion of chromite ore (FcCr204) with
(A) (NH.th Cr201 (B) NaN3 Na2CO3 in air gives a yellow solution upon
(C) NI-LiN02 (D) (NH4)2 (C204) addition of water. Subsequent treatment with
H2SO4 produces an orange solution. The yellow
51. When heated in air, brown copper powder and orange colours, respectively, are due to the
turns black. This black powder would turn formation of (19 Nov, 2017]
brown again when heated with rs
Nov, 2017] (A) Na2Cr04 and Na2Cr201
(A) CO (B) 02 (B) Cr(OH)J and Na2Cr201
(C) H2 (D) NH3 (C) Cr2(CO3)., and Fe2(SO-1h
(D) Cr(O!-1)3 and Na2CrO4
52. The geometry and magnetic property of
[NiC4] 2~, respectively, are [5 Nov, 20171 59. Hybridization and geometry of [Ni(CN)4]2- arc
(A) tetrahedral, paramagnetic 119 Nov, 20171
(B) tetrahedral, diamagnetic (A) sp2d and tetrahedral
(C) square planar, paramagnetic (B) sd 3 and square planar
(D) square planar, diamagnetic (C) sp3 and tetrahedral
(D) dsp2 and square planar
53. Among (i) [Cr(cn)3)3\ (ii) trans-[Cr(cn)2 Cht ,
(iii) Cis·[Cr(en)2 Cbt (iv) [Co(NH3)4CJit the 60. The total number of geometrical isomers possible
optically active complexes are rs Nov, 2017] for an octahedral complex of the type [MA2B 2Ci]
(A) i and ii (B) i and iii is
(C) ii and iii (0) ii and iv (M = transition metal ~ A, B and C are
monodentate ligands) [19 Nov, 20171
54. Phosphorous reacts with chlorine gas to give (A) 3 (B)4
a colourless liquid, which fumes in moist air (C) 5 (D) 6
to produce HCI and fl9 Nov, 201711
(A) POCh (B) H3P03
(C) PH3 (D) H3P04

www.puucho.com
CP Publication KVPY chapterwise with www.puucho.com

ANSWERS
1. (A) '2. (C) 3. (D) 4. (C) 5.(A) 6. (B) 7. (A)
~-
8.(D) 9.(C) 10. (A) 11. (B) 12. (C) 13. (B) 14.(C)

15, (D) 16. (A) 17. (A) 18. (C) 19. (C) 20. (B) 21. (C)

22. (A) 23.(C) 24. (B) 25. (D) 26. (C) 27. (C) 28. (C)

29. (C) 30. (C) 31.(B) 32.(B) 33. (D) 34, (D) 35. (B)

36. (A) 37. (B) 38. (A) 39. (B) 40. (D) 41. (A) 42. (C)

43, (D) 44.(C) 45. (D) 46. (B) 47. (A) 48. (D) 49. (D)

50. (D) 51.(C) 52. (A) '


~ .
.:.j:
·.53, (B) 54. (B) 55. (C) 56. (B)

57, (A) 58. (A) 59, (D) 60. (C)


' ,'\ r

t"'l'W:

1. Isomers of [Co (dien) Ch] is ma3b3 type 5. Correct structure of PChF:2 is


complex therefore it shows two cis & trans F
.i.-·"4,Ct
·mb -~b
isomers
.,,... - ~
'
Ci+.... P ~ ·: Trigonal biprymidal

b
···t '•
.. I

F Ct

a~b a a for minimum repulsion between atoms.


· a cis trans
6. Cyclic or ring silicates have general formula
2. CN" accept electrons from metal ion in its (Si-0/ln or (Si-01)/n-
vacant rr.0 ABMO. -l, .J,.
.
S13 og6- s·l6o18 12-
3. Bond order of o/-
Total electron = 18 7. (I) H20--)o H20 Afi=44kJ
Configuration= KK cr (2s)2 o* (2s)2 o.,(2p,J2 (R) (g)
1e(211x)2 1e(2py) 2 n:* (2p,.)2 n*(2py)2 ·,. 1
(2) 2B + - 02 --> B203 MI= - 1273 kJ
' 3
Bond order= Nb -N. = 8 - 6 = LO
2 2 · (3) B2H6 + 302--> B2O3 + 3H2O
(g)
4. Three bond pair & two lone pair present in ClF3 Ml=-2035kJ
molecule. (4) H2 + 1/2 02----) H10 Mi =-286 kJ
F (£)
. -- - l equation (2) + (4) x 3 + (1) x .3 - (3)
_..,..,'ct-F
I
. F MI= (-1273) + (-286) x 3 + (44) x 3
T-shape -(-2035) = 36 kJ

www.puucho.com
CP Publication KVPY chapterwise with www.puucho.com

t INORGANIC CHEMISTRY
8. Ni(C0)4 12. XeF6 +3H2.0 Xe03 +6HF
Ni0 = 3d84s 2
J, J.
sp3d 3Hybridisation sp3 Hybridisation
3d 4s 4p
I1L 11L 11L I 1 I 1 I IIl I I (distorted octahedral) (Pyramidal)

By effect ofS.F.L. CO.


13. (A) 0 2 , B.O = 2 {B) Fi, B.O = I
4s 4p (C) 0/ B.O = 2.5 (D) Ft B.O = 0.5
11111111111 r~D
___I _I~~ 14.
sp3 J-I ybri.
/~,
Cr+2 = 3d44s 04p0 CJ I Cl
Cl
4p µ._ .t-0
3d 4s
11 11 11 11 I D I I 15. [MCl..i]2- Tctrnhcdral = sp3 hybridisation

l 3d
I1L 11 11 1
4s 4p
M'"l
·: µ = Jn(n+ 2)B.M. =3.83
D I I n=J
Means configuration ofM42 = Jd7
so, M = 3d74s2 =21Co
9. Complexation step
r
Ag2S + CN,:;--+ [Ag(CN)2 + s-2
16. Bond Length c c - - - -
1
.J, + Zn Reduction step Bond Order

[Zn (CN)4 r + Ag
2 CO,B.0.=3
O=C= 0, B.0.=2
,,,o·
10. 0 =C B.0. = l.33
'o-
11. [Mn+2 (CN)6 r 4

Mn 12 --+ 3d54s 04p 17. [Ni(CO)-i]

CN" is strong ligands so creates back paring Ni0 = 3d84s2


CO = SFL, Ni = 3d164s'1
0

effect of (n - l) d orbitals configuration


3d _ _ _ 4s 4p
I1L I1~ I 1 I.. I·· I B I.. I·· I.. I 1~1~1~1~·11~·11~ 11~ 10 I I -I__=:jI
So, unpaired c-""' l 1
sp,
µ = ../n(n+2) B.M Tetrahedral
µ = 1.73 B.M Unpaired dcctmn = 0, Diamagnetic
And in [MnBr4r2
18. Na+ Mg2+ y- 02-
Br is a weak ligands so no back pairing
effect on (n - 1) d orbital so, unpaired c- is= 5 IOc- I De- IOc- lOc-

µ= ~5(5+2) = ..)35=5.92B.M
HlO Na,CO,
19. BCl 3 >HJB03 Na2B401

www.puucho.com
CP Publication KVPY chapterwise with www.puucho.com

TOPIC-WISE KVPY [STREAM SX] Questions with Solutions

~
25. Total 3 geometrical isomers are possible -
F F
20.
~f
a::::::.xe.:::::J)
Cl
I F F {1)
F
3 R.p. 2 R.p.

NH,
NH1
Cis
en
Cl
(2)

Cr

NH3 Cl
-·--~- . NH1
In Border no. ofb'aron atom is 4. Trans
·/ ,·.
,
en
In B--0-B unit iJ 5. (3) NH3

Cr
22. Reagent which is reduced by aldehyde is
Tollens reagent (AgiO) so metallic nitrite must Cl "---,------"
NH3
beAgN03 • Cl
Tra,is

24. [Co(H20)6] 2+ + 02 + NH3--------------) [Co(NH3) 6]3+ 26. Aluminium form amphoteric oxide with

X y oxygen (Ab03)

Co 2+ ----j- 3d7 4s 2
27. Ni exist in zero oxidation state so its
Co3+-----)> 3d6 4s 0
. configuration is -
Acc. toCEFT

In complex X Co.,--> 3d7 ---E =+'t=


its configuration is -
2sNi = [Ar] 3d8 4s 2

11ql11~11111 00 I I I I
--t+- 3dg 4:-2 4po
n =3
- CO is strong ligand so pairing of electron

In complex Y Co+3 ----j- 3d6 ~ == possible and configuration will be

~=ti 11i!ll1H1~11~1 Cl I I I I
++- ,4s 4p 1
n=O sp /Tetrahcdral
3

n::3 n=O - Number of unpaired electron are zero hence it


(X) · (Y)
is diamagnetic in nature.

www.puucho.com
CP Publication KVPY chapterwise with www.puucho.com

•28.
INORGANIC CHEMISTRY

Energy of dz 2 is greater than dxy in case of 32.


H-C-H H-N-H H---0-H
octahedral crystal field while energy of dzJ is
less than .d~, in case of tetrahedral splitting ® @ @
B.P.=4 B.P.=3 B.P.=2
dxlyl dzi
L.P. = 0 L.P.= I L.P.= 1
,
,
,, Hybridization sp3 s/ sp3
,' Bond angle I 09°, 28' I 07°.1 104.5°
,,
1 1 I I I ~--~--------
Average energy•, 2/5 6.o 33. OW·
of the d-orbital in ',,
spherical crystal fieli'-,,,
,---L--~,------,
34. H,S gas is passed through a hot acidic aqueous
,.._.__...._.tzg -
solution containing Al +3 , Cu +"·, Pb +2 and N+2
1
dxy dyz d,,. II group elements give ppt~ CuS, P~S
[Spliting in octahedral crystal field]

Pb+.:- Cu +2 Cd +2 Br·
"' Hg+'• As+3 Sb+ 3 Sn+ 2 Sn·4
,
1 I I I I ~/ _______ _
Average energy'', 35. Na= 1s 2 2s 2 2p6 3s 1
of the d-orbital in',, 3/5 llt Na· -t 1s 2 2s 2 2p 6 [Ne]
spherical crystal fieici'',,
[Ne] is inert gas, so, electron removal ii; very
difficult so I.P. is very high.
Na+2 ~ ls2 2s 2 2p 5
[Spliting in tetrahedral crystal field]

36. Electronegath~ty oc .% s character


e CHJ - CH3 < CH2 = -~H2 < Cl-J;; 5 -CH·
29. O=Xe=O sp3i'Pyramidal geometry
II sp3 . · . spr -,: sp ., '
0
25% ·~J.3% ~- '; 5Qf°/o• I
XcF 6 + 3H--0H-------> 6HF + Xe03
s character ,s charat,,t,er 's character'"
38
30. Lewis acid strength ofBBr3, BCl3 and BF3 is in
the order of
Mn02 +K01 Oxidized
In Air
) dark green compound PP
BF3 < BCl3 < BBr3 MnO :;2 & K2Mn04
Due to back bonding

F-B-f:
Disproportionation l
I .. +4
F Strong 2p - 2p Back bonding Purple colour (Y) + MnO 2
+7
KMn0 4
31. 0-2 is isoelectronic with Mg+ 2
(X)-> K2Mn04
o-2 ~8+2= IO e-
(Y)~KMn04
Mg'"2~ 12-2 = 10 e.

www.puucho.com
CP Publication KVPY chapterwise with www.puucho.com

TOPIC• WISE KVPY [STREAM SX] Questions with Solutions

39. Al 46. NaCl+ AgNO3-----* NaNO 3 + AgCI is not a


DIIIICI
X ----H2t
DilNaOH
oxidation-reduction reaction because there is no
(Metal) change in oxidation state of any element.

NH,Cl jExcess
NH,OH
lnHCI
solution
NaOH
NH,Cl

(Y)
47. Thermal stability of polyvalcnt anion salt like
Co/- increase down the group due to increase
in ionic character
Al(OH)3
(Y) 48.
Al(OH) 3 (X) -----+ Al
White ppt (Y)-----+ Al(OH)J

40. SC~:::::> 4b.p + I £.p.


Cl
/

Cl
Cl
Borazolc or Inorganic benzene
41. It has +ve & -ve overlap both simultaneous. So
it leads to non-bonding overlap. 49. Due to stcric hinderance SFG is not hydrolysed
by lhO at 25°C
42.
50. (NH.,)z Cr101----+ N2 + Cr203 + H20
en
2NaN3----+ 2Na + 3N2
Nl-L;N02-> N2 + H20
(Nl-l4hC204 - ? 2NH3 + H2C204
en Cll

51. H 2 ls reducing agent

Cl 52. [NiC4J 2 sp 3 hybrid, Tetrahedral

43. µ oc ~E-N 53.


So HF has highest value of dipole moment en en

44. Greater is the tendency to donate £.p more .


stable will be the lewis. acid-acid-base adduct.

45. [IO 2F5-]r ion


Hybridisation is sp3d 3 shape is pentagonal
optically active optically active
bi pyramidal
Double bond cause more repulsion so they
would be on Axial position 180° angle to each 54. P~ + Ch PCb
co lour! ess Iiq uid
other so shape is

H,01 m•-
F~F
H,PO; + HC!
F~F
0 F

www.puucho.com
CP Publication KVPY chapterwise with www.puucho.com

• INORGANIC CHEMISTRY

55.

56.
XeF6 + 3H20 --4- Xe03 + 6HF
whlte
explosive compound

These dissolve in water to form hydroxy acid.


59.

[
NC'-
NC/
Ni
/CN
"-cN
1
Hybridisation is dsp2 and shape is square planar
2
-

Stronger acidic oxide react more faster Acidic


strength increase with increase in EN 60,

57. Ma3b3 Type exist in facial and meridional (mer)


a b B~tr l¥r liJ
lµ Jg C
A
B C
A
C IV.
B
C

58.
a
facial b
b

Example [Co (N02)3 (NH3)3]


b
Meridional

8Na 2 C03 +4FeCr2 0 4 + 70 2


a

m: ti~r C
B
B
C

~ Na 2 Cr0 4 + 2Fe 2 0 3 + 8 02
yellow colour
2H+
2Na2Cr04 Na2 Cr201 + 2Na+ + H20
yellow orange

www.puucho.com
CP Publication KVPY chapterwise with www.puucho.com

Organic Chemistry
- - - ~ - , .====··-.- I
1. Friedel-Crafts acylatiou is- (20101 4. The cnontiomeric pair among the following
(A) a-acylation of a carbonyl compound four structures- [2010]
{B) acylation of pheaols to generate esters
(C) acylation of aliphatic olefins I.
HO~
(D) acylation of aromatic nucleus

2. The order of acidity of compounds I-IV, is-


[20101 OH
11.~H

(1) (II)
III.
HO~

H3C--OOH Q-so.,H H H

(III) (IV)

(A) I < III < II < IV (B) lV < l <ll < III IV. H O ~
(C) III< l <II< IV (D) II< IV< III< I

3. The most stable conformation for n-butane is- (A) I&II (B) l & IV
(2010) (C) II & lll (D) II & IV

(A)
H~H
(B)
H~~ s. The aromatic carbocatlon among the following
is- (2010)
II H H H
CH3 H
(A)o<B (B) o<B
CH3

(C)
H
H ~ CH3
H
(D)
&'
H
H
H
H
(C)o<B (D)OEB
www.puucho.com
CP Publication KVPY chapterwise with www.puucho.com

f ORGANIC CHEMISTRY
6. Cyclohcxcne is reacted with bromine in CCl4 in 9_ Amongi-iv
the dark. The product of the reaction is- (20101
Br ~
~~r
CN
(A) (B) d _ B r

~)6
Br

(C) O Br (i) (ii)

7. Two isomeric compounds I and II are heated


with HBr - [2010]
OH OH (iii) (iv)

o~OH oOCH,
(I) (II)
the compound that does not undergo JXJlymeriz.ation
under radical initiation, is
(A)i (B) ii (C) iii
(2011]
(D) iv

The products obtained arc-


10. Two possible stcrcoisomcrs for
(A ) ~ ;(H
VcH2Br UoH )C)=<COOH
HO H
Are (2011]
(A) enantiomcrs (B) diastereomers
(C) conformers (D) rotamers

11. The major product of the following reaction is:


[2011]

cr~H __H_+_

(B)o(
s_ The number of possible cnatiomcric pair(s)
produced from the bromination of I and 11,
respectively, are [20101
Mc\>--Mc
H3C CH3 H 1C (A)D
.H j=cH2
H3C

(A) 0, 1
(C) 0, 2
(n
ClI3 H3C

(B) 1, 0
(D) I, 1
(II) Aze
(C)u Mc ~)o--M, Me

www.puucho.com
CP Publication KVPY chapterwise with www.puucho.com ' ,

TOPIC-WISE KVPY [STREAM SX] Questions with Solutions

12. For the transformation (2011] 0 0


Br
crel4 8Dd cfoH
0
(ill)

the reagent used is


s e
N2Cl
(iv)
er ,..cH,
0

N
I
CH3
and cfoH
0

(A) LIA~ (B) H3P02


(C) H30+ (D) Hll'Pt (A) i (B) ii (C) iii (D) iv

13. The reaction 16. In the reaction sequence,

&CHO
OH OH

6+CHCI, NaOH/Hcat
0
---Y

is known as (2011) the major products X and Y, respectively, are-


(A) Perkin reaction (2011]
(B) Sandmcycr reaction
(C) Reimer-Tiemann reaction
(D) Cannizzaro reaction

14. Among i-iii


OH (i)

~OH

~OH
OH
(i) (ii} (iii) 0 OH O ()
~N,, ~N~
the boiling point follows the order
(A) ii < i < iii (B) iii < ii < i
[2011]
(ii) V and V OH
(C) i < ii < iii (D) ii< iii < i

15. In the following conversion,

V
~ C N (i) MeMgBr
(ii) e3o' x
NaOH/h
H.,o· Y

the major products X and Y, respectively, are-


{2011]
0 CH3

(i) u
~N/CH1
I and cfH 0
r97(0H
Jv
Ct
~N_,.OH
H
0 (iv) V and

(II) crNH, and


~NH2
V (A) i (B) ii (C) iii (D) iv

www.puucho.com
CP Publication KVPY chapterwise with www.puucho.com

• ORGANIC CHEMISTRY
17. Optically active (S)-a-methoxyacetaldehyde on 20, The maJor final product in the following
reaction with MeMgX gave a mixture of alcohols. reaction is - {2012]
CH CH CN (l}CH1Mgl3r
The major diastereomer 'P' on treatment with l l (2) H10·

Mel/K.2C03 gave an optically inactive compound. NH


Pis- [2011] (A) H3C- ll
"-./ "cHJ
OCH3 (B) H 3C~N/CH3
/'-._,/CH;
H3C ! 0
OH (C) H3C- JI
"-.../"cH3
(i) (ii) 0
(D) H3C~N/CH3
H

21. The major product in the following reaction is-


(2012]
(ill) (iv) H~HNaNH~

(A) i (B) ii (C) iii (D) iv H~Br

18. In the reaction benzene with an electrophile E\


the structure of the intermediate cr- complex
{A) H---=~~-H (B) H>=<H
H
-
NH2
can be represented as (2012]

(A)UH
E
(B) HOE H 0
(C)
H2C-CH2
/
H2N
\
NH2
(D) H3C,/"'-.NH2
H

(C)
a;E H (D)
Of H
22. The major product of the following reaction is-

~Conc.HBr
(2012)

H H
HO
Br
19. The most stable conformation of 2, 3-dibromobutane
is - [2012] (A)HO~

"0~
Br Br Br CH;
(A) (B) ~H {B)Br~
H H H H
H H
(C)Br~Br
Br Br

A/n,
H Br

(C)
"(!)"
H
Br
H (DJ~··
H
H
H (D) H3C

www.puucho.com
CP Publication KVPY chapterwise with www.puucho.com

TOPIC• WISE KVPY [ST REAM SX] Questions with Solutions

23. The reaction that gives the following molecule 25. The final major product obtained in the following
as the major product is - [2012} sequence of reactions is- 120121
CH3
H,C,,.._i ' Pb
__-_-_-_ 1.NaNI-{i.NH,
2.CH3I
Hc /"-..._o/CH3
3 3.H2.Pd/C

Ph>==<H
(A) -
H CH3

Ph>==<CH3
(B) -
H H

/'.... _.......CH3
(D) Ph/ '-/

24. The major product of the following reaction is - 26. (R)-2-bromobutane upon treatment with aq. NaOH
(2012] gives - [2012]
rrYCl-hCH3 OH
V + (CH3C)2 CHCH2Cl
AICh I

(A)

CH2CH3 H3C
(A)
~ OH
(H3C)3C OH

(D)
Y #
CH2CH3 (B)

H3C
CH,+ H3C ~ · ; H .
I : l mixture 3

CH2CH(CH3)2

~ C H2CH3
(C) ~
(H3C)2HCH2C .-9'
CH2CH3
(D)
C(CH3)3

www.puucho.com
CP Publication KVPY chapterwise with www.puucho.com

t ORGANIC CHEMISTRY
27. Phenol on treatment with diL HN03 gives two 30. 1be reaction of ethyl methyl ketone with Clz/excess
products P and Q. P is steam volatile but Q is Olr gives the following major product - 12013 l
(A) ClCH2CH2COCI-l3
not. P and Q arc, respectively- (2012]
(B) CH3CH2COCCh
OH OH (C) ClCH2CH2COCH2Cl

(A) u
~N02
and
¢ 31.
(D) CH3CCl2COCH2Cl

The compound 1hat readily tautomcrizes is -


12013]
N0.2 (A) CH;COCH2C02C2Hs
(B) CH3COCH2CHCH3
(C) CH:iCOCH2CH2CH3

¢
OH
(D) (CH3)3CCOC(CH3)3

32. D-Glucosc upon treatment with bromine-water


(B) ,ad
gives - l2013]
OOH HO
N02 OH OH
(A)H H (B)H H
OH OH H OH H OH
OH OH

(q y
AroH
and
ArN02

y
CH20H

HO
CHO

HO
NOz OH H

OH OH

(D) ~ and
A COOH

lAN0 lAOH
33. The number of peptide bonds in the compound-
2 12013]
O CH3 H 0
28. The compound which reacts with excess bromine
to produce2,4,6-tribromophcnol, is- (2013]
H,cAN~(X:NHNH2H 0
is -

(A) 1, 3-cyclohcxadicnc l-13C CHJ


(B) 1,3-cyclohexanedione (A) l (B) 2 (C) 3 (D) 4
(C) salicylic acid
34. The order of SNl reactivity in aqueous acetic
(D) cyclohcxanone acid solution for the compounds (2013]
0
29. Ethyl acetate reacts with NH2NHCONH2 to fonn- II
I-I;C-C-CH2-Cl
(2013] 1
(A) CH3CONHCONHNH2
(B) CH3CON(NH2)CONH2
(C) Cl13CONHNHCONH2
(A) I >2 > 3 (B) 1 > 3 > 2
(D) CH1CH2NHNHCONH2 (C)3>2> I (D) 3 > I >2

www.puucho.com
CP Publication KVPY chapterwise with www.puucho.com

KVPY [STREAM SXJ Questions with Solutions

·oc
TOPIC• WISE

35. The major product obtained in the reaction of 39. In the following reaction- (20141
aniline with acetic anhydride is -

cr
(2013]
l. :mnolysis X
NHCOCH 3 2. OH
(A) the major product Xis-
.d-
o

(A)Q (B)~O

40.
~ieY
Given the structure ofD-(+)-glucose as [2014]
CHO
H OH
HO H
36. The maximum number of isomers that can result H OH
from monobromination of2-methyl-2-pentene with H OH
N-bromosuccinimide in boiling CClcis - (2013 J
(A) I (B) 2 (C) 3 (D) 4 CH20H
The structure ofL-(-)-glucose is
37. The compound X (C,H9N) reacts with CHO CHO
beczenesulfonyl chloride to give Y (CnH13N02S)
which is insolube in alkali. The compound X is -
HO H HO H
[2013] HO H H OH

cr
(A) H OH (B)HO H
NHCI-1 3

v.
(B) ~ ' ,CH, H OH HO H
(A)
,d-
CH20H CH20H
CHO CHO
H OH HO H
HO H H OH
(C) H OH (D) HO H
38. Aniline reacts with excess BrlH20 to give the
major product [2014] HO H H OH

NH2
- ¢rBr
NH2

I 41. Major products formed in the reaction oft-butyl

y
(A) B r ~ B r

Br
,: CBl
/4
Br
Br
methyl ether with HI are -
(A) ll3C -

(B)
I and

)= and H3C-OII
+ 120141
OH

NH2

(C)Br6Br
I /4
NH2
(D) 0
BrVBr
(C) H 3C-OH a n d + I

(D) h and H3C- OH


Br I
Br

www.puucho.com
CP Publication KVPY chapterwise with www.puucho.com

1 ORGANIC CHEMISTRY
42. 4-Formylbcnzoic acid on treatment with one 0
equivalent of hydrazine followed by heating ANH
with alcoholic KOH gives the major product·
12014)
(A) ¢ 9
Br Br
0
(A)

06:l
ANH NH2

y y
OK (B) B r ~ B r Br~Br

Ilr Br
(D)

0E9
OK
(C)(f ~
NH2 0

Br OH
43. The major product of the following reaction

Ur Cu, 4 is 12014] ANH


0 0
ANH
(A)% 45.
(D)Br~Br


HO~OH
u
Calcium butanoatc on heating followed by
treatment with 1,2-ethanediol in the presence of
catalytic amount of an acid, produces a major
(B) product which is : [2014)
I\
I
(A)~
I

(C) ~OH
(B)~
I

(D)0 rPYI (C)


HO~
~
______rOH

~
44. In the reaction sequence ~
~ OH
6 I. (CH,COhO.pyridinc
1. Br2 /CH 1C01H
)

X and Y are, respectively,


X · Aqueous wnc. N.OH )

[2014]
y
(D)

~
www.puucho.com
CP Publication KVPY chapterwise with www.puucho.com

TOPIC• WISE KVPY [STREAM SX] Questions with Solutions


46. Which of the following molecules can exhibit 51. Reaction of aniline with NaN02 + dil. HCl at 0 °C
optical activity? [2015] followed by reaction with CuCN yields [2015]
(A) l -bromopropane (B) 2-bromobutane
(C) 3-bromopentane (D) bromocyclohexane

47. The structure of the polymer obtained by the


following reaction is . [2015}

U;L"'.:
I dibenzoyl .,,,~~de ) Polymer

~~2
I~
n
heol;,

i~ ~'
6
Q Ill IV
I II m IV
(A) I (B) II
(A) I (B) II (C) III (D) IV
(C) Ill (D) IV
48. The major product of the reaction between
CH3CH2ONa and (CIIJ) 3CC1 in ethanol is 52. In the reaction
[2015]
60H
6 px
u)y
(A) CH3CH20C(CH3)3 (B) CH2 = C(CH3)2 iii)z )

(C) ClhCH2C(CH3)3 (D) CH3CH:ccCHCH3 I~


&

49. If a-D-glucosc is dissolved in water and kept x,y and z are (2015]
for a few hours, the major constituent(s) present (A) x = Mg, dry ether; y = CH,CI; z = H20
in the solution is (are) (2015] (B) x = Mg, dry methanol; y = CO2; z = dil.HC!
(A) a-D-glucose (C) x = Mg, dry ether; y = CO 2; z = di!. HCI
(B) mixture of j3-D-g1ucose and open chain (D) x = Mg, dry methanol; y"' CH3Cl; z = H2O
D-glucose
(C) open chain D-glucose 53. An organic compound having molecular formula
(D) mixture of a.-D-glucose and P-D-glucose C2H 60 undergoes oxidation with K2Cr201/H2S04
to produce X which contains 40% carbon, 6.7%
50. The major product of the reaction (2015)
hydrogen and 53.3% oxygen. The molecular

~·~H,O ~ Products 1s . formula of the compound Xis


(A) CI-120 (B) C1H.i02
[2015]

(C) CiH40 (D) C2H602

~H
I
~ II
54. The maximum number of cyclic isomers

d3H
(positional and optical) of a compound having

~
molecular formula C3H 2Ch is [2015]
(A) 2 (B) 3
(C)4 (D) 5
III IV
(A) I (B) ll (C) Ill (D) IV

www.puucho.com
CP Publication KVPY chapterwise with www.puucho.com

t ORGANIC CHEMISTRY
55. The reaction ofD-glucosc with ammoniacal AgN03
produces [2016]
CHO 02H
H OH H OH
(A)HO H (B) HO H
H OH H OH
H OH H OH
CHO C02 H
02H ,..,02H
~OH
1I OH H OH
(qHO H (D) HO H (C)HjC~OH
H OH H OH 0 0
H OH H OH
O~CH3 .
CH 2 0H CHO ~ 011
(D) I d- H
56. The reagent (s) used for the conversion of
0
benzene diazonium bydrogensulfate to benzene
is/arc- {2016] 59. The compounds contaning sp hybridized carbon
atom arc . 120 I 6]
(A) H20 (B) H3P02 + HiO
(C) H2S04 + H20 (D) CuCI I HCI (ii)o
N
57. The major product obtained in the reaction of
(iii) H3C-CN (iv) H2C=C=CHCH3
toluene with l-bromo-2-methyl propane in the
(A) (i) and (ii) (B) (iii) and (iv)
presence of anhydrous A1Ch is (2-016] (C) ( ii) and (iii) (D) (i) and (iv)

60. Upon heating with acidic Kl\1n0 4 an organic


(A) H , C ~ ( R ) ~ compound produces. hexan-1,6-dioic acid as the
major product the starting compound is [2016]
(A) benzene
(B) cyclohexene
(C) 1-mcthylcyclohcxcnc
(D) 2-mcthylcyclohcxcne

58. The major product in the following reaction is 61. The number of stereoisomer possible for che
f2016] following compound is [2016]
YYOH (CH)COhO Clh-CI !=Cl I-Cll(Ilr )---CHi-GI l3
CH 3COOH
~OH (A) 2 (B) 3
0 (C)4 (D)S

www.puucho.com
CP Publication KVPY chapterwise with www.puucho.com

TOPIC-WISE KVPY [STREAM SXI Questions with Solutions


62. X, Y and Zin the following reaction sequence
are [2016]

~-~0~1---X--H~-0----Y+Z

65. In the following reactions (2016]


~
(Il)X= 0 OOH 0Y=
OH
Z= ,/'---.
0 + Go _,._ A"'"•
0

x ~ -• Y
~
(C)x=O OH Y=O, Z= A OOH <;? 0
X and Y, respectively, are

(A)~CO,H rum c¢ 0

63. The reagent required for the following two step


transformation are [2016]

v--··c1
(A) (i) HBr, benzoyl peroxide; (ii) CH 3CN (B)
(i) HBr, (ii) NaCN
(C) (i) Br2 , (ii) NaCN
(D) (i) NaBr, (ii) NaCN

64. In the reaction sequence [2016] ~~


(D)v and~

CC CHO

CHO
l. cone, KOH
2, HiO•
,
X H'
y 66. The maJor product formed in the following
reaction is
H
(5 Nov, 2017]

0{NyCH3 Conc.HN03 1
The major product X and Y, respectively, are ~ Q ConclI2S04
0('NHCOCH3
(A) ~
02N
(YNHCOCH3

(B)y
~o N02
0

www.puucho.com
CP Publication KVPY chapterwise with www.puucho.com

• ORGANIC CHEMISTRY
72. In the following reaction sequence

C1 1. B2H6
2. H102/NaOH
X CrOil-hS0,1 y

X and Y are [5 Nov, 2017]


CHO
(A)X=UOHY=U

(YOH ("YCOOH
67. Among the a-amino acids - threonine, tyrosine, (B)X=~ Y""~
methionine, arginine and tryptophan, those
which contain an aromatic group in their side
chain are 15 Nov, 20171
(C)X=UOH Y=C1
(A) threonine and arginine
(B) tyrosine and tryptophan
(C) methionine and tyrosine
(D)X=UOH Y=U
(D) arginine and tr1ptophan 73. In the following reactions
0
I. NaBI-Li
68. The number of stereoisomers possible for the )l X
H 3C CH 3
following compound is
Cll]-CH=CH -CH(OH)-Clh [5 Nov, 2017]
0
(A) 1 (B) ::!
)l y
(C) 3 (D) 4 H 3C CH 3
X and Y are [5 Nov, 2017]
69. In electrophilic aromatic substitution reactions HOD DOH
of chlorobcnzenc, the ortho/para-dirccting (A)X= A Y= x._
H3C CH3 H3C CH3
ability of chlorine is due to its [S Nov. 2017}
DOH HOD
(A) positive inductive effects (+I)
(B)X= A Y= A
(B) negative inductive effect (-1) H3C CH1 H3 C CH3
(C) positive resonance effect (+R)
(D) negative resonance effect (-R) (C) X.=

70. Among the following, (D)X=

~
CPO ~ 0
~NI
IV H
74. Which of the following alkenes can generate
optically active compounds upon
I II III V
hydrogenation? [5 Nov, 2017]
the antiaromatic compounds are [5 Nov, 2017]
(A) I and IV
(C) II and V
(B) III and V
(D) I and Ill
~~
11
71. Upon reaction with CH;MgBr followed by
protonation, the compound that produces
~~
III IV
ethanol is IS Nov, 20171
(A) CH3CHO (B) HCOOH {A) 1, Ill and IV (B) 11 and ill
(C)HCHO (D) (CHO)2 (C) I and III (D) II and IV

www.puucho.com
CP Publication KVPY chapterwise with www.puucho.com

TOPIC· WISE KVPY [STREAM SX] Questions with Solutions


75. The major product formed in the following 81. The most abundant metal fon present in the
human body is (19 Nov, 20171

orHO
reaction is [19 Nov, 2017]
(A) Zn2+- (B) Ca2+
HCI gas (C) Na· (D) Fen

v~
excessMeOH

~-
82. In the reaction

(A) ~· OH (B) OMc


O
Cl....!±..
2.y
I
CO2H

~-
,&

x and y are 119 Nov, 2017}


OMe (A) x = H2, Pd/BaSO4; y =NaOAc, Ac2O
(C)~OMe (D) Me (B) X = LiAll-4 ; y = NaOAc, Ac2O
(C) x =H2, Pd/C; y =NaOH, Ac2O
(D) x= LiAIH4; y = NaOH, AC20
76. Which among the following is n non-benzenoid
aromatic compound? [19 Nov, 20171 83. In the following reaction
(A) o-Xylcnc (B) Phenanthrene
(YCN l.SnCb/HCI X dil.NaOH
(C) Indole (D) Thiophcne
V 2.H30•
----+¥
HJC_____r-\ no
77. Natural rubber is a polymer of [19 Nov, 2017J ~CH3
(A) Neoprene (B) Chloroprene X and Yare [19 Nov, 2017)
(C) Isoprene (D) Styrene
(A)X= V
~Cl

78. The following tripeptide [19 Nov1 20171

~ J~,Y,'(cooH Y-
H3C)(__)
~V
'lPh
can be represented a~
(A) Tyr-Val-Thr
O OH

(B) Phe-Ala-Ser
(B) X= UCI
(C) Phe-Leu-Cys (D) Lys-Ala-Ser
. Y-~
79. The sugar units present in natural DNA and V ~CH3
RNA, respectively, are 119 Nov, 20171 ~CHO
(A) D-2-deoxyribose and L-ribose (C)X= V
(B) L-2-deoxyribose and D-ribose
0
(C) D-2-dcox.yribose and D-ribosc
(D) L-2-deoxyribose and L-ribosc
Y-11,~
80. The major product formed in the following ~CHO
reaction is (19 Nov, 2017] (D) X= V
CH3Br + CH3CH2ONa--+
(A) CH~H~Hz()H
(B) CH;OCH3 Y-~
(C) CH3CH20CH3 ~CH3 V
(D) CH3CH2OCH2Br

www.puucho.com
CP Publication KVPY chapterwise with www.puucho.com

• ORGANIC CHEMISTRY

84. Acetophenone (PhCOClh) reacts with


perbenzoic acid to produce a compound X.
Reaction of X with excess CH3MgBr followed
by treatment with aqueous acid predominantly
produces fl9 Nov. 20171
0 0

(A) PhAOH (B) PhAOCH,

www.puucho.com
CP Publication KVPY chapterwise with www.puucho.com

TOPIC- WISE KVPY [STREAM SX] Questions with Solutions

ANSWERS
1. (D) 2. (A) 3. (A) 4.(B) 5.(C) 6. {B) 7. (A)
8. (A) 9. (D) 10. (A) 11. (C) 12. (B) 13.(C) 14. (C)
15.(C) 16. (C) 17. (*) 18. (D) 19. (C) 20. (C) 21. (A)

22. (B) 23. (B) 24. (A) 25. (D) 26. (C) 27. (A) 28. (C)

29.(C) 30. (B) 31. (A) 32. (A) 33. (A) 34. (C) 35. (A)
36. (D) 37. (A) 38. (A) 39. (A) 40. (B) 41. (C) 42. (A)
43. (A) 44. (A) 4S. (A) 46. (B) 47. (A) 48. (B) 49. (D)

50. (A) 51.(C) 52.(C) 53. (B) 54.(C) 55. (C) 56. (B)

57. (C) 58. (B) 59. (B) 60. (B) 61. (C) 62. (D) 63. (B)

64. (A) 65. (A) 66. (A) 67. (B) lilt (D) 69. (C) 70. (B)

71. (C) 72. (B) 73. {A) 74. (C) 75. (B) 76. (D) 77. (C)

78. (B) 79. (C) 80. (C) 81. (B) 82. (A) 83. (D) 84. {C)

c-=----~
1. Friedel craft reaction used for introducing an
• SOLUTIONS

alkyl or acyl group in benzene nucleus by an
4. THJ CH1
,.I
alkylating or acylaring agent in presence of a
H~C-H H--C-OH
I l
suitable catalyst. ~-H H-1-H
2. H--T-H H-y-H
0 No. resonance
8
CH,--0 < -Oe<0~
Clh ~ // 0 ~ b C--00
CH3 CH3

Mirror image & not superimposable on each


otheL While chiral center absent in ll and HI.
0

<O~e
less stable
5.

ooo~
less acidic nature

0
More-stable
more acidity

Not follow sp 3 'C' 6 n c- sp 3 'C'


3. Most stable conformer of n-butanc is Huckcl rule: present & planer present
(nonplaner) molecule (non-planer)

OBr
(Aromatic)
H):H

H>fZH
CH3
6, 0 + Br2 Dark
cc~
Br

Br2 molecule ionizes on interaction with 1t bond


because dihedral angle between CH3 group is 180° Br2 ------t Bt + : Br-
E!ectrophile Nuclcopbilc

www.puucho.com
CP Publication KVPY chapterwise with www.puucho.com

• ORGANIC CHEMISTRY

11.
Me Me
OH Me OH 5 Mc
7.
EB

- - ~7
OH 2

A
©
H-Br
8
A 40!,-B,
(More strain in 3 4
Ring Expension
Offi Ll.
UoH 4 member ring)

VocHJ
l
c±74M,
Me

8.
CH3
I 3 4
Br-C-Cll3
I
Br-C-CH~

Meso
I
CH3
JvM,
(Not enantiomer's)
\___}Me
l
A ~~
CH3
*I
CH3 B Br-C-CHr-Br 12.
\_____ r2
; - C i r z - CHz
I
VN U 2wc1~
CH~CH2 I
Cl-13 H 3P0 2 As a reagent used because other arc
chiral center present strong reducing agent

9.
13. O OH
+CHCh
· NaOil/heat 1/
11
(YOH CHO

::::,,..
I

Reimer - Tiemann Reaction - Phenol react


10. H OOH
with CHCh in presence ofNaOH given product

Carbon

Enantionmcrs Formed because cyclohcxa dicne


14. (Y OH
.
(YOH OH

~
l ¢ OH
(i) (ii) (iii)

shows optical activity. (iii) > (ii) > (i)


In (ii) Intra H-Bonding is not formed because
ring fonnation is not stable

www.puucho.com
CP Publication KVPY chapterwise with www.puucho.com

TOPIC- WISE KVPY [STREAM SX] Questions with Solutions

0
. II 22.
~CN (i)McMgBr ~C--CH3
15.
0 (ii) H,o•
0
NaOU/1,
H,o•
I
Jdnform
Reaction

a .
0
II
C--OH v-Br-Dr/'v'

OH
23.
16. @ 7

· 24.

H
AlCh
li3C-C-CH2+
I
I
CH3

1Hydride Shift
H3 C-C+ -CH3
17. Incorrect question
The statement optically active (s) - a - methoxy
l
CH3
acetaldehyde is incorrect.

18.

20.
ZS. PhC = CH NaNit 21Nll3 ) PhC 5 c-

, PhC = C-CH3 H;z.Pd/C >

21. CH 2 =CHBr NaSK, ) CH 2 CH


-HBr

www.puucho.com
CP Publication KVPY chapterwise with www.puucho.com

• ORGANIC CHEMISTRY

26. 30. Example of iodoform reaction


C2Hs-C-Clh ClilExc.css CHrCH2.....C-CCh
l OHe l
lOHS
CHr-CH2-C-cf3 + CHCl3
II

NSN OH AqNaOH
2 (invcrsion)
0

s 31.

(S) -2 bromo butane


Aceto Acetic ester

27,
32.

6 [YNO,
OH
OH OH HO 'OOH

9
OH OH
Dil.HNO, + H H H
Br,IH20
0 H OH H
(Intra molecular 1- OH H
H~bonding) N02
CH20H CH2---0H
(Inter molecular
Glurouicacld
JI-bonding)

33. Peptide bond is formed when two amino acid


28.
combines it is amide linkage
CH3
0

f><->plide
lxmd

29.
34. Rate of SN l reaction oc stability carbocation.
!SNAC re:i.ction

0
II NH 2
CH3-C-NH-NH-C-NH2 (CH,CO),0
36.
~
A«tylatior

www.puucho.com
CP Publication KVPY chapterwise with www.puucho.com

TOPIC-WISE KVPY [STREAM SXJ Questions with Solutions

37. 41. CHO i"-l CHO


~
:;,:j
H3C-H2C-HC=C---CH3
I H OH HO H
HO OH
j NBS
CH3
Br
H
H
H
OH
OH
H
HO
HO
H
H
I
CHryH-CH=r-CH3 + CH~H2---CH=r-CH2 CH20H CH 20H
Br CH 3 CH3 D(+) Glucose ~ L(-) Glucose
+ (I) + (III)
Br - Br Enantiomer
l I
CH.,CH=CH--C---CI-h CH3-CHrCH-C=CH2
I - I 42.
(11) CH3 (IV) CH1 '
CH,:'
I .;
CH 3 -C-+O-CH 3
Benzene
38. C 7 H 9N ------+ [Y] Insoluble in KOH I :
[ X] Sulfonyl chloride CH 3 ',
1

It must be 2° amine
If one of the alkyl group is 3°. Then mechanism
is SN 1 and nuclcophile attach to the carbon
where carbocation more stable.

Insoluble in KOH

43.

39. ·cooH COOH coo°I-14'

A.JC.KOH

Br
2,4,6-Tribromoaniline CHO CH=N-NH2

¢:
40. This is example ofwolfkishner reduction which

0( (i) Ownolysis
converts.

~C=O
/
But do not

QH
8
Intramolecular
aldol reaction
l 44.
reduce ---COOH group.

Cu,ll.

This is example of
Ulman reaction which
gives product like Wurtz
reaction.

www.puucho.com
CP Publication KVPY chapterwise with www.puucho.com

t ORGANIC CHEMISTRY
45.
48.

NH2 e ai

~o
lS:JJ
(i) (CH1C0)20, Pyridille
Acetylation
CH3--CH:rO Na
Alk.oxide
(base)

Br,/CH3COOH
Bro ml nalio n
.l 3° (halide)
Alkoxide ion is strong nucleophile and strong
base & with 3° Alkyl halide Alkenes is the
NH2 major product [E2 Elimination]
A(I. Cone. NaOH
CH,COOH+¢ Hydrolysis
Dcacctylation 49. a-D Glucose ~ Open chain ~ P-D
Br
Structure Glucose
[i] 35% Glucose 65%

~ ~
46.
H"'
(CH3CH2CH2C00)2 Ca ~ 50.
CH3--CH2--CHrO-CH2-CH2-CH3

-CH2
1H~
~ 6t.-
-H20 I 1r9 -Ha
--CH2

CHrCHz-CH//C~Hr-CH2-CH3
0 0
I I
Br NaNOi + di! HC!
I 51. H-0-No:O
47. CH3-CH-CH:rCH3 2 Bromo butane
* Benzene
This molecule contain 1 chiral centre and diazonium
chloride
molecule having one chiral carbon do not have
any type of symmetry so it is optically active lCuCN
CH3
I
Dr,T'H CH2CH3
Cynobemene
(ill)
Non superimposable on mirror image

www.puucho.com
CP Publication KVPY chapterwise with www.puucho.com

TOPIC- WISE KVPY [STREAM SX] Questions with Solutions

52. @
Br
(;) Mg, drydhcr @ Be
Tollen's

grnup [-~-
reagent

H)
oxidise

in to cacboxylic acid
aldehyde

Grignasd
reagent

h-OHJ
56.
0
)I 0 EB

~O.H H'1' ~Mg~c

~ (DilHCI)
Benzene

53. AIC!i EB
57 CHrCH---CH3 --A-IC_h_Br- CHrCil-Cl-h
CHrCl!i-OH CH7r-O-II . I I I
(C2H60) Br CI-I1 CH1
0
Alcohol
(CnH2n+20)
Carboxylic acid l hydride shift

Total wt= 60 @
CH3-C-CH3
C = 24 (40% carbon) I
CH3
0 = 32 (53.3%)

54.
H=4 {6.7%) Toluene

fsubs.l y fE©l
[Act As a Ee]

/Cl-h
H1C /fJ'.-r -CH3
~ " - C HJ
[Reaction is Fridel Craft Alkylution]
Racemic mixture (Enantiomcr) pair
55. 58.
H OH 0
I I II
CHr-C, O 0
C=O C=O CH,-C_,...
!I
, II
0 ~0-C--CI-[3
H OH H OH
HO H Ammonical
HO H CH,-G-OH
~C-OH .
I-I
H
OH
OH
AgNO,
(To!len's test)
•H
H
OH
OH
0
n
II
0
CH20H CH20H Salicyclic [ Acctyla tion] Acctyl Salicylic
D-Glucosc acid Acid
[Aspirin]

www.puucho.com
CP Publication KVPY chapterwise with www.puucho.com

ORGANIC CHEMISTRY
•59. (1) sp3 (II) sp2

l !
0
(Ill)
UV) H~C=C- CH-CH3
~ l f
l sp' sp'
p

Ill & IV compound contain sp hybridised carbon

60.
·o acidic
KMnO,
C l CHO

CHO (0)

Cyclohexene
C COOH

COOH

Hexane-1,6-dioic acid

/01 63.
*
61. CHrCH=CH-CH-CH2-CH3
1 t I
GI Br
n = 2 [No. of stereogenic area]
Total stcreoisomcr = 2n
[When symm. is/are absent]
Total stereo isomer= 2 2 = 4

62. 0
II e e

64.
@( CHO
Cone.KOH
(Cannizorro
~C--OK

CHO re.action)
~CHrOH
[K]
!Ha
0
0
t-i{)H \

Mech----,), 0
h •
=Q----!!!....+ 0- 0

@jo _:20
[Esteri fic.ation]
@( CH;aH \ .•....
[Y]

www.puucho.com
CP Publication KVPY chapterwise with www.puucho.com

- M SX] Questions with Solutions


65.~o~-------~-----_:'.T~OP".'.l::c-:_1:w".'.tS:E~KV~P~Y~[~S~T~R~E~A~~~~~~~~~~:':_.

nOQl,
L_J
~¢"0~ e

O A!Cl,
68. CH,-~l-?H-CHs
. G.l/ Ch' l
( OH · ' .•
0 0 No. ofS. l ~ 2" ~ 2' ~ ~ra cenl[e ,_

j c;s R
TransR
e Cis s
~HrCHrG--0-AICI Trans S
o II ,
0
..
69.

6 : CI: ---')- it sho"."s +R effect that is

/2'
o, P directing gro
up.
.
why it act as

GQ-
F-- _ _ H, OH OH

co ~
Ir]
70. m.n y4rce-

Al1Cs2h.
O
' P ybnd anti aromatic

r;;::;yNyCH
~
I-I
3 Conc.HNOi
V. 'y 41te-
66. 0 Conc.H2so4 ' 0
. All C ' sp2 hybrid
. anti aromatic
0

ff II
NH-C-CH,
71.
~
H-C-H+Cil
II I
. ;iMgBr~H-C-H
CH 3

0 10 e
N0 2 0 MgBr

67. !H-0-H
T y r ~ s i n0e
CH3
Q OH I
H-C-H
HO NH 2
I
OH
Tryptophan 0
Ethanol
OH
H2

www.puucho.com
CP Publication KVPY chapterwise with www.puucho.com

t ORGANIC CHEMISTRY
72. 77.
~
~ -H,0,/NaOH
B,Hs
~OH

X
CrO, I H,SO,),
CH
I J
CH~ = C _CH= CH2 Polymerisation
2 Methyl 1,3 bu1adicnc
.I /r cH, I
~H 2
3

CHj
UCOOH [lsoprene] Namral Rubber
Poly isoprcne
y ( cis form)

73. 78.
COOH
0 O-D I
I. NaB}-4
Jl. 2. D30 4 ~ Phenyl alanine NH 2 - CII - Cifi - Ph
CH3
H
I
Alanine NH 2 - CH - COOH
0 0-H
I. NaBD4 Serine .NH,-CH-COOH
)l
2. H30+
,A . I
D CHrOH

Hv~H
79.
0
74. ~ H,lNi:,,. ~ CH20HOOH
H H
optically actiH~
H

~ H,/Ni i,, ~ optically act ivc


OH OH OH II

j)-D Ribose (in RNA) ~-2 Deoxy Ribose

75. It is example of N ucleophilic addition in which 80. It is Williamson synthesis Reaction (SN 2 rex 8 )
Alcohol attack as Nuclcophile and final product 8 \j)
CH1-Br+CH3 -CH 2 -0 Na
is Acetal
+0
CHrCH2 - O......CHr ...... Br Transition
5-

t
CH3-CH2 - 0 - CH3
state

Ether

81. Calcium Mainly present in Bones all other are


required in lower amount

82.

@I~, H,, Pd/BaSO4


Roscnrnund
Reduction
Benzal dehyd e

NaOAc l(CH CO)z0 3

(CH3COONa)
76. Perkir. rexn

D .s.
It is Non-Benzenoid compound
~ C H = CH - COOH

cinnamic acid

www.puucho.com
CP Publication KVPY chapterwise with www.puucho.com

TOPIC• WISE KVPY £STREAM SX] Questions with Solutions

83. 84.
r,:_yC=N SnCb+HCI ~Cll=NII
~ Stephen reduction 0
W ln,o•
H-C'©

./

www.puucho.com
CP Publication KVPY chapterwise with www.puucho.com

Mathematics
[ToP1c-w1se KVPY [STREAM-SX] Questions with Solutions]

www.puucho.com
CP Publication KVPY chapterwise with www.puucho.com

,•

Trigonometry
I
1. Let ABC be an equilateral triangle, let KLMN 7. The sum of a1i x E [O, n} which satisfy the
be a rectangle with K, L on BC, M on AC and
N on AB. Suppose AN/NB = 2 and the area of equation sin x + ½cos x = sin 2 ( x + ~) is (2012]
triangle BKN is 6. The area of the triangle
ABC is- (2010] (A) 2: (B) Sn (D) 21t
(A) 54 6 6
J!!.fl08 8. Let ABC be an acute-angled triangle and let D
(C)48
(D) not determinable with the above data be the midpoint of BC If AB = AD, then
tanB
- - equal- (2013]
2. The number of roots of the equation tanC
cos 70 - sin40 = I that lie in the interval [O, 21t] (A) fi (B) ..fi (C) 2 JB) 3
is- _,c I2010]
.[jr:J2 (B)3
(C) 4 (D) 8
9. The angles a, p, y of a triangle satisfy the
equations 2sina + 3cosp = 3 ,,)2 and
3. The product (l + tan 1°) (1 + tan 2°) (1 + tan 3°) 3sinp + 2cosa = 1. Then angler equals- [2013)
.... (1 + tan 45°) equals- {20101 (A) 150° (B) 120° (C) 60° ~O''
(A) i2 1 (B) 222
.,)¥)123 (D) 22.'l 10. Let P be a closed polygon with 10 sides and l 0
vertices (assume that the sides do not intersect
4. In triangle AJJC, we are given that
except at the vertices). Let k be the number of
3 sin A + 4 cos B = 6 and 4 sin B + 3 cos A = I.
interior angles of P that are greater than 180°,
Then the measure of the angle C ts - f2011]
The maximum possible value ofk is- (20131
~(A)30" (B) 150"
(C) 60° (D) 75° (A)3 (B)5 ,,fj;r1 (D)9

11. Let XY be the diameter of a semicircle with


5. Let A= {8 R I cos 2 (sin 8) + sin 2 (cos0) = 1}
E
/.,,, centre 0. Let A be a variable point on the
and B = {0 E R I cos(sin0) sin(cos8) = O}.
Then An B (2011]
semicircle and B another point on the
J:_Aj is the empty set semicircle such that AB is parallel to XY. The
(B) has exactly one element value of LBOY for which the inradius of
(C) has more than one but finitely many triangle AOB is maximum, is (2013]
elements
(D) has infinitely many elements ~4-'(.Js2-l) (B) sin-1( .Js2-1)
6. The value of tan8 l O - tan63 ° - tan2 7° + tan9°
(D) 2!.
LS 120121 5
(A) 0 (B) 2 (C) 3 ./(0) 4

www.puucho.com
CP Publication KVPY chapterwise with www.puucho.com

TOPIC• WISE KVPY [STREAM SX] Questions with Solutions

12. Suppose that the earth is a sphere of radius 19. X = {x E R : cos (sin x) = sin (cos x)}. The
/ 6400 kilometers. Tbe height from the earth's numycr of elements in X is - (20161
surface from where exactly a fourth of the
earth's surface is visible, is [20131
\.(.KJ
O (B) 2
(C) 4 (D) not finite
(A) 3200 km (B) 3200 .fi. km
(C) 3200"3 km (,gy£;oo km 20. A sphere with centre O sits atop a pole as
shown in the figure. An observer on the ground
13. Let ABC be a triangle _such that AB = BC. Let is at a distance 50m from the foot of the pole.
F be the midpoint of AB and X be a point on She notes that the angles of elevation from the
IlC such that FX is perpendicular to AB. If
observer to points P and Q on the sphere are
BX= 3XC then the ratio BC/AC equals [2014]
30" and 60°, respectively. Then, the radius of
(A) .J3 (B) ,/2 (C' )I_ (D) I the sphere in meters is - 120161
Yit2
14. The number of solution.'> to the equation
4 1 . -1 1
cos X + --~- Sill X + - ~ in the
cos- x sm- x
interval [0, 21t] is/ f2014J
(A)G ~4 (C)2 (D)O

The number of real numbers A for which the


- sin(Aa)
cosp,a) Id cc
equa I1ty = ,_ - 1, 110 s 1or
'I
. -
sma
cosa 50m
all real a which are not integral multiples of
(B) 50-J6
1[/2 is - 120151 (A)IOO(l- ~ )
3
(A) I (B) 2
J9f3
16. Suppose ABCDEF is a hexagon such that
(D) Infinite
yrso(1- ~) (D) 100,J6
3
AB = BC = CD = I and DE = EF = FA= 2. If
the vertices A, B, C, D, E, F arc concylic, the 21. Let AB be a sector of a circle with centre O aJJd
radius of the circle passing through them is -
12015] rndius d, LAOB = 8 ( < ~), and D be a point

(A) J% ~~ ' (C) fl- (D) .J2 on OA such that BO is perpendicular OA Let E
be the midpoint of BO and F be a point on the
17. The sides ofa right-angled triangle are integers. arc AB such that EF is parallel to OA. Then the
The length of one of the sides is 12. The largest ratio of length of the arc AF to the length of the
possible radius of the incircle of such a triangle arc AB is - f2016]
is [2015]
(A)2 (B)3 (C)4 ._J.P}-) (A) _l__ (B) Q_
2 2

18. Let C(8) = f


n=O
cos(ne)
n!
I .
(C) -sm8 ~;m. -1( e) I .
~sm
2
Which of the following statements is FALSE ?
12015] 22. The number of real solutions of the equation
(A) C(O).C(tt) = I 2sin 3x + sin 7x - 3 = 0 which lie in the
(B) C(O) + C(1t) > 2 interval [-21r, 21t] is __ /_ JS Nov, 2017]
~9'C(O) > 0 for all 8 ER 2
(A) 1 (.BJ 2
y) C'(9) '1= 0 for all 9 ER (C) 3 (D)4

www.puucho.com
CP Publication KVPY chapterwise with www.puucho.com

t TRIGONOMETRY
23. One of the solutions of the equation 25. The number of solutions of the equation
8 siii39 - 7 sin 9 + .fi cos 0 = 0 lies in the sinB + cos8 = sin2B in the interval [-re, 11] is
interval /_ [S Nov, 20171 119 Nov, 2017)
(A)(O, 10°] gJ) c10°, 20°1 (A) 1 .fBfz
(C) (20°, 30°) (D) (30°, 40°] (C)3 (D)4

24. The integer part of the number


44 I
I----is 119 Nov, 2017)
k=o cos k 0 cos(k + l)°
(A) 50 (B)52
i.q,57 (D) 59

www.puucho.com
CP Publication KVPY chapterwise with www.puucho.com

TOPIC•WISE KVPY [STREAM SX] Questions with Solutions

ANSWERS
1, (B) 2. (A) 3. (C) 4. (A) 5. (A) 6. (D) 7. (C)

8. (D) 9. (D) 10. (C) 11.(A) 12. (D) 13. (C) 14. (B)

15. (B) 16. (B) 17. (D) 18. (D) 19. (A) 20. (C) 21. (D)

22. (B) 23. (B) 24. (C) 25. (B)

1.
::::> sin (A+ B) = _!_
2

=> A+B=~
6
2x
X 5x
y ::::> C = 6 (wrong)
B-------------C
K
60°
L 51t 7C
:::,. A+B=-:::,.C=-
y=-
./Jx 6 6
2 because C = S1t does not follow equation
X 6 .
z=-
2 3 sin A + 4 cos D = 6
!yz =6 =c>x2 = ~ 5, For AnB
2 -.fj
AreaofdABC cos (sin 8) = 1 or-1 & sin(cos 8) = 0
which is not possible
= 6 + 6 + 2xy + .!.. (2x)(2x) sin 60° or cos (sin 8) = 0 & sin (cos 0) = I or-1
2
also not possible
./3x
= 12+ 2 x - +2x -
2 -./3 so A n B is an empty set
2 2
=12+2J3 x ~ ... ~ 6. tan8 l 0 - tan63 ° - tan2 7° + tan9°
J3 . = tan(90° - 9°)- tan(900- 27°)- tan 27° + tan 9°
= cot 9° - cot 27° - tan 27° + tan 9°
2. cos7 9 = 1 +sin4 9 L 1 hut cos7 9 s I By solving we get
so cos 9 = 1; sin e = o =4
IS= O; 2~

3. (1 + tan8) (1 + tan(4S -8)) 7. . x + 1 cosx -sm


sin 2
, 2 (.x+ 1t)
4
= (1 + tan8) (1 + 1+l-tane)
tan0
=2
::::> sinx +½cosx =½(1-co{~ + 2x ))
(1 + tan 1°)(1 + tan 44") = 2 etc
so product= 222 (1 + tan 45°) = 223 :::,. sinx +!cosx = !o +sin 2x)
2 2
4. Square & add both equations => 2 sin x + cos x = 1 + sin x cos x
9 + 16 + 24 sin (A+ Bf= 37 => 2 sin x cos x - 2 sin x + 1 - cos x = 0

www.puucho.com
CP Publication KVPY chapterwise with www.puucho.com

• TRIGONOMETRY
=> (1-cos x)-2 sin x(l -cos x) 0 For example
=> (l-cosx)(l-2 sinx) =0
=>1-cosx=O or 1-2sinx""O
. 1
=>COSX=l or smx=-
2
1t 5,r
::::;,x=O, or X=-- 11.
6' 6
1t 51t
Sum= O+-+- = re
6 6

8.
A
Let radius of semicircle is 'R'
. Ie, r
. o f.meire
Radrns = -11
s
where AB = 2R cos0
2R+2Rcos0
s=-----
2
Draw AM1-BD
= 2R(2eos 2 ~ )
BM=MD= -2.
2 2
AM 2AM 28
tanB= = --
~ =2Rcos -
x/2 X 2
AM 2AM
tanC= - - = - -
A= ..!_R2 sin20 = R2 sinB eos0
3x 3x 2
Rsin Bcos0
2 r=----
:. tanB = 3
2cos2~
2
tanC
2Rsm-.cos-
. 0 B[ I-tan2 -BJ
2 2 2
9. Square and add given equations, we get 2 0
2cos -
~e
l+tan--
4+9+ 12sin(a+P)= 19 2 2

~ BJ
. 1
sm(a.+ P)= - 1-tan--
2 B 7
=Rt.an-
2 [ 28
-
a,+ J3 = 3ou, 150° l+tan 2
But a, + ~ cannot be 30°
0
as if a. 5 30° or ~ s; 30° Lett.an- =x
2
then eq.(i) is not satisfied
3
so a, + J3 = 150° r=R[x-x )
I +x 2
y = 30°
~ = R[(l + x 2 )(1-3x 2 )-(x-x 3 )(2x))
10. Total sum of angles = 81t dx (I+x 2 )2

·: [(n-2)rc] = R [ 1- 2x 2 - 3x 4 - 2x 2 + 2x 4 )
Maximum can be 7n+ (l+x 2 )2
(71t) +x= 8x
= R[-x 4 -4x 2 +l)
sum of remaining three angles can be less than 1t (l+x2)2

www.puucho.com
CP Publication KVPY chapterwise with www.puucho.com

TOPIC• WISE KVPY ISTREAM SX] Questions with Solutions

~=Oatx2 = 4 ±-.JW =2± Js I= 32y 2 -AC 2


dx 2 3 32y 2
at x2 = 2 + J5, ris maximum 64y = 96/ - 3AC2
2

3Ac2=32y
1-x 2 ""5-1
:. cos0= --~ = - -
1 + x~ 2 AC=4%y
12. For a cone solid angles.
Total solid angle full round= 41t
for 1tr2 -+ rr (Because one foUTth of area is
covered)
p
4 • 4 I l
14. cos x-smx= - - - - -
sin2 x cos 2 x

~~~J
( cos x -sm• 2 ) 2
( cos 2x . 2 )
-sm x = -'-------
x
(m = solid angle)
sin x co:/· x
2

4cos2x
'\.. ,/ cos 2x= ,
..... __ .... sin- 2x
ro = n = solid angle cos 2x (I -4 cosec22x) = 0
ro = 2n(l - cosO) cos2x=O
1t
2x = 2mt±
2
T
I 1[
I x=nrt±-
:R
l
4
TT
l ' Atn=O , x= -4
=> cos9 = - 1
2 5rr 3n
=> 9 = 60" I n= I· x= - --
, 4 ' 4
7rr.
R=64~0 n=2 x= -
' 4
6 O"
----
t ::,_.....;,-
~~ 15. sin(11.a)
S!IlO'..
cos().a)
COS O'..
1.- l
cos 60" = __B:_ => e= 6400 By observation
R+£
13. sin().a) cosa- cos (11.a.) sina
A = (A. - 1) sina cosa.
sin(}. - 1)a = ().. - 1) sina cosa.
Clearly i = 1, A= 3 is solution

16.
2

16y 2 + I 6y 2 -AC 2
Al so cos B = --'~----=-------
2.4y.4y .

www.puucho.com
CP Publication KVPY chapterwise with www.puucho.com

• TRIGONOMETRY
From the figure:
18. C(0) = i: cos ne
--
smv= - l&.
"(l
smn= -1 n• 0
n!
2r r
3 ·x- (2f:l) + (2a) X 3 = 360° cos 9 cos 20 cos39 cosn0)
un +--+--+--+ ... + - -
= l~l
8 +ex= 60" · n 1! 2! 3! n!
' 1 1 1 1
Now, cos(0 +a.)= - C(O)= l+ -+-+-+ ... uptoootem1
. ' ' 2 l! -:!! 3!
. 9. since.
=> cos9. coscx - sm .
= -I =e
2 l I 1
C(1t) = 1- -+---+ ... up to co term
=> ~ ~1- 1 - _!_..!. =_!_ =e-1
I! 2! 3!
f-41 r 2r r 22
Clearly C(O). C(n) = 1
=> ~4r 2 -l ~r 2 -1-l=r 1
=> (4r - 1) (r2 -1) = (r2 + 1) 2 C(O) . C(rt) = c + - > 2
e
=> 4r4 - 5r2 + 1= r'\ + 2r +l
C'(e) =- sin8 _ 2 sin28 _ 3sin36 + ...
-,'=> 31"4 = 7r2
1! 2! 3!
2 7
=>r = - ... up to oo term
3
And that value is equal to zero at 0 = 0
=>r=~ 19. cos (sin x) = sin (cos x)

17,
::::::, sin ( "i ± sin x) =sin (cos x)
T~x, => COS X Ilrt = + (- t (1+sin1 X) ' n E I

!~I+--- y ------+l
=> cos x ± sin x = nrt + (-1)'1.::, n e I
2
Clearly A£ LHS E [ -.fi.,.fi_ ], and it does not satisfies
x2 -y2= 144 RHS
(x -y)(x + y) = 144, X, y E l So No solution possible
So factorsize 144 into two even factors
X + y=72 X + y= 36 X + y = 18
20.
x-y=2 x-y=9 x-y=8
x=37 x=20 x= 13

:1, ~l:¾
y=35 y=16 y=5 D-

! ti\
i--35--i
~ 1
~
¾,, !
l 6 ----i>I
~
i---5 - - i
h

r=Ms r=Ms r=!:JJs C B 50m A


l/2x12x35 I/2x12xl6 DE=BC=r
r=-,-----..,... r=--,-----..,... r=2
h
(12+3;+37) (16+220+12) tan30°= -
50
12x35
r:=:--
84
12x16
r=--
48 Ih = Ei
r= 5 r=4 r
h +-
tan 60 • -_ -
50-r

www.puucho.com
CP Publication KVPY chapterwise with www.puucho.com

TOPIC• WISE KVPY [STREAM SXJ Questions with Solutions

=> .fi (50- r) = h + r 23. 6 sin9 - 2 sin 30 - 7 sin 0 + .fi cos 0 = 0


r;; 50
=>·<JJ (50-r) = -+ r ../3 cos 9 - sin O=2 sin 39
-./3 Jt can be written as
;:::> 3(50 - r) = 50 + ,J3 r 2 (sin (60° - 8)) =2 sin 39
=> 100 = (3 +.J3 )r sin (60° - 0) = sin 30
100 60° = 48
;:::>f= - -
3+.J-i' => 0 = 15° is one of the value

=>r= 100(3-./3) =50(1--1) I 1 1


.. 6 ..fj 24. ----+
cos O°cos 1°
+ - - - - + ...
cos 2° cos 3°
cos l"cos 2°
21. I
cos 44° cos 45°
multiply & divided by sin I 0

-I- [ - -sinl
- - + - -sinl
- - + ... - -sinl
0 0 0
- - -]
sinl" cosO°cosl 0 0 cosl cos2° cos44°cos45°
_l_[ sin(l -0°) + sin(2 -1)" + ... sin( 45°-44°)
0
l
sinl 0 cosO°cos1° cosl 0 cos2° cos44°cos45" j
I
"'":""""'10""' [tanl O -tanOO + tan2° - tanl 0 + ... tan4S' - tan44°]
SID

CF= r(l - cos9 sec qi) 1


= -.- [tan 45°]
EC= r(simp- cosG tan¢,) s101°
CD = r cos0 tan <Ii
EC+CD=ED - -1- - = 57.2987
0.0174524
• A r sin 0
rsm.,,= - - Integral part= 57
2
..,_
,,-sm. -i(sin0)
-- 25. Squaring sin29 + cos 20 + 2 sin 29 ~ sin 2 20
2 .:::;:, sin2 28 - sin29 -1 = 0

22. only possible when sin 3x = 1 & sin 7x = 1


sin 3x = 1
sin20 = - ( ~- I), sin29 ~+ 1 (reject)
sin 3x = sin (4n + 1) ~, n e I two solution exist between
2 -7t -JI: 1t
7C 7t - to - & - to O (2 solution)
3x=(4n+ 1)- ::;;>x=(4n+ 1)-· 2 4 4
2 6
sin 7x = sin(4m + 1)~, me I
2
l't
x=(4m+ 1)-
14
for common solution
1t n
(4n + 1)- = (4m + 1) -
6 . 14
Solving these I = 3m - 7n
First solution is m = 5, n = 2
Second solution ism= 12, n = 5
So two solutions are possible

www.puucho.com
CP Publication KVPY chapterwise with www.puucho.com

~ ~ ,.
.•.

2 ;':.
t;u·-_--

..
,;: ·
Algebra~~-
·a
ff ~--~·f<.:· ~ .\.

1. Let A denote the matrix ( 0i O , where i2 =- 1, i) 5. Three vertices are chosen randomly from the
seven verti~es of a regular 7-sidcd polygon.
The probability that they form the vertices of

and let I denote the identity matrix ( ~ ~)-


an isosceles triangle is- (2010]

(A) -1 (B) -l (C) -3 ~


(D -
Then l +A+ A2 + ... + A2010 is- (2010] 7 3 7 S

(A) (~ ~) (B) (~ ~)
6. How many six-digit numbers are there in which
no digit is repeated, even digits appear at even

~)t ~) (D) ( ~I ~1) places, odd digits appear at odd places and the
number is divisible by 4?
(A) 3600 (B) 2700 (C) 2160 ~ O
(20101

2. Suppose the sides of a triangle form a 7. The number of natural numbers n in the interval
geometric progression with common ratio r. [1005, 2010] for which the polynomial
Then r lies in the interval- [2010] l + x + x2 + x3 + .... xn-1 divides the polynomial

(A>(o. - 1;,'5]
l +x 2 +x 3 +x 4 + .... +x 2010 ~
• 12 010]
(A) 0 (B) 100 ,59,:;m (D) 1006

Let 8-0 = 0 and a:i = 3an- 1 + I for n ~ 1. Then


(1\,/s, 2+2,/s]
(B)
8.
the remainder obtained dividing a201o by 11 is-
12010]

y{-•;,ls. 1\,/s] ~ (B)7 (C)3 (D)4

2\,/s ..,)
(D) (
9. Arrange the expansion of (x 112 + 2), 4 J in

decreasing powers ofx. Suppose the coefficient


of the first three terms form an arithmetic
3. The number of rectangles that ~an be o~ained progressioIL Then the number of terms in the
by joining four of the twelve vertice of a expansion having integer powers ofx is- {2010]
12-sidcd regular polygon is- 2010} (A) 1 (8) 2
~ (D) more than 3
(A) 66
(C)24 y~s
(B) 30 I
1
10. Let r be a real number and n e N be such that the
polynomial 2x2 + 2x + 1 divides the polynomial
4. Let 1, oo and ol be the cube roots of unity. 'The
(x + tt-r. Then (n, r) can be- 120101
least possible degree of a polynomial, with rJa1
coefficients, having 2fil, 3 + 4co, 3 + 4oo2 and
5 - w- 002 as roots is- . / I20 IOI
.,.~oo. 4 100°) ~f(ooo, ,!oo )
4

(A)4
(C) 6
JS)5
(D) 8
(c(lOOO
) 4 • 1)
411JOO (D) (4000, - 1- )
4000

www.puucho.com
CP Publication KVPY chapterwise with www.puucho.com

TOPIC-WISE KVPY [STREAM SX] Questions with Solutions


11. Suppose m, n are positive integers such that 17. Thesumof(l 2 -l+l)(ll)+(2 2 -2+1)(2!)+
6m + 2m+n 3"' + 2° = 332. The value of the .... + (n 2 - 11 ·!· l)(n!) is - 12011]
e(Axp)ression m(B2 )+ mn + n,((2CCisl,,-l· .,,.,. (D) [20101 (A) (n + 2)! .,,(Bf{n - l)((n + 1)!) + 1
7 . 13 r-/ g 21 (C) (n + 2)! - l (D) n((n + I)!)- 1

18. Let Xbc a nonempty set and let P(X) denote the
12. Suppose log,b + logi,a = c. The smallest
collection of all subsets ofX. Define
possible integer value of c for all a, b > 1 is -
1, if x e A
[2011) f: xx P(X) -> R by f(x, A)_, { 0, if Xf~ A
(A)4 (B) 3 (D) 1
Thenf(x, A v B) equals - (20111
13. Suppose n is a natural number such that (A)J(x, A)+ j(x, B)
(B).0,r, A)+ f(x, B)- l
Ii+ 2P + 3i 3 + ... + nin I= 18-/2, where i is the·
l9}1Cx, A)+ J,-.:, B)-f(x, A)f(x, B)
square root of- l. Then n is - [2011]
(A)9 (B) 18 yo" (D) 72
19.
(D)f(x, A)+ lf(x, A)-f(x, B)I

Let A and B be any two n x n matrices such


14. Let P be an m x ,n mattix such that p2 = P. that the following conditions hold : AB = BA
Then (1 + P/ equals - [2011] , and there exist positive integers k and P. such
(A)'/+ P (B) I+ nP that Ak = I (the identity matrix) and Bt"' 0 (the
(C) J + znp (f;l){+ (2" -1 )P zero matrix). Then- (2011J
,.,/ (A) A+ B =l
15. Consider the cubic equation x3 + ax2 +bx+ c"" 0, .)BJdct (AB)"' 0
where a, b, c are real numbers. Which of the (C) <let (A+ B) '# 0
(D) (A+ Br "' 0 for some integer m
following statements is correct? [2011)
(l}Ylf a 2 - 2b < 0, then the equation has one
20. The minimum value of n for which
/ real and two imaginary roots
22 +42 +62 + .... +(2n)2 I0
(B) If a 2 - 2b 2: 0, then the equation has all real 2 2 z. z< . l f2011 I
l +3 +5 T .... + (2n - })
roots
(A) is 101 (B) is 121
(C) If a2 - 2b > 0, then the equation has all real
~s151 (D) docs not exist
and distiact roots
0,
(D) If 4a3 - 27b 2 > then the equation has real
21. TI1e arithmetic me.an and the geometric mean of
and distiact roots two distinct 2-digit numbers x and y are two
integers one of which can be obtained by
16. Two players play the following game : A writes reversing the digits of the other (in base I 0
3, 5, 6 on three different cards ; B writes 8, 9, representation). TI1en x + y equals- 12011]
10 on three different cards. Both draw
randomly two cards from their collections.
(A) 82 (B) 116 730 (D) 148

Then A computes the product of two numbers 22. Three children, each accompanied by a
guardian, seek admission in a school. The
he/she has drawn, and B computes the sum of
principal want to interview all the 6 persons
two numbers he/she has drawn. The player '
one after the other subject to the condition that
getting the larger number wins. What is the
no child is interviewed before its guardian. ln
probability that A wins? [2011)
how many ways can this be done I2012J
5 /4 (A) 60 (B)'~O (C) 120 (D) 180
(n) - EC) - (D) J_
9 / 9 9 J

www.puucho.com
CP Publication KVPY chapterwise with www.puucho.com

t ALGEBRA
23. In the real number system, the equation 29. The number of ordered pairs (m, n), where
m, ll E {l, 2, J, ...... ,50}, SUCh that 6m + 9n is
~x+3-4-h-l +~x+S-6.Jx-l =lhas [20121
a multiple of 5 is [2012)
(A) No solution A
v,1;[50 (B) 2500 (C) 625 (D) 500
(B) Exactly two distinct solutions
(C) Exactly four distinct solutions
_J') Infinitely may solutions
30. Suppose ai, a::, a3, ....... , a2012 are integers
arranged on a irclc. Each number is equal to
the average of its two adjacent numbers. If the
24. The maximum value M of 3" + 5x - 9~ + 1Sx - 25\
sum of all even indexed nwnbers is 3018, what
as x varies over reals, satisfies (2012]
is the sum of all numbers ? f2012J
(A) 3 < M < 5 ,iJ3)·0< M < 2
(A) 0 (B) 1509 (C) 3018 JD)-'6036
(C) 9 < M < 25 (D) 5 < M < 9

31. Let S = {1, 2, 3, .. , n} and


25. Let a, b, c, d be numbers in the set {I, 2, 3, 4, 5, 61
such that the curves y = 2x3 + ax + b and A = {(a, b) 11 ~ a, b ~ n} = S x S. A subset B of
y = 2x3 +ex+ d have no point in common. The A is said to be a good subset if (x., x) e B for
maximum possible value nf (a - c)2 + b - dis every x ES. Then the number of good subsets
[2012] of A is (20121
(A)O (C) 30 (D)36 (A) 1 (,12 11" (C) 2o(1t--ll (D) 2" 2

26. Let n be a natural number and let 'a' be a real 32. Suppose a, b, c arc real numbers, and each of
number. The number of zeros of the equations x2 + 2ax + b2= 0 and x2 + 2bx +
x2n+ 1 - (2n + I) x. +a= 0 in the interval [-1, 11 c2 = 0 has two distinct real roots. Then the
IS I [2012] equation x2 + 2cx. + a2 = 0 has (2012]
(A)2 ifa > 0 (A) Two distinct positive real roots
(B) 2 ifa < 0 (B) Two equal roots
/>
._,,.(e) At most one for every value of a (C) One positive and one negative root
(D) At least three for every value of a ~,£real roots

~ The coefficient of x 2012 in


I\
1+ x
27. Let Sn = 2) denote the sum of the first n
/ -- (l+x 2 )(1-x)
is
k=l
positive integers. The numbers S1, S2. S3, ... S99 (20121
are written on 99 cards. The probability of (A) 2010 (B) 201 l (C) 2012 (D) 2013
drawing a card with an even number written on
34. A man tosses a coin lQ times, scoring 1 point
it is (2012]
for each head and 2 points for each tail. Let
(A) 1/2 (B) 49/100
P(K) be the probability of scoring at least K
~99 (D) 48/99
points. The largest value of K such that
28. A purse contains 4 copper coins and 3 silver P(K) > ½ is [2012]
coins. A second purse contains 6 copper coins (A) 14 j..l3)'15 (C) 16 (D) 17
and 4 silver coins. A purse is chosen randomly
and a coin is taken out of it. What is the 35. The ~um of non-real roots of the polynomial
pr~6ility that it is a copper coin [20121 equation x3 + 3x.2 ·1· 3x + 3 = 0. (20131
.(,A) 41/70 (B) 31/70 (A) equals 0
(C) 27/70 (D) 1/3 (B) lies between O and 1
Jy)"ties between -1 and 0
'(D) has absolute value bigger than 1

www.puucho.com
CP Publication KVPY chapterwise with www.puucho.com

TOPIC• WlSE KVPY {STREAM SX] Questions with Solutions


36. Let n be a pos1t1ve integer such that 41. For an integer n let So {n + 1, n + 2, ...., n + 18}.
log,ilogilogilogilogi(n) < 0 < log2logilogzlog.(n). Which of the following is true for all n z 10?
Let .f! be the number of digits in the binary .(20131
expansion of n. Then the minimum and the (A) S 0 has a multiple of 19
ma~um possible values of/! arc- (2013] (B) Sn has a prime
J)iJ 5 and 16 (B) 5 and 17 (C) Sn has at least four multi pies of 5
(C) 4 and 16 (D) 4 and 17
~ h a s at most six. primes

37. Let ro be a cube root of unity not equal to L


42. Let a, b, c, d ·· be real numbers such that
Then the maximum possible value of
n
la+ b[i) + col Iwhere a, b, c E {+l,-1} is- I,(ak 3 +bk 2 +ck+d)= n4 , for every natural
[2013] k=I
(A) 0 ~ number n. Then lal + lbl +[cl+ ldl is equal to
(C) ..J3 (D) 1 + -./3 (2013}
,st>,rf5 (B) 16 (C) 31 (D) 32
38. A box. contains coupons labeled l, 2, 3, ..... , n.
A coupon is picked at random and the number x x2 x3 x.4
x is noted. The coupon is put back into the box 43. Let f(x) = !+-+-+-+-. The number
and a new coupon is picked at random. The
1! 2! 3! 4!
new number is y. Then the probability that one ofreal roots of f(x) = 0 is (2013]
of the numbers x, y divides the other is (in the ~ (B) 1 (C)2 (D) 4
options below [r] denotes the largest integer
less than or equal tor) [2013] 44. A box contains coupons labelled 1, 2, ..... , 100.

(A)!
2
(B) ~t[~J
n k=l k
Five coupons are picked at random one after
another without replacement. Let the numbers
on the coupons be x 1, x 2 , •••• , x 5 • What is the
l 21 £..,; -
(C) --+
n n
~[n] k 7
I 2
m/--+2
n n
:2: .!!.k
n [
l::=t
] probability that x. 1 > x2 > x3 and X3 < "4 < X5?
k=I [20131
(A) 1/120 (B) 1/60
39. Let n <:: 3. A list of numbCIS O< x1 < x2 < ... < x.,_ (S,JP-d (D) 1/10
has mean µ and standard deviation cr. A new
list of numbers is made as follows : }'1 = 0, 45. In a tournament with five teams, each team
Y2 = X2, •••.• ,y,,....1 = Xn-1, Yn = xi + Xo, The mean plays against every other team exactly once.
and the standard deviation of the new list arc µ Each game is won by one of the playing teams
and & . Which of the following is necessarily and the winning team scores one point, \vhile
truy A A 12013] the losing team scores zero. Whlch of the
._;A') µ = µ , cr s; cr following is NOT necessarily true? (2013]
(B) µ = µ, cr <:: cr (A) There are at least two teams which have at
(C) cr cr most two points each

(D) µmayor may not be equal to µ (B) There arc at least two teams which have at
least two points each
40. The number of integers n with 100 s; n s; 999 (C) There are at most three teams which have at
and containing at most two distinct digits is- least three points each
/ ~o~ (D) ~re are at most four teams which llav~ at
~ef 252 (B) 280 (C) 324 (D) 360 cJ"most two points each

www.puucho.com
CP Publication KVPY chapterwise with www.puucho.com

t ALGEBRA
46. Let Co be a circle of radius l. For n ;:;: 1, let C0 51. Let S = {(a, b) : a, b e Z, 0 s a, b S: 18}. The
be a circle whose area equals the area of a number of elements (x:, y) in S such that

square inscribed in Ca- 1• Then L:0 A.rca (C1) 3x + 4y + 5 is divisible by 19 is


(A) 38 ~ 9 (C) 18 (D) 1
[2014]

equals [2014]
2
(B) 1t - 2 (C) _1 52. For a real nwnber r let [rJ denote the largest
(A)1t2 (D).;l--
7t2 rt 2 /rt-2 integer less th, · or equal to r. Let a > 1 be a
./ real number which is not an integer and let k be
47. For _a real number r \Ve denote by [r] the largest the smallest positive integer such that
integer less than or equal to r. If x, y are real [/] > [a]'. Then which of the following
numbe~ with x, y ~ 1 then which of the statements is always true? [2014]
following statements is always true'? 120141 (A) k S: 2 ([a]+ 1)2 y,k s ([a]+ 1)4
(A) [x + y] s [x] + [y] (B) [xy] s [x] [y] (C) ks zluJ+l (D) k $'. - 1- +1
(C) [2x] S 2[x] y{;] S ~:~
a - [a]

53, Let X be a set of 5 elements. TI1e number d of


ordered pairs (A, B) of subsets of X such that
48. For each positive integer n, let
A -1' ~' B* ~, An B =~satisfies (2014]
An = max { (: JIO s rs n}. TI1en the number of (A) 50 :-: ; d :-: ; I 00
(B) 101 sd.s 150
elements n m { 1,2, ... ,20} for which (C)•151 s d s 200
y
(D) 201 :S: d
[2014]

(A)9 (B) IO ../(C)'l I (D) 12 54. Let n :::: 3 be an integer. For a permutation
cr = (a1, a2, ....... , an) of(l, 2, ....... , n) we let
49. Two persons A and B throw a (fair) die (six- fa(x) = a 0 xn- 1 + 8n-1xn-2 + .....+ a2x + a1. Let Scr
faced cube ,0th faces numbered from l to 6) be the sum of the roots of f;,.(x) = 0 and let S
alternately, starling with A. TI1e first person to denote the sum over all permutations cr of
get an outcome different from the previous one
( l, 2, ..... , n) of the numbe~c-- Then - [20141
by the opponent wins. The probability that B
(A) S < n! .....(£; - n! < S < 0
wins is f2014J (C) 0 < S < n! (D) n! < S
(A) .?_
6
(B)"~
../ 7
(C) 7._
. 8
(D) !9
55. If n is a positive integer and co~ I is a cube
roct of unity, the number of possible values of
50. Let n 2. 3. A list of numbers x 1, x 2 , ••. , Xn has
(2014]
meanµ and standard deviation CY. A new list of
numbers Yi, Y:.,· .. , Yn is made as follows:

Y1 =
X 1 +X 2
----'---=- v, =
x 1 +x,
- and y·.1 =x for
(A) 2 (B) 3 fr {D)6
2 ' ." 2 .1

j = 3, 4, ... , n. The mean and the standard 56. Suppose a parabola y : ax 2 + bx + c has two x
intercepts, one positive and one negative, and
deviation of the new list are µ and 6- . TI1cn
its vertex is (2, -2). Then which of the
which of the following is necessarily true?
following is true'? ~ [2014]
[2014]
(A) ab> 0 -~BJ be> 0
(A) µ = µ and CY $'. & yl'(µ
= µ and CY ;;;: 6- (C)ca>O (D)a+b+c>O
(C) cr = cr (D) µ -,. µ

www.puucho.com
CP Publication KVPY chapterwise with www.puucho.com

TOPIC-WISE KVPY [STREAM SX] Questions with Solutions


57. The number of integers n for which 63. There are 6 boxes labelled Bi, B2, •.... , 8 6 . In
3x3 - 25x + n == 0 has three real roots is - (2014( each trial, two fair dice Di. D2 are thrown. IfD 1
(A) l (B) 25 y)'Ss (D) infinite shows j and D2 shows le, then j balls are put
into the box Bk. After n trials, what is the
58. Choose a number n unifonnly at random from probability that B 1 contains at most one ball?

+ 1he set {l, 2, ..... , 100}. Choose one of the first


seven days of the year 2014 at random and
consider n con.o;ccutivc days starting from the
chosen day. What is the probability that among
12015]

the chosen n days, the number of Sundays is


different from the number of Mondays? [2014]
(A) _!_ (B) 3_ (C) g_ (D) 43
2 7 49 175

59. If z is a complex number satisfying


[z3 + z~31S:. 2, then the maximum possible value
of ]z + z- 1! is - [2015]
(B) ifi (C) 2 Jz (D) I 64. Iflo,&Jx- I) (x-2) = log(9 ,.,_6 x+I) (2x2 -10x - 2),
then x equals - (20151
60. The largest perfect square tl1at divides
20143 - 2013 3 + 2012 3 - 2011 3 + .... + i3- i1 (A) 9-.Jl5 ~./ii
is- {20151 {C)2+ ./5 (D) 6- ../5
(A) i2 (B) 2 2
~?2 (0)20142 65. Suppose a, b, c arc positive integers such that
2a + 4h + 8., = 328. T lten -
a+2b+3c . l
- - - 1s equa to-
61. Let p(x) be a polynomial such that abc
p(x) - p'(x) = xn, where n is a positive integer.
[20151
Then p(O) equals - (2015]
~! (B)(n - 1)! (A).!. (B) ~ (C~ (D) ~
2 8 - . / :t4 6
(C) _!_ (D) 1
· n! (n-l)!
66. Let x= (./so+ 7) 113 - ( .Jso -7)1f3. Then-
62. An urn contains marbles of four colours : red,
[20151
white, blue and green. When four marbles are ",.,('A)x=2
(B) x=3
drawn without replacement, the following
(C) xis a rational number, but not an integer
events are equally Likely : (2015]
(D) xis an irrational number
( 1) the selection of four red marbles
(2) the selection of one white and three red
67. Let (I+ x + x 2)2° 14 = ao + a 1x + a2x 2 + a 3x3 + ...
marbles
(3) the selection of one white, one blue and two
.....+ a402gX4021!, and let (20151
A= ao - a3 + 116- .... + a4026,
red marbles
(4) the selection of one marble of each colour B = ai-14 + a1 - ..••••• - a.io21.
The smallest total number of marbles satisfying C = a2-a5+ as - .... + 84o2e,
Then-
the briven co.~tion is
(A) 19 :)6) 21 (C) 46 (D) 69 (A) IAI = ]Bl > ICt (B) IAI = IBI < !Cl
(C) IAI = ICI > JBI ~>'rAI = ICI < !Bl

www.puucho.com
CP Publication KVPY chapterwise with www.puucho.com

."
t ALGEBRA
68. Let Xn = {1, 2, 3, ... , n} and let a subset A of Xn b2
be chosen so that every pair of elements of A (A)~
v/r+b (E) (r + b)2
differ by at least 3. (For example, if n = 5, A
can be 0, {2} or {1,5} among others). When (C) b +1 (D) b(b + l)
n = 10, let the probability that 1 e A be p and r+b+l (r+b)(r+b+l)
let the probability that 2 e A be q. Then - (20151
1 The number 0f noncongruent integer-sided
(A) p > q and p- q = - triangles whose sides belong to tbe set
6
{10, 11, 12, .... , 22} is - (2016]
1
(B) p < q and q -p = -
6
(A) 2J,3 (B) 446 <t"s-
(~048 (D) 449
~>qandp-q= J_
10 74. Suppose we have to cover the xy-plane with
I identical tiles such that no two tiles overlap and
(D) p < q and q - p = -
10 no gap is left between the tiles. Suppose that
we can choose tiles of the following shapes;
69. The remainder when the determinant equilateral triangle, square, regular pentagon,
2014 2014 20152015 2016 2 6 (11
regular hexagon. Then the tiling can be done
20172017 201 g20lB 20192019 is divided by 5 with tiles of - [2016)
(A) all four shapes
20202020 2021 2021 20222022 (B)~actly three of the four shapes
'v
is - [2015] (C) exact! y two of the four shapes
(A) 1 (B) 2 (C) 3 io)"<l (D) exactly one of the four shapes

70. The number of triples (x, y, z) of real numbers 75. The remainder when the polynomial
. x4 + y4 + z4 + 1 = 4 xyz
'fyi ng the equation
sat1s l + x 2 + x4 + x6 + .... + x 22 is divided by
IS [2016] 1 + x + x7 + x3 + .... + x 11 is - [2016]
(A) 0 ..(B(4 (A) 0
(C) 8 [D) more than 8 (8) 2
(C) 1 +x 2 TX. 4
+ .... +x
10

71. For any real number r, let A, = {einm : n is a (D);.2(1 + x2 + x4 + .... + x 1~


.:./ '

natural number} be a set of complex numbers.


Then- (2016] 76. For each positive real number A, let A,. be the set
(A) A1, A 1 , Ao.3 arc all infinite sets of all natural numbers n such that
7! lsin(,,ln+l- sin(~)I <?•. Let Ai be the
(B) A1 is a finite set and Al , Ao.3 are infinite complement of A;, in the set of all natural
-
7! numbers. Then - [2016]
sets (A) A 1 , A 1 , A 2 are all finite sets
(C) At, A 1 , Ao.3 are all finite sets -
2 3
--
5

y)< AQ: arc finite seL<; and A; is an infinite


(B) A I is a finite set but A 1 , A 2 arc infinite
3
-
2 5
:sets
sets
~~-.A!, Ai are all finite sets
72. A box contains b blue bails and r red balls. A ball 2 3 5
is drawn randomly from the box and is rctW11cd (D)A 1 ,A 2 are finite sets and A I is an infinite
to the box with another ball of the same colour. - - -
3 5 2
The probability that the second ball drawn from set
the box is blue is - [2016]

www.puucho.com
CP Publication KVPY chapterwise with www.puucho.com

TOPIC-WISE KVPY {STREAM SX] Questions with Solutions

77. Three schools send 2, 4 and 6 students, 82. Suppose p, q, r arc real numbers such that
respectively, to a summer camp. The 12 q = p (4 -p), r = q (4 - q), p = r (4- r).
srudents must be accommodated in 6 rooms The maximum possible value ofp + q + r is
numbered 1, 2, 3, 4, 5, 6 in such a way that [5 Nov, 20171
(A) 0 (B) 3
each room has exactly 2 students and both from
the same school. The number of ways, the
$)Y (D) 27

students can be accommodated in the rooms is - 83. A shooter can hit a given target with
[2016]
probability _!__. She keeps firing a bullet at the
(A) 60 (B) 45 4
rOO (D)2700 1arget until she hits it successfully three times
and then she stops firing. The probability that
78. Let a be a fixed non-zero complex number with she foes exactly six bullets lies in the interval
[5 Nov, 2017)
I a I < 1 and w === ( -z-aJ . a
- _ - , where z 1s (A) (0.5272, 0.5274) (B) (0.2636, 0.2638)
1-az (C) (0.1317, 0.1319) ,~,)'(o.0658, 0.0660)
complex number. Then [2016]
(A) there exists a complex number z with 84. Consider the following events :
I z I < 1 such that ] w I > 1 E 1 : Six fair dice are rolled and at least one die
_ ~~~v I > I for all z such that I z I < 1 shows six.
~ Iw I < 1 for all z such that I z I < I E2 : Twelve fair dice are rolled and at least t\vo
(D) there exists z such with I z I< 1 and I w I=== I dice show six.
Let p1 be the probability of E 1 and p2 be the
79. Ten ants are on the real line. At time t = 0, the probability of E 2• Vv'hich of the following is
k-th ant starts at the point k2 and travelling at tr~i~ [S Nov, 20171
~ P i > P2 (B) PI= p2= 0.6651
uniform speed, reaches the point ( 11 - kl at
(C) P1 < P2 lD) Pt= P-1 = 0.3349
time t === 1. The number of distinct times at
which at least two ants are at the same location
85. For how rna.ny different values of a does the
is (2016]
following system have at least two distinct
(B) 11 solutions?
(D) 9 ax +y = 0
X +(a+ JO))'= 0 [5 Nov, 20171
The number of solution pairs (x, y) of the (A) 0 (B) 1
80.
(C) Jr-s (D) Infinitely many
simultaneous equations
log113 (x + y) + log3 (x - y) = 2 and 2/ = 5 IY+t
;r
is [5 Nov, 20171
86. Let a, b, c, cl, c, be real numbers such that
(A) 0 ~ a + b < c + d, b + c < d + e, c + d < e + a,
d+ z:,a+ b. Then [5 Nov, 2017J
(C) 2 (D) 3
JM The largest is a and the smallest is b
(B) The largest is a and the smallest is c
81. The fractional part of a real number x is x -[ x ],
(C) The largest is c and the smallest is e
,vhere Ix] is the greatest integer less than or (D) The largest is c and the smallest is b
equal !Ox. Let F 1 and F 2 be the fractional parts
87. If a fair coin is tossed 5 times, the probability
of (44 ~ .J2017 )w17 and (44 + ,J2017 )~017 that heads does not occur two or more times in
respectively. Then F1 + F 2 lies between the a row is [5 Nov, 20171
numbers (5 Nov, 2017] (A)~
(B) 0.45 and 0.9 25
(~:~d0.45
(9'1.9 and 1.35 (D) 1.35 and 1.8 (C)~
zs

www.puucho.com
CP Publication KVPY chapterwise with www.puucho.com f •

• ALGEBRA

. 1 fi . 92. Let
88. t+-;-' Ofl 1,2, ....... , 20. Put
1 a2+62+c2 }
S:= { ----:a,b,ceR,ab+bc+ca;t;Q ,
I ab+ be+ ca
p= - (a1 + a2 + ....... + a20) and
20 where R is the set of real numbers. Then S
119 Nov, 2017)
q = _I
20
(_!_+-
a1
1 + ...... +-1-)'- Then
a2 a 20
equals
(A) (---oo, - 1] u [I, OCl)
(B) (---oo, 0) u (0, oo)
{5 Nov, 20171 (C) ( - w, -1] V [2, cc)

~ (o.222~P)
,_j)ifJ(-oo,-2] U [1, co)

93. Let S be the infinite sum given by


(B)qe ~ , ( 22-p 2(22-p)) 21 S- ~ an
- L.., 10 2n

(C)q E (
2(22-_p)
2}
22-p)
,-7
n~o
where {a.,}""° is a sequence defined by a-0 = ar =I
and aj = 20aH -108 aj--2 for j ~ 2.

(D) E (22-p 4(22-p)) If S is expressed in the form ~, where a, bare


q 7 ' 21 b
coprime positive integers, then a equals
[19 Nov, 20171
89. Let x,y,z be pos1t1ve integers such that
(A) 2017 (B) 2020
HCF (x,y,z) = I andx2 + y2=2z 2. Which of the (C) 2023 (8)-2025
following statements are true?
I. 4 divides x or 4 divides y. 16x 2 -96x+ 153
II. 3 divides x + y or 3 divides x -y. 94. Define a function f (x) = _ for
x-3
Ill. 5 divides z(x2 - / ) !5 Nov, 2017) all real x * 3. The least positive value off(x) is
(A) I and II only J8J
II and lil only [19 Nov, 20171
(C) II only (D) III onJy (A) 16 (B) 18
(C) 22 (~-24
. 2 4 6 34
90. The quotient when 1 + x + x + x + ...... + x 95. Let n > 2 be an integer and define a polynomial
·
1S ct·lVl·ctedb y l + X + X1 + X3 + ...... + ·s
X 17 I p(x) = xn + aa- 1 xn-- 1 + ...... + a1x + ao
[5 Nov, 20171 where .1-0, ai, ........ an-- 1 are integers. Suppose we
know that np(x) = (1 + x)p'(x). lfb = p(l), then
(A) x 17 -x 15 +x 13 -x 11 •••••• +x
[19 Nov, 20171
(B) x11+x15+xn+x11+ ...... +x (A) bis <livisfolc by l 0
( C) x 17 +x16 +x 15 +x 14 + ...... + 1 (B) b)s divisible by 3
~1 -xl6 +xl5 _xt4 + ...... -1 _(i;.)'b is a power of2
tb) bis a power of 5

91. Among the inequalities below, which ones are


96. The number of 5-tuples (a, b, c, d, e) of positive
true for all natural numbers n greater than I 000 ?
integers such that
[19 Nov, 2017) I. a,b,c,d,e are the measures of angles of a
I. n ! s nn convex pentagon in degrees ;
II. (n!) 2 :::;; nn 11. a~b5:c5:d5e;
Ill. IOn:::;; n! Ill a, b, c, d, e are in arithmetic progression is
/ [19 Nov, 2017]
IV. nn :s; (2n)! (A) 35 .....(8) 36
(A) I and IV only g}Y( III and IV only (C) 37 , (D) 126
(C) ll and IV only (D) I, II, III and IV

www.puucho.com
CP Publication KVPY chapterwise with www.puucho.com

TOPIC~ WISE KVPY [STREAM SX] Questions with Solutions


97. • Thirty two persons Xi, X2, ..... ,X32 are [' 2017 2]
randomly seated around a circular table at equal 102. Let A- 1 = 1 2 0 11 4 . Then 12AI - 12A-11is
intervals. Two persons Xi and Xj are said to be 1 2 0 18 8
within earshot of each other if there are at most
three persons between them on the minor arc eau!,\.lto 119 Nov, 20171
joining X 1 and Xj- The probability that X1 and (B)-3
--{~3_,
X2 are within earshot of each other is, £C)12 (D)-12
fl9 Nov, 20171
(Here(;}= (n _n;) !r!) 103. Let z 1, z 2, ...... ,z7 be the vertices of a regular
heptagon that is inscribed in the unit circle with
(3;}0! centre at the origin in the complex plane. Let
w= :~::Z zj, then lwl is eq-ual to
1
(B) 4(32!) l:Si<j:S7

4 (19 Nov, 20171


(D) 31 JA')O (B) 1
(C) 2 (D)3
98. Let n be the smallest positive integer such that
104. Let m, n be two distinct integers chosen
t+.!.+!+ ....... +!~4- randomly from the set { 0, I, 2, ..... ,99}. Then
2 3 n the probability that 4111 -1- 4 11 + 3 is divisible by
Which one of the following statements is true? 5 lies in the interval (19 Nov, 2017)
/ [19 Nov, 20171 {b,)'(O, 0.25} (B) (0.25, 0.5J
!){)20 <nS: 60 (B) 60 < n S: 80 [C) (0.5, 0.75] (D) (0.75,l)
(C) 80 < n ~ 100 (D) 100 < n :s; 120
105. Let A be the set of all permutations a1, a2, .... ,a6
99. A pair of 12-sided fair dice with faces of I, 2, .... , 6 such that ai. a2•..•. ak is not a
numbered 1,2,3, ..... , 12 is rolled. The permutation of 1, 2, .... , k for any k. 1 ~ k ~ 5.
probability that the sum of the numbers Then the number of elements in A is
appearing has remainder 2 when divided by 9 is (19 Nov, 20171
(19 Nov, 20171 (A) 192 (B) 408 (C) 312 y1-528
(A) J_ (B) 2_
72 48 106. Let f(x) be a polynomial with integer
coefficients satisfying f(l) = 5 and f(2) = 7.
(C) _!_!_ (D)~ The smallest possible positive value off( 12) is
144 7 9 119 Nov,2017)
(A) 5 (B)7
100. Let Xi. x:!, ...... , x 6 be the roots of the yr21 (D) 15
polynomial equation x 6 + 2x5 + 4x 4 + SxJ +
107. _Suppose four balls labelled I, 2, 3, 4 are
16x2 + 32x + 64= 0. Then (19 Nov, 2017]
randomly placed in boxes Bi. B2, BJ, B4. The
(A) [xd = 2 for exactly one value ofi
(D) ~ = 2 for exactly two values ofi probability that exactly one box is empty is
GGJ1xd =2 for all values of i [19 Nov, 2017}
(D) lxil = 2 for no value of i
(A) 2!6 03(.96
101. In the complex plane, let z 1 = ./3 +i and
(C) 27 (D) _!_
z2 = ../3 -
i be two adjacent vertices of an 256 64
n-sided regular polygon centered at origin
Then nequals / [19 Nov, 2017]
(A) 4 (Bj'6
(C) 8 (D) 12

www.puucho.com
CP Publication KVPY chapterwise with www.puucho.com

t ALGEBRA

ANSWERS
1. (C) 2. (C) 3. (D) 4.(B) 5. (D) 6. (D) 7.(C)
8. (A) 9. (C) 10. (B) 11. (C) 12. (C) 13.(C) 14. (D)
15. (A) 16. (C) 17. (B) 18. (C) 19. (B) 20. (C) 21. (C)
22. (B) 23. (D) 24. (B) 25. (B) 26. (C) 27. (C) 28. (A)
29. (A) 30. (D) 31. (8) 32. (D) 33., (*) 34. (B) 35. (C)
36. (A) 37. (B) 38. (D) 39. (A) 40. (A) 41. (D) 42. {A)
43. (A) 44. (C) 45. (D) 46. (D) 47. (D) 48. (C) 49. (B)
50. (B) 51. (B) 52. (D) 53.(C) 54. (B) 55. (C) 56. (B)
57.(C) 58. (*) 59. (A) 60.(C) 61. (A) 62. (B) 63. (D)
64. (B) 65. (C) 66. (A) 67. (D) 68. (C) 69. (D) 70. (B)
71. (D) 72. (A) 73. (C) 74. (B) 75. (D) 76. (C) 77. (C)
78.(C) 79. (C) 80. (B) 81.(C) 82. (C) 83. (D) 84. (A)
85. (C) 86. {A) 87. (B) 88. (A) 89. (B) 90. (D) 91. (B)
92. (D) 93. (D) 94. (D) 95. (C) 96. (B) 97. (C) 98. (A)
99. (D) 100. (C) 101. (B) 102. (C) 103. (A) 104. (A) 105. (D)
106.(C) 107. (B)

e SOLUTIONS

[oi Oi] '· [-1 ::as - J

1. A= A2 =
0
o] ·
-I '
rE [ 1--JS ' 1+2..J5)
2 ... (1)

A3 = lO - i]
-i O '
· I= [I OJ
0 l
ar2 ·! ar > a
r2 + r - 1 > 0

I+A+A2 +A3 = [~ ~l A4 = [~ ~] =I
1
-1+-E
r> - - , r < - -
2 ~
-1-../s
2
... (2)

ar- + a > ar ; r· ~ r + 1 > 0 always trnc


:. I+A+A2+Al+ ..... A2010
Solving (1) & (2)
=(I +A+ A2 + A3)+ A4(1 +A+ A2 +A3) +...
... + A2oos(I +A+ A2) rE[-./5-!
2 '
.f5+1)
2
= G ~)
2. 3. Number of dfagonals passing through centre= 6
Number of rectangles= 6C2 = 15

6
a+ ar > ar2
ar2
4.
a
2

[3
6=5-(w+ ci/)=5-(-1)=6
~
roots --t 2(1) , 3 + 4ro, 3 + 4ro·, 5 - co- co·
y o
If a = 2Cil is a root then 2w has to be a root too.
?

r2 -r-l<O total-)- mitL 5 roots, hence min. degree-)- 5

www.puucho.com
CP Publication KVPY chapterwise with www.puucho.com

TOPIC-WISE KVPY [STREAM SX] Questions with Solutions

s.

G®B
A 9.

1\, T2, T1 4 AP
2 ncl _o,.., nc2
F C
---Lo+-~
2 22

D -E n-1 = n(n-1)
........'-----.0... ~ n= 8
MGB, 6.AFC & MED are isosceles 8
16-3r
P=
7 C x3
I
7x3
=--- 3 - - = Integers
4
k-= - -
0, 4, ~
. 7 C1 7x6x5 5
3x2
10. 2x2 + Zx + 1 =O
6. -l+i -1-i
x=-~ ~~

2 ' 2
I3ways I 3ways I 4ways I 4ways I Sways I2ways I x satisfies (x + 1)'1-r =O
(1,3,5,7,9) (2,6)
=3 X 3 X 4 X 4 X 5 X 2 = 1440 ( --l±i
2-+t
)n -r=O
7. 1 + x2 + x 4 + ..... x2010

1(1- x2m2)
l-x 2
= (l- x1006 )(I+ x•006)
(1-x)(l + x)

= (l+x1oor,/ (l-xs01))[(l+xsoo))
l
(1- x) (l + x)
1006,,_(1
= (1 +x , +x+x + .... xS02)
2
RHS =reai
(1 - x + x2 - x3 + .... x502) LHS = real only when n = multiply of 4
n=4000
this is divisible by I + x + x2 + ..... xn-- 1
ifn-1 = 502
r= ( Jz)4000 - -41000-
In= 5031
8. an= 3an-l + 1 11. 6m + zm~n 3w + 2° = 332
a2010 = 3a2009 + I
maximum possible value ofm is 3
= 3(3awos+ I)+ l = 32 a200s + 3 + I checking form= 3, 2 and 1
= 33 a2001 + 3 + 3 + 1 we get m = 2, n = 3, w = 2
m 2 +mn+n 2 =4+6 +9= 19

J2010 ao+ (3+3+ ...3) + 1 12.


'-------.r-----
2009 timei,

""0+ 6027+ 1
=6028 13. S=i+2i 2 +3i 3 + ..... +ni"
·s = 1•2 + z·1t + ..... + (n- l)•n1 +ru•n+I
i.

Remainder ( 6~~ 8) = 0 S(l - I') =!+l


• •2 +t•3 +..... +!•n -m· n+l

www.puucho.com
CP Publication KVPY chapterwise with www.puucho.com

e ALGEBRA
S(l-i) i(l-in) -niu+l I if xeAuB
18. f(x,A uB)= { 0 if
1-i xiAuB
1-in nin+i
. => S=----- if xeA, xeB}
-2i 1-i if xeA, x 1B
-!, ,J. if x!i!!!A, x eB
z1(say) z2(say) => f{x, A u B) = 1
None of the option (A,B,D) satisfy
I z1 l = ~ orO; [z2I = ';;; =!!....fi
...,2 v2 2 ifx Ji!!! A, x Ii!!! B
=> f(x, A uB) = 0
!!.. ... 18 => n =36
2 => C(only C satisfy)

14, p2 ... p Ak =I, Bt = 0 (det (B) =O)


19.
p-lp2 = p-1 p
=> det (AB) = 0
P=I
(I -:1- Pt= (2Pt =2npn
X
=211P 2D.
2n(2n + 1) (4n + I) < l.Ol
= P+ (211 - 1) P
6 -x
=I+ (211 - l) P
2.0lx < (l.Ol) 2u(2n + 1)(4,i+ 1)
15. f(x) =x3 + ax.2 + bx+ c 6
f' (x) =3x2 + 2ax + b
2 ,0l. 4n(n + 1)(2n + 1) < (1.0l) 2n(2,r +1)(4n + 1)
D =4a2 -4. 3. b =4 (a2 -3b) 6 6
1fa2 <2b => a2 < 3b => D < 0 ·

uflx)
2.01 4n +1
=> f '{x) =0 has non real roots - < - - => n> 150.5
1.01 211+2

x+y
_ _ _ _ _ _:=;..._ _ _ x 21. - 2 -=lOa+b, (a,beN)

..Jxy =IOb+a
Hence f(x) =0 has 1 real and two imaginary xy = (I Ob+ a) 2
roots
(x-y)2=4 (I la+ lib) (9a-9b)
=4. ll .(a+b).9(a-b)
16. For A to win, A can draw either 3, 6 or 5, 6. If
A draws 3, 6 then B can draw only 8 & 9 ::::) a + b = 11, a - b = I
1 1 1 =>a=6,b=5
Prob.= - . -=- ((x -y)2 is perfect square ofan integer)
3 3 9
If A draws 5, 6 then B can draw, any two x+y= 130
1 1
Prob. = - . 1 = -
3 3 22.
Prob. = .!. + .!. =
9 3 9
i
17, Tn=(n2 -n+ l)n! 23. ~x+3-4~ +~x+S-6~ =l;x~l
= (n2 - l) n! -(n-2) n! ~(x-l)-2x2.Jx-l +4 +~(x-1)-6.Jx-l +9 == l
Tn=(n-1) (n+ 1) !-(n-2)nl
Sum= 1 + (n- l)(n + 1) ! I.Jx-l-21+J.Jx-l-31 =I

www.puucho.com
CP Publication KVPY chapterwise with www.puucho.com

TOPIC• WISE KVPY [STREAM SXJ. Qm~stions with Solutions


Case-I Total even tcnns = 48 + 1 = 49
.Jx-1-2+.Jx-l-3 =l when x~lO Probability = 49
.2~=6 99
X=IO
Case-II 28. P1 : 4 copper coins 3 silver coins
.Jx-1-2-.Jx-l +3=1 when S~xSlO P2 : 6 copper coins 4 silver coins
Case -Ill E = Event of copper coin
-.Jx-l +2-,,/x-l +3=1 when 1:-:; x~5 P(E) = P(P1). P(E/P1) + P(Pz). P(!W2)
2.Jx-l =4 1 4 1 6 41
=-x-+-x-=-
x=5 2 7 2 10 70

24. M =a+ b-a2 + ab - b2 29.


2 b2 61 =6 91 =9
but a + ~ab 62 ::::6 9 2 ::::I
2
63 =6 9 3 =9
=> a2 + b2 ~ 2ab or-{a2 +·b2) S: -2ab 6 4 =6 9 4 =1
MS:a+b-ab m can be a~y value and n will be odd number
SoM<l-(a-l)(b-1) then swn is multiple ofS
So maximum value of M can be I. 50x 25= 12SO
at x = 0, else where the value of the function is
less than I. So answer must be option (B) but 30. · ai. a2, a3, ..•.•..,a2012 = 3018 ... (I)
KVPY has mentiop.ed option (C) as answer in a1 +a3
-2-=82
its answer key.
2a2 + 2a4 + 2a«, + .... + 2a2012 = 6036
y=2x 3 +cx+d (a1 + a3) + (a3 +as)+ (as+ a7) + ... +(aim + a1)
25. y=2x 3 +ax+b
=6036
No Solution 2{a1 + a3 + as+ ... + a2011) = 6036
*
2x 3 +ax+ b 2x3 +ex +d a,+ a3 +as+ .... + a2011 =3018 ... (2)
ax+b;ta cx+d for no real x Add (1) and (2)
Sum of all number= 3018 + 3018 ::;, 6036
(a-c)xctcd-b
d-b
xctc-- 31. As total nwnbcr of elements in set A will he n2 •
a=C
a-c Now set B will be
(a-c)2 +(b-d)=0+6-1 =5 Set B = {(I, 1), (2, 2) ....... (n, n)}
Number of elements in B is 'n'.
f(x) = x 2 n+J -(2n+ l)x +a So number of good subsets= zn
26.
So answer is (B) although KVPY has
f'(x)= (2n+l)x 2 " -(2n+l) Mentioned option (C) in its answer key.
=(2n +l)(x 20 -1) .:5: 0 when x e[-1, 1]
32. x 2 +2ax+b 2 =0 x 2 +2bx+c2 =0
f(x)is strictlydecreasing in [-1, IJ
D 1 >0 D 2 >0
f{x) cut x axis at most one point in given 4a 2 +b 2 >0 41:>2 -4c 2 >0
interval a2 > b2 ......... Q) b 2 > c2 ......... -(2)
From (I) and (2)
1.7. 1,3.6, 10, 15,21,28,36,45,55,66, 78,91, a2 > b2 >c2:::) a2 >c2:::) c2 -a2 < 0
105 till 98 terms
x 2 +2cx+a 2 =0
48 tenns are even and 48 tenns odd
D ""4c 2 -4a2 < 0 No real roots
99lh term:::: 99 xlOO even
2

www.puucho.com
CP Publication KVPY chapterwise with www.puucho.com

• ALGEBRA

f 2012.1s - -
Coeff.ox (l+x)2 37. la + b(ro + ro2) I S: ]al + lbl I@ + ui21
33. ---
(l+x2)(1-x2) S:l+l.l
=(1+x)\l-x 4 r 1
38.
S:2
Note: No. of numbers divisible by 'k' among
=(I+ 2x+x2)(l-x4rt
Coeff. of x2012 + 2 Coeff of x 2011 + Coeff of
x 20 1D in the expansion of (l-x4 1 r
{I, 2, 3, ...•. n} arc[:]
x 2011 and x 2610 llO~OSSible Ill(} - x4 1 r Somecascfory Lctx= l,y= 1
= only coeff. ofx 12 in the expansion of y=l
(1-x4rl y==2
""1 i·5o:i-ic503 =l
x=n, y=n
34. Ways to make the sum K is coefficient of~K in comes two time so that
(x + x2)10 . - .
Coefficient of xK in x 10( l + x) 10
Coefficient ofxK-IO in (I +·~.)1°
y=n I 2 ° [ ~]
--+2
n n kocJ k
r
which is IOCK-10 adding then
So ways to make sum minimum K is
10c
K-10 + 10c K-9 + 10c'K-S + ..... + me ID 39.
Probability
P(K)= IOC K-10+ toeK-9+ ..••••••••••• + toe 10
2 2
X1 +x2 +...... +xn2
210
cr=
n
P(K) 2 10 - (toe o + ......+ IOc
K-11
)
Yi = 0, Y2 = x2, Y1 = X3, •••••• , Yn--1 = x.... 1,
210 y.,=x, +x11
to Co+ .......+ IOCK-11 1 Clearly, µ =µand
=1 ------->-
2'0 2
+ IOC I + ...... lOC K-11 <2') 02 +xi+ x~ + ..... + x~_f+ (x1 + xn) 2
S0 !OC0 6- =
SoK-11 =4 n
::::>K= 15 2 2 2
XI +xz + ..... + Xn +2X1X 11
n
35. f(x) = x 3 + 3x2 + 3x + 3
Socr>cr
f'(x) = 3x2 + 6x + 3
f'(x) = 3(x2 + 2x += 1) ~ 0 40. Required integers = 900 - 9 x 9 x 8
regular increasing function = 900-648
and f(-2) f(-3) < 0 :=:252
so real root lies betwcen-2 and-3
i.e. -2 < a< - 3 41. (A) Put n = 19 Sn= {20, 21, ....... 37}
a.+J3+y=-3 So (A) is wrong
J3 +y=-3 -a (B) By observation we can say there exist
-l<l}+y<O some S,, don't have any prime number
(C) Let n = 10 {II, 12, ....... 28}
36. After solving logarithmic inequation so option C is wrong
24 <n <216 (D) Let n = 10, S11 = {11, 12, ....... 28}
number of binary digits in 2" = n + 1 prime number= 11, 13, 17, 19, 23
:.5<l<16 (five prime)
(at most six)

www.puucho.com
CP Publication KVPY chapterwise with www.puucho.com

-
42.
n
Ica1c 1 +bk 2 +ck+d)= n4
TOPIC• Wl~E KVPY IST~EA!VI SX] Que~tions

46.
w
LArea(Ci)
with Solutions

= m/ + m/ + rr.r./ + nrl +..... co


k=l i-o
a(n 2 )(n 2 + 2n + 1) bn(2n 2 + 3n + 1)
:::>-------~---+-----
4 6
2
c(n +n) dn
+ -----------"-- + = n4
2
a
(1)-=l ::::)a=4
4
2a 2b Area of Cn = m,/ = ( ,.fi rn-1)2
(2) - + - :::: 0 => b =-6
4 6 2_ 2 2
fn - - rn-l
a 3b C 1t
(3) + 4 6
+ =O 2 2 2 2
so r 1 = -ro,
C n:
:::> 1-3+.- =O

::::>c=4
2
rl= .!.r/ = ~(~rt)
1( 7r :It

b
r32= -(r2)
2 2 = -2(2 2 2)
C
(4) - + - + d = 0 ~ d = -1 --ro
6 2 X 1t 1t1t
:. lal + lbl + lcl + Jdl = I 5

43. Minimum value of f(x) is +ve. ;,,o


So f(x) doesn't have any solution. - [2ro +-ro
- 22 +---ro
Jl 222 + ....m]
TC 1t 1t

'lt~2
,,,,_o_
1 2
44. l--
20 7C
n2r:2
= -- 0 'v ro = 1
45. Total point = 5C2 = 10 rr-2
Maximum points of one point= 4 n:2

possible distribution of points rr-2

4 4 2 0 0 47. (A) [x+y]~Ix]+[y]


4 4 1 0 Let x = 0.l, y = 0.9
4 3 2 I 0 [0.1 + 0.9] S' [0.1] + [0.9]
4 3 3 0 0 I::;; 0 + 0 wrong
4 3 1 1 I (B) [xy]:::;; Lx] [y]
4 2 2 2 0 1
4 2 2 x=2;y= 2
3 3 3 0
3 3 2 2 0
3 3 2 I
2 2 2 :::> l ~ 0 wrong
3
2 2 2 2 2 (C) [2"'] s 2''1
X = 0.99 [2'1·9!1] S: 2 [0 99]
Clearly answer is "D" [2°·99] s 2° ::::: l wrong

www.puucho.com
CP Publication KVPY chapterwise with www.puucho.com r.

1 ALGEBRA

(D) [!_]y ~ [x]


[yJ
A2
(J -
2
Y1 +Y2
2
+ .... +yn
---=--~--- - µ
n
2
2

givenx, y 21
. (X1+X2) 2+ (X1+X2) 2
---'----'-- + X32+ .....xn2
ifx<y f;l =O 0 s.; [xJ true
[y]
2 2
----'-----------'-----------'------------'----------- µ2
n

if x ~ y [~] S: [ x] always true


y [y]

48. Case (1) n = even

~ = ncn/2 =2
A n-1 n-lc
n-1-1
2 2 2
_ X 1 +x 2 -2x 1x 2
so for all n even given relation is true.
2n
Case (2) n = odd
(x1-X2)2 ~0
ncn-1
A,, -
-
2n 2n
- - - - -2-
An-I
n-lc
n-! n+l => cr:C::6- & µ=µ
2
which satisfies only for n = 19 51. 3x + 4y + 5 = 19 I
5 S: (3 X + 4y + 5) S: 131
49. P=~ ~ +~ _!_ _!_ ~ 5:::;191::::;:131
6.6 6·6·6·6 Case (i) Case (ii)
6 l l l l 5 3x +4y+ 5 = 19 3x + 4y+ 5 = 38
+ -.- .-. - .-.- + ..... 00
6 6 6 6 6 6 3x+4y= 14 3x + 4y= 33
5 5 5 14-3x 4y= 33 -3x
= - + 3 + 5 + ..... . y=
6 6 6 4 33-3x
y=
5/6 30 6 x=2 4
X =J, 7, 11
1 _....!_ 35 7
36 Case (iii) Case (iv)
3x +4y = 52 Jx + 4y= 71
52-Jx 4y=71-3x
50. x=
4 71-Jx
y=
X = Q, 4, 8, 12, 16 4
x = I, 5, 9, 13, 17
Case (v) Case (vi)
3x +4y = 90 3x+4y= 109
11 90-3x 109-3x
y= y=
4 4
µ= Xi +x2 +x3 + ..... +x,, =µ=>Iµ=µ I
n x=6, 10, 14, 18 x = 15 is only possibility.

cr2
'°'
= _L,_x_~, _ µ z Total Solution= 19
n
1 1 1
52. By taking different values of a & k.
2 X1 + X2 + .... + Xn 2 option (B) is possible .
cr = -µ ...(1)
n

www.puucho.com
CP Publication KVPY chapterwise with www.puucho.com

53. 5 Ci. 2! + 5C3(~x2!)


1! 2! .
TOPIC•WISE

56.
KVPY [STREAM SXJ Questions with Solutions
The graph acc~rding tot/le question is.
y-·
+ C4 -4t- x2!+-·-x___:
5 [ 41 2r]
I! 3! 2! 2! 2!

+ 5C5 [ -51· x21+-·


5! x2!
1!4! 2!3!
l 2
X
= 10(2) + 10(6) + 5(8 + 6) + (10 + 20) rI ••

=20+60+70+30= 180 I

54. S -{A-a
- A-a!l_1 ....... + A-a 1
+----=-.c..+
an
0

an-I a,
l I :
Clearly it can be observed
c<0
VA= a1 + a2 + ....... +an
a>0
S ={(a 1 +a 2
1 + .... +1-
+ .... +anl(_!_+-
a. a2 an
)-n] -b
->O
a
:::::,. -b>0 :::::-b<0
S=n-(a 1 +a 2 + ... +an)(_!_+-1-+ .... +-1-)
a1 a2 an
f(l) < 0
=> a+b+c<O
from A.M. :?: H.M. ab< 0
ac < 0
be> 0

j S::.'.-n(n-1) I 57. x(3x2 - 25) = - n

n
3x( x2 - 235] =-n
55. "\'nc
L.i kro k -_nc o+ nc100+ .•...+ ncn©n
k~O

= (1 +co?= (-al)°
= (-I)° 0020
I\

[eH)" .,1. I= lc(-,.-.1)" I --i',50


9

= (-cos sin~)"
e
~-i
3 3

nn; . . mt
eos-.+,s,n-
. e 3 3

n,,;
= cos 3
e
can have values
:. ne (-z~o. 2:0)
={e 1, ein, e- 112' e- 1}
:. But n EI
Four values. Total integer= 55

www.puucho.com
CP Publication KVPY chapterwise with www.puucho.com
i

1 ALGEBRA
- -- " - D - - D·l - D--2
59. [z3 + z-31-;;; 2 61. LctP(x)=aox +a1x +a2x +... +an
P'(x) = DBoXn--1 + (n- l)a I x»-2 +... +a.,...1
lz3 + : 3 152 P(x)P'(x) = BoXn + (~1 - nao) xn-- I
- 2
+ (a2-(n- l)a1) xn-- +...
..
l(z+;J(z + : -1) s2
2
2
-·.........
.:...+ (au-Bn-1)
given P(x) - P'(x) = aox0 •
so that
(z+~)((z+ff-3] ~2

lz+~(z+;r-3 52. a2 - (n- l)a1 = 0 :::> a 2 =(n-1)


a,

an - an-1 = 0 :::> ~ =1
lz+~{F+~, -3} ~2 an-I

{ ·.- !z1 - z2I 2:. llz1l- lz2II} P~O) = 8 ~ = (~)( 80-1


8 0-1 3 n-2
J---(~) 80

tlt2 -3[s2 {t2:.0)wheret= lzj_!J = 1 x 2 x· 3 x ... x n


-;J J.
=n!

i'
"' Ji
t~ 0:St<Ji
62. Let Red Balls = x
White Balls = y
t<r-3) :S 2 t(3-r):s2 BlueBalls=z
t3-3t-2 :s 0 3t-t3 52 Green Balls = w
(t-2)(t+ 1)2:-i;;o t1-3t+2 2:. 0 xc xc ,YC
-----'4-=_ 3 1
(t-1)2 (t+ 2) ~ 0
t-250 t2:.-2
152 t e [O, Ji) x+y-tz+wc4
;· .
xc 4 ccc"C/C1
x-3 =4y
x=4y+3
60. 2{(2014) 3 + (2012)2 +...+ i} ;.
- {(2014)3 + (2013)3 +... 3} +] "C3 Yc1 =xC2 Ye, zc,
=2 X 8 {(1007)2 +{1006)2_+ ... 7 13 }
x-2=3z

=2 X gX ( (I 007~1008) r
- {(2014)3 + (2013}2+ ...+ 13 }

~
·· x=3z+2

l<c/c/c, =x ctctctc,
.. - '

-
+

_ ( (2014~2015)
.,
J x-1=2w
x=2w+ I

Clea1y for y = I not possible


. = 2 X 8 X (1007)2 (1008)2_ (2014)2 (2015)2 .• "
. 4 4 at y = 2 X = 11 .
z=3 x= 11
= (1007)2 (2016)2- (10.07)2 (2015/
w=5 x=ll
= (1007)2 {2016 -2015} {2016'+ 20.15}
= (1007)2 (4031} (• -- . . so, minimum number of Ball
= 11 + 2 + 3 + 5 = 21
= divisible by (1007)2

www.puucho.com
CP Publication KVPY chapterwise with www.puucho.com

-
91. ·. (A) ~
n!
=(E_)
n
(....E_)
n-1
(-n
n-2
) . . . . . (!:)I ~ 1
TOPIC• WISE KVPY [STREAM SX] Questions with Solutions

n" ~ n ! (correct) 1 l 27
S - l - - = - ( S - 1 ) - - S.
(C) ~ = (n)(n-l)(n-2) ..... . 100 5 2500
10n (10) (10) ....... n times (10)
S - 1 - - 1- = .!_ S - .!. - _E_ s.
given that n > l 000 100 5 5 2500
nr
clearly - ·
10"
~ I s(i-.!..+-32....)=-.!..
5 2500 5
+ 1 + - 1-
100
n! ::>: 10n
S (2500-500+27) =~
2500 100
(D) (I .2.3.4 ........n)(n + l)(n + 2)(n + 3) ...... (n + n)
nu S = 81x25
2027
S = 2025
2027
clearly~ L
2n! ~ nn 16x 2 -96x+l53
94. y-==-----
x 3
92. Case-I
make it quadratic in x
(a--b}2+(b-c)2+(c-a/ 2:0 16 X2 - X (96 +y) + (153 + 3y) = 0
a2 + b2 + c2 - ab - be - ca 2: 0
D~O
a2 + b 2 + c2 ~ ab + be +ca Solve y2 -576 ~ 0
Ifab+bc+ca>O
y E (- co, -24] V [24, oo)
a2 +b2 +c2
Tben----~l So, least positive is 24
ab+ bc+ca
Case-ll 95. n[xn + an-1 x11 • 1 + an-2 xn-2- a1x + 8-0]
Let ab-+ be+ ca< 0 = (1 + x) (nxn-l + an-1(n - l)xn-~
+ a11:-2 xn-3(n - 2)
(a+~+c)2 ;s;o ' ~

ab+ bc+ca + a,,_3(n-3)x + ....... )


compare coefficient of ~n-l
a 2 + b2 +c 2 +2(ab+ bc+ca) 5 0
nan-1 = (n-I)an-1 + n
ab+ bc+ca Solve an- 1 =nor nc 1
a2+b2+c2 compare coefficient of xn-2
----+2:-;;o
ab+ bc+ca na,,_2 = (n-2)an-2 + (n-l)an-1
a2 +b2 + c2 _ n(n-1) _ "C
----:::;;-2 an.z- - ~ - 2
ab+ be +ca 2
So, Range is (- oo, -2] u [1, co) similarly an-3 = nc3 & So ..... on
b = P(l) = I + an-1 + an-2 + ..... a1 + ao
93. an = 20an-1 - 108an-2 = nco+ nc1 + "C2 +....... °Cn=2n
an : - 20an-l 108an-2
10 2n - ~ - 10 2n 96. a+b+ c+d+e=540
Let say firsl tenn = a
an _ 20 an-I 108 an-2
Common difference d
10 2" - 100 10 2<n-!) - l 0000 102(n- 2)
n
apply summation 2 [2a + (n- l)d) = 540
~ a+ 2d= 108

www.puucho.com
CP Publication KVPY chapterwise with www.puucho.com

• ALGEBRA
Case-I d=O x2 x3
a=l08 98. l+x <l+x+-+-+
2! 3!
(108, 108, 108, 108, 108) I+ X <ex
Case-2 d= I In (l+x) < x
(106, 107, 108, 109, 110) 1
x=-
Similarly it goes up to d = 35 Y
ford> 35, interior angle> 180° 1 1
ln(l + -) < -
which is not possible y y
So form d = 0 to d,...., 35 I
In (y + l) - In y < -
total 36 tuples are possible y
put y= I
97. Case-I In (2) - In (1) < lI
No person between X 1 & X2
put= y= 2
In 3 -ln2 < 1
2
put= y= 3
l
ln4-ln 3 < -
30!x2 ! 2 3
31!
--
31
puty= n
1
Case-2 ln(n+l)-Inn< -
4
One person between X 1 & X 2 1 I 1
ln(n+l)-ln(l)< 1+ --i- -+ .... -
2 3 n
In (n +l) .s" 4
nS e4-l
nS60
3°C 1 (29) !x2 ! = 2_
31! 31 99. X1 + X2 = 11 or
Case-3 possible cases
When 2 person between X 1 & X 2 (1,10) (8,12)
(2,9) (9,11)
(3,8) (10,10)
(4,7)
(5,6)
10 6 16 1
+-=-=-
3°C 2 x28!x2!x2! = -3._ 144 144 144 9
31 ! 31
100. It form an G.P.
Case-4
When 3 person between X1 & X2
30 C3 x27!x2!><3! = 2-_
31 ! 31

Total=_! solve that


31
x1 = 27
x="

www.puucho.com
CP Publication KVPY chapterwise with www.puucho.com

TOPIC- WISE KVPY [STREAM SX] Questions with Solutions


101. Now, Case l + Case 2
(43,1) :::;, 2352 + 98 = 2450
Total number of ways m, n can be selected
= 100 x 99
=9900
(43,-1)

2rc 1t Proba bI·1·tty= -


2450
- = 0.2474
9900
n 3
n=6 106. f(x) = ax+ b
5=a+b ... (i)
102. 23 IAI -2 3 -
1- 7=2a+b .. ..(ii)
IAI solve a=2
1 2017 2 b=3
IA- 1= 1 I 2017 4 f(x) = 2x + 3
2018 8 f(l2) = 24 +3
f(12) = 27
1
-2-:::;, IA:=
' 2
--I
IAI
Put the value answer is= 12
107. required probability =
4c 1X 4!
2 X
t X 31•
103. given expression can be written as (I!) x2! 2!
1 ------------ - -169
44
(z 1 +z 2 + .... +z 7 f- 2:zf
i....:1 ~
-------~~ = L./iZj
2 l:s'.i<_i!;'.-7
Sum of all seven root of unity= 0 (By property)
zf + Zi + Zi + .... + z~ = 0 (By property)
104. possible case
Case-I
If unit digit is sum of 4m + 4n is 7
possible case are
(m, n) = (O, 2) (0, 4) .......... (0, 98) = 49
(n, m) = (0,2) (0, 4) ............ (0,98) = 49
total= 98
Case-2
Unit digit is sum of 4"' + 4n is 2
Possible case
(m, n) = (2,4) (2,6).......(2, 98) = 48
(m, n) = (4, 6) (4, 8) ........ (4.98) = 47

(m, n) = (96, 98) :::=- !Case


same case repetition (n, m)

2 X (
48 ; 49 ) =2352

www.puucho.com
CP Publication KVPY chapterwise with www.puucho.com

2D-Coordinates
I
1. A circle touches the parabola .; = 4x at (1, 2) 4. Among all cyclic quadrilaterals inscribed in a
and also touches its dircctri'C. They-coordinates circle of radius R with one of its angles equal to
of the point of contact of the circle and the 120°. Consider the one with maximum possible
directrix is- {2010] area. Its area is- J2010J

(A) .ff_ (D)2 (A) .Ji R2 (B) -J3 R2


(C) 2 R2 (D) 2/J R2
(C) 2../2 (D) 4

2. Let P be an arbitrary point on the ellipse


s. All the points (x, y) in the plane satisfying the
cquation.1:2 + 2x sin (.ly) + 1 = 0 lie on - [2011]
x2 y2
- 2 +-? ==I, a> b > 0. Suppose F 1 and F2 are (A) a pair of straight lines
a b- (B) a family of hyperbolas
the foci of the ellipse. The locus of the centroid (C) a parabola
of the triangle PF 1F2 as P moves on the ellipse (D) an ellipse
is- [2010)
(A) a circle (B) a parabola 6. Let A = (4, 0), B = (0, 12) be two points in the
(C) an ellipse (D) a hyperbola plane. The locus of a point C such that the area
of triangle ABC is 18 sq. units is - [2011]
(A) (y+ 3x + 12)2 = 81
3. Suppose a, b arc real numbers such that ab ~ 0.
(B) (y + 3x + 81)2 = 12
Which of the following four figures represents {C) (y + 3x - 12)2 = 8 l
the curve (y- ax - b)(bx2 + a/- ab)= 0? (D)(y+3x-81) 2 ""' 12
12010]
7, ln a rectangle ABCD, the coordinates of A and
Bare (I, 2) and (3, 6) respectively and some
diameter of the circumscribing circle of ABCD
has equation 2r -y + 4 = 0. Then the area of
the rectangle is - (20111
(A) 16 (B) 2..{to
(C) 2,JS (D) 20

8. In the xy-plane, three distinct lines 11, 11 , l3


concur at a point (A., 0). Further the lines / 1, / 2,
/3 are normals to the parabola y2 = 6x at the
points A = (xi, Yi), B = (x2, Y2), C = (x3, y3)
respectively. Then we have - f2011]
(A) "'<- 5
(C) - 5 < A < - 3
(B) 11. > 3
(D) 0 < 11. < 3

www.puucho.com
CP Publication KVPY chapterwise with www.puucho.com

TOPIC· WISE KVPY [STREAM SX] Questions with Solutions


The locus of the point P = (a, b) where a, b are 15. Two line segments AB and CD are constrained
real numbers such tbat the roots of to move along the x and y axes, respectively, in
x3 + ax 2 + bx + a = 0 are in arithmetic such a way that the points A, B, C, D are
progression is- [20111 concyclic. If AB = a and CD = b, then the
(A) an ellipse locus of the centre of the circle passing through
(B) a circle A, B, C, D in polar coordinates is- f2013]
(C) a parabola whose vertex in on the y-axis , , 2 2
(D) a parabola whose vertex is on the x-axis (A) r2 = a· + b" (B) r2 cos29 = a - b
4 4
10. The smallest possible positive slope of a line (C) r2 = 4(a2 + b2) (D) r2cos20 = 4(a2 - b2)
whose y-intercept is 5 and which has a common
point with the ellipse 9x2 + l 6y2 = 144 ls [2011] 16. Consider a triangle ABC in the xy-plane with
vertices A= (0, 0), B = (1, I) and C = (9, 1). If
(A) i (B) 1
the line x = a divides the triangle into two parts
4
of equal area, then a equals- [2013)
(C) .± (D) ~ (A) 3 (B) 3.5
3 16
(C) 4 (D) 4.5
11. Suppose t'No perpendicular tangents can be
drawn from the origin to the circle 17. The vertices of the base of an isosceles triangle
x2 + l - 6x - 2py + 17 = 0, for some real p. lie on a parabola y2 = 4x and the base is a part
then IPI = l2012) of the line y = 2x - 4. [f the third vertex of the
(A) 0 (B) 3 triangle lies on the x-axis, its coordinates are
(C) 5 (D) 17 [2013)

12. Consider the conic (A)(i,o) (B) (l• 0)


ex2 + ny2- 2c 2x - 2rr2y + c 3 + n3 = ne. Suppose
P is any point on the conic and S1, S2 are the
(C)(~,oJ (D) (121, o)
foci of the conic, then the maximum value of
(PS1 + PS2) is [2012]
(A) 1tC (B) & 18. In a triangle ABC, let G denote its centroid and
let M, N be points in the interiors of the
(C) 2J;. (D) 2..,le
segments AB, AC, respectively, such that M,
G, N are collinear. If r denotes the ratio of the
13. Let (x, y) be a variable point on the curve area of triangle AMN to the area of ABC then
4x2 + 9y2 - 8x -36y + 15 = 0. Then
(2013]
min(x2 - 2x + y2 - 4y + 5)
1 I
+ max (x 2 - 2x + / - 4y + 5) is (A) r= - (B) r> -
12012] 2 2
4 1 4
(A) 325 (B) ~ (C) - :-,; r < -
9 2
(D) - <r
9
36 325
(C) Q (D)~ 19. Let b, d > 0. The locus of all points P(r, 9) for
25 13
which the line OP (where O is the origin) cuts
14. If a, b are positive real numbers such that the the line r sin 9 = b in Q such that PQ = d ism
lines ax+ 9y = 5 and 4x +by= 3 are parallel, 12014}
then the least possible value of a + b is- [2013] (A) (r - d) sine = b (B) (r ± d) sine = b
(A) 13 (B) 12 (C) (r - d) cos 0 =b (D) (r ± d) cos 0 =b
(C)8 (D)6

www.puucho.com
CP Publication KVPY chapterwise with www.puucho.com

f 20-COOROJNATES
20. Let C be the_circle x2 + y2 = 1 in the xy-plane. 24. Let S = {(a, b)ia, b E Z, 0 s a, b s 18}. The
For each t ~ 0, let L1 ·be the line passing through number of lines in R2 passing through (0, 0)
(0, 1) and (t, 0). Note that Li intersects C in two and exactly one other point in Sis - [2014]
points, one of which is (0, 1). Let Qi be the (A) 16 (B) 22
other point. As t varies between 1 and l + -,Fi, (C)~ ~)TI
the collection of points Qi sweeps out an arc on
C. The angle subtended by this arc at (0, 0) is 25. The number of ordered pairs (x, y) of real
[2014] numbers that satisfy the simultaneous equations
x-'-y2 = x2 + y = 12 is 120151
(A ..:C.
. ) 8 (B) 2: (A) 0 (B) 1
4
(C)2 (D)4
(C) ~ (D) 3rr
3 8
26. Suppose OABC is a rectangle in the xy-plane
where O is the origin and A, B lie on the
21. In an ellipse, its foci and the ends of its major
parnhola y = x 2• Then C must lie on the curve -
axis are equally spaced .• rf the length of its
(2015]
semi-minor axis is 2-/2, then the length of its (A) y=x 2 + 2 (B) y = 2x 2 + J
semi-major axis is [2014] (C) y = -x2 + 2 (D) y = - 2x 2 + 1
(A)4 (B) 2/3
(C) ,Jw (D) 3 27. Circles C1 and C2, of radii rand R respectively,
touch each other as shown in the figure. The
line e, which ls parallel to the line joining the
22. Let n ~ 3 and let C 1, C2, .. , .,, Cn, be circles with
centres of C 1 and C 2 , is tangent to C 1 at P and
radll r1, rz, ..... , rn, respectively. Assume that C;
intersects C2 at A, B. If R 2 = 2r2, then LAOB
and C;+1 touch externally for 1 s i s n - I. It is equals - [2015]
also given that the x-axis and the line C2
y = 2.fi.x + 10 are tangential to each of the
circles. Then ri, r2 , ..... , rn arc in - [2014]
(A) an arithmetic progression with common
difference 3 + ./2,
(B) a geometric progression with common ratio
3 +,Ji.
(C) an ari1hmetic progression with common
difference 2 + .fj (A) 22 .!_ 0 (B) 45°
(D) a geometric progression with common ratio 2
2+/3 (C) 60° (D) 67 _!_ 0
2
23. An ellipse inscribed in a semi-circle touches the 28. A mirror in the first quadrant is in the shape of
circular arc at two distinct points and also a hyperbola whose equation is xy = 1. A light
touches the bounding diameter. Its major axis is source in the second quadrant emits a beam of
parallel to the bounding diameter. When the light that hits the mirror at the point (2, 1/2). If
ellipse has the maximum possible area, its the reflected ray is parallel to the y-axis, the
eccentricity is - [2014] slope of the incident beam is - 12015]
(A) - 1 (B) .!_ (A)~ (B) ]_
-./2 2 8 4

(C) ~ (D) H (C) ~


8
(D)2

www.puucho.com
CP Publication KVPY chapterwise with www.puucho.com

TOPIC· WISE KVPY [ST REAM SX] Questions with Solutlo ns

Suppose the tangent to the parabola 34. Let BC be a fixed line segment in the plane.
y = x 2 + px + q at (0, 3) has slope -1. Tl1en The locus of a point A such that the triangle
p + q equals 12016] ABC is isosceles, is (with finitely many
(A) 0 (B) 1 (C) 2 (D) 3 possible exceptional points) 15 Nov, 20171
(A) a line
30. Let O = (0, 0); let A and B be points (B) a circle
respectively on x-axis and y-axis such that (C) the union of a circle and a line
LOBA = 60°. Let D be a point in the first (D) the union of two circles and a line
quadrant such that OAD is an equilateral
triangle. Then the slope of DB is - (2016]
35. Let A= (ai, ai) and B = (bi, b2) be two points
(A) .J3 (B) .fi. (C) _}i (D) ~ in the plane with integer coordinates. Which
one of the following is oot a possible value of
the distance between A and B ? (5 Nov, 20171
31. Suppose the parabola (y - k)2 = 4 (x - h), v.ith
vertex A, passes through O = (0, 0) and L = (0, 2).
(A) .J65. (B) .J74
Let D be an end point of the latus rectum. Let (C) ,/8'] (D)#i
the y-axis intersect tbc ax.is of the para bola at P.
Then L PDA is equal to (20161 36. How many different (mutually noncongruent)
(A) 1an·1 _!_ (B) tan" 1 ~ trapeziums can be constructed using four
19 19 distinct side lengths from the set {l,3,4,5,6}?
(C) tan 1 _i_ (D) tan-I _!_ 15 Nov, 2017)
19 19 (A) 5 (B) 11
(C) 15 (D) 30
32. In a circle with centre 0, suppose A, P, B are
three points on its circumference such that P is
the mid-point of minor arc AB. Suppose when 37. Let R be the region of the disc x2+y2s:I in the
first quadrant. Then the area of the largest
LAOB = O, area(AAOB) = ,./5 + 2 . possible circle contained in R is 15 Nov, 2017]
· area(AAPB)
lf LAOB is doubled to 28, then the ratio
(A) n(3 - 2-J2) (B) 1e(4-3./2.)
arca(AAOB) is 120161
area(AAPB) (C) ~ (D) n(2Ji. - 2)
6
(A) _l (B) ,./s - 2
.js
38, Consider a rigid square ABCD as in the figure
(C) 2F3+3 (D) -.Js -1 . with A and Bon the x and y axis respectively.
2 [19 Nov, 20171

33. A wall is inclined to the floor at an angle of


e
135°. A ladder of length is resting on the wall. C
As the ladder slides down, its mid-point traces
an arc ofan ell[pse. Then the area of the ellipse
D
1S [2016]

When A and B slide along their respective


axes, the locus of C forms a part of
? (A) a circle
(A) 1t€M (B) xf (B) a parabola
4 (C) a hyperbola
(C)41t£2 (D) 2x£ 2 (D) an ellipse which is not a circle

www.puucho.com
CP Publication KVPY chapterwise with www.puucho.com

t 2D-COORDINATES
39. An ellipse with its minor and major axis 41. Let AB be the latus rectum of the parabola
parallel to the coordinate axes passes through y2 = 4ax in the xy-plane. Let T be the region
(0,0), ( 1,0) and (0,2). One of its foci lies on the bounded by the finite arc AB of the parabola
y-axis. The eccentricity of the ellipse is and the line segment AB. A rectangle PQRS of
[19 Nov, 2017J maximum possible area is inscribed in T with
(A) _13-l (B) ..}5-2 P, Q on line AB, and R, S on arc AB. Then
area(PQRS)/area(1} equals [19 Nov, 20171
(C) -./2 -l (D) .J3 -l
2 (A) J_ (B) l_ (C) _l (D) _l
2 3- ,,fi Ji
40. The soW1d of a cannon firing is heard one
second later at a position B than at position A. x2 y2
If the speed of sound is uniform, then 42, An ellipse 2 + -., = I, a > b and the parabola
a bk
{19 Nov, 20171
(A) The positions A and B are foci of a
x2 = 4(y + b) are such that the two foci of the
ellipse and the end points of the latus rectum of
hyperbola, with cannon's position on one
branch of the hyperbola parabola are the vertices of a square. The
(B) the position A and B are foci of an ellipse eccentricity of the ellipse is I19 Nov, 20171
with cannon's position on the ellipse (A) _l_ (B) 2
(C) One of the positions A,B is focus of a -m Ju
parabola with cannon's position on the
parabola
{C) _l_
.Ju
(D) }i
(D) It is not possible to describe the positions
of A, B and the cannon with the given
information

www.puucho.com
CP Publication KVPY chapterwise with www.puucho.com

TOPIC•WISE KVPY (STREAM SX] Questions with Solutions

ANSWERS
1. (C) 2.(C) 3. (B) 4.(B) S.(A) 6. (C) 7. (A)
8. (B) 9. (C) 10. (B) 11. (C) 12. (C) 13. (A) 14.(B)
15. (B) 16. (A) 17. (C) 18. (C) 19.(B) 20. (8) 21. (C)
22. (D) 23. (D) 24. (*) 25. (D) 26. (A) 27. (B) 28. (C)
29, (C) 30. (D) 31.(B) 32, (A) 33, (A) 34. (D) 35. (C)
36. (B) 37. (A) 38. (D) 39. (C) 40. (A) 41. (D) 42. (B)

1.
• .·- ..... •
SOLUTIONS

h= acos8+ae-ae. cose= 3h
3 ' a
(-1,a.) k= bsin8 _ sin 8 "" 3k
3 ' b
cos2e + sin28 = l
9h 2 9k 2 x2 yz
-+-=l => --+--=l
x=-1 a2 b2 (a 2 /9) (b 2 /9)
(Ellipse)
i=4x
x:z y2
2y dy =4 3. y=ax+ band - + - = l
dx a b
2 2 slope= a
mr= 1.= 2 =I for the line, y intercept = b
Circle-t S + i.L=O Fig.I for line~ a< 0, b > 0
(x + 1)2 + (y-a)2+ 1,,(x +I)= 0 .... (1) Hence the other fig. cannot be an ellipse
di ffcrcntiatc Fig.2 a > 0, b < 0 -
2(x + 1) + 2(y-a.) dy + 11.= O Hence the fig. is a hyperbola
dx. Similarly you can cl1eck rest 2 options
x=l,y=2
4 + 2(2 - a) mr + A. =0 4.
}.. = 2a.- 8 ....(2)
(1, 2) satisfies eq.(l)
22 + (2 - «)'- + 2}.. = 0
a 2 -4a. + 8 + 2{2a-8) = 0
a'-=&
a.= 2..fi

2. P -+ (a cos a, b sin 9)
A= 2 x .!_ x. ./3 R x R
G 4 (!:xi3 • l:y;)
3
2
= ..JiR2
F1-+ (ae, 0) F2-+ (-ae, 0)

www.puucho.com
CP Publication KVPY chapterwise with www.puucho.com

• 2D-COORDlNATES

5. .:-<.2 + 2 x sin (xy) + 1 = 0 I


Slope of BC= - -
2 sin (xy) = -(x +l) 2
£(AB)= ,/4 + 16 = 2/s
R.H.S. ~ 2 or s; -2 distance between 2x - y + 4 = 0 & 2x - y = 0
u u :::::,-
4
L.H.S. = R.H.S. = 2 sin(xy) = -1 ,,/5
x=-1
U (x = I)
Area= 2-Js. Js = 16
sin (-y) = l
sin y=-1 8. Any normal
Jl y = mx - 2am- am:; Herc a= 3/2
Y = 2nn: - - neI
2' through ().., 0)
Hence pair of straight lines. 0 = ITIA- 2am-am3
m = 0, ). = 2a + am2
6.
2 ),
y m=--2>0
a
B (O, 12)
l > 2a =:, ), > 3

9. Let roots a - <l, a, a+ d

-------1--......l..---x
A Sum 3 a = - a :::::, a. = - -a
0 (4, 0) 3
Product
1 X }'
a(a2 - d2) = -a
l 1 0 12 = ± 18 a 2 -d 2 =3
2
1 4 0 pair product b = cl - ad + rx 2 + ad+ a-2 - d2
:::::, 1(-48)-x(-12) + y(4) =±36 b =2ci+ 3
:::::, 12x + 4y- 48 =± 36 b-3 = ~ a2
:::::, 3x +y - 12 '"" ± 9 9
:::;, (3x +y- 12)2 = 81 :::::, Locus x·' = -(y-3)
9 Parabola
2
7.
. y·, x·'
10. Ellipse-+-= l
9 16
xcos0 ysin0
I
Any tangent - - + - - = 1
I 4 3
I

y=2x~ y intercept= 5 :::::, sin 0 = ¾; 0 e ( f, re)


" " .... .... _ - :::::, cos e = --4
s
2x-y+ 4 =O y =J
tangent:::::>- -X +-
4 5 5
Slope of AB = - = 2 :::::, slope= l
2

www.puucho.com
CP Publication KVPY chapterwise with www.puucho.com

TOPIC-WISE KVPY [STREAM SX] Questions with Solutions

11. (x-3)2 +(y-p) 2 =9-17+p 2 15.


Director circle is
(x-3)2 +(y-p)2 =2(p 2 -8)
Passes through (0, 0)
9+ r2=
2p 2 - 16
=> p2 = 25 => p = ± 5 => IPI = 5

12. X
2 '1)
.., ')
e(x -2ex +e-)+rc(r -2ey+x- = ri:e

~ .
x-e) - + (y-11Y =l
~
(
Equating radius
TC e

a 2 =rc=>a=J;; rr >e
PS 1 +PS 2 = 2a
PS 1 + PS 2 = 2,J";
Ivlajor axis is II to axis
X 2 -V 2

= [a 2
-b
4
2
l
13. 4x 2 +9y"-8x-36y+15=0
'

4(x2 - 2x) + 9(/-4y) = - 15


4(x2 -2x -t- I)+ 9(/-4y +4) =-15 + 4 + 36
4(x- 1)2 + 9(y-2)2 = 25 16.

(ff (ff
(x-1)2 + (y-2)2 =1
.. (!)
B(l,l) M (a,l)
C(9,1)

x-~ - 2x + y-' - 4y + 5
= (x- 1)2 + (y-2)2
A(0,0)
- '
mm ot ((x - I)
2
+ (y- 2)2) = -25
9

max of((x-1)2 + (y-2)2) = 2}


I
2 · 2
area ilAJJC = - 8 x I = 4
:. minof((x-1) +(y-2) )+ 2
max of((x- 1/ + (y- 2/)
= 25 + 25 = 325
9 4 36
a2
=>9-a-a+- =4
a 9 9
14.
4 b al
=> - -2a+5=0
=>ab= 36 9
We know => a2 - I Sa+ 45 = 0
AM.?:G.M.
=> (a - l5)(a - 3) =0
a+b ~.,/;J; =>a= 15, 3
2
::::>a= 3
a+ b ;;>: 12
(a+b)mb=l2

www.puucho.com
CP Publication KVPY chapterwise with www.puucho.com

...

• 20-COORDINATES

17. Let point satisfying parabola and line is (t2, 2t) 19.
ii I

which is vertex of triangle. (x, y)


y
2t2-2t-4 = o P(r, 0)
d
t2-t-2=0
t=-1,t=2
(4, 4)

Equation of OP is y = x tan 9
Point Q is (b cot 0, b)
(Xo, 0) :. Point P is y = b ± d sin 8
r sin B= b ± d sin B
(xo - 1)2 + 4 =(x0 - 4)2 + 16 (r + d) sin O=b
-2xti+ 1 +4=-8Xo+ 16 + 16
6XQ= 27 20. x2 + y2 = I
27 9 Lt ~ ·t· 2'.. = I
Xo ---
6 2 t 1
X
18. y=l--
t
A 2 x2 2x _
x+l+----1
t2 t

x2(1 +..!..)
t2
- 2xt = 0
B-----------"C N X = 0, X (1 +1-)
t2
= 3_
t
ar(MMN)= a y=l
2t
x=--· y=l---
2
2 t2 + 1 ' t2 +I
I t 2 -1
=> Tma,,: < - (0, l) y=--
2 t2 + l
A
Qi[-3.:....
l+t t +1
~) 2' 2

1:;;t:;;1+"2 t = tan 0, Q1 (sin 20, - cos 28)

8 e ( 45°, 67 ~) lies on circle C


B~-----C
D
AG 2
-
AD
- -3
ar(t.AMN) = ±
ar(MBC) 9
z2
= - =r.
32 llllll

4 l
- ~r<-
9 2
7t
so angle at.centre = -
4

www.puucho.com
CP Publication KVPY chapterwise with www.puucho.com

TOPIC- WISE KVPY (STREAM SXJ Questions with Solutions

21. . 0 1
=> Sill - =-
2 fj
=> ON= ./3r1
A'S' = SS' = SA
---- In l\OPQ,
2ae= a-ae . 0 r2
sm-=--=---
3ae=a 2 ON +r1 +r2
e= 1/3
1 f2
b2 l b2 8
l--=-::::>-=-
a2 9 a2 9
=> .fi - /3r1 + r1 + r2

8 8 => ..fir1+ r1 + r2 = .,J3r2


- - = -9
~ a2
=> r1 ( ..fi +1) = r2 ( /5-1)
=-a=J
r2 _ Ji +1
22. => ~ - /3-1
y
tan0 = 2,/5. = (,fi +1)2
2
=2+-,J3

23.
y M p
X y

2tan~
tan9= ~ ~ -2-
2 e
l-tan -
2
0
2tan-
2
2 0
I-tan -
2 x2 y2
Let ellipse - 2 + -, = I
l2 tan2 ~ + tan 8 -Ji= 0 a b-
2
and circle x2 + (y + b)2 = r2 {let radius= r}
. 9
tan -
-1±.Ji+s
2 2 a 2 y2
2 2,./2 putx =a - - -
b2
-1±3 1 or -.Ji 2 '.'
= 2~ =,Ji. in circle a~ - ~ + (y + b)2 = r2
62 .

tan - =-;=
8 I ~I =>
( 2)
ll-: 2 y2+2by+(a2+6 2 -r1)=0
2 •,./2
./2
2 a4
Inti.OMN, D=O=>r = - -
a2-b2
0 . r1
sm-=-
2 ON
=>b=a g 2

www.puucho.com
CP Publication KVPY chapterwise with www.puucho.com

• 2D•COORDINATES

Area= Li =nab= na 2 g 2
So, t1t2 =- 1
Now ~= ti +t~
2 2
dll 2 2r 2 11 + t2 = b ... (I)
-=0::::>a = -
da 3 also tl + tl = k
(t1 + t2}2 - 2t1t2 = k
=:>a=~r
h2 + 2 = k
Locus is x2 + 2 = y
... b= a~l-¾ = E 27.

=> c=J¾
25. X+ y2 = x 2 + y = 12
Curve (1)
x+y2= 12
y2 =-(x-12)
Intersection on x-axis ( 12, 0)
Intersection on x-axis = (±Jfi, 0)
Curve (2) Chose AB subtend 90° at centre.
x2 +y= 12
2 so that AB subtend 45° at O (circumference of
X =-(y-12)
circle)
Intersection on y-axis (0, ± Ju )
Intersection on y-axis= (0, 12) 28.
Normal

I
Curvey=-
x
Four intersections dy = - -
dx x2

26. Slope of tangent cc: - ¼at ( 2, ½)


y
y=x2 Slope of normal = 4

where foot of normal is ( 2, ½)


Let slope at incident beam is 'm'

l1~-4:l=I~
4-m I
1+4m
--=±-

15
4

m=-
OB.LOA 8

www.puucho.com
CP Publication KVPY chapterwise with www.puucho.com

TOPIC• WISE KVPY [STREAM SX] Questions with Solutions


29. - (0, 3) lies on the curve 32.
Soq=3
Now, dy =2x+p;
dx
P( cos!, sin!)
(dy)
dx (0, 3)
=p=-1
---------A
2 2

,', p + q =-1 + 3 = 2

30.

A(AOB) = 2 + ..JS
AAPB
.!. .1,sin8
----.--2~------,- = 2 + .Js on solving
I O 1
I 0 . 8
-cos- sm- 1
2 2 2
cos8 sin8 I
I 8
Meo=-
.,fi cos-
--·=2-=--=2+../s
31. l-cos-
e
y D{h + l,3) 2
0 I+.../5
=> cos-=--
2 4

=> So cos 0= - -
-./s-1
(b, l) !' (h, k)A M(b+ I, I) 4
If8->28
MOB cos8 1
=> .6.APB =1- cos 8 = ../s
Curve, S : (y- k)2 = 4(x - h)
LLR = 4; Clearly k = l ;
33.
=> A(h, I) & 1M1 is focus (h + 1, I)
So D (h +l, 3)
Seo, o) = O => k2 = - 4h
-14 (34' )
=>h= -=> D - 3

!_2 {x, OJ
3 2
m 1 -m'J
Now·tana=
'
~ =,~---1=-
I +m m2 8
l+-><2
3
19 Mid point (h, k) = ( X Xi ,~ i)
3-1 2 8 Now (x + x,)2 +x1 2 = f!. 2
where, m1 = -3- = 3 = 3
--0 As 2h + 4k= x + x 1, 2k= x,
4 4 So required locus is
3-1 4(h + 2k)2 + 4k2 = e2
=> m2= - = 2
I

www.puucho.com
CP Publication KVPY chapterwise with www.puucho.com

• 20-COORDINATES
2 36.
:::!> h2 + 5k2 + 4hk = !_ C
4

::::, x 2 + sy2

Whose area is -·-


+ 4xy= -

nP 2
4
e2
4
t a
la-cl <b + d <a+ c
34. (a, c) (b, d) (1, 3) (5, 6)
Case (i): (1,3) (4,5)
A
and (l, 3) (4, 6)
Now make different combination. Total of 11
combination are possible.

37. Required eq1L1.tion of circle


B~--~~c (x - h)2 + (y - h)2 = h:2
If LB=LC
Both circle touch internally
locus of A is .l bisector of BC C1C2 = Ir, -r1l
So it is straight line
Jh 2 +h 2 =Jh-11
Case (ii):
A
Solve this h = .Ji - 1
Arca n(-fi - 1)2 = n (3 -2.fi)

38,

B ,____ _ __.__~c
lf LA=LC
DC fixed B(a, 0), C(U, a)
BC=AB
So, (x - a}2 + ..J = 2a2
Circle C(h, k)
Case (iii): h = a sin O :::::> sin O"" ~ ... (i)
LA=LB a

AC=BC
k = a sine + a cos 9
k=h+a cos 0
~h 2 +(k-a) 2 = J2a 1 k-h
- - = cos e ....(2)
x1 + (y - a)1 = 2a2 a
also a circle sin28 + cos20 =I
So union of two circle an<l a line. h2 (k-h) 2
-+---=I
a2 a2
35. AB= ~(a 1 - b 1) 2 + (a 2 - b2 ) 2 h2 +(k-h)2=a2
locus
Square + Square = f6s possibJc when x 2 + (y-x}2 = a2
=64+ I solving we get
..fii =49+25 -I
x2 + + x 2 - 2xy = a2
2x2 + y-2xy = a2
m =s1+16 check h:2 -ah< 0
But .Jii not possible ellipse

www.puucho.com
CP Publication KVPY chapterwise with www.puucho.com

TOPIC• WISE KVPV [STREAM SXJ Questions with Solutions


39. 41.

Centre is

(x-f)' + (y-p)' = I ( at~ ,2at2) B


a2 b2
pass through (0, 0) (0, 2) & ( 1, 0)
T= lo 2..)a ..Jx.dx= -
a 8a2
3
Distance between F 1 & F:i.:::) a·= 2ae
ti= t2
a 2 =4a2 e2
Pass through (0, 0) Area ofPQRS
(l2 p2
:::>-+-=I = 2a It,- t2I Ia- atr I
X

4a 2 b2
But t1 =-h
0.2
-=e2 = 4a2 t1 (1- t;)
4a 2
Pass through (0, 2) Differentiation with respect t 1
(2-p)2
-0.2 + ___ _
......__ -1
4a 2 b2 We will get t~ "".!.3
from these two :;. ~ = 1
Use properties PF 1 + PF2 == 2a . Now put t 1 = Ji1 get Area of
F 1(0,1) Fl(o., I) P(O, 2)
1 + ~o. 2 +1 =2a PQRS= 8a2
. 3.fj
F,(0,1) F2 (a., 1) P(l, 0)
. B 1
M + ~(a-1) 2 +1 =2a ratto ecomes ,Ji
From these two
a =- 2a. + 2..fi.a 42.
put et in any ofabove two equations --4--
-5.+l ~ (-2, l - b ) D ( 2 , F-b)
a= - - :;.2a= '1'2+1
2 \l(E!ll-2)2 +(1-b)'

o.= 1
a= 2ae
(-ae, 0) (ae, 0)
u=l,2a= Ji+l
find e= ,.fi, As it is square
ae=2 ·
40. 4 = j-(a-c"""'"--2-)2_+_(_1___
b)-
2
p
vi/\ v.,,; b=-3 I b=5
a2= 13 a2=29
(from realation b2 =a2(1-e2)
(t)
Al \B (t+l)
ae =2
·

. 2
PB-PA=Vt+v-vt e=-
PB-PA= V (canst) .JlJ
locus of Pis Hyperbola
A & Bare foci cifHyperbola

www.puucho.com
CP Publication KVPY chapterwise with www.puucho.com

Differential Calculus
I
1. Let f : R --)> R be a differentiable function such
that f (a)= 0 = f (b) and f '(a) f '(b) > 0 for
some a < b. Then the minimum number of
roots off' {x) = 0 in the interval ( a, b) is- (201 O]
J (\
(A) 3 (B) 2 (C) I (D) 0 a b a b
Fig.I Fig.2
2. The roots of
(x-41)4 9 + (x- 49)41 + (x -2009)2009 = 0 are -

(A) all necessarily real


[2010}
rJ V
(B) non-real except one positive real root
a b a b
(C) non-real except three positive real roots
Fig.3 Fig.4
(D) non-real except for three real roots of
which exactly one is positive (A) Fig. 1 (B) Fig. 2 (C) Fig. 3 (D) Fig. 4

3. The figure shown below is the graph of the 5. Let V 1 be the volume of a given right circular
derivative of some function y = f '(x). (201 OJ cone with O as the centre of the base and A as
its apex. Let V 2 be the maximum volume of
y=f'{x) the right circular cone inscribed in the given
cone whose apex is O and v,rhose base is
parallel to the base of the given cone. Then the
a b C
ratio V2N1 is- 120101
Then- (A)~ (C) ~ 8
25 27 (D) 27
(A) f has local minima at x = a, b and a local
maximum at x = c
(B) f has local minima at x = b, c and a local 6. Let/ (x) = cos 5x + A cos 4x + B cos 3x
maximum at x = a + C cos 2x + D cos x + E, and
(C) f has local minima at x = c, a and a local
maximum at x = b
(D) the given figure is insufficient to conclude
T=J(O)- r(; )+ r( )-r( )+ . . .
2; 3;

any thing about the local minima and local


maxima off
... r(S5 )-r( 9;)
n Then T (20111

(A) depends on A, B, C, D, E
4. The following figure shows the graph of a
(B) depends on A, C, E but independent of B
differentiable fLmction y = f(x) on the interval
andD
[a, b] (not containing 0).
(C) depends on B, D but independent of A, C, E
y = f(x) (D) is independent of A, B, C, D, E

~ 7. Which of the following intervals is a possible


domain of the fimctionf(x) = logM [x] + l.o~xl {x},
where [x] is the greatest integer not exceedingx
a b
and {x} =x- [x]? [2011]
Let g(x) = f{x)/x which of the following is a
possible graph of y = g(x) ? [20101
(A)(O, I) (B)(l, 2)
(C) (2, 3) (D) (3, 5)

www.puucho.com
CP Publication KVPY chapterwise with www.puucho.com

TOPIC-WISE KVPY [STREAM SX] Questions with Solutions


8. If_t(x) = (2011 + x)", where x is a real variable 14. Let f: R 4 R be the function
and n is a positive integer, then the value of f(x) = (x - a1)(x - a2) + (x- a2)(x a3)
j .,. (0) r(n-1) (0) + (x a3) (x a1)
/(0)+ f'(O)+--+ .... +· is [2011)
2! (n -1)! with a1, a2, a3 "' R. Then f(x) ~ 0 if and only if
(A) (2011 )" (B) (2012)" (2012]

cq c2012r- 1 co) 11(2011)" (A) At least two of a 1, a2, a 3 are equal


(U) a1 = a2 = a3
9. The minimum distance between a point on the (C) a1, a2, a3are all distinct
(D) a 1, a 2, a3 , are all positive and distinct
curve y = e" and a point on the curve y = log.,.t"
is- [20ll]
15. A polynomial P(x) with real coefficients has
(A)-1 (B) -..12 the property that P"(x) ,t: 0 for all x. Suppose
.fi. P(O) = 1 and P'(O) = -1. What can you say
about P(I)? (2012]
(C) Jj (D) 2./2 (A)P(l)~O (B)P(l)ctcO
1 1
(C) P(l) ~ 0 (D) - - < P (1) < -
10. Let f{x) = x 12 - x9 + x4 - x + 1. Which of the 2 2
following is true? [2011)
16. Define a sequence (a 0 ) by
(A) fis one-one a1 = 5, a 11 = a I a2 ... al}-- 1 + 4 for n > L Then
(B) fhas a real root
(C) f' never vanishes
-
11m-- F: (2012]
n----,co an~l
(D) flakes only positive values 1
(A) equals - (B) equals 1
2
11. The maximum possible value of ')
(C) equals ..::. (D) docs not exist
x2 + ·:/ - 4x - 6y, x, y real, subject to the 5
condition ] x + y I + I x - y I = 4 [2011]
(A) is 12 (B) is 28 x+l
17. Let f(x) =- -
for all x :;c 1. Let
(C) is 72 (D) does not exist x-1
f 1(x) = f(x),f 2 (x) = f(f(x)) and generally

sin(x-a)+sin(x+a) h fn(x)=f(f"- 1(x)) forn> 1


12. L et f( x) =- - - - - - - , t en [20121
cos(x-a)-cos(x +a) Let P = f(2)f\3)f\4)f(5)
Wbich of the following is a multiple of P ?
(A) f(x + 2n) = f(x) but f(x + a) ,t. f(x) for any (20121
0 <a< 2rr (A) 125 (B) 375
(C) 150 (D) 147
(B) f is a strictly increasing function
( C) f is strictly decreasing function 18. Let f : R ~ R be a function such that
(D) fis a constant function Jim f(x) = M > 0. Then which of the
X----l-00

following is false ? 120131


13. The mid- point of the domain of the function
(A) lirn x sin(I / x)f(x) = M
X-HO
f(x) = ~4--hx+5 forrcal xis 12012]
(B) lim (f(x)) = sin M
X-----)<ll

(A)_!_ (B) I (C) Jim xsin(c-x)f(x) = M


4 2
2
(C) 3. (D)-- (D) lim sin x. f(x) = 0
3 5 X-----)<0 X

www.puucho.com
CP Publication KVPY chapterwise with www.puucho.com

• DIFFERENTIAL CALCULUS .

19. Let f(x = ~2- x -x 2 and g(x) = cos x. Which 61 2


23. The value of the limit lim ( ~ ) " is -
of the following statements arc true? [2013] x-~O Sinx
(D Domain of f((g(x)) 2) = Domain of ftg(x)) (201SJ
(II) Domain of f(g(x)) + g(f(x)) ,,., Domain of (A) e (B) e-1
g(f(x)) (C) e-116 (D) e6
(ill) Domain of f(g(x)) = Domain ofg(f{x))
(IV) Domain of g((ftx))') = Domain off(g (x)) 24. Among all sectors of a fixed perimeter, choose
(A) Only (I) the one wi1h maximum area. Then the angle at
(B) Only (I) and (II) the center of this sector (i.e., the angle between
(C) Only (Ill) and (IV) the bounding radii) is - [2015]
(D) Only (I) and (IV)
(A) 2:.
3
(B) !2 (C) Ji (D) 2

20. Let f(x) be a non-constant polynomial with real


25. Let a > 0 be a real number. Then the limit
·coefficients such that{½) = 100 and f(x):::; 100
[2015]
for all real x. Which of the following statements
is NOT necessarily true? [2013] 4
(A) 2 log a (B)- - a
(A) The coefficient of the highest degree term 3
in f(x) is negative 2
2
(B) f(x) has at least real roots (C) ~ (D) - (1-a)
2 3
1
(C) If x * -.,_ then f(x) < 100
"
(D) At least one of the coefficients of f(x) is 26. Let f(x) = a. x2 - 2 + .!. where a. is a real
X
bigger than 50
constant. The smallest a for which f(x) ;::-: 0 for
all x > 0 is - [2015]
21. Consider the function
(B) f_
X= {X + S if X;i: 2 33
/( ) x-2
I
·r
l X =
2
(D) ~
33
Then/(f(x)) is discontinuous [2014]
(A) at all real numbers
(B) at exactly two values of x 1.7. If P(x) be a polynomial with real coefficients
(C) at exactly one value ofx such tbat P(sin2x) = P(cos2x), for all x e [O, x/2].
(D) at exactly three values ofx Consider the following statements: 12016]
I. P(x) is an even function.
22. Let/: [O, tt] -;, R be defined as II. P(x) can be expressed as a polynomial in
sin x, if xis irrationaland xe[0, 1t] (2x- 1)2
{ I. P(x) is a polynomial of even degree
f(x)= tan 2 x, ifxisrationalandxe[0,1t] ·
Then.
The number of points in [O, 1t] at which the (A) all are false
function/is continuous is [20141 (B) only I and II are true
(A) 6 (B) 4 (C) only II and 1H arc true
(C) 2 (D) 0
(D) all are true

www.puucho.com
CP Publication KVPY chapterwise with www.puucho.com

TOPIC• WISE KVPY [STREAM SX] Questions with Solutions


28. Nwnber of integers k for which die equation 32. The range of the polynomial p(x) =4x3 - 3x as
x3 - 27x + k = 0 has at least two distinct integer
roots is - [2016) x vanes 1, 1
· overthe .interva I ( - 2 2 J·is [2016]
(A) 1 (B) 2
(C)3 (D)4 (A) [-l, I) (B) (-1, IJ

29. The graph of the function f(x) = x +.!.sin (27tx).


(C) (-1, I) (D) (-½,½)
8
0 S x s 1 is shown below. Define 33. Let f{x) be a non-negative differentiable
f,(x) =f{x), f 0 ... ,(x) = f{fn(x:)), for n ~ I
fimction on [O, co) such that f'(O) = 0 and
1
f '(x) :s; 2f(x) for all x. > 0. Then, on [O, co)l2016]
(A) f(x) is always a constant function
(B) f(x) is strictly increasing
0.5 (C) f(x) is strictly decreasing
(D) f '(x) chanses sign

34. The value of the limit lim (..J4x 2 -x +2x)


ll-+-00
0.5
is (5 Nov, 2017]
Which ofthe following statements are true ?
I (D) _!_
I. There are infinitely many x e [O, l] for (A)-a:, (B) - - (C) 0
4 4
which lim fn(x) = 0
n-t,:o
II. There are infinitely many x e (0, l] for 35. Let R be a rcfation on the set of all natural
numbers given by a Rb ~ a divides b 2•
which lim fn(x) = _!_
n--t<.0 2 Which of the following properties docs R
III. There are infinitely many x E [O, 1] for satisfy? [5 Nov, 20171
which lim fn(X) = 1 I. Reflexivity
11-toCO II. Symmetry
rv. There are infinitely many x e (0, I] for III. Transitivity
which lim fn(x) does not exist (2016] (A) I only (B) Ill only
II-too
(C) l and III only (D) I and II only
(A) I and III only (B) II only
(C) l, ll, III only (D) I, ll, Ill and IV
36. Let R be the set of real numbers and f: R 4 R
30. Limit lim x 2 le0
x 1
1 -x
1
dt equals (2016J be defined by f(x) = {x} , , where [x] is the .
X--1-00
l+[xr :·
(A).!_ (B) 2 (C) a; (D) ~ greatest integer less than or equal to x, and ,,
3 3 {x} = x - [x]. Which of the following
,t
t
statements are true ?. '
31. The polynomial equation
I. The range off is a closed interval
x3 - 3ax2 + (27a2 + 9)x + 2016 = 0 has [20161
II. f is continuous on R.
(A) exactly one real root for any real a
(B) three real roots for any real a
111. f is one-one on R. I5 Nov, 20171
(A) 1 only
(C) three real roots for any a 2:: 0, and exactly
(B) Uonly
one real root for any a < 0
(C)III only
(D) three real roots for any a 5: 0, and exactly
(D) None ofl, II and Ill
one real root for any a > 0

www.puucho.com
CP Publication KVPY chapterwise with www.puucho.com

• DIFFERENTIAL CALCULUS
37. Let x 0 = (2 11 + 3nt2n for all natural numbers n. (A) I fa true and II is false
Then [5 Nov, 20171 (B) I is false and II is true
(A) Jim Xn <XI (C) I and II both arc true
1\---l«)
(D) 1 and 11 both are false
(B) lim
n-wo
Xa =Ii
(C) lim
n-i-ao
Xn =./3 + Ji 41. The number ofeontinuous functions f:[O,l]----)R
that satisfy [5 Nov, 2017)
(D) lim Xn = Js (A) 0 (B) I
n~co
(C) 2 (D) infinity
38. Consider the following parametric equation of
a curve: 42. A sector is removed from a metallic disc and
x(8)=1cos40 lcos8 the remaining region is bent into the shape of a
y(8) = I cos 48 I sin e
circular conical funnel with volume 2-JJ re.
for Os 0 s 2:n:
The least possible diameter of the disc is
Which one of the following graphs represents
[19 Nov, 2017)
the curve? [S Nov, 20171
(A) 4 (B) 6
1.0 y (C) 8 (D) 12

43. Let an = f"' Ix - I Icos nx dx for all natural


-n

numbers n. Then the sequence (an)n2.o satisfies


I19 Nov, 2017}
(A) Jim a 11 = co
n-i-co

(B) lim a 0 = -a)


tl-t<'l

(C) Jim an exists and is positive


ll-+C<l

(D) lim an
n-~i::o
=0
39. A solid hemisphere is mounted on a solid
cylinder, both having equal radii. If the whole 44. Let f(x) = log(l + x.2) and A be a constant such
solid is to have a fixed surface area and the that I f(x) - f(y) I S" A for all x, y real and
maximum possible vol umc, then the ratio of the I x-yl
height of the cylinder to the common rndius is x ¢ y. Then the least possible value of A is
(5 Nov, 2017] [19 No,·, 2017)
(A) 1:1 (B) 1:2 (A) equal to I
(C) 2:1 (D) Ji,: 1 (B) bigger than l but less than 2
(C) bigger than O but less than 1
40. Let R be the set of real numbers and f: R--+R (D) bigger than 2
be given by f (x) :::: -vfxf - log(l + jxj) . We now
make the following assertions:
I. There exists a real number A such that
f{x) 5.A for all x.
TI. There exists a real number B such that
f{x} ~ B for all x. [5 Nov, 2017]

www.puucho.com
CP Publication KVPY chapterwise with www.puucho.com

TOPIC-WISE KVPY [STREAM SX] Questions with Solutions

ANSWERS
1. (D) 2. (B) 3. (C) 4. (B) 5. (C) 6. (C) 7. (C)
8. (C) 9. (B) 10. (D) 11. (B) 12. (D) 13. (B) 14. (B)

15. (B) 16.(C) 17.(B) 18. (C) 19. (B) 20. (C) 21.(B)
22. (D) 23. (A) 24. (D) 25. (D) 26. (D) 27. (C) 28. (B)

29. (D) 30. (A) 31. (A) 32. (C) 33. (A) 34. (D) 35. (A)

36. (D) 37.(B) 38. (A) 39. (A) 40. (8) 41. (B) 42. (B)

43. (D) 44. (A)

1. f'(a) . f '(b) > O '( +. . (c+h)f'(c+h)-1


g C J = 11 m - - - - - - : : : - - - < 0
so either both are positive or both are negative b--.0 (C + h)2
f(a) = f{b) = 0
(f'(c + h) < 0)

0
a~b 5.
hence fig.(2)

f '(x) = 0 A

2. (x - 41) 49 + (x - 49)41 + (x - 2009)2°09 = 0


f(x) = (x -41 )4 9 + (x -49) 41 + (x - 2009)w09
f' (x) = 49(x-41) 48 + 41(x -49)4°
+ 2009 (x - 2009)48 > 0
hence f(x) will cut x-axis only once. 0 R
It real rooij MBC and MOP are similar
.!1_ = H ::::>h= rH
3. f'(a) = f '(b) = f'(c) = 0 r R R
f'(a-) < O f'(a +) > 0] minima at a & c
f'(c-) < 0 f'(c~) > 0
I __2 1t • 2 ( 1--
V2=-m(H-h)=-rH
3 3
r)
R
f'(bl > 0 f'(bl <O ] max. at b.

4.
dV2 3r 2 2R
- =2r- - =0::) r= -
(T\ V,
dr
"" 41tR2H
R 3

'' ~max 81
a C b 1tR 2 H
f'(c) = 0 f'(c-) > 0 f'(cl < 0 V1=--
3
g(x) f(x) ::::> g'(x) = xf'(x)-1 V2 = _i_
X x2 V1 27

www.puucho.com
CP Publication KVPY chapterwise with www.puucho.com

, DIFFERENTIAL CALCULUS

6. Clearly f(n + x) + f(n -x) 10. f(x)=xg(x3 -l)+x(x3-I)+ l


(every term contain cosine) positive for x ~ I or x ::; 0
{f)=r(95x} {25x)={s;), =1-x+x4 -x9 +x 12
positive for x E ( 0, I)

t( x)= r(7;), 1(~n }= 1( t)


3
5
6
f(x) is always positive

11.
T=f(0)-2 [r(i)+r(3; )] (0, 2)
y
(2, 3)
• ., , ;

+2 (r( )+{:x )]-f(n)


2; ,. ,. .
,. ,.
,. .,
f{O) - f{n:) = 2(1 + B + D) ., ,.
.,.
,. .,. 0 (2,
1(~)+ 1(3;)=1(f)-1 4
sn:)
C,,. ,.
. ,. .,
.
=2(l+Bcos 3; +Dcos~) / (-2, -2)

r( n) + { :x) =r( x)- r(3 n)


25 25 5 1x + y I+ Ix - y I=4 represent a square
x2 + y2 -4x-6y= (x-2)2 + (y-3}2- 13
1t)
=2(1+ Beas 6; + Deas 25 = (distance point on square from (2, 3))2-13
Maximum= (- 2 -2)2 + (-2-3)2-13
T => contains only B, D tenns
=28
7. x If I & [x] > 1
c:=> x E (2, 3) only option satisfy. 12. f(x) = sin(x - a)+ sin(x + a)
cos(x- a)- cos(x + a)
8. (201 It+ nC,(2011),.._1 + 0 Q(2011)n--2 +... 2sin(x).cos a
=----=cot a
.... + 0Cn-12011 + PCn-1 2sinx.sina
= (2011 +It-I
9.
13. f(x)=~4-~/2x+5
4 - ,/2x + 5 ~ 0, 2x+5:2'.0

,Jzx+ S ~4. x~--


s
2

XS .!_!
2

X 6 [-~
. 2' 2
!_!]
5 11
--+- 3
Ilnd curve Mid point = 2 2 =-
2 2
y'= ..!.
X
14. Only when a! = a2 = a3
x= l =} point (1, 0) In other cases f{x) wm take both positive and
similarly 1st => point (0, I) negative values
distance = .Ji

www.puucho.com
CP Publication KVPY chapterwise with www.puucho.com

TOPIC• WISE.KVPY [STREAM SX] Questions with Solutions


15. AB P"(x) ¢ 0 for all x. So P(x) can be a
18. (A) lim l<.Sin(.!_)f (x)
polynomial of degree greater than or equal to 2. x-+ao X
but situational benefit must be given to P(x:) as
quadratic expression. liin (sin(l/x)) lim f(x) =M
x-+iii 1/ X x--tao
So let P(x) = ax2 + bx+ c
=-
P(O) = 1 c = 1 (B) lim sin(f(x)) = sin[ Jim f(x)] = sin M
X-+CO X--t00
P'(O) '-"-~ ~ b=-1
P"(0} ¢ 2a ;1, 0
-
- (q Jim xsine- 1 f(x)= iim (~)( sin_e-xJ(f(x))
So P(x) =ax2 -x + l x-.oo ,....... e11 l.. e x
:. P(l) =a¢0
So P(l) ¢ 0 (D) lim smx .f(x) =0
x--toc X
but if we consider P(l<) as polynomial of degree
than 2. then all options (A), (B), (C). (D) can be
correct answers, as given information will not be 19. ftx) = .J2-x -x2 ; g(x) = cos x
sufficient to find the result. So option (B) is the Domain, Dr => x2 + x - 2 S 0
most appropriate answer. Although KVPY has => (x + 2)(x - 1) S: 0 => x e [-2, 1)
[o, f]
mentioned only option (C) as answer in its answer
key. Rangeoff e

16. a1 =5 DeeR
a11 = a1az ..... an--1 + 4 R11 e [-1, I]
az=a1 +4= 9 (I) Now; D((f(g(x))2) :::>-1 S cos2 x s 1
=
a.3 = a1.a2 + 4 5 x9 + 4 =49 => cos2 x 5: l
114 = a1a2a3 + 4 = 2209 also D (f(g(x)))::::, cos x e (-1, 1] ~ true
as = a1a2BJB.t + 4 = 4870849 = (2207)2 (II) Drog R & Dgor is [-2, I]
as =(114-2)2 Dro3 (') Dgor = Dgof
14 = (49-2)2 = (83 -2)2 (III) Clearly false from above
a3 = (9-2)2 =(a2 -2)2 (IV) DgofE [-2, 1]
an = (ao-1 - 2)2 D,08 E R Hence false.
Ja: =an-1-2
.Ja:
--=--=-~-
-2an-I 20. r(½) = 100
an-I an-I

= 1- Il. f f l 2
--

·: lffl an-I _, a.,
f(x) = a(x- ~) (aoXn-1 + a1xn-2 + a2xn--3 + ...
n-~:,o aD-I D-ioa>
... + an--1) + 100
=1
lf f{x) S 100 v' x e R
17. f(x) = x+l :. a < 0 and f(x) must be even degree
x-1 polynomial.
x +I l Since there may be more value of x at which
f 2 (x) =f(f(x))=f(x+l)= ~+ x f(x) attains its maximum so C maybe false.
x-1 x+I_ 1
x-1 21. Discontinuous at x =2
f 3 (x) = f(x) = x+ 1 ( x+S +s)
x-1 ftf(x)) ... Jx+S)= N
f 4 (x)=x ·'\x-2
(x+S _ 2)
P = f(2).f 3 (3)f3 (4)f 4 (5) x-2
6x-5 6x-5
5
P=3x3x-x5=75 - -x+9
- - = 9-x
--
3
Multiple of Pis 375 At x = 9 it is discontinuous

www.puucho.com
CP Publication KVPY chapterwise with www.puucho.com

DIFFERENTIAL CALCULUS
• sinx x!l!Q . a x£n a - a3-x£na
22. f{x) = { 2 Q = 11m
tan X XE
x--+l -a 3-"lna - _!_a "12lna
ifis continuous at x = 0, re 2
so 2 points = a2 ina-aina = a 2 -a =~(I-a)
sinx = tari2x ::::) sin x(cos2x - sin x) = 0 a 3a 3
-at'na --lna --
sinx=O 2 2
x=O,x
sin2x + sinx - I = 0 2 1
26. f(x)=ax -2+ -
-1±./5
sinx=---
X

2
f(x)= axJ-2x+l 'v'x(O,oo)
. .Js-1
smx = - - 2 values
X

2 so cj(x) =a.x:3 - 2x + I should be positive


Total 4 points f(X) = a.x3- 2x + I
f'(x) = 3ax.2-2 = 0
23. lim
JC--+O
(-x-) sinx
6 2
"' (l""form) => X=± /2
v~
li,n _!_(2....-1)
_
-e :1.2 sinx
.1;--tiQ

. 6 (x---,;inx)
Clearly x = .[!; point of minima
Inn- --
~ :11.-tO ::.?
-e sinx

{&)~o
&{a. 3:-2} + 1~o
.{!;(-j)+1 ~o
&(1)~1
&5¾
=e•
-2 s 3-2
2

3o. 4
p
24. Given that 2r + r8 = P ::::) r = - - a.~ 32
2+0 27
27. P(sin2x) = P(colx)
P(sin2x) = P(l - sin2x)
P(x) = P(l -x) V x E [0, 11
Differentiable both sides w.r.t. x
P'(x)=-P'(l -x)
25.
lim a" + a3-" - (a2 + a)
x-~2. a 3-x - ax/2
(Q)0 So P '(x) is symmetnc. a bout pomt
. x,= -I
2
Apply L hospital rule So P '(x) has highest degree odd
=> P(x) has highest degree even

www.puucho.com
CP Publication KVPY chapterwise with www.puucho.com

TOPIC-WISE KVPY [STREAM SX] Questions with Solutions


l l • l
28. Let f(x) = x 3 - 27x . 2ex + e" + 3x'ex
= hm 1 ,
f'(x) = 3x2 - 27 = 3(x2 - 9) x--+:,, 3ex + 9x 3e"
l1. 3+3xJ l
-54 m--- - -
X--+OO 3 + 9x 3 3

31. f'(x) = 3x2 - 6ax + 27a2 + 9


-3 3 = 3[x2 - 2ax + 9a2 + 3]
= 3((x - a}2 + 8a2 + 3)
-54
:. f'(x) is+ ve for x E R so f(x) is monotonict
for x ER.
As sum of the roots is zero, so if two roots are
integer then 3 rd root has to be integer
Now putx = 6t
32. P'(x) = 12x2 - 3 = 3 (4x2 - I)
216 t3 - 27 X 6t + k = Q
54 ( 4r' - 3t) + k = 0
1n(-½,½) P'(x)<O

Putt=cosO ~ P(x) is decreasing


54 cos 38 =-k ~ Range E (P(-1), P(l))
Now for 30 = 0, 2n we get integral solution Range E (-1, I)
So two values of'k'
33. f'(x) .$; 2 f(x)
29. lim f 11 (x) = f(f(f( ........ .oo times (x)) f'(x)e-2< ~ 2 f(x)e· ix
n->o:,
~(f(x)e-ix) ~ 0
Nowforx 1 E (o, ½) dx
g(x) = f(x)e-2xis non-Increasing function
· f(x1) > x 1 as f(x) is concave - downward x~O
1 g(x) ~ g(O)
Thus f11 ---+ - as n ---+ oo
2 f(x)c-2x ~ f(O)c-0
f(x)e-h .$; 0
Similarlyforx1 E (½, 1) t(x):;; 0 but given f(x) is not negative
f(x1) < x 1as f(x) is concave upward :. f{x) =0 Constant function
1
Thus f11 ---+ - as n ---+ oo 34. Rationalise
2
X

x 2 Jeildt
lim ((J4x 2 -x +2x)x 4 x 2
)\7"~
.J
J4x 2 -
-xx --2xx)
2

30. Jim o
x--+«> ex'
Apply L Hospital

J
X

2x e 11 +x 2 e"1
lim [
X-+--«l
R ]
lxl 4---2x
X
1 at x---+----a:) lxl =-x

lim u
X-+"l 3X:2 ex)

2f etl +xexi
.
x~~
[
R
-x 4-- -2x
-x

X
]
=2+2=4
l l

= 1im~
0---
x-+,., 3xexl

www.puucho.com
CP Publication KVPY chapterwise with www.puucho.com

• DIFFERENTIAL CALCULUS

35. (I) This relation is reflexive relation because


every natural no. divides square of itself
a Rao a divides a2
40. -
graph of given function actually look like this

(II) not symmetric cg. 5 R 10::::, 5 Divide 100


2
Hut 10 R 5 -;;;p I ODivide 25
(IlI) Not transitivity for example -9 0 x=9
if 8 R 4 & 4 R 2 =t> 8 R 2
only (I) Option
Clear from graph option (B) is right.
{x}
36. f(x) = I +[x]2 41. Given equation can be written as

X +}
-l$x<0 - 31 = J - 2 -x )2 dx- J
J(f(x) l X 2dx
2 ' 0 0
x·• 0$x<l
x-1
f(x) = - -; l:,';x<2
2
x-2
-- 2::;;x <3
5 '

So-on
So, no of continuous function is I.
Now'check accordingly

42.

)~(3")"'"[(¾)' +If"
Q
37.

Put lim ../3


n-+""

for a cone
38. Make graph and observe yourself
r will be cone slant height
V= 2-J3
39. S = 2rrR2 + 2rcRb + rrR2
Let x radius of cone
(R = radius of hemisphere & cylinder)
h be height then
2 3 ~
V= -1tR +1tR-b 1 2 r,;
3 -m h =2v3 11:
3

x2h = 6-./3 ~ x2 = 6,/3


V= ~1tR3+1tR2 X (5-31tR2)
3 21tR h
least Diameter::::, least slant height of cunc
dV =Z1tR2+ ~- 9'1t R2
dR 2 2 £2=x2+ h2
r2 = x2 + h2
r. •
tor maximum & nurumum
. . -dV =O
dR r2= 6../3 + h2
51tR2 = S h
51tR2 = 31tR2 + 21tRh Diff. w. to x
2R=2h dr
2r- = - - +2h
-6-./3
h:R=l:1 dh h2

www.puucho.com
CP Publication KVPY chapterwise with www.puucho.com

TOPIC-WISE KVPY [STREAM SX] Questions with Solutions

dr 2x
for maximum & minimum - = 0 44. f'(x) =
dh l+
h= Jj => h2 = 3 2x
Range of--~ e [-I, lJ
2
X =--
6.J3 1+x-
.J3 i f(x)- f(y) I s; A means
x 2 =6
I x-yl
r 2 = 6+3
2
r =9 maximum value of I f(x) - f(y) I is always
r=3 lx-yj
d=2r
d=6 less than or equal to A.
So, least value of A is 1
I n
43. an= J-cx-l)cosnx dx + J(x-l)cosnx dx
-n l
I II
Solve I & II part by I LATE
2 . mi: 2 2
an = rr sm - + --:;- cos Ill[ - 2 cos n
n n- n
lim an
l(_--;,<O
=0

www.puucho.com
CP Publication KVPY chapterwise with www.puucho.com

Integral Calculus
1. The following figure shows the graph of a
continuous function y= f(x) on the interval [1, 3].
----=====- -
5.
2r.
The value of Jmin{lx-nl,cos-1 (cosx)} dx
I
0
The points~ B, C have coordinates (1, I), (3, 2),
is- 120101
(2, 3) respectively, and the lines L 1 and 4. are
2 2 2
parallel, with L1 being tangent to the curve at (A)!:..._ (B) ~ (C) !:..._ (D)1t2
C. If the area under the graph of y = ftx) from 4 2 8

x = I to x =3 is 4 square units, then the area of


tbe shaded region is- 120101
6. Let/: (2, oo) ---i> N be defined by/ (x} = the
8
largest prime factor of [x]. Then Jf(x)dx is
2
equal to- (2011]
(A) 17 (B) 22 (C)23 (D)25

7, Let [x] denote the largest integer not· exceeding


1 3
x and {x} =x- [x]. Then
(A) 2 (B) 3 (C)4 (D) 5 2012 cos( 1Ci.x})
J (xx,e}) +e_
ecos
C ( })
cos,-; .x
dx is equal to12011)
I 0
2. Let In = J(logxt
o
dx, where n is a nan- {A) 0 (B) 1006
(C) 2012 (D) 2012ri:
negative integer. Then 120m - 2011 I2ow is

l
equal to- 12010) 8. The value of
(A) I moo + 999 l99s (B) ls90 + 890 lss9
. ( --,==
1tnl l l
+---;::::==+ 1
.... +---;====
(C) l100 + 100 lw (D) lsJ + 54 ls2
n--1>®.J4n 2 -1 .J4,1 2 -4 .J4n 2 -n 2
is - [2011]
3. Consider the regions A= {(x, y) I x2 + y2 S 100}
and = {(x, y) I sin (x + y) > O} in the plane. (A)_!. (B)~ (C) ~ (D) ~
4 12 4 6
Then the area of the region An B is- 12010]
(A) IO 1t (B) I 00 9. Let f(x) = x 3 + ax2 +bx+ c, where a, b, care
(C) I001t (D) 50n real numbers. If f(x) has ~ local minimum at
x =I and a local maximum at x = - _!_ and
4. Let f : R 4' R be a continuous function 3
I

J Jf (x) dx equals-
X

satisfying f(x) = x + f(t) dt, for all x e R. f{2) = 0, then 120111


-I
0

Then the number of elements in the set (A) 14 (B) -14 (C) 2 (D) -7
S = {x E R; f(x) = O} is- 3 3 3 3
[20101
(A) 1 (B) 2 (C) 3 (0)4

www.puucho.com
CP Publication KVPY chapterwise with www.puucho.com

topic- ~ISE ~PY [STREAM SX] Que~~i_o~_s with_ Solut~C?hs


10. For each positive integer n, define 16. for x, t e R, let
xn (1-xt)
fo(x) = minimum ( - , - - - , for O s; x s; I.
p1(x) = (sin t) x 2 (2cos t) x + sin 1 be a family
n! n! of quadratic polynomials in x with variable
1

Let In=
l
Jf (x)dx, n ~ 1.
11 Then In=
~

1) 11 is coefficients. Let A(t) = fPt (x)dx _ Which of


0 n~l 0
equal to- [2011] the following statements are bue? f-2013]
(A) 2-Je-3 (B) 2.Je-2 (I) A(t) < o for all t
(C) 2Fe-1 (D) 2-fe (II) A(t) has infinitely many critical points
(III) A(t) = 0 for infinitely many t
,r./2 (IV) A'(t) < 0 for all t
J(sin x)-fi+i dx (A) (I) and (II) only (B) (II) and (III) only
0
11. The value rt/2
lS [2012] (C) (Ill) and (IV) only (D) (IV) and (I) only
J(sinx)Ji-•dx
0 17. For real x with -10 S:: x :<; 10 define
(A) /'i.+I (B) .fi.-1 X

Ji-1 .fi +1 f(x) == Ji 11 dt, where for a real number r we


-IO
(C) ,12+1 (D) 2-.fi denote by [r] the largest integer less than or
,Fi equal to r. The number of points of
2012 discontinuity off in the interval (-10, 10) is-
12. The value of f(sin(x )+x
3 5 +l)dx is (2012] [2013]
-20ll (A) 0 (B) 10 (C) 18 (D) 19
(A) 2012 (B) 2013
(C) 0 (D) 4024
18. For a real number x let [x] denote the largest
13. Let [x] and {x} be the integer part and integer less than or equal to x and {x} = x - [x].
fractional part of a real number x respectively. The smallest possible integer value of n for
5 n
The value of the integral J[x]{x}dx is [20121 which J [x]{x}dx exceeds 2013 is~ [20131
0 I
(A) 5/2 (B) 5
(A) 63 (B) 64 (C) 90 (D) 91
(C) 34.5 (D)35.5

The value of the mtegral . f-cos-x


1f


,

dx, where
19. The area bounded by the curve y = cos x, the
14.

a> 0, is
-It
l+ai,:
12012]
line joining (-¾, cos(-¾)) and (0, 2) and

(A) 11: (B) an (C) rJ2 (D) 21t


the line joining (:, cos(:)) and (0, 2) is-
15. Consider
(2013]
L = 1/2012 + V2013 + .... + \/3011
R =~lzo13 +1/2014+ .... +~3012 (A) (4+/2}-.Jz (B) ( 4+8.Jz}+.Jz
3012

and I = P.,fxctx [2012]

(A) L+R< 21
2012
Then

(B) L+R>2I
(C) ( 4+/}-.Jz (D) ( 4+/)x+,/2
(C) L+R= 21 (D) ..fLR=I

www.puucho.com
CP Publication KVPY chapterwise with www.puucho.com

• INTEGRAL CALCULUS

20. Let n be a positive integer. For a real number x, 24. For a real number x let [x] denote the largest
let [x] denote the largest integer not exceeding integer less than or equal to x and {x} x- x]. r
t•l
x and {x} = x - [x]. Then JI
n+I ( {
x
}
dx is
Let n be a positive integer.
n
Then

equal to
[X]
[20131
Jcos(2n[x]{x}) d.x is equal to (20141
0
(A) loge(n) (B) _1_ (A) 0 (B) I (C)n (D)2n-1
n+l
(C) _n_ 1 l
(D) l+-+ ... +- rr/2
n +1 2 n 25. Let In = Jx n cos x dx , where n is a non-
o
21. For a real number x, let [x] denote the largest
negative integer.
number less than or equal to x. For x E R, let
/(x) = [x] sin n. Then (2014] "' (I
Then """"' .....!!.. + rn-:l. ) equals -
~ n! (n -2)!
(20141
(A) f is differentiable on R.
(B) f is symmetric about the line x = 0. (A) erra - 1 ~; (B) eefl. - l
(C) J~/(x)dx = 0 . (D) errn.
(D) For each real a, the equation/ (x) - a =0
bas infinitely many roots.
26. For a real number x let [x] denote the largest
22. Let/: IO, I] ---:), IO, oo) be a continuous function integer less than or equal to x. The smallest
l positive integer n for which the integral
such that J f(x)dx = 10. Which of the n
J [x][.,/;Z]cL1: exceeds 60 is - (20141
0

following statements is NOT necessarily true? l

[2014) (A)8 (B) 9 (C) 10


I
(A) fc" f(x)dx S: 10 27. Define a function f: R -4 R by f(x) = max {lxl,
0 Ix - II, ... , Ix - 2nl} where n is a fixed natural
(B) J(I+f(x)x)2 dx ::; 10
I

number. Then
2o

Jf(x)dx is - {2015]
0 0
l
(D) 3n2
(C) - IO S: Jsin(IOOx) f (x) dx:::; IO (A) n (B) n2 (C) Jn

0
28. If p(x) is a cubic polynomi'al with p(l) = 3,
I
(D) J f(x)2 d'< S: 100 p(O) = 2 and p(-1) = 4, then
1
Jp(x)dx is -(2015]
0
-1

(A)2 (B) 3 (C) 4 (D) 5


23. A continuous function f : R ----t R satisfies the

J
X
29. Let x > 0 be a fixed real number. Then the
equation f (x) =x + f(t)dt. Which of the
J
,r,
0
integral e-1 I x - t 1dt is equal to - [2015]
following options is true? [2014)
0
(A) f (x + y) = /(x) +f(y) (A)x+2e-"-1 (D) X - 2e-;s + J
(B)f(x + y) = /(x)f(y) (C) X + 2e-"+ 1 (D) - x 2e-~ + 1
(C)f(x + y) =/(x) +f(y) +f(x.)J(y)
(D)f(x + y) =/(xy)

www.puucho.com
CP Publication KVPY chapterwise with www.puucho.com

TOPIC-WISE KVPY [STREAM SX] Questions with Solutlons


30. Let f : R ---+ R be a continuous function Where [x] denotes the largest integer not
exceeding x. The positive integer n for which
J"
satisfying flx) + tf(t)dt + x 2 = 0, for all x e R
J~ (g(x)- f(x)) dx = I 00 is (2016}
0
Then- [2015)
(A)IOO (B)l98
(A) lim f (x) = 2 (C) 200 (D) 202
X->"l

(B) lim f(x) - -2


X->---«> 35. Let f be a continuous function defined on [O, I]
(C) f(x) has more than one point in common
with the x-axis such that J/1 2 (x)dx = (J/(x)dx
J )2 . Then
(D) flx) is an odd functions
the range off [2016]
31. The figure shows a portion of the graph (A) has exactly two points
y = 2x - 4x3 • The line. y = c is such that the (B) has more than two points
areas of the regions marked I and II are equal. (C) is the interval [O, 1]
1f a, b arc the x-coordinates of A, B (D) is a singleton
respectively, then a+ b equals - [20151
36. The parabola y2 = 4x + 1 divides the disc
x2 + y2 ~ 1 into two regions with areas A 1 and
A2. Then I A1 -A2I equals 15 Nov,2017)

Y=C
(A) _!_ (B) ~ (C) ~ (D) ,:!.
3 3 4 3

37. Let f(x) = max {3, x 2 , x12 } for ½s x s 2. Then


2
(A) 2_
..fi
(B) 2_
,fi
(C)~
-,fi
(D) .2_
..fi
the value of the integral Jf(x)dx is [5 Nov, 20171
l/2

32, The area of the region bounded by the cl!n'e (A).!..!_ (B) ~ (C) .!i (D) _!i
3 3 3 3
y""" Ix' -4x + 3x I and the x-axis, 0 s x s 3, is
2
3
[2016] 38. Define g(x) = Jf(x.-y)f(y)dy, for all real x,
(A) 32, (B) 2. (C) 37 (D) 0 -3
6 4 12 I, ostsl,
where f(t) ={
33. The number of continuous function 0, elsewhere.
f : [O, 1] ---> [O, 1] such that f{x) < x for all x
2
Then [5 Nov, 20171
II
and f(x)dx = - is:
o
1
3
[2016] (A) g(x) is not continuous everywhere
(B) g(x) is continuous everywhere but
(A) 0 (B) 1 differentiable nowhere
(C)2 (D) infinite (C) g(x) is continuous everywhere and
differentiable evcryw here except at x = 0, I
34. On the real line R, we define two functions f
(D) g(x) is continuous everywhere and
and gas follows :
differentiable everywhere except at
f(x) = min{x- [x], 1-x + [xJ}
X = 0, 1,2
g (x) = max{x - [x], 1 - x + [x]}

www.puucho.com
CP Publication KVPY chapterwise with www.puucho.com

• INTEGRAL CALCULUS

39. J
Let In = ~e-Y l dy , where n is a non-negative 41. Let g(x) = J \i<l214
0
1
t 213 sin- dt , for all real x.
t
. f, In
mtcger. Then L., ~
.
JS (19 Nov, 2017} Then lim g(x) is equal to [19 Nov, 2017J
n=I n! ~-tO X

l (A) oo (B) ---00


(A) l (il) 1--
c (C)O (D) I
4
(C) .!. (D) 1+ .!_
e e

40. The area bounded by the cUIVe y = .!_ 14 - x2!


4
and y = 7 - ]x[ is (19 Nov, 2017)
(A) 18 (B)32
(C)36 (D)64

www.puucho.com
CP Publication KVPY chapterwise with www.puucho.com

TOPIC• WISE KVPY [STREAM SX] Questions with Solutions

ANSWERS
1. (A) 2. (C) 3.(D) 4. (A) 5.(B) 6. (B) 7.(B)

8. (D) 9. (B) 10. (A) 11. (D) 12. (D) 13.(B) 14.(C)
15. (A) 16.(B) 17. (A) 18. (D) 19, (A) 10. (C) 21. (D)
22. (D) 23. (C) 24. (B) 25. (A) 26. (B) 27. (D) 28. (D)

29. (A) 30. (.B) 31, (A) 32. (C) 33. (A) 34. (C) 35. (D)

36. (B) 37. (C) 38. (D) 39. (C) 40, (B) 41, (C)

. 2-1 e
1. Equat1on of £2---+ y- 1 = - ( x -1)
3-1 2. In= JI.(logxt dx
In I
2y-2=x-1
2y-x= I
=> In = (I ogx)D x le1 - Je -
n(logxr-I
- - . x dx.
l I X
=> slope of l1 "" -
2 => I0 =c-O-nl0 _,
Equation of l1 = y- 3 = .!_(x-2) => In + n In--1 = c
2 => l:2001 +2011 ho10"" e
2y-6 = x-2
P,oo + 100199 = 9
2y-x=4

D -+ ( l, I) , E -+ ( 3, i) 3, x2 + y2 s; 100 -+ inside of a circle


sin(x +y) > 0
x +ye (O,n:) u (21t, 3rt) .....
x + y = c -+ equation of a line

(l,Q)G F(3,0)
+(--..;i,.)
2

Arca under f(x) =4


Shaded area"" area of trapezium DEFG
-area under f{x)

=.!..(~+I..)
2 2 2
X2-4

=6-4
Required area= shaded region
=2
=-I x(l0) 2 = 50 1t
2

www.puucho.com
CP Publication KVPY chapterwise with www.puucho.com

t INTEGRAL CALCULUS
4, f" (x) = l + f(x);;::, f(x) = ex - 1 rr/2

f(x)=O ;;::,ex= 1 I x=O I One solution


11. 11 = J(sinx) 12 .sinxd.x
0
rt/2

5. I= Jxdx+
n/2 n
Jcn:-x)dx+ j(x-n:)d,x
3,r/2
12 = f(sinx)'12 - dx 1

0
0 rr./2 ,r
\,r/2
2n
+ f 2n:-xdx r, = [ (sin x) Jj Jsin xdx J,
3n:./2
1t1 1t1 n:2 n:2 1t2
=-+-+-+-=-
8 8 8 8 2 -Irr.l?(l-J2(sin xr'2-l cos XJsin Xdx )
0
8

6. Jrcx) dx = 2 + 3 + 2 + s + 3 + 7 = 22
{
= -,cos x (sin x) JI£\.,;~-
2
it/2

I cos11.r
f
+ -J2 (sinx)"12- 1 (1-sin 2 x)dx
7. 1=2012 J e+
Cil5.TT.l' -COS ITT
dx 0

OC e
Using king property

1=2012 I -cos,._,· +
I

e
e-00011.,

ecos ,tt
dx
0 2012

;;::, 21 = 2012 ;;::, I= 1006 12. f(sin(x )+x 3 5 +I)dx


-2012
2012 20n 2012
. ~
8. I1m 1
n-->w r=l ~411 2
]' 1 {,
£..J-;::::=== = 1m - £..J
-r 2 ,,...,, Jl ,,.]
1
}4-(rl n) 2
= J
-20\2
(sin(x 3 )dx + f
-2012
x\ix + J dx
-2012
= 4024

J
= l - -dx- = [·-•l(x))
Sm -
1 rr
S 5
o ~4-x2 2 o 6
13. J [x]{x}dx =J[x](x -(x])dx

( , 2 1)
(I 0

9. f'(x)=3 x-- 3 x- 3 =3x 2 -2x-l l 2 3

3 ~ .
J J
= O.dx + l.(x - l)dx + 2(x - 2)dx J
f(x) = x - x- - x + A 0 ! 2

f(2) = 8 - 4 -2 + 1c = 0 =:- l = -2 4 5

ft'.x) = x3 - x2 - x - 2 J
+ 3(x -3)dx + J4(x-4)dx

J1cx)dx=-2Jcx
l l .
2
( 1 l 14
+2r -
3 4

=( (x~l)'X +z[ (x~2)1


+ 2)dx-= -2 3 3
~ 0 .

10. I
n
= -X I n! dx + f -'-------'--
112 n
( n! dx I '\
-X
)"

+{<x~3)'I +{ (x~4)'I
O 112

= (n+l)l
l [r·,,2).,,n+l + (12Jn+l] = Gr
+ (n 1
1 2 3 4
=-+-+-+-
2 2 2 2
"'5

"'
~In= [1/2
- 1 (1/2)1 -+ ......J-':>
+--
2
r _3
- .. ..,;e
n-1 2. 3.

www.puucho.com
CP Publication KVPY chapterwise with www.puucho.com

TOPIC-WISE KVPY [STREAM S)ij Questions with Solutions


K 2 X
cos x
I= J --dx
.1 +ax
.... (1) 17. f{x) = Ji 11 dt
-n: -IO
b b
For -10 :S: x <-9
Since Jr(x)dx=Jf(a+b-x)dx X

K
a
2
a
f(x) = Ji- 10 dt
I=Jcos (-x)dx ... (2)
-IO

,-;.,
l+a-" = r 10(x + lO); -10 :S X <-9
It X 2 For-9:S:x<-8
I= Ja cos xdx
f
X

-n:
l+a" f{x) = z[t] dt
add equation (I) and (2) -IO
-9 X
K
2I:::::Jcos
-n'
I + - ax -) dx
2 x( - -
l+a" I+ax = J z-10 dt + Jr 9 dt
-ID -9

:::) r = J
" r./2
J
cos 2 x dx = 2 cos 2 x dx =; f(x.) = T 10(I) + 2---9 (x + 9); -9 ~ x <--&
For-8:,; x:,; _7
0 0

J
X

15. L = V2012 +V2013 + .... +V301 l ... (1)


f(x) = iitJ dt
-10
R = ¾'2013 +V2014 + .... +V3012 ... (2) --9 -8 X

[=
3012
Jx 113 dx
= J i- 10 dt + J 2-9 dt + Jie J dt
1

-10 -9 -6
2012
Let f(x) = x 113 = i-10 + r 9 + Z- 11(x + 8);
--8 s; x:,;-7
Similarly all
0 = b-a = 3012-2012 = 1000
f(-9-) = r10 = f(-91
h I
f(-81 = z-ll) + T 9 =f(-81
I= (b-a) [f(a)+f(a+h)+f(a+2h)+ ... so no discontinuity
h
... +f(a+(n-l)h)] n
= [1{2012) + {(2013) + ..... + f(301 l )] 18. f[x]{x}dx
I = (2012) 113 + (2013) 113 + .... + (30l1) 113
+
21 = 2(2012/13 2(2013)1(3 + .... + 2(3011)1'3 2
:::) J[xJ{x}dx+ j[xi{x}dx+ ...
3

= {2012)1'3 + (2012)1'3 + 2(20I3t3 •••


I 2
... + (2)(3011) 113 + (3012) 113 + (3012) 113 n
= (2012)113 + L + R + (3012)w ... Jlx]{x}dx.>2013
:::> L + R < 21 n-1
l
:::) Jx(l+2+3+ ..... (n-l))dx >2013
16. A(t) = (sin t) ! - 2 ccs t x .!. + sin t 0
3 2

A(t)= isint-c-ost;A(t) e

Clearly A'(O) = 0 has many zeroes.


[-¾,%] c, (n ~l)n ( •:J: > 2013

So, infinitely many critical points :::) n(n-l)x_!>2013


& A(t) = 0 also (~r t e many real nos. 2 2

3'35]·.
::::, n(n- 1) ~ 8052
A'(t) c:: [ -- 5
SO Ilmin = 91

www.puucho.com
CP Publication KVPY chapterwise with www.puucho.com

t INTEGRAL CALCULUS
19. 21. f(x) = [x] sin 11:x
y
C(0,2) 2

-3 -2 -1 0 2 3

-3m'2 311/2 X
-2

Not differentiable for V x e R.


Area of trapezium ABPCQ = ½[¾] [~ + 2] Not symmetric about x = 0.
3

= .!_x~[l+2./2)
J
-3
f(x)dx ;co
2 4 ..fi.
f(x) = a will have co Solution
.
Reqmred area = 8[ Ji.
r;::)
11 1+2'v2 - !
n/4
cos x dx
22. ·: f{x) ;;:-: 0
I

-_ -11[1+2/i.)
8
--=-- - [.
Ji.
')it/4
SIDX..k) Jf ( x) 2 dx. ~ 100 not necessarily true.
0

= ~[1+
8 .fi.
2Ji]- _I.Ji because (f(x))2can take very high values then
area hounded by (f(x))2, x-axis & x = 0 to 1
may cross 100.
= ~[1 .fi.2Jzl _Ji.
+
4
J
X

23. f (x) = x + /(t)dt

= ~[8+~~]- ~ 0
f'(x) = 1 + f(x)
=} f'(x) -f(x) = I

n+I ( {X} ix] dx ~ e-xf'(x)- f{x) e-x = e-x


20.
f [x]
I ~ ~ (f(x) e-x) = e-x
dx
-I{x} + J-{x}2-
_ 2
dx
3 2
dx •
.
J-
4 {x}l
3- dx + ..... ~f(x)c-x=.:._+c
-x

I 2 3 -1
{xt ~ f(x) =-1 + cex
... + I ---;-dx
n+l

D
f(O) =O =-1 + ce0
:::::> c= l
= J[{x} + {x}22 + {x}33 + ... + {x}n)
n
dx f(x) = e• - I
0 . f(x) + f(y) + f(x) f(y)
1 1 1 1 = ex - I + eY - 1 + (e" - 1) ( eY - I)
= -+-+-+ ....+ - - - = ex - I + eY - 1 + ex ,eY - eY - ex + 1
2 2.3 3.4 n.(n + 1)
11111 1 I = e\el-1
= 1--+---+---+ ... +---- = ex+ Y -1
2 2 3 3 4 n n+l
= /(x + y)
=1--1-=-2!._
n+l n+1

www.puucho.com
CP Publication KVPY chapterwise with www.puucho.com

TOPIC-WISE KVPY [STREAM SX] Questions with Solutions

J
D

24. cos(2n[x]{x})dx
0
I 2
=J cos(O)dx + Jcos(21t(x-l))dx,
0 1 '
3
+ Jcos(4n(x - 2)) dx + ..... .
2
n
... + Jcos(2rr(n-l)(x-(n-l)))dx n

o-1
2 3
26., Let 1 = J [x]hlx]dx

J
= (I - 0) + cos2nx.dx + cos4xxdx + ... J k0:x<4 [rx]=1
I 2
n
... + Jcos(2rr(n - l)x) dx 4sx<9 [£]=2
o-1

2
9sx<I6 [£]=3
-- l + sin21tx1 + sin41cxr + ... Now,
21t I 41t 2
2 3 4 S 6 7

sin 2rr(n - I)x 0


... + - - - - - ,
J
1
J
2
J
I = dx + 2dx + Jdx + 8d.x + l Odx + l 2dx
3
J
~
J
S
J
6
21t(n -1) n-l
8 9 10 11
=1+0=1
+ Jl4d"\ + J16dx + J21dx + J30dx .....
7 8 " '1 Ill

25. I= 1 + 2 + 3 + 8 + 10 + 12 + 14 + 16 = 66
So n=: 9

27. f(x) = max {Ix.I, Ix - 11,.,., Ix - 2nl}


ll I
rr
= ( 2) -0- lnx"- (-cosxt
I n/2
·
x2n l
x<n

- I ~12
n(n-l)x 0 - (-oosx)dx
2
f(x) = Ix]
f
lx-2nl

= U.nf(x)dx J2n f(x)dx


0

11
J02n
f(x)dx
'
+
D

= (
2
1t )
-O-n(n:-1)
.
fo
n./2 2
x""'" cosxdx
= J0nIX - 2n] dx +
J2nn IX ldx

In= (~r-n(n- l)In- 2 = fn(2n -


0 x) dx +
J2n x.dx
n

~(1"n! + (n-2)!
::i ln-2 )

- "
f:i
~ [(i)"
~----+
-n(n-1)10-2
n!
1
o-2
(n-2)!
l 3n 2 3n 2
=-+-=3n2
2 2

www.puucho.com
CP Publication KVPY chapterwise with www.puucho.com

• INTEGRAL CALCULUS

28. Let P(x) = ax 3 + bx 2 -.;,- ex+ d


a+b+c+d=3 30. f(x) + fox t f(t)dt + x2 = 0
d=2 =:,. f'(x) + xf(x) + 2x = 0
-a+b-c+d=4 f'(x}
2b + 2d= 7 => =-X
f(x)+2
2b+4=7
2b=3
=> J f'(x) dx
(f(x) + 2)
=-J xdx
·
b=~ x2
2 => in (ftx) +2) =- -+ C
2
J I
J(ax +bx
3 2 +cx+d)dx = 2J (bx 2 +d)dx => f(x) + 2 = e-,..2/2 + c
~ 0 =:,. f(x) = k c_": 12 -2

-2 [< +dx ~2 (%+d) 1 wherex = 0


212
ftx) = 2 (e-" -1)
f(x) = 0

clearly lim f(x) = -2


X->-<Xl
k=2

=2(}+2)=5
31.
a,

29. f(x)=Je- 1 lx-tldt


0
(l
X m

ftx)= Je-1 (x-t)dt+Je-1 (t-x)dt


0

r
X

f'(x) = e-x (x-x)-e 0 (x-0). O


(2x -4x 3 )dx = 2 (b-a)c
J
X

+ e-1 (1) dt + 0- e-x (x-x).1 + J-c-t dt


~

(x 2 -x 4 ): =2 (b-a)c
0 X

= [-e-1 ]~ +[c-1] ; (a+ b)(l -(a2 + b2)) = 2c


= - c-x + 1 + 0- e-x (a+ b) (I - (a+ b/ + 2ab) = 2c ... (1)
again 2x 4x3 = c
f'(x) = l -2e-x
dy=1-2e-"dx
y=x + 2e-x-!· c
f(x) = x + 2e-x + c
4x3 - 2x + c"" 0 f a
b
0.
.., a+b+a=O
f(O) = Je- 1 t dt ::::>a+ b =- a ... (2)
0
ab+(a+b)a=-.!.
"' 2
= [(-e- )t]; + Je- dt
1 1

0 =:,. ab = a.2 - .!.


2 ... (3)
= [O-e- 1}~
= 0+ 1 aba=- ~
4
f(O) = 1
f(O) = 1 = 0 + 2e-0 + C ::::> C =-1
f(x) = x + 2e-x - 1
=:>c=-4u(a 2 -½) .. (4)

Put value of(a + b), ab, c from eq. (2), (3), (4)
in equation (1) and solve it

www.puucho.com
CP Publication KVPY chapterwise with www.puucho.com

TOPIC• WISE KVPY [STREAM SX] Questions with Solutions


0 I
{1-u +2( u -½J}=-8u( u -½J
2 2 2 36. A1 J
= 2 .J4x+l dx +
-l/4 -
zJ ~1-x
0
2 dx

::::} I - a? + 2u 1 = 8ct2 - 4
2-

: : } u= 8u 4
2 2-

:::> 7a.2 = 4
2
=} a= .fi
I 3
32. A= Jf(x)dx - Jf(x)dx
0 1
I 3
J
= (x3 -4x 2 +3x)dx- (x 3 -4x 2 + 3x)dx J
0 l
37
12
33. :. f(x) is always positive for x e [O, l]
I I

:. f(x) < x2 ::::} J f(x)dx < x 2J 37. Given Integral can be distributed into
11.fj 1 .fi 14
0 0
J- 2 • dx+ dx+
2
JJ3
x 2 . dx = _
J
I1<½ I 112 X lt/j .fj

38. Definition can be break as


But it is given that I =.!. which is not possible I
3
g(x) = Jf(x -y) dy
0
34.
X- y = t ; -dy . dt
X
2 -
1 ~ g(x) = Jf(t) dt

!~
Y"'f(X) O~ x-l
x::;;O

y=g(x)
l~M ~ -
g(x) =
2-x
0 x>2
n I 3
Jg(x)dx =nJ g(x)dx =-n · Now, check yourself
O O 4
n
Jf(x)dx =~ 39. In"' J:e-Y yndy
0 4
In= - .!. + n Io-1 (by reduction formula)
e
35. By Cauchy Schwarz inequality

{tb f(x)g(x)dx} 2~ r: (f(x))2dx r (g(x)) 2 dx


In -n Io-1 = -

.!n_ -
-1 .
e

=--I-
Here g(x) = l In-1.
n! n-1! e(n!)
and equality holds only when f(x) =)..
g(x) .!!__~=- 1
n=l ...(i)
So, f(x) is constant H O! e

www.puucho.com
CP Publication KVPY chapterwise with www.puucho.com

t INTEGRAL CALCULUS
l
•.. (ii) 40.
e(2!)

n=3 ~-~ =- l ...(m)


3! 2! e(3!)

n=4 ~-~=- I ... (iv)


4! 3! e(4!)
eq.(l) + cq.(2) X 2 + eq.(3) X J + (eq. 4) X 4 + ......
I1 I2 l3 .!4} -Io
- {-+-+-+-
I! 2! 3! 4! Required Area

=- .! {1+1 +_!_+..!..+ ...}


e 2! 3! A• {!((7-x)-¼(4-x')f'
.., I
Let S= L...1!..
n=I n! + [(<7-x)-¼(x' -4)dx)]

-S-Io=--I xe solve this= 32


e
S=l-lo 41. Apply L hos.pital rule
1 -)
g'(x) = Ix 1112 sin ( -
S= 1- te-Ydy
IX 1314
Jim g'(X)
S"" I+ [e-Y ]:1. x---.o

S= + 1 [i-1] lim Ix 11' 2 sin(-


lMO
1-) = O
lxl3'4
1
S=-
e

www.puucho.com
CP Publication KVPY chapterwise with www.puucho.com

Vector& 3D
_I
1. J
Let ii "" 2i - + k , v= - 3J+ 21< be vectors in (A) Vi +v2 +v3+Y4 =O
R3 and w be a wnt vector in the xy-plane. (B) There exist i, j with I :S: i < j :S: 4 such that
Then the maximum possible value of vi + vj is in the first quadrant
I (ti x v) .w I is- [2010J J.9,Thcrc exist i, j with I :s: i < j .:s" 4 such that
(A) .fs (B) Ju , I. J < 0\'· v-
{D) There exist i, j with I :S: i < j :s: 4 such that
(C) .Ji5 ~ff? vi. vj > 0
2. Let ABC be a triangle and P be a point inside
6. What is the angle subtended by an edge of a
ABC such that PA + 2PB + 3PC :.:: 0. The ratio regular tetrahedron at its center? (2014J
of the area of triangle ABC to that of APC is-
(2010] (A) cos-• (-/) (B) cos- 1 [ 1z)
(A) 2 (B) I
2 ?~ (-/) (D)cos-• (~J
3. Let ii,b,c be three vectors in the xyz space 7. The shortest distance from the origin to a
such that ax jj = b x c ""- cx a¢ o If A, B, C are variable point on the sphere
(x-2)2+(y-3) 2 +(z-6)2= I is (2015]
points with position ·vectors a, b,c (A) 5 (B)1'°
respectively, then the number of possible :)k7,.... (D) 8
positions of the centroid of triangle ABC is
.................... --->--->--->-+
(2011] =i
y{i (B)2 (C)3 (0)6
8. Leta =6i -3j

Suppose that
-+
a =b
7

.....
6k andd
-~ -+
+ c where b is parallel
+j +k.

..... ---> 4 -+
4. Let H be the orthocenter of an acute - angled to d and c is perpendicular to d . Then c is
triangle ABC and O be its circumcenter. Then f2015]
4 ..... .....
HA+HB+HC 120121 (A) 5 i - 4 j - k
(A) is equal to HO
_,,.,.,if i-2j..... -5k --Jo

(B~equal to 3~ 4 ..........
(C) 4 i - 5 j + k
-..(e) is equal to 2HO -+ ..... -+

(D) is not a scalar multiple of HO in general


(D) 3 i + 6j - 9k

9. Let v in the plane


be a vector such that
5. Let v1 , v2 , v3 , v4 be unit vectors in the
jv-il=lv-2il=lv-}I-Then !vi lies in the
xy-plane, one each in the interior of the four
interval- [2016]
quadrants. Which of the following statements (A)(O, 1] (B) (1, 2]
is necessarily true? f2013J y(c2,31 (D) (3, 4]

www.puucho.com
CP Publication KVPY chapterwise with www.puucho.com

t VECTOR&3D
10. Let ABC be an acute scalene triangle, and 0 11. A spherical ball is kept at the corner of a
and H be its circumcentre and orthocenter rectangular room such that the ball touches two
respectively. Further let N be the midpoint of (Perpendicular) walls and lies on the floor. If a
OH. The value of the vector sum point on the sphere is at distances of 9, l 6, 25
-> -> -Jo from the two walls and the floor, then a
NA+NB+NC is .,, [5 Nov, 2017] pos;,!PJe radim of the sphere is [19 Nov, 20171
-Jo
,.(A) 13 (B) 15
(A) 0 (zero vector) (B) HO
(C) 26 (D) 36
!91HO (D) .!.oH
2

www.puucho.com
CP Publication KVPY chapterwise with www.puucho.com

TOPIC• WISE KVPY [STREAM SXJ Questions with Solutions

ANSWERS
1. (D) 2, (D) 3.(A) 4. (C) 5. (C) 6.(C) 7. (B)

8. (B) 9. (C) 10. (C) 11. (A)

t. uxv = (2i - ]+ k)x (-3]+ 2k) 4. G is centroid


G = A+ B + C ; G = 20 +H
= -6k-4j-2i+3i 3 3
= i-4j-6k 20 +H=3G
Let w = ai + bj HA+HB+HC=A-H+B-H+C-H
a2+ b2 = l :::A+B+C-3H
a = cos 0; b = sine
::,:3G-3H
ux v. w = a - 4b = cos 8 - 4 sin 9
= 26+ fr-311
max. value ~1 2 +(----4) 2 ffe =20-2H
=2HO
2.
A(a)
S. v1 , v2 , v3 , v4 lies on unit circle
i.e. v1 =cos Eh i + sin 91 J 81 in I Quadrant
v2 = cos 82 i + sin 92 J 82 in II Quadrant

V3 = COS 93 i + Sin 93 J 83 in m Quadrant


B(b) C(c)
v4 = cos 94 i + sin 84} 94 in IV Quadrant
PA+2PB +3PC = 0
(A) v1+v 2 + v3 + v4 = 0 not necessarily true for
(a-p) + 2(b-p) +3(c-p) =o
all Vt, V2, V3 V4
_ a+2b+3c
p= (B) vi+vj (I SiSjS4)
6
}_-- __ _ because
Area 6.ABC = 2 !axb+bxc+cxal vi +vi= (cos Oi + cos 9j)+ i(sin 0i + sin Oj))
Area ti.APC
.!.1 axp+pxc+cx al If x coordinate is positive i.e.
2
Oi is in I quadrant
_ a+zb+ 3c
_put p::,: Sj is in IV quadrant
6
but in this case y coordinate is not
ratio= 3
necessarily positive.
3. (C) v;·'\ = cos 9i cos 9j + sin Si sin 8j
a+ c= A, b Let . Oi is in I quadrant
8j is in III quadrant
Hence a+ b+ c=O
u ,\.vj < 0; V1.V3 <0
only 1 position of centroid

www.puucho.com
CP Publication KVPY chapterwise with www.puucho.com

t VECTOR&3D
(D) vi.vi = cos 0i cos 8j + sin 8i sin 8j 7. Spherex2 +y2 + z 2 -4x-6x-12z +48 = 0
ii
Centre (2, 3, 6)
= cos (8i - 8j) is positive O< l8i-8j) < 1t Ra~us = ,J_4_+_9_+_3_6___
4_8 = I
2
Distance between centre and origin
3rc
=- <l9i-8j)<2'1t = .J4+9+36 =7
2
Not positive for all values of vi, V2, VJ, v4 Shortest distance =7 - 1
= 6 (Origin lies outside the sphere)
6. ----> """""'I- ---Jo ---li-4
8. b = 11. (i+ j+k) =:> a = b + c::
-j, -) - - A.

c= a-11.(i+j+k)
-j, AA A AAA

=:> c = (6i-3j-6k) -A (i+ j+k)


-j, A A A

c = (6-A)i+(-3-),,)j+(-6-A)k
-Jo -j,

C. ).. =6-J..-]-A.-6-11.=0

a, b, c are unit vectors ]Ji.= 3


:A.=-l
a"h=b"c=c"a= 'It .......

c = 7i-2j-Sk
A ""I, _..

Ccntrep (O+i;b+C) 9. V is the circumccntre of A.ABC 'v A~ (1, 0),


B ~ (0, l), C(2, 0)
Now angle between AP & BP --- Let V(x, y)
VA=VB=VC
AP-BP (x- 1)2 +.J=x2 + (y- 1)2 = (x-2)2 + y2
cosO= ~ _
IAPIIBP] (x,y)=
33)
(2'2 3i+3j
=:> V=-2-

_ (a+:+c a){a+:+c b) IV I= ..fi. e (2,3)


3

- a+:+c -•11•+:+c bl 10. · Circumcenter (origin 0)


A(a)
(h+c-Ja)-(i+ c-Jii)
lh+c-Ja I ·la +c-Jiil
a-ii+ ii-c-Jh 2 +a·c +c2 -Jb·c-3a 2
-3a-c+9a-li
(b 2 + c2 + 9a 2 +2b-c-6a-c-6i. h) B(h) C(c)
!+!-3+.!.+1-I-J-~ +2- 2 1
2 2 2 2 2 2
1+1+9+1-3-3
-5+3 1
--=~~
6 3

:::) 0 = cos -1(-31) - 1


N ;= - (a + b + c)
-
4

www.puucho.com
CP Publication KVPY chapterwise with www.puucho.com

TOPIC• WISE KVPY [STREAM SX] Qu estlons with Solutions

11.
(9, 16, 25)

(r-9/+(r-16)2 +(r-25)2=r2
solving this
2r2-100 r+ 962 = 0
r2-50r+481=0
r=37,r=l3
possible radius "" 13
According to option

www.puucho.com
CP Publication KVPY chapterwise with www.puucho.com

Biology
[ToP1c-w1sE KVPY [STREAM-SX] Questions with Solutions]

www.puucho.com
CP Publication KVPY chapterwise with www.puucho.com

Diversity in the Living World,


Structural Organisation in
Plants and Animals
I. In I 670, Robert Boyle conducted an
I
5, Four species of birds have different egg colors :
experiment where in he placed a viper (a [1] white with no markings, [2] pale brown
poisonous snake) in a chamber and rapidly with no markings. [3] grey-brown with dark
reduced the pressure in that chamber. Which of streaks and spots, [41 pale blue with dark blue-
the following would be true? [2010) green spots. Based on egg color, which species
(A) Gas bubbles developed in the tissues of the is most likely to nest in a deep tree hole?
snuke 120101
(B) The basal metabolic rate of the snake (A) I (B) 2 (C) 3 (D) 4
increased tremendously
(C) The venom of the snake was found to 6. Rodents can distinguish between many
decrease in potency different types of odours. The basis for odour
(D) The venom of the snake was found to discrimination is that- (20111
increase in potency (A) they have a small number of odorant
receptors that bind to many different
odorant molecules
2 Bacteria can survive by absorbing soluble
(B) the rnechanoreceptors in the nasal cavity
nutrients via their outer body surface, but
are activated by different odorant
animals cannot, because (20101
molecules found in the air passing through
(A) Bacteria cannot ingest particles but animals
the nostrils
can
(C) the part of the brain that processes the sense
(B) Bacteria have cell walls and animals do not
of smell has many different receptors for
(C) Animals have too small a surface area per odorant molecules
unit volume as compared to bacteria (D) a large number of different chemoreceptors
(D) Animals cannot metabolize soluble arc present in the nasal cavity that binds a
nutrients variety of odorant molecules

3. Path-finding by ants is by means ot: [20101 7. Selection of lysine auxotroph (bacteria which
(A) Visually observing landmarks requires lysine for growth) from a mixed
(D) Visually observing other ants population of bacteria can be done by growing
(C) Chemical signals between ants the bacterial population in the presence of-
(D) Using the earth's magnetic field [2011]
(A) lysine (B) penicillin
4. Bacteriochlorophylls are photosynthetic (C) lysine and penicillin (D) glucose
pigments found in phototrophic bacteria. Their
function is distinct from the plant chlorophylls 8. Increasing the number of measurements of an
in that they- I2010] experimental variable will- 12011]
(A) do not produce oxygen (A) increase the standard error of the sample
(B) increase the mean of the sample
(B) do not conduct photosynthesis
(C) decrease the standard error of the sample
(C) absorb only blue light
(D) result in all of the above
(D) functiou without a light source

www.puucho.com
CP Publication KVPY chapterwise with www.puucho.com

TOPIC• WISE KVPY rsTREAM SXl Questions with Solutions


9. Nocturnal animals have retinas that contain [2011] 16. A desert lizard (an ectotherm) and a mouse (an
(A) a high percentage of rods to increase endotherm) are placed inside a chamber at
sensitivity to low light conditions l 5 °C and their body temperature [T(L) for the
(B) a high percentage of cones so that nocturnal
lizard and T(M) for the mouse] and metabolic
color vision can be improved in low light
conditions rates [M(L) for the lizard and M(M) for the
(C) an equal number of rods and cones so that mouse] arc monitored. Which one of the
vision can be optimized following is correct - [2014]
(D) retinas with the photoreceptor layer present (A)T(L) and M(L) will fall while T(M) and
in the front of the eye to increase light
M(M) will increase
sens iti vi ty
(B) T(L) and M(L) will increase while T(M)
10. Some animals excrete wic acid in urine and M(M) will fall
(uricotelic) as it requires very little water. This (C) T(L) and :tvl(L) will fall, T(M) will remain
is an adaptation to conserve water loss. Which same and M(M) will increase
animals among the following are most likely to (D) T(L) and M(L) will remain same and T(M)
be uricotelic? (2012]
(A) fishes (B) amphibians and M(M) will decrease
(C) birds (D) mammals
17. Insectivorous plants such as Venus fly trap
11. Vibrio cholerae causes cholera in humans. catch and digest insects in order to supplement
Ganga water was once used successfully to the deficiency of f2016]
combat the infection. TI1e possible reason could
(A) Sulphur (D) Nitrogen
be- [20121
(A) High salt content of Ganga water (C) Potassium (D) Phosphorus
(B) Low salt content of Ganga water
(C) Presence ofbacterophases in Ganga water 18. Which of the following species makes the
(D) Presence of antibiotics in Ganga Water largest true flower in the world ? (2016]
(A) Amorpltophallus tihmium
n. Fission yeasts are (2013] (B) Raffiesia amoldii
(A) Archacbacteria (B) Eubacteria (C)Nelumbo nucifera
(C) Prokaryotcs (D) Eukaryotes (D) H elianthus amums

13. In orange and lemon, the edible part of the fruit 19. Match the type of cells given in Column I with
IB (20JJJ organisms given in Colurrm II. Choose the
(A) placenta appropriate combination from the options
(B) thalamus below. (5 Nov. 20171
(C) hairs of the ovary wall Column I Column II
(D) succulent Mesocarp (P) Flame cells (i) Sponges
(Q) Collar cells (ii) Hydra
(R) Stinging cells (iii) Planaria
14. Part of epidermis that keeps out unwanted
(A) P-iii, Q-i, R-ii (B) P-iii, Q-ii, R-i
particles is called' [2013J
(C) P-i, Q-ii, R-iii (D) P-ii, Q-iii, R-i
(A) columnar epithelium
(B) squamous epithelium 20. Amphimixis is 119 Nov, 2017)
(C) ciliated epithelium (A) A fusion of pronucJei of male gametes.
(D) cuboidal epithelium (B) a fusion of pronuclei from male and female
gametes
15. Which of the following results from (C) a fusion ofpronuclei of female gametes
conjugation in Paramecium? [20141 (D) the development of a somatic cell into an
(A) Cell death (B) Cell division embryo
(C) Budding (D) Recombination

www.puucho.com
CP Publication KVPY chapterwise with www.puucho.com

DIVERSITY IN THE LIVING WORLD, STRUCTURAL ORGANISATION IN PLANTS AND ANIMALS

21. Botanical names of plants arc given in Column-I,


and the family/order name in Column-II.
Choose the appropriate combination from the
options below [19 Nov, 2017]
Coiumn-1 Column-II
(P) Tamari11dus indica (i) Arecaccac
(Q) Cocos m1C1fera (ii) Liliaceae
(R) Colchicum automnale (iii) Solanaccac
(S) Withania somnifera (iv) Papilionaceae
(A) P-iv, Q-i, R-ii, S-iii
(B) P-iv, Q-ii, R-iii, S-i
(C) P-i, Q-ii, R-iv, S-iii
(D) P-iv, Q-i, R-iii, S-ii

www.puucho.com
CP Publication KVPY chapterwise with www.puucho.com

TOPIC~ WISE KVPY rsTREAM SXl Questions with Solutions

ANSWERS
l. (A) 2.(C) 3.(C) 4. (A) 5. (A) 6. (D) 7.(D)

8.(C) 9. (A) 10. (C) 11. (C) 12. (D) 13. (C) 14. (C)

15. (D) 16. (C) 17. (B) 18. (B) 19. (A) 20, (B) 21. (A)
~~"':"",'":. : :-.-...-.-..-.----,-,. ,., .,./ ~,. ,·'"'"· ,.,-.~-,---~~,.,.,.,.,,,.,.,,.,.,,.,-....,....~----,--,-=·=·""""'-··~ - - - - - ~ ~ ~ - - - - - - - .
r·:·>L:F>><
L,0:,;,2:.··'·. ·· ~--·· ·:;~·:___. ,,v___.___
. ----~·-···"-'····.··~·.:.~~
:•mi: :tsotuT10NsUH:.1;.;·=·,,=======~=-========·
, :·""··
:·,;: :. :· ~: ., :· : ·:.;
~f"/'~:-":"j

··/,.';,;.,·/.:'1;.;:::j

10. Uricotelic organism excrete uric acid or its salts In homeotherms metabolic rate 1s inversely
as a result of dcamination. Uricotelic organisms proportionarto environmental temperature.
included terrestrial arthropods, Lizards, Snakes So at 15°C T(M) remain same and M((M) will
and Birds. increased.

11. Large number of bacteriophage are present in 17. Insectivorous plants grow in Nitrogen deficient
Ganga water which destroy bacteria. soils and in order to compensate the deficiency
they catch and digest insects and obtain N from
12. Fission yeast cg. Schizosaccharomyces chitin (NAG).
octosporus are ascomycetes member of fungus
18. Rafjlesia arnoldii is the total root parasite and
13. Edible part of fruit in Orange and Lemon are forms the largest flower in angiosperms while
juicy hairs. Amo1phophallus titanium is the largest
inflorescence in the angiosperms
14. Ciliated epithelium keeps the harmful particles
out of the body by trapping them with their 19. Flame cells are for secretion and
osmoregulation in flatworms.
cilia. Example nasal epithelium.
Collar cells line spongocoel and cavars in
sponges and creat water amount.
15. Conjugation in paramccmm results m Cnidocytcs arc stringing cells and help in
recombination defense, affiance and capture of prey these cells
also help in attachment with substrate
16. Desert lizard is an ectothcrm (poikilothcnn)
20. Amphimixis--+ Formation of offspring due to
whose body temperature varies according to
environmental temperature.
fusion of O & ~ gametes
So at 15uc T(L) will fall due to fall in T(M)
While mouse is an cndothcrm (Homeotherm) 21. Fact based answer
whose body temperature remains com,tant
always due to variation is metabolic rate

www.puucho.com
CP Publication KVPY chapterwise with www.puucho.com

~
9

81 b2_J
Cell : Structure & Functions iI
1. Ribonucleic Acids (RNA) that catalyze 6. In which phase of the cell cycle are sister
enzymatic reactions are called ribozymes. chromatids available as template for repair?
Which one of the following acts as a ribozyme? [2011]
120101 (A) GI phase (B) G2 phase
(C) S phase (D) Mphase
(A) Ribosome (B) Amylase
(C)tRNA (D) Riboflavin
7. If you fractionate all the organelles from the
cytoplasm of a plant cell, in which one of the
2. Of the following combinations of cell
following sets of fractions will you find nucleic
biological processes which one is associated acids? 12011!
with embryogencsis? [2010] (A) nucleus, mitochondria, chloroplast,
(A) Mitosis and Meiosis cytoplasm
(B) Mitosis and Differentiation (B) nuclem,, mitochondria, chloroplast,
(C) Meiosis and Differentiation glyoxysome
(D) Differentiation and Reprogramming (C) nucleus, chloroplast, cytoplasm and
peroxisome
(D) nucleus, mitochondria, chloroplast, Golgi
3. The distance between two consecutive DNA
bodies
base pairs is 0.34 run. If the length of a
chromosome is 1 mm, the number of base pairs
8. The Na+/K+ pump is present in the plasma
in the chromosome is approximately- [2010]
membrane of mammalian cells where it- [2011]
(A) 3 million (B) 30 million (A) expels potassium from the cell
(C) 1.5 million (D) 6 million (B) expels sodium and potassium from the cell
(C) pumps sodium into the cell
4. A bust cell has intracellular bacteria symbionts. (D) expels sodium from the cell
If the growth rate of the bacterial symbiont is
always 10% higher than that of the host cell, 9. The process of cell death involving DNA
after 10 generations of the host cell the density cleavage in cells is known as- (2011 J
of bacteria in host cells will increase - (2010) (A) necrosis (B) apoptosis
(C) cytokinesis (D) endocytosis
(A) by 10 % (B) two-fold
(C) ten-fold (D) hundred-fold
IO. At which stage of Meiosis I does crossing over
occur? 120111
5. Which of the following correctly represents the (A) lepoptene (B) zygotene
results of an enzymatic reaction? Enzyme is E, (C) pachytene (D) diplotene
substrate is Sand products are Pl & P2. (2010]
(A) Pl+ S Q P2 + E 11. Human chromosomes ·undergo structural
(B) E + S ¢:,. Pl + P2 changes during the cell cycle. Chromosomal
structure can be best visualized if a
(C) Pl+ P2 + E ~ S
chromosome is isolated from a cell at (2012]
(D) E + S <:> Pl + P2 + E
(A) G 1 phase (B) S phase
(C) G2 phase (D) M phase

www.puucho.com
CP Publication KVPY chapterwise with www.puucho.com

TOPIC-WISE KVPY rs:rREAM SXl Questions with Solutions

12. A hormone molecule binds to a specific protein 18. Two tubes labelled 'P' and 'Q' contain food
on the plasma membrane inducing a signal. The stun: Tube 'P' gave positive test with Benedict's
protejn it binds to is called- (2012} solution while tube 'Q' gave positive test v.ith
(A) ligand (B) antibody Nitric acid. Which of the following is correct?
(C) receptor (D) histone (2014)
(A) Tube 'P' contains sugar; tube 'Q' contains
13. Which one of the following is true about protein
enzyme catalysis? (2012] (B) Tube 'P' contains protein; tube 'Q' contains
(A) The enzyme changes at the end of the sugar
reaction (C) Bo~ tube 'P' and lube 'Q' contain sugar
(B) The activation barrier of the process is (D) Both, tube 'P' and tube 'Q' contain protein
lower in the presence of an enzyme
(C) The rate of the reaction is retarded in the 19. A diploid plant has 14 chromosomes, but its
presence of an enzyme egg cell has 6 chromosomes. which one of the
(D) The rate of the reaction is independent of following is the most likely explanation of this?
substrate concentration
(2014]
{A) Non-disjunction in meiosis I and II
14. Nucleolus is an organelle responsible for the
{B) Non-disjunction in meiosis 1
production of (20131
{C) Non-disjunction in mitosis
(A) carbohydrates (B) messenger RNA
(D) Normal meiosis
(C) lipids (D) ribosomal RNA

20. Following the cell cycle scheme given below,


15. Genetic content of a cell reduces to half rl;uring
what is the probability that a cell would be in
[2013]
(A) meiotic prophase I M-phase at any given time? (20141
(B) mitotic prophasc GI-phase
(C) meiotic prophase Il
(D) meiotic telophase

G2-phasc
16. The two enzymatic activities associated with
RUBISCO are [20131
(A) oxidase and oxygenase
(B) oxygenase and carboxylase (A) l/24 (B) 1/12 (C) 1/6 (D) 1/2
(C) oxidase and carboxylase
(D) oxygenasc and carbamylation 21. After meiosis-II, daughter cel1s differ from the
parent cells and each other in their genotypes.
17 Which one of the following statements about This can occur because of which one of the
nitrogenase is correct ? [20131 following mechanism(s)? [2014]
(A) It is sensitive to CO2 and therefore present {A) Only synaptic crossing over
in isolated nodules {B) Only crossing over and independent
(B) It requires 0 2 and therefore functional assortment of chromosomes
during the day (C) Only crossing over and chromosomal
(C) It is sensitive to 02 and therefore is segregation
functional in anaerobic invironmcnts
(D) Crossing over, independent assortment and
(D) It is sensitive to light and therefore function
segregation of chromosomes
only in dark

www.puucho.com
CP Publication KVPY chapterwise with www.puucho.com

• CELL : STRUCTURE & FUNCTIONS


22. Which one of the following bas phosphoric 29. The mitochondrial inner membrane consists of
acid anhydride bonds? [2015) a number of infoldings called cristae. The
(A) Deoxyribonucleic acid increased surface area due to cristac helps in :
(2015)
(B) Ribonucleic acid
(A) Increasing the volume of mitochondria
(C) dNTPs
(B) Incorporating more of the protein
(D) Phospholipids complexes essential for electron transport
chain
23. Phospholipids are formed by the esterification (C) Changing the pH
of (20151 (D) Increasing diffusion of ions
(A) three ethanol molecules with three fatty
30. Which one of the following pairs of molecules
acid molecules
never forms a hydrogen bond between them?
(B) one glycerol and two fatty acid molecules
(20161
(C) one glycerol aod three fatty acid molecules (A) Water and water (B) Water and glucose
(D) one ethylene glycol and two fatty acids (C) Water and ethanol (D) Water and octane
molecules
31. Lactase hydrolyses lactose into [2016]
24. Given the fact that histonc binds DNA, it (A) Glucose + glucose
(B) Glucose+ galactosc
should be rich in [2015]
(C) Galactose + galactose
(A) arginine, lysine
(D) Galactose + fructose
(B) cystcinc, methionine
(C) glutamate, aspartate 32. Which of the following statements is incorrect
(1?) isoleucine, leucine regarding biological membrane ? (20161
(A) It is composed of ~ipids and proteins
25. Which one of the following can be used to (B) Peripheral proteins are loosely associate
with the membrane
detect amino acids? [20151
(C) Integral proteins span the lipid bilayer
(A) Iodine vapour (B) Ninhydrin
(D) Lipids and membrane proteins do not
(C) Ethldium brnmide (D) Bromophenol blue provide structural and functional
asymmetry
26. Which of the following processes leads to DNA
ladder formation? (2015) 33. Which of the following statements about
(A) Necrosis (B) Plasmolysis nucleosome is true? [2016]
(A) It consists of only DNA
(C) Apoptosis (D) Mitosis
(B) It is a nucleus-like structure found in
prokaryotes
27. Co-enzymes are components of an enzyme (C) It consists of DNA and proteins
complex which are necessary for its functiorL (D) It consists of only histone proteins
Which of these is a known co-enzyme? [2015]
(A) Zinc (B) Vitamin B12 34. Epithelial cells in animals are held by
(C) Chlorophyll (D) Heme specialized junctions, one of them being "Gap
junction". Function of a "Gap junction" is to
(2016]
28. The peptidoglycans of bacteria consist of
(A) Facilitate cell-cell communication by rapid
[2015) transfer of small molecules
(A) sugars, D-amino acids and L-amino acids {D) Cement the neighbouring cells
(B) sugars and only D-amino acids (C) Stop substances from leaking
(C) sugars and only L-amino acids (D) Provide gaps in the tissue to facilitate
(D) sugars and glycine uptake of small molecules across tissues

www.puucho.com
CP Publication KVPY chapterwise with www.puucho.com

TOPIC-WISE KVPY rSTREAM SXl Questions with Solutions

35. Match the following organelles in Group I with (A) P and Q are peripheral membrane proteins.
the structures in Group II. Choose the correct (B) R is a peripheral membrane protein.
corµbination. [2016] (C) P and Q arc integral membrane bound
Group I Group II proteins.
P. Mitochondrion L Cisternae (D) Pis peripheral and Q is integral membrane
Q. Golgi ii. Cristae protein.
R. Chloroplast iii.Thylakoids
S. Centrosome iv. Radial spokes 40, How many different proteins consisting of 100
(A) P-ii , Q-i, R-iii, S-iv amino acids can be fanned from 20 different
(B) P-iii, Q-i, R-ii, S-iv amino acids ? [5 Nov, 2017]
(C) P-iv, Q-i, R-ii, S-iii
(A) 20100 (B) 10020
(D) P-iv, Q-ii, R-i, S-iii
(C) z20 (D) 20 xIO0
36. Which of the following best describes the DNA
content and the number of chromosomes at the 41. Choose the CORRECT combination of
end of S and M phases of the cell cycle in statements given below related· to cysteine
mitosis, if the DNA content of the cell in the residue in proteins. [5 Nov, 2017)
beginning of cell cycle (GI phase) is i. Cysteine can be linked to tyrosine by ·s-O
considered as C and the number of bond.
chromosomes 2N '? [2016] u. Cysteine can be linked to another eysteine
(A) 2C and 2N for S phase ; 2C and 2N for M by S-8 bond.
phase iii. Cysteine can complex with Zn2+_
iv Cysteine can be linked to methionine by
(B) 2C and N for S phase ; 2C and N for M
S-S bond
phase
(A) i and ii (B) ii and iii
(C) 2C and 2N for S phase; C and 2N for M
(C) iii and iv (D) i and iv
phase
(D) C and N for S phase ; C and 2N for M
phase 42. Which ONE of the following is a hetero-
polysaccbaride ? [19 Nov, 20171
37. The rigidity of cellulose is due to 15 Nov, 2017].
(A) Glycogen (B) Starch
(A) coiled structure of glucose polymer
(B) j3(1 -), 4) glycosidic linkage (C) Cellulose (D) Hyaluronic acid
(C) hydrogen bonding with adjacent glucose
polymer 43. Bacterial plasmids are genetic entities that,
(D) cross-linking between gtucose and peptides [19 Nov, 2017]
(A) are non-transferable to the same bacterial
38. Which ONE of the following statements is species.
INCORRECT about restriction cndonucleases? (B) are capable ofindcpcndent replication.
[5 Nov, 2017[ (C) have RNA as genetic material.
(A) They serve as primitive form of immune (D) always require integration in the genome
system in bacteria. for their replication.
(B) They digest the DNA non-randomly.
44. Which ONE of tbe following statements is
(C) They digest the DNA at specific location.
INCORRECT for translation in cytoplasm?
(D) They digest the DNA from free ends. [19 Nov, 2017]
(A) One codon codes for only one amino acid.
39. Proteins P, Q, and R are associated with i-')tact (B) One amino acid may be coded by many
organcllar membrane in a cell. 1f the intact codons. ·
organeltel is treated with a high ionic strength (C) More than one amino acids are coded by
buffer, only protein R remained associated with one specific condon.
the membrane fraction. Based on thls, one (D) There are some codons that do not code for
could conclude that 15 Nov, 20171 any amino acid.

www.puucho.com
CP Publication KVPY chapterwise with www.puucho.com

t CELL: STRUCTURE & FUNCTIONS


45. The protein P, the oligosaccharide 0, and the 46. An octapeptidc (NH2-Asn-Glu-Tyr-Lys-Trp-
oHgonucleotide N are composed of 100 amino Mct-Glu-Gly) is subjected to complete protease
acid residues, I 00 hcxosc residues, and l 00 and chemical digestion. Based on the results
nucleotides, respectively. Which ONE of the obtained, choose the rNCORRECT option from
following orders of molecular weights is below. [19 Nov, 20171
CORRECT? [19 Nov, 2017) (A) Trypsin generates mixtures of dimer and
(A) P > 0 > N (B) P > N > 0 trimer
(C)N>O>P (D)O>P>N (B) Trypsin generates tetramers only
(C) Cyanogen bromide generates a hcxamcr
and a dimer
(D) Chymotrypsin generates mixture of dimer
and trimers

www.puucho.com
CP Publication KVPY chapterwise with www.puucho.com

TOPIC· WISE KVPY fSTREAM SXl Questions with Solutions

ANSWERS
1. (A) 2. (B) 3. (A) 4. (H) 5. (D) 6. (D) 7. (A)
8.(D) 9. (B) 10. (C) 11. (D) 12. (C) 13. (B) 14. (D)

15. (D) 16. (B) 17. (C) 18. (A) 19. (B) 20. (B) 21. (D)

22. (C) 23. (D) 24. (A) 25. (B) 26. (C) 27. (B) 28. (A)

29. (B) 30. (D) 31. (B) 32. (D) 33. (C) 34. (A) 35. (A)

36. (C) 37. (C) 38. (D) 39. (A) 40. (A) 41. (B) 42. {D)

43. (B) 44. (C) 45. (C) 46. (A)

. -
:_ .. ~-~--- :·,

"":-~~~·~ .:.·· . SOUJTiONS


- • -~
11. Chromosome structure is best seen at 20. Total time for cell cycle= 24 hrs.
mctaphase which is a sub stage of M-phase of Time for M-phase = 2 hrs
the cell cycle. So, probability of cell in M-phase at any given
. . 2 l
12. Hormone receptors for protein honnone are t1me1s - ::::) -
24 12
present on the surface of plasma membrane of
''' cell. 21. After meiosis-II, daughter cells differ from the
parent cells and each other in their genotypes
14. Nucleolus are also termed as ribosomal
due to Crossing over, independent assortment
factories and syntl1csizcs rRNA.
and segregation of chromosomes

15. DNA content reduces to half each time during


meiotic telophase i.e. during separation of 30. Hydrogen bond is a weak bond between two
homologous chromosome and also during molecules resulting from an electrostatic
separation of sister chroma ti ds. attraction between a proton in one molecule and
an electronegative atom in another molecule.
16. RUDISCO --j> Ribulose-1, 5-bis phosphate This is not possible in case of water and octane
carboxylase/oxygcnasc

17. Nitrogcnase enzyme is sensitive to oxygen and


31. Lactose Glucose+ Galactose
so requires anaerobic environment for its
functioning. 32. Proteins provide asymmetry to plasma
membrane, as they are of 2 types i.e. peripheral
and integral.
18. --)> Nittic acid reacts \Vith proteins to form

yellow nitrated products. 33. Nucleosome is the smallest unit of DNA


-> Benedict test is used to test the presence of packaging containing 200 nitrogen bases and
monosaccharide and reducing sugar. four types of histone proteins i.e. H2A, H2B, H3
and f4. H I type of hi stone is used in plugging.
19. Egg cell is formed by meiosis. Less number of
34. Gap junctions are cytoplasmic communications
chromosome indicates non-disjunction in
bet\veen two epithelial cells for rapid transfer of
meiosis 1.
some ion'! and small molecule

www.puucho.com
CP Publication KVPY chapterwise with www.puucho.com

• CELL : STRUCTURE & FUNCTIONS

35. Cristae are invaginations of inner mitochondrial 42. Hyaluronic acid is polymer of D-Gulucomc
membrane. Cisternae is unit of golgi body acid, N-Acetyl, D-Glucose amine. So
where glycosidation and glycosylation takes heteropolysaccharide.
place. Grana of chloroplast are composed of
43. It is extra chromosomal genetic material
thylakoids. Centrosome contains centrioles
capable of independent replication.
with radial spokes.
44. One specific codon codes for only one
36. G1 -----t S -----t G2 -------;, M aminoacid.
DNA C 2C 2C 2C ---t C
Content No. of 2N 2N 2N 2N ---t 4N -), 2N 45. Monomer of protein is amino acid
Chromosome oligosaccharide is sugar & oligonuclcotidc is
nucleotides order of its molecular weight is
37. Bundle of cellulose fibre are rigid to I-I-bonds. Nucleotide> A-1onosaccbaride > Amino acid

38. Endonucleases do not cut from free ends. 46. Cleavage site:- for Trypsin-After Lys & Arg
For chymotrypsin - After Phe, Trp, or Tyr
39. Peripheral proteins can be detached earily For cyanogen bromide- After met

40. 20100

41. Fact base answer

www.puucho.com
CP Publication KVPY chapterwise with www.puucho.com

Plant Physiology
I
1. If the total number of photons falling per unit 5. A water molecule can from a maximum of
area of a leaf per minute is kept constant, then .......... hydrogen bonds. (2011}
which of the following will result in maximum (A) 1 (B) 2 (C) 3 (D)4
photosynthesis ? [2010)
(A) Shining f,>Teen light 6. Wl1ich sequence of events gives rise to flaccid
(B) Shining sunlight guard cells and stomata} closure at night?
(C) Shining blue light 120111
(D) Shining ultraviolet light (A) low [Glucose] => low osmotic pressure
=> low pH => high pC02
2. Fruit wrapped in paper ripens faster than when (B) ]ow pH :::::,. high pC0 2 =::, low [Glucose]
kept in open air because- [2010] =::, low osmotic pn::ssure
(A) Heat of respiration is retained better (C) low osmotic pressure => high pC02 => low
(B) A chemical in the paper helps fruit ripening
pH~ low [Glucose]
(C) A volatile substance produced by the fruit
(D) high pC0 2 :::::,. low pH :::::,. low [Glucose]
is retained better and helps in ripening
:::::,. low osmotic pressure
(D) The fruit is cut off from the ambient
oxygen which is an inhibitor to fruit
7. A ripe mango, kept with uruipe mangoes
rcpening
causes their ripening. This is due to the release
of a gaseous plant hormone- [2012}
3. If you dip a sack full of paddy seeds in water
(A) auxin (B) gibberlin
overnight and then keep it out for a couple of
(C) cytokinine (D) ethylene
days, it feels wann. What generates this heat ?
[2010]
8. Which of the following is true for TCA cycle in
(A) Imbibation
eukaryotes ? (2012]
(B) Exothermic reaction between water and
(A) takes place in mitochondrion
seed coats
(B) produces no ATP
(C) Friction among seeds due to swelling
(C) takes place in golgi complex
(D) Respiration
(D) independent of electron transport chain

4. You marked rn'o ink-spots along the height at


9. Plant root,;: are usually devoid of chlorophyll
the base of a coconut tree and also at the top of
and cannot perform photosynthesis. However,
the tree. When you examine the spots next
there are exceptions. Which of the following
year when the tree has grown taller, your will
plant root can perform photosynthesis ?
see- [2010]
(20121
(A) the two spots at the top have grown more
(A) Arabidopsis (B) Tinospora
apart than the two spots at the bottom
(C) ruce (D) Hibiscus
(B) the top two spots have grown less apart
then the bottom t\vo spots
10. Insectivorous ptauts that mostly grow on
(C) both sets of spots have grown apari to the
swampy soil use insects as a source of- (20121
same extent
(A) Carbon (B) Nitrogen
(D) both sets of spots remain un-altercd
(C) Pbosporous (D) Magnesium

www.puucho.com
CP Publication KVPY chapterwise with www.puucho.com

• PLANT PHYSIOLOGY

11. In green leaves, the light and dark reaction 17. The percentage of sunlight captured by plants is
occur in [2013] [2016]
(A) stroma and grana respectively (A) 2-10% (B) 10-20%
(B) grana and stroma respectively (C) 60-80% (D) 100%
(C) cristae and matrix respectively
(D) both occur in cytoplasm 18. If 14CO2 is added to a suspension of
photosynthesizing chloroplasts, which of the
12. In an experiment investigating photoperiodic following will be the first compound to be
response, 1he leaves of a plant arc removed. radioactive ? [2016]
What is the most likely ouleome ? (2014] (A)ATP (B) NADPH
(A) Photoperiodism is not affected (C)NADH (D) 3-phosphoglyccratc
(B) Photoperiodic response does not occur
(C) The plant starts flowering 19. In an experiment represented in the schematic
(D) The plant starts to grow taller below, a plant species was grown in different
day and night cycles and its phmoperiodic
13. Name the terminal acceptor of electrons in the flowering behaviour was noted. This species is
mitochondrial electron transport chain (2014] a (2016]
(A) Nitrate (B) Furnarate Light Dark
(C) Succinate (D) Oxygen
~ No flower

14. If the humidity of the atmosphere suddenly ~ No flower

increases substantially, the water flow in the 16 hrs 8 hrs No flower


xylem will - (2014]
(A) increase
1111a 9hrs Flower

(D) decrease IIIB'l!IIII IO hrs Flower


(C) remain unaltered 1111111'1!111 11 hrs .,.- Flower
(D) increase sharply and then reduce slowly to
the preexisting level
~Flower
15. If mitochondria isolated from a cell arc first
10 hrs 10 hrs Flower
placed without carbon source in a buffer at pH
12 brs 10 hrs Flower
8.0 and then transferred to a buffer at pH4 , it
will lead to - 12014] 8 hrs NoFhwcr
(A) an increase in intra-mitochondrial acidity 10 hrs No Flower
(B) a decrease in intra-mitochondrial acidity
12 hrs No Flower
(C) blockage of ATP synthesis
(D) synthesis of ATP
(A) short day plant and actually measures day
16. Which of the following molecules is a primary length to flower
acceptor of CO 2 in photosynthesis ? f2016] (B) short day p!ant and actually measures night
(A) Pymvatc length to flower
(R) 3-phosphoglycerate (C) long day plant and actually measures night
(C) Phosphoenol pyruvate length to flower
(D) Oxaloacetatc (D) long day plant and actually measures day
length to flower

www.puucho.com
CP Publication KVPY chapterwise with www.puucho.com

TOPIC-WISE KVPY rSTREAM SXl Questions with Solutions


20. In a chemical reaction, enzymes catalyze the 26. If a pure chlorophyll solution is illuminated
reaction by JS Nov, 2017) with ultraviolet light, the solution appears
(A) lowering the activation energy. [19 Nov, 2017}
(B) increasing the activation energy.
(A) green (B) violet
(C) decreasing the free energy change between
(C) red (D) black
reactants and products.
(D) increasing the free energy change between
reactants and products. 27. Nitrogen fixation is inhibited by oxygen.
However, in aerobic nitrogen fixing bacteria,
21. The number of net ATP molecules produced nitrogen is fixed in the presence of oxygen.
from I glucose molecule during glycolysis is Nitrogenase in such organisms is protected by
[5 Nov, 2017] which ONE o the following mechanisms
(A) 1 (B) 2 (19 Nov, 2017)
(A) channelizing oxygen to form ozone
(C) 3 (D)4
(B) removal of oxygen by metabolic activity
(C) utilizing oxygen for membrane remodelling
22. Which ONE of the following coenzymes is
(D) utilizing oxygen for synthesis of
required for the conversion of L-alaninc to a pentapeptide chain in peptidoglycan
racemic mixture of D-and L-alanine '?
JS Nov, 2017)
(A) Pyridoxal-6-phosphate
(B) Thiamine pyrophosphate
(C) Cocnzymc A
(D) Flavin adenine dinucleotidc

23. The cyclic electron flow during photosynthesis


generates [5 Nov, 2017]
(A) NADPH alone.
(B) ATP and NADPH.
(C) ATP alone.
(D) ATP, NADPH and 02.

24. In photosynthesis, oxygen is produced by


[5 Nov, 2017]
(A) photosystem l from carbon dioxide.
(B) photosystem II from carbon dioxide.
(C) photosystem I from water.
(D) photosystem II from water.

25. Which ONE of the following molecules serves


as a substrate for direct synthesis of ATP ?
fl91'"ov, 2017]
(A) 1, 3-bisphosphoglycerate
(B) Glucose 6-phosphate
(C) Pyruvate
(D) Fructose 1,6-bisphosphate

www.puucho.com
CP Publication KVPY chapterwise with www.puucho.com


k~,---_--_«·: I

•PLANT PHYSIOLOGY

ANSWERS
1. (C) 2. (C) 3. (D) · 4. (A) 5.(D) 6. (D) 7. {D)
8. {A) 9. (B) 10. (B) 11.(B) 12. (B) 13. (D) 14. (B)
15, (D) 16. (C) 17. (A) 18. (D) 19. (B) 20, (A) 21. (B)
22. (A) 23. (C) 24. (B) 25. (A) 26. (C) 27. (B)

,. The only gaseous hormone out of these four 16. In C4 plants the primary acceptor of CO2 is
options is ethylene. Its main function is phosphoenol pyruvate, a 3C compound. e.g. In
ripening. Maize, Sugarcane etc.

8. TCA (Tricarboxylic acid cycle), also known as 17. Plants capture 2-10 % of PAR
Krebs' cycle, takes place in matrix of
_mitocbondrion. 18. During Calvin cycle, the 14C02 is incorporated
with a,SC compound Ribulose 1-5 Biphosphate
9. Tinospora and Trapa are plants which have and forms 2 molecules of 3C compound 3-
assimilatory or photosynthetic roots. phosphoglycerate.
10. Insectivorous plants are grown in nitrogen
19. SDP (Short day plants) or LNP (Long Night
deficient soil and they get nitrogen by trapping
Plants) flowers only when photopcriod is below
insect e.g. Utricularia, Dionea, Nepenthes.
critical day length (Critical photoperiod) or
they are responsible to night length and flower
11. Grana 4 light reaction
when night length is above critical dark period.
Strorna -+ Dark/Light independent reaction
In this experiment plant flowers when dark
12. Leaves are the site for photoperiodic period is above 8 hrs.
perception. So, it is SOP and actually measures night
length to flower.
13. Oxygen is the terminal acceptor of electrons in
mitochondrial ETS. 20. Enzyme decreases activation energy in
reaction.
14. Humidity of atmosphere is inversely 21. In glycolysis net ATP produced
proportional to transp~tion (water flow). =4-2=2ATP
Increase in humidity will
decrease the water flow in the xylem. 22. Information based question

15. Mitochondria synthesizes ATP based on 23. In cyclic photophosphorylation only ATP is
chemiosmosis when mitochondria is transferred formed.
from a buffer pH 8.0 to buffer pH 4.0
24. photosynthesis at Ps II in lumen oftbylakoid

2S. 1,3-biPGA
7\~ 3PGA

ADP ATP
26. Fact based answer
LowpHm~
HighH' ions 27. Excess 0 2 is used for metabolic activity.

www.puucho.com ....
CP Publication KVPY chapterwise with www.puucho.com

l'
Human Physiology l
---l!li/Ei:--i:=:-==--·-·-"·--- I
1. Wben a person is suffering from high fever, it 4. Estimate the order of the speed of propagation
is sometimes observed that the skin has a of an action potential or nerve impulse - [2010J
reddish tinge. Why does this happen? [20101 (A) nm.ls (B) micron/s
(A) Red colour of the skin radiates more heat (C) emfs (D) mis
(B) Fever causes the release of a red pigment in
the skin 5. Which of the following graphs accurately
(C) There is more blood circulation to the skin represents the insulin levels (Y-axis) in the
to keep the body warm body as a function of time (X-axis) after eating
(D) There is more blood circulation to the skin sugar and bread/roti ? [20101
to release heat from the body Sugcr Roti

2. Athletes often experience . m~scle cramps. (A) (B)


Which of the following statements is true
muscle cramps'? [2010]
(A) Muscle cramp is caused due to conversion
Roti Suger
of pyruvic acid into lactic acid in the
cytoplasm (C) (D)
Suger Roti
(I3) Muscle cramp is caused due to conversion
of pyruvic acid into lactic acid in the C D
mitochondria
(C) Muscle cramp is caused due to 6. The major constituents of neurnfilaments are-
nonconvcrsion of glucose to pyruvate in [2011]
(A) microtubules
the cytoplasm
(B) intermediate filaments
(D) Muscle cramp is caused due to conversion
(C) actin filaments
of pyruvic acid into ethanol in the
(D) protofi.Jaments
cytoplasm

7. A person has difficulty in breathing al higher


3. The fluid part of blood flows in and out of
altitudes because- [2011]
capillaries in tissue to exchange nutrients and
(A) oxygen is likely to diffuse from lungs to blood
waste materials. Under which of the following
(B) oxygen is likely to diffuse from blood to
conditions will fluid flow out from the
lungs
capillaries into the surrounding tissue? [2010]
(C) partial pressure of 02 is lower than partial
(A) When arterial blood pressure exceeds blood
pressure of CO2
osmotic pressure
(D) overall intake of 02 by the blood becomes
(B) When arterial blood pressure is less than low
blood osmotic pressure
(C) When arterial blood pressure is equal to 8. The CO2 in the blood is mostly carried- [2011]
blood osmotic pressure (A) by hemoglobin in R.BCs
(D) Arterial blood pressure and blood osmotic (B) in the cytoplasm ofWBCs
pressure have nothing to do with the (C) in the plasma as bicarbonate ions
outflow of fluid from capilleries (D) by plasma proteins

www.puucho.com
CP Publication KVPY chapterwise with www.puucho.com

t HUMAN PHYSIOLOGY
9. Circadian Rhythm is an endogenously driven 15. Vitamin A deficiency leads to night-blindness.
cycle for biochemical, physiological and Which of the following is the reason for the
behavioral processes. In humans, the disease? (2012]
approximate duration of rhis 'biological clock' (A) rod cells are not convened lo cone cells
lS- (2011] (B) rhodopsin pigment of rod cells is defective
(A) 1 Hour (B) 6 Hours (C) melanin pigment is not synthesized in cone
(C) 12 Hours (D) 24 Hours cells
(D) cornea of eye gets dried
10. An electrode is placed in the axioplasm of a
mammalian axon and another electrode is
16. A reflex action is a quick involuntary response
placed just outside the axon. The potential
to stimulus. Which of the following is an
difference measured will be- [2011]
example of BOTH, unconditioned and
(A) 0 (B)-70 mV
conditioned reflex ? 12012]
(C)-70 µV (D) +70 µV
(A) knee Jerk reflex
11. Although blood flows through large arteries at (B) secretion of saliva in response to the aroma
high pressure, when the blood reaches small of food
capillaries the pressure decreases because- [201 I] (C) sneezing reflex
(A) the valves in the arteries regulate the rate of (D) contraction of the pupil m response to
blood flow into the capillaries bright light
(B) the volume of blood in the capillaries is
much lesser than that in the arteries 17. \Vhy hydrogen peroxide is applied on the
(C) the total cross-sectional area of capillaries wound as a disinfectant, there is frothing at the
arising from an artery is much greater than site of injury, which is due to the presence of an
that of the artery enzyme in the skin that used hydrogen peroxide
(D) elastin fibers in the capillaries help to as a substrate to produce- [20121
reduce the arterial pressure (A) Hydrogen
(B) Carbon Dioxide
12. Peptic ulcers arc caused by- [2012]
(C) Water
(A) a fungus, Candida albicans
(D)Oxygen
(B) a virus, cytomegalovirus
(C) a parasite, Trypanosoma brucei 18. Persons suffering from hypertension (high
(D) a bacterium, Helicobacter pylori
blood pressure) are advised a low-salt diet
because- {2012)
13. By which of the following mechanisms is
(A) More salt is absorbed in the body of a
glucose reabsorbed from the glomerular filtrate
patient with hypertension
by the kidney tubule? (20121
(B) High salt leads to water retention in the
(A) osmosis (B) diffusion
blood that further increases the blood
(C) active transport (D) passive transport
pressure

14. In mammals, the hormones secreted by the


(C) High salt increases nerve conduction and
pituitary, the master gland, is itself regulated by increases blood pressure
(2012] (D) High salt causes adrenaline release that
(A) hypothalamus (B) median cortex increases blood pressure
(C) pineal gland2 (D) cerebrum

www.puucho.com
I
CP Publication KVPY chapterwise with www.puucho.com

TOPIC· WISE KVPY fSTREAM SXl Questions with Solutions


19. · When a person beings to fast, after some time 23. Brown fat is a specialized adipose tissue with
glycongen stored in the liver is mobilized as a abundant mitocilondria and rich blood supply.
source of glucose. Which of the following Brown fat 120131
(A) insulates animals that are acclimatized to
graphs best represents the change of glucose
cold.
level (y axis) in his blood, starting from the (B) is the major source of heat production in
time (x -axis) when the beings to fast? (20121 birds. ·
(C) provides energy tomusclcs.

(A) Glucose ~ 24.


(D) produces heat without producing ATP.

In whieil of the following types of glands is the


secretion collected inside the · cell and
Time
discharged by disintegration of the entire

~
gland? (2014]
(A) Apocrine (B) Merocrine
(B) Glucose
(C) 1folocrine (D) Epicrine

Time 25. Where is the third ventricle of the brain


located'? (2014]
(m Cerebellum
(
(A) Cerebrum
(C) Glucose (C) Pons varoli (D) Diencepbalon

26. Which of the following is secreted at the ends


Time of an axon'? (2014]
(A) Ascorbic acid (B) Acetic acid

(D) Glucose
L Time
27.
(C) Acetyl choline (D) Acetyl CoA

The two components of autonomous nervous


system have antagonistic actions. But in certain
cases their effects arc mutua}ly helpful. \Vhich
of the following statement is correct? [2015]
20. Human fetal haemoglobin differs from the (A) At rest, the control of heart beat is not by
adult haemoglobin in that it has- [2013]
the vagus nerve
(A) higher aflinity for oxygen
(B) During exercise the sympathetic control
(B) lower affinity for oxygen
(C) two subunits only decreases
(D) is glycosylated (C) During exercise the parasympathetic
control decreases
21. Stroke could be preventecVtreatcd with [2013] (D) Stimulation of sympathetic system resull~
(A) balanced diet (B) clotting factors in constriction of the pupil
(C) insulin (D) blood thinners
28. A person has 400 million alveoli per lung with
22. In vertebrates 'glycogen' is stored chiefly in
an average radius of 0.1 mm for each alveolus.
(2013]
(A) heart and blood Considering the alveoli arc spherical in shape,
(B) spleen and stomach the total respiratory surface of that person is
(C) bones and lymph closest to l201SJ
(D) liver and muscles (A) 500 mm2 (B) 200 mm2
(C) 100 mm2 (D) 1000 mm2

www.puucho.com
CP Publication KVPY chapterwise with www.puucho.com

• HUMAN PHYSIOLOGY
29. Which of the following statements is true about 36. Which of the following graphs best describes
glandular epithelium in salivary gland ?[20161 the oxygen dissociation curve where p0 2 is the
(A) It consists of isolated single cells partial pressure of oxygen? [2016]
(B) lt consists of mutlicellular chmter of cells
(C) Its secretions are endocrine
, .,
(D) It consists of squamous epithelial cells ,,
., ,
30. Wruch one of the following ion pairs is
, ,,
involved in nerve impulses? (2016]
(A) Na+, K+ (B) Na\ er
(C) K+, Cr (D) K+, Ca2+

31. · Which of the following hormones that controls ''


blood pressure is secreted by human heart? ''
(2016] ''
(A) Erytbropoietin ''
(B) Atrial natriuretic factor '
(C)ACTH
(D) Glucocorticoid

32. Oxytocin and vasopressin are synthesized in


[2016] ---->
(A) Hypothalamus (B) Adrenal gland
,
,,
(C) Pituitary gland (D) Ovary
I
I
33. If you exhale multiple times into a conical flask I

containing lime water through a single inlet


,I

fixed through a stop cork, lime water will?


{2016)
(A) Become cooler ----;>
(B) Tum milky
(C) Remain unchanged
(D) Turn yellow I '
I
I
,
34. The path of passage of stimulus when you
accidentally touch a hotplate is ·(20161 p02
(A) Receptor 7 Brain 7 Muscles 37. Leydig cells synthesize (5 Nov, 20171
(B) Muscles 7 Spinal cord 7 Receptor (A) insulin (B) growth hormone
(C) Muscles 7 Brain 7 Receptor (C) testosterone (D) estrogen
(D) Receptor"'? Spinal cord 7 Muscles
38. Glucagon increases the blood glucose
concentration by (5 Nov, 2017)
35. Which of the following anions neutralize the (A) promoting glycogenolysis.
acidic pH of the chyme that enters into the (B) increasing the concentration of fructose
duodenum from the stomach? [2016] 2,-6-bisphosphate.
(C) increasing the concentration of pyruvate
(A) H 2 P04 (B) HS04 kinase.
(D) inhibiting gluconeogenesis.
(C) HC0 3 (D) CH3 Coo-

www.puucho.com
CP Publication KVPY chapterwise with www.puucho.com

TOPIC-WISE KVPY rSTREAM SXl Questions with Solutions

39. Concentration of Na+ ions outside a nerve cell is 45 If the number of alveoli in an individual is
-100 times more than inside. The concentration doubled witbout changing the total alveolar
of K+ ions is more inside the cells. The levels of volume, the gas exchange capacity of the lungs
Na' ions and K+ ions are maintained by
will f19 Nov, 20171
IS Nov, 2017]
(A) increase for both 02 and CO2
(A) free diffusion of Na+ ions and pumping of
K~ ions across the membrane. (B) decrease for both 02 and CO2
(B) Na+ and K+ pumps in the membrane. (C) remain unaltered for both 02 and CO2
(C) free diffusion of K+ ions and pumping of (D) increase for 0 2 and decrease for CO2
Na+ ions across the membrane.
(D) water channels formed by lipids in the 46. Which ONE of the following graphs hest
membrane. describes the blood press urc (BP) change when
blood moves from aorta to capillaries?.
40. Compared to the atmospheric air, the alveolar [19 Nov, 20171
air has IS Nov, 2017] (i) (ii) ,.....
,-,.
(A) more p02 and less pC02
(B) less p02 and pC0 2
t,J)
:.r:: '
'' ~
t:IJ
,, -
E ''I 6 t
t

(C) more p0 2 and more pC02 5 ,::; t

'' v '
(D) less p02 and less pC02 /J-, /J-,
co '' i:Q ,,

41. Resting membrane potential of a neuron is Nnta->Capillaries Aorla-+Capillaries


approxima tel y (19 Nml, 20171
(A)-70 mV (B) +70mV (iii) (iv)
'bi] '@ -' ,-
(C)-0.7 V (D)+0.7V ::r: ::r:: '' '
g - ----- -· - .. _- - ''
i::: s ''
a
..__
42. Activation ofsympathctic nervous system /J-, ~
I

' J '
CQ co ~-'
[19 Nov, 20171
(A) decreases blood pressure. Aorta...+Capillaries Aorta-+Capillarics
(B) causes pupil contraction. (A) (i) (B) (ii)
(C) (iii) (D) (iv)
(C) increases heart rate.
(D) causes bronchoconstriction.

43. At physiological temperature, sterols in


biological membranes (19 Nov, 2017]
(A) increase Lhcir fluidity.
(B) decrease their fluidity.
(C) increase their permeability to water.
(D} decrease their permeability to water.

44, In diabetic patients, the pH of blood plasma can


decrease leading to acidosis. This is because
tissues catabolise f19 Nov, 20171
(A) amino acids leading to loss of buffering
capacity of the blood
(B) stored glycogen leading to the
accumulation of pyruvic acid
(C) stored fatty acids leading to the
accumulation of beta hydroxybutyric acid
and acetoacetic acid
(D) nucleic acid pool leading to decrease in
blood pH

www.puucho.com
CP Publication KVPY chapterwise with www.puucho.com

• HUMAN PHYSIOLOGY

ANSWERS
I. (D) 2. (A) 3. (A) 4, (D) 5. (A) 6. (D) 7. (C)

8. (C) 9. (D) JO. (B) 11.(C) 12. (D) 13. (C) 14. (A)

15. (B) 16. (B) 17. (D) 18. (B) 19. (A) 20. (A) 21. (D)

22. (D) 23. (D) 24. (C) 25. (D) 26. (C) 27. (C) 28. (D)

29. (B) 30. (A) 31. (B) 32. (A) 33. (B) 34. (D) 35. (C)

36. (D) 37. (C) 38. (A) 39. (D) 40. (B) 41. (A) 42. (C)

43. (A) 44.(C) 45. (A) 46. (A)

• • : • ~- < ••••

12. Peptic ulcer is mucosa! erosion equal to or greater 26. Synaptic bulbs of axon have vesicles which are
than 0.5 cm of GIT. 70 - 90% of peptic ulcer are filled with acetylcholine.
associated with helicobacter pylori spiral shaped
27. Parasympathetic control decreased during
bacterium that lives in the acidic environment of
exercise.
stomach.
28. Radius of spherical alveoli= 0.1 mm
13. Glucose from glomerular filtrate is reabsorbed Total No. of Alveoli= 400 x 2 = 800 million
from proximal convulatcd tubule via active Surface area of sphere = 4rcR 2
transport. Total respiratory surface Area
= 4ri:R2 x 800 million
15. Aldehyde form of vitamin A (Retinal) is required ""1000 mm2 (approx)
tor synthesis of rhodopsin pigments of rod cells.
defkiency of vitamin A will lead to defective 29. Salivary gland is multicellular exocrine gland,
formation ofrhodopsin. made up cuboidal epithelium.

20. Fetal haemoglobin has has highr affinity with 30. Na 1 - K+ pump is essential for impulse
oxygen as compound to adult haemoglobin. conduction

22. Glycogen is stored primarily in liner & 31. ANF [Anti-Natriuretic Factor] is antagonist of
muscles. Ren in Hormone and act as Vasodilator to
reduce blood pressure. ANF is secreted from
heart.
23. Brown fat stored in adipocyte cells acts as
tbermogenic tissue & fat metabolism produces 32. Oxylocin and Vasoprcssin synthesised in
heat rather than producing ATP hypothalamus then comes into
ncurohypophysis of pituitary gland to release in
24. Holocrine gland distingrate completely for blood
discharge of secretion. [Given answer by KVPY is C]

33. Exhaled air bas CO2 in it.


25. Cavity ofDiancephelon is called as diococl or
Ca(OH)2 + CO2 ----t CaC03 + H20
third ventricle.
Lime water Exhaled air Milley white ppt
So lime water become milky white.

www.puucho.com
CP Publication KVPY chapterwise with www.puucho.com

TOPIC- WISE KVPY rsTREAM SXl Questions with Solutions


34. Reflex arch 42. Sympathetic nervous system stimulate Sino-
Receptor---+ Sensory Neuron---+ Spinal cord---+ Atrial Node.
Motor neuron ---+ Muscle [Effector]
43. Cholesterol in eukaryote & hapanoid in
35. Duodenum receives Bile and Pancreatic juice prokaryoic decreases membranes fluidity
alkaline in nature due to high amount ofHCO 3-
anion which helps in neutralization of acidity of 44. Lack of blood gulcose CEAO to break down of
chyme. Fat which produce acetoacetic Acid and
36. Oxyhaemoglobin dissociation curve is sigmoid 13-hydroxy butyric acid which decrease pH of
shaped. blood.

37. Also know as interstitial cells present in testis 45. Surface area will increase
between somniferous tubule.
46. Blood pressure decrease as it channelise in
38. Promote breakdown of Glycogen in liver.
numerous fine blood vessel.
39. Energy derived pumps are present in plasma
membrane.

40. Pressure gradient is essential for movement of


gases in and out of the body.

41. Resting membrane potential is potential


deference across the plasma membrane when
neuron is at rest.

www.puucho.com
CP Publication KVPY chapterwise with www.puucho.com

Reproductton,Genettcs
and Evolution
1. A horse has 64 chromosomes and a donkey has (A) Both the DNA molecules would denature at
I
62. Mules result from crossing a horse and a the same rate .
donkey. State which of the following is (B) The infonnation given is insufficient to
INCORRECT? [2010] draw any conclusion
(A) Mules can have either 64, 63 or 62
(C) DNA molecule given to student B would
chromosomes
denature faster than that of student A
(B) Mules arc infertile
(D) DNA molecule given to student A would
(C) Mules have well defined gender
(male/female) denature faster than that given to student B
(D) Mules have 63 chromosomes
6. The amino acid sequences ofa bacterial protein
2. A couple went to a doctor and reported that and a human protein carrying out similar
both of them are "earners" for a particular function are found to be 60% identical
disorder, their first child is suffering from that However the DNA sequences of the gene.~
disorder and that they are expecting their coding for these proteins are only 45%
second child. What is the probability that the identical. This is possible because- (2010)
new child would be affected by ' the same
(A) Protein sequence does not depend on DNA
disorder? (2010)
sequence
(A) 100 % (B) 50 %
(C) 25 % (D) 75 % (B) DNA codons having different nucleotides
in the third position can code for the same
3. Restriction endonucleases are enzymes that amino acids
cleave DNA molecules into smaller fragments. (C) DNA codons having different nucleotides
Which type of bond do they act on ? 12010] in the second position can code for the
(A) N-glycosidic Bond · same amino acids
(B) Hydrogen bond (D) Same DNA codons can code for mulliple
(C) Phosphodiestcr bond amino acids
(D) Disulfide bond

7. The following DNA sequence (5' ---) 3')


4. In a diploid organism, there are three different
alleles for a particular gene. Of these three specifies part of a protein coding sequence,
alleles one is recessive and the other two alleles starting from position I. Which of the
exWbit co-dominance. How many phenotypes following mutations will give rise to a protein
are possible with this set of alleles ? (2010) that is shorter than the full-length protein?
(A) 3 (B) 6 (C) 4 (D) 2 [2010]
J 2 3 4 5 6 7 8 9 10 11 12 13 14 15
5. Two students arc given two different double
stranded DNA molecules of equal length. They ATGCAAGATATAGCT
are asked to denature the DNA molecules by (A) Deletion of nucleotide 13
heating. The DNA given to student A has the (B) Deletion of nucleotide 8
following composition of bases (C) Insertion of a single nucleotide between 3
(A:G:T:C:35:15:35:15) while that given to and4
student Bis (A:G:T:C::12:38:12:38). Which of
(D) Insertion ofa single nucleotide between 10
the foHowing statements is true? 120101
and 11

www.puucho.com
CP Publication KVPY chapterwise with www.puucho.com

TOPIC· WISE KVPY fSTREAM SXl Questions with Solutions


8. Consider a locus with two alleles, A and a. If 14. Puns in the polytene chromosomes of
the frequency of M is 0.25, what is the Drosophila melanogaster salivary glands
frequency of A under Hardy-Weinberg represent- [2011]
equilibrium? [20101 (A) transcriptionally active genes
(A) l (B) 0.25 (C) 0.5 (D) 0 (B) transcr:iptionally inactive genes
(C) heterochromatin
9. In humans, the composition of a zygote that (D) housekeeping genes
will develop into a female is- [20111
(A) 44 A+ XX (B) 44 A+ XY 15. According to the original model of DNA, as
(C) 22 + X (D) 23 A proposed by Watson & Crick in 1953, DNA is
a- (2011]
(A) left handed helix
10. A protein with l 00 amino acid residues has
(B) helix that makes a full tum every 70 nm
been translated based on triplet genetic code.
(C) helix where one turn of DNA contains 20
Had the genetic code been quadruplet, the gene
bascpairs
that codes for the protein would have been-
(D) two stranded helix where each strand has
(2011]
opposite polarity
(A) same in size
(B) longer in size by 25 % 16. Rice has a diploid genome with 2n = 24. If
(C) longer in size by 100 % crossing-over is stopped in a rice plant and then
(D) shorter in size selfed seeds are collected, will all the
offaprings be genetically identical to the parent
11. If the sequence of bases in DNA is 5'- plant ? (20111
ATGTATCTCAAT-3', then the sequence of (A) yes, because crossing-over is the only
bases in its transcript will be- [2011} source of genetic variation
(A) 5'-TACATAGAGTTA-3' (13) no, because stopping of crossing-over
(B) 5'-UACAUAGAGUUA-3' automatically increases rate of point
(C) 5'-AUGUAUCUCMU-3' mutation
(D) 5'-AUUGAGAUACAU-3' (C) yes, only if the parent plant 'Nas a
completely inbred line ·
12. Modem evolutionary theory consists of the (D) yes, only if the parent plant was a hybrid
concepts of Darwin modified by knowledge between two pure-bred lines
concerning- f2011J
(A) population statistics
17. E.coli about to replicate was pulsed with
tritiated thymidine for 5 min and then
(B) Mendel's laws
transferred to normal medium. After one cell
(C) the idea of the survival of the fittest
division which one of the following
(D) competition
observations would be correct ? (2011]
(A) both the strands of DNA will be radioactive
13. Soon after the three germ layers arc formed in a
(B) one strand of DNA will be radioactive
developing embryo, the process of
(C) none of the strands will be radioactive
organogenesis starts. The human brain is (D) half of one strand of DNA will be
formed from the- (20111 radioactive
(A) ectodcrm
(B) endoderm 18. For a human male what is the probability that
(C) mcsodcrm all the maternal chromosomes will end up in
(D) partly endoderm and partly mesoderm the same gamete? f20llj
(A) 1/23 (B) 2 23 (C) 246 (D)(l/2) 23

www.puucho.com
CP Publication KVPY chapterwise with www.puucho.com

REPRODUCTION, GENETICS AND EVOLUTION

19. The length of one complete ~ of a DNA 26. The following sequence contains the open
double helix is- [2011] reading frame of a polypeptide. How many
(A) 34 A (B) 34 nm (C) 3.4 A (D) 3.4 µm amino acids will the polypeptide consists of?
(20121
20. Fossils are most often found in which kind of 5'-AGCATATGATCGTJT
rocks ? [2012] CTCTGCTITGMCT-3'
(A) meteorites (B) sedimentary rocks (A) 4 (B) 2
(C) igneous rocks (D) metamorphic rocks
(C) 10 (D) 7

21. Transfer RNA (tRNA) - (2012]


27. The sequences of four DNA molecules are
(A) is present in the ribosomes and provides
structural integrity
given below: [20131
(B) usually has clover leaf-like structure (i) TATATATA TATATA
(C) carries genetic information form DNA to (ii) TITCCCGGGM ATATATATATATA T
ribosomes AAAGGGCCCTTT
(D) codes for proteins (iii) TIGCGTTGCGCC
(iv) GCCGGATCCGGC AACGCMCGCGG
22. DNA mutations that do not cause any CGGCCTAGGCCG
functional change in the protein product are Which one of these DNA molecules will have
known as - I2012] the highest melting temperature (Tm) ?
(A) nonsense mutations (A) i (B) ii
(B) missense mutations (C) iii (D) iv
(C) deletion mutations
(D) silent mutations 28. lf DNA codons are ATG GAA, insertion of
thymine after the first codon results in, 12013]
23. lfthc sequeuce ofbases in sense strand of DNA
(A) non-sense mutation
is 5'-GITCA TCG-3', then the sequence of
(B) mis-sense mutation
bases in its RNA transcript would be~ [2012)
(C) frameshift mutation
(A) 5'-GTICATCG-3'
(D) silent mutation
(B) 5'-GUUCAUCG-3'
(C) 5'-CAAGTAGC-3'
(D) 5'-CMGUAGC-3' 29. According to
me.nde1, ...................... segrega le
24. ln cattle, the coat colour red and white are two and .............. assort independently. f2Ul3]
dominant traits, which express equally F 1 to (A) alleles ofa gene; alleles of different genes
produce roan (red and white colour in equal (B) alleles of different genes; alleles of a gene
proportion). If F 1 progeny are selfbred, the (C) dominant traits; recessive traits
resulting progency in F2 will have phcnotypic (D) recessive traits; recessive traits
ration (red : roan: white) is - (2012]
(A)l:1:1 (R)J:9:3 30. Morphogcnctic movements take place
(C) l : 2 : l (D) 3 : 9 : 4 predominantly during the following embryonic
stage [2013]
25. The restriction cndonuclease EcoR-1 recognizes (A) Blastula (B) Monda
and cleaves DNA sequence as shown below
(C) Gastrula (D) Fertilized eggs
5' -GA AT T C-3'
3' -C TT A A G-5'
31. The only organ which is capable of producing
What is the probable number of cleavage site:.
iliat can occur in a 10 kb long rnndom DNA Fructose in humans is- f2013J
sequence ? [2012J (A) liver (B) pancreas
(A) 10 (B) 2 (C) 100 (D) 50 (C) seminal vesicles (D) muscle

www.puucho.com
CP Publication KVPY chapterwise with www.puucho.com

TOPIC- WISE KVPY rsTREAM SXl Questions with Solutions


32, In a large isolated population, alleles p and q at 37. Testosterone is secreted by which endocrine
a locus are at Hardy Weinberg equilibrium. The part of testis ? [20141
frequencies are _p = 0.6 and q = 0.4. The (A) Leydig cells
proportion of the heterozygous genotype in the (B) Seminiferous tubules
population is: [2013] (C) Tunica albugenia
(A) 0.24 (B) l (C) 0.48 (D) 0.12 (D) Sertoli cells

33. Watson and Crick model of DNA is [2013] 38. The mutation of a purine to a pyrinudine is
(A) B-form DNA with spiral length of 34 A known as [20141
(A) transition (B) frame shift
and a diameter of 20 A
(C) nonsense (D) transvcrsion
(B) A-form DNA with a spiral length of 15 A
and a diameter of20 A
39. Which one of the following 1s the
(C) Z-form DNA with spiral length of 34. A and
complementary sequence for the DNA with
a diameter of20 A
5'-CGTACTA-3' (2014]
(D) B-form DNA with spiral length of 28 A
(A) 5'-TAGTACG-3' (B) 5'-ATCATGC-3'
and a diameter of 14 A
(C) 5'-UTCUTGC-3' (D) 5'-GCUAGCA-3'
34. From an earty·amplubian embryo the cells that
would give rise to skin adults were transplanted 40. A flmver with Tt genotype is cross-pollinated
into the ·developing brain region of another by TI pollens. What will the genotypes of the
embryo. The transplanted cells developed into resulting endosperm and embryo, respectively,
be? [2014]
brain tissue in the recipient embryo. What do
(A) TIT, (TT+ Tt) (B) (TIT+ TTt), TT
you infer from this experiment? [2013]
(C) JTt, Tt (D) Tit, (TI+ Tt)
(A) Cell fate is permanently determined during
early embryonic development
41. A new life form discovered on a distant planet
(B) Developmental fate of donor cells is
has a genetic code consisting of five unique
influenced by the surrounding cells
nucleotides and only one stop codon. If each
(C) Developmental fate of donor ccUs is not codon has four bases, what is the maximum
influenced by recipient cells number of unique amino acids this life form
(D) Any cell which is tramplanted into another can use? [2014]
embryo always develops into a brain. ( A) 624 (B) 20 (C) 124 (D) 3124

35. Which of the following is the final product of a 42. A spontaneous mutation results in a couple
gene? (2014] having only female progeny. ,vhen the
(A) a polypeptide only daughter marries and has children, none of
(B) an RNA only them are males. However, in the third
(C) either polypeptide or RNA generation there are few male offspring. What
(D) a nucleotide only 1s the most likely explanation of this
observation - [2014]
36. Forelimbs of \vhales, bats, humans and cheetah (A) The mutation reverses spontaneously in
are examples of which of the following the third generation
processes? (2014] (B) The mutation occurs on the X chromosome
(A) Divergent evolution and is both recessive and lethal
(C) The mutation occurs on the X chromosome
(B) Convergent evolution
and is both recessive and dominant
(C) Adaptation
(D) The mutation occurs on an autosome and is
(D) Saltation
dominant

www.puucho.com
CP Publication KVPY chapterwise with www.puucho.com

• REPRODUCTION, GENETICS AND EVOLUTION

43. In Griffith's experiments mice died when (A) DNA with 15% guanine
injected with - [2014] (B) DNA with 30% cytosine
(A) heat killed S-strain (C) DNA with 40% thymine
(B) heat killed S-strain combined with R-strain (D) DNA with 50% adenine
(C) heat killed R-strain
(D) live R-strain 51. Mutation in a single gene can lead to changes
in multiple traits. This is an example of [2015]
44. Which one of these substances will repress the (A) Heterotrophy (B) Co-dominance
lac operon? [2015] (C) Penetrance (D) Pleiotropy
(A) Arabinose (B) Glucose
(C) Lactose (D) Tryptophan 52. Which one of the following is used to treat
cancers? (20151
45. Assume a spherical mammalian cell has a (A) Albumin (B) Cyclosporin A
diameter of 27 microns. If a polypeptide chain (C) Antibodies (D) Growth hormone
with alpha helical conformation bas to stretch
across the cell, how many amino acids should it 53. Consider the following pedigree over four
be comprised of? [20151 generations and mark the correct answer below
(A) 18000 (B) 1800 (C) 27000 (D) 12000 ahout the inheritance of haemophilia. (2015]

46. In a random DNA sequence, what is the lowest


frequency of encountering a stop codon? (20151
(A) 1 in 20 (B) l in 3 (C) l in 64 (D) 1 in IO

47. The two alleles that determine the blood group


AB of an individual are located on [2015]
(A) two different autosomes
(B) the same autosomc
(C) two different sex chromosomes
(D) one on sex chromosome and the other on
D Normal male O Normal female
an autosome • Haemophilic male @Haemophilic female
(A) Haemophilia is X-linked dominant
48. Spermatids are formed after the second meiotic (B) Haemophilia is autosomal dominant
division from secondary spermatocytcs. The (C) Haemophilia is X-linked recessive
ploidy of the secondary spermatocytes is [2015] (D) Haemophilia is Y -linked dominant
(A) n (B) 2n (C) 3n (D) 4n
54. If a double stranded DNA has 15% cytosine,
49. If molecular weight of a polypeptide is 15.3 what is the% of adenine in the DNA? [2015]
kDa, what would be the minimum number of (A) 15% (B) 70% (C) 35% (D) 30%
nucleotides in the mRNA that codes for this
polypeptide? Assume that molecular weight of 55. The activity ofa certain protein is dependent on
each amino acid is 90 Da. [2015] its phosphorylation. A mutation in its gene
(A) 510 (8) 663 (C) 123 (D) 170 changed a single amino acid which affected the
function of the molecule. Which amino acid
50. Melting temperature for double stranded DNA change is most likely to account for this
is I.he temperature at wh1ch 50% of the double observation? (20151
stranded molecules are converted into single (A) Tyrosine to Tryptophan
stranded molecules. Which one of the (B) Lysine to valinc
. following DNA will have the highest melting (C) Leucinc to isoleuciue
temperature? [20151 (D) Valine to alanine

www.puucho.com
CP Publication KVPY chapterwise with www.puucho.com

TOPIC-WISE KVPY rSTREANI SXl Questions with Solutions


56. Enzyme X calalyzes hydrolysis of GTP into
GDP. The GTP-bound form of X transmits a
signal that leads to cell proliferation. The GDP D D
- bound form does not transmit any such
signal. Mutations in X arc found in many
cancers. Which of the following alterations of
X are most likely to contribute to cancer?
12015]
(A) Mutations that increase the affinity of X
forGDP
(8) Mutations that decrease the affinity of X AUG UAC
forGTP (a) (b)
(C) Mutations that decrease the rate of G1P
hydrolysis
(D) Mutations that prevent expression of moieties
enzymeX.

57. The hard outer layer of pollens, named exine, is


made of [2,0161
(A) cellulose (B) tapetum
(C) sporopollenin (D) pectin

CC G C C U - - - - - > anticodons
58. In the presence of glucose and lactose,
Escherichia coli utilizes glucose. However, (c) (d)
lactose also enters the cells because 12016]
(A) low level of permcasc is always present in (A) a, b, c, d (D) b. a, c, d
the cell (C) c, d, a, b (D)b,a, d, c
(B) it uses the same transporter as glucose
(C) if diffuses through the bacterial cell 61. If the initial number of template DNA
membrane molecules in a PCR reaction is 1000, the
(D) it is trnnsported through porins number of product DNA molecules at the end
of20 cycles will be closest to (2016]
59. A human population containing 200 individuals (A) I a3 (B) I06
has two alleles at the 'T' locus, named T and t. (C) 109 (D) 1012
T, which produces tall individuals, is dominant
over t, which produces short individuals. If the 62, The allele for black hair (B) is dominant over
population has 90 IT, 40 n and 70 tt brown hair (b) and the allele for brown eye (E)
genotypes, what will be the frequencies of
is dominant over blue eye (e). Out of the
these two alleles in this population? 120161
offsprings obtained upon mating a black-haired
(A) T, 0.50 ; l, 0.50 (8) T, 0.55 ; t, 0.45
and brown-eyed individual (BbEe) with a
(C) T. 0.45; t, 0.35 (D) T, 0.90; t, 0.10
brown-haired and brown-eyed individual
(bbEE), the ratio of brown-haired and brown-
60. An mRNA is transcribed from a DNA segment
eyed individuals to black- haired and brown-
having the base sequence 3'-
eyed individuals is [2016]
TACATGGGTCCG-5'. What will be the
correct order of binding of the four amino acyl- (A) 2 : I (B) 3 : I
tRNA complexes given below during (C) 1 : l (D) 1 : 2
translation of this mRNA? [2016)

www.puucho.com
CP Publication KVPY chapterwise with www.puucho.com

REPRODUCTION, GENETICS AND EVOLUTION


63. For a particular gene that determines the coat 68. When a pure bred, red flower-producing plant
color in a diploid organism, there are three of genotype RR is crossed with a pure bred,
different alleles that are codominant. How white flower-producing plant of genotype rr,
many different skin colors are possible in such all the F I plants produced pink flowers If all the
an organism? [5 Nov, 2017] plants in each generation from F 1 to .F6 are
(A) 9 (B) 6 selfed, what will be the percentage of plants
(C) 4 (D) 3 with red and white flowers in the final
population consisting of a large number of
64. Two genetic loci controlling two different traits individuals '! (Consider that flower colour has
are linked. During the inheritance of these no effect on reproduction and survival.)
traits, the Mendelian Jaws that would be IS Nov, 2017)
affected is/are J5 Nov, 2017] (A) 3-4 (B) 12-13
(A) Law of dominance, law of segregation and (C) 49- 51 (D) 97-100
law of independent assortment
(B) Law of segregation and Law of 69. Two homozygous parents harboring two
independent assortment different alleles of a gene, exhibiting
(C) Only Law of independent assortment incomplete dominance for flower colour were
(D) Only Law of segregation used for a genetic experiment. Which ONE of
the following statements is INCORRECT?
65. Which ONE of the following statements is f19 Nov, 2017]
INCORRECT? [5 Nov, 2017) (A) The F2 generation will consist of plants of
(A) Alleles are different forms of the same three different flower colours
gene. (B) The genotypic and phenotypic ratios
(B) Alleles are present at the same locus. obtained in the F2 generation will be
(C) Alleles code for different isqforms of a different
protein. (C) The F1 generation will be of a different
(D) Alleles are non-heritable. flower colour compared to both the parents
(D) The genotypic ratio obtained in the F2
66. The genetic distance between genes A and D is generation will be the same irrespective of
10 cm. An organism with Ab combination of whether it is complete dominance or
the alleles is crossed with the organism with aB incomplete dominance
combination of alleles. What will be the
percentage of the gametes with AB allele 70. Which ONE of the following is an essential
combination by an Fl individual? (5 Nov, 2017) condition for a population to be at Hardy-
(A) I (B) 5 Weinberg equilibrium ? [19 Nov, 2017]
(C) 10 (D) 50 (A) Random mating
(B) lmnugration
(C) Emigration
67. The minimum number of plants to be screened (D) Geographical isolation
to obtain a plant of the genotype AabbCcDd
from a cross bctccn plants of genotypes 71. Inbreeding in a population leads to
AaBbCcDd and AABbCCDd is [5 Nov, 20171
[19 Nov, 2017)
(A) decrease in recessive disorders
(A) 8 (B) 16
(B) hetcrosis
(C) 32 (D) 64
(C) increase in homozygosity
(D) increase in heterozygosity

www.puucho.com
CP Publication KVPY chapterwise with www.puucho.com

'
TOPIC- WISE KVPY rSTREAM SXl Questions with Solutions
72. Suppose the three non-linked autosomal genes 73. The following two pedigrees describe the
A, B and C control coat color in an animal and autosomal genetic disorders P and Q in Family
the dominants alleles A, B and C are 1 and Family 2, respectively (19 Nov, 20171
responsible for dark color and the recessive
Family I Family2
alleles a, b and care responsible for light color.
1f a cross between a male of AABBCC
genotype and a female of aabbcc genotype
produce 640 off springs in the F2 generation,
how many of them are likely to be of the
parental genotype? (19 Nov, 2017] Choose the CORRECT statement from the
(A) 10 (B) 20 following options.
(C) 160 (D) 640 (A) Both P and Q are dominant traits
(B) P is a dominant trait and Q is a recessive
trait
(C) Both P and Qare r~cessive traits
(D) P is a recessive trait and Q is a dominant
trait

www.puucho.com
CP Publication KVPY chapterwise with www.puucho.com

REPRODUCTION, GENETICS ANO EVOLUTION

ANSWERS
1. (A) 2.(C) 3. (C) 4. (C) 5. (D) 6. (B) 7. (B)

8.(C) 9. (A) 10. (B) 11. (D) 12. (C) 13. (A) 14. (A)

15. (D) 16. (C) 17. (B) 18. (D) 19. (A) 20. (B) 21. (B)

22. (D) 23, (B) 24. (C) 25. (B) 26. (D) 27. (D) 28. (C)

29. (A) 30.(C) 31. (C) 32. (C) 33. (A) 34. (8) 35.(C)

36. (A) 37. (A) 38. (D) 39. (A) 40. (A) 41. (A) 42. (B)

43. (B) 44. (B) 45. (A) 46. (A) 47. (B) 48. (A) 49. (A)

50. (B) 51. (D) 52, (C) 53,(C) 54. (C) 55. (A) 56. (C)

57. (C) 58. (A) 59. (B) 60. (Bonus) 61. (C) 62. (C) 63. (B)
64. (C) 65. (D) 66. {B) 67. (C) 68. (D) 69. (B) 70. (A)

71. (C) 72. (B) 73. (B)

§i, • , ... 'I"··


. \_, ... SOLUTIONS J::t;,:\.
12. Propounded by Herbert Spencer for natural 23. The direction of RNA sequence is also from 5'-
selection process. This explains that 3'. The sequence of sense strand is -
adaptability has the genetic basis which proves 5'-GTICATCG-3'
fitness of an organism and nature selects We know the sequence ofm-RNA is similar to
organism for its fitness and allow to produce its
sense strand. Only uracil is present instead of
progeny in large number.
Thymine.
Hence the m-RNA sequence will be -
13. Organogenes_is begins with the process of
neurulation. This neurulation begin with the 5'-GUUCAUCG-3'
formation of primitive streak in epiblast
(Ectoderm) which leads to formation of neural 24. This is an example of Ca-dominance. (Result is
tube (nerve cord) 1 Red: 2 Roan: 1 white).

20. Sedimcntary rocks are type of rock that are 25. Eco RI is an example of six-cutter restriction
formed by deposition of material at the earth's endonuclease. It usually cleaves once in every
surface and within bodies of water. It forms
4096 bp.
only 8% of total volume of crust fossils are

21.
mostly found in these.

The clover leaf model is the 2-D model. Given


(±J- 4;96
Given length of DNA fragment = 10 Kb
by Holley. ILs 3-D model is a L-sbaped model.
= 10000 bp
Hence,
22. Silent mutations also called same - sense
mutations arc not lethal because in these Probable no. of cleaving sites === 1OOOO "' 2.44
mutations, the amino acid do not get changed. 4096

www.puucho.com
CP Publication KVPY chapterwise with www.puucho.com

TOPIC- WISE KVPY rsTREAM SXl Questions with Solutions


26. 5' AGCAT A TG 34. The fate of cells during embryonic life depends
'-------,-'
1 on their surrounding cells
A TC G IT T CT C TG C TI
,._____, ' - r - - ' ' - v - - ' ,._____, ' - v - - '
2 3 4 5 6 35. Gene is a segment of genetic material which
T GA T GAACT' 3' produces either polypeptide or a RNA [rRNA
'-------,-' '-..,--------'
7 or.tRNA]
5'AGC
36. Fore limbs of whale, bat, human & cheetah are
-~--G A
AU A U U CG U U U C
' - . . . - ' '-------...--' ' - . . . - '
2 3 4
U
homologous organ which represents divergent
evolution.
C U G C U UU G AACU3'
' - . . . - ' '--------V-------- - ~ - -
s 6 Stop
37. Leydig cells or Interstitial cells of testes
So this sequence is present in mRNA and
secretes tcstosteron hormone.
translation start from start codon 'AUG' so 6
Amino acid will be formed close to 7
38. Mutation of purine to purimidine is knov.rn as
So answer can (D)
transversion. 8 possible transversion can
occurs.
27. More the hydrogen bond-., more will he the
melting temperature (Tm)
39. Double stranded DNA has antiparalllcl strands
(iv) DNA molecules has highest number ofC = G
and complementary N-bases
bonds
so, 5' CGTACTA 3'
3' GCATGAT 5'
28. Insertion or deletion of a single nucleotide
,(--
causes shifting in complete framework of
amino acids thus, answer is 5'TAGTACG3'

29. Segregation is related to separation of alleles of 40. Tt x TT (Pollen means male plant)
same gene whereas independent assortment is Endosperm ----t 2 polar nuclei (should be same)
'-----,,-------
related to separation of different genes. f=ial ~

+ l male nuclei
30. Morphogenic movements takes place during So, TIT
gastrulation that results in formation of 3 germ · Embryo-> Egg cell + 1 male nuclei
'--------v-----
layers ectodcnn, mesodcrm & endoderm. female
So, either TT or Tt
31. Fructose is produced by seminal vesicle, it
helps in providing energy to sperms. 41. No. of unique nucleotide= 5
No. of bases in codon= 4
so, total combinations·:::::> 54 :::::> 625
32. p = 0.6 q = 0.4
No. of stop codons= l
Proportion of heterozygous genotype m . so, unique amino acids ( maximum) = 625 - I
population= 2pq :::::>624
2pq =2 X 0.6 X 0.4 = Q.48
42. The mutation that occurred is on x-
33. Watson and Crick model was based on chromosome and is both recessive and lethal
common form of DNA i.e. B-fonn which has since, female is carrier it survives but due to
hemizygous condition male is unable to
spiral turn= 34A
survive.
Diameter= 20A
xx_m --;)> carrier (survives)
xmy--;), die5

www.puucho.com
CP Publication KVPY chapterwise with www.puucho.com

• REPRODUCTION, GENETICS AND EVOLUTION

While in third generation, the possibility of 51. Pleiotrophy : A single gene affects more than
male child occurs. one phenotype.

43. In Griffith experiment, mice died when injected 52. Monoclonal antibodies arc used for treatment
\Vith combination of cancer.
heat killed Htrain + Live R strain
which resulted in transformation of R II into S 53.
y
l1I form.

44. Presence of glucose inhibit the lac opcron

45. No. of amino acid in one term in a. helix= 3.6


Female parent is carrier and male and female
Pitch length for a Helix = 5.4 A°
offspring is affected.
:. The No. of Amino acid in polypeptide
3·6 X 27 X 10-6 54. Cytosine = 15%
5.4x 10-IO
According to cbargaff principle
= 1.8 x 104 Amino acid A=T &G=C
= 18000 amino acid A+ T+G+C= 100
Thus A will be 35%
46. Total no. of codon= 64
Functional codon= 61 55. Serine, tyrosine, threonine and histidine are
Stop codon =3 most common target for phosphorylation in
:. frequency of encountering stop codon eukaryotes.
3 l If T1Tos inc get mutated in tryptophan than
=-l:::-
61 20 phosphorylation of gene does not occur and
gene can not be activated
47. Allele of the same gene are present at same
gene locus on homologous chromosome 56. GTP -X Complex-)> Active cell division
GDP -X Complex~ No cell division
48. Secondary spermatocyte formed by meiosis-I. and GTP Hi'!!roly;is > GOP
Xen:,yme

49. Molecular weight of polypeptide= 15.3 kda Mutation cause the decrease of rate of
hydrolysis ofGTP. Thus GTP-X complex
Molecular weight of amino acid= 90 Da
remain present for long time and cell division
:. No. of Amino acid in polypeptide become uncontrolled.
= 15.3xl0 3 - l?0
57. Sporopollcnin forms the exinc of pollen grain
90
which is resistant to acids, high temperature
·: One amino acid is coded by = 3 Nitrogen and radiations.
base
170 amino acid would be coded by 58. The preferred molecule is glucose first, then
= 170 x 3 = 510 Nitrogen base lactose is used by E.coli because ultimately
lactose is also broken down into galactose and
50. Melting temperature of DNA oc GC content glucose. Permease is a carrier protein which
belps in facilitated diffusion. Lac operon is
always operational at low levels.

www.puucho.com
CP Publication KVPY chapterwise with www.puucho.com

TOPIC-WISE KVPY rSTREAM SXl Questions with Solutions

59. A population with 200 individual has 90TT, 40


63.
n(n + 1) = 3(3 + 1) =6
Tt and 70 tt genotypes 2 2
i.e. dominant allele (T) is IT + Tt
i.e. 90 + 90 + 40 64. Linkage in exception of law of independent
=>220 assortment
Recessive allele (t) is Tt + tt
i.e. 40 ·I 70 + 70 65. Alleles are alternate form of gives of one
=> 180 character present oµ same locus.
Total allele= 400
Dominant allele (T) frequency = 220 = 0.55 66. Rccombinants formed = l 0% (AB 5% & ab
400 5%)

Recessive allele (t) .frequency= ISO = 0.45 67. AaBbCcDd x AaBbCCDd


400
J..
59. DNA sequence AabbCcDd
l 1 I 1 1
- X --X - X - =-
60. 3' TAC ATG GGT CCG 5' 2 4 2 2 32
mRNA
5' AUG UAC CCA GGC 3' 68. After selfing homozygous indivisiual produces
All white & red offspring and heterozygous

~ Lc~J~~J ~
also produces 50%. Red & White flower.

69. Both genotype & phenotype ratio same l : 2 : l

UACAUGGGLJCCG 70. For Random mixing ofalleles.

71. Inbreeding is the production of offspring from


5'AUG UAC CC A GC 3'
the mating or breeding of individuals or
mRNA organisms tbat are closely related genetically
Direction of translation
None of the option is correct 72. ~ x 640 = 20 (Trihybrid cross)

Lin L
As option (D) diagram is incorrect 64

73.
Itsbauld bo place of
I
AaFamily
aa
G G U CC U
61.

62.
During PCR1 the number of DNA molecules
increases by zn. Where 'n' is number of
divisions

Parents - BbEe x bbEE


x•lH-fi~Aa
aa aa aa aa Aa Aa aa aa

Black haired Brown haired


Brown eyed Brown eyed
J. ,!,
Gametes-
BE Be bE be
BbEE BbEe bbEE bbEe bE
Ratio of Brown haired and Brown eyed to
Black haired and Brown eyed is 2 : 2 or l : 1

www.puucho.com
CP Publication KVPY chapterwise with www.puucho.com

Biology in Human Welfare


and Biotechnology
l. · When of the following class of
I
Sometimes urea is fed to ruminants to improve 6.
their health. It works by- [20101 immunoglobulins can trigger the complement
(A) Helping growth of gut microbes that break cascade? (2012]
down cellulose (A) IgA (B) IgM
(B) Killing harmful microorganisms in their gut (C) lgD (D) lgE
(C) Increasing salt content in the gut
(D) Directly stimulating blood cell proliferation
7. Diabetes insipidus is due to - [2012]

2. Conversion of the Bt protoxin produced by (A) hypersecretion of vasopressin


Bacillus tlmringiensis to its active form in the (D) hyposecretion of insulin
gut of the insects is mediated by- (2010] (C) hypersccrction of insulin
(A) acidic pll of the gut (D) hyposccrction of vasopressin
(B) alkaline pH of the gut
(C) lipid modification of the protein 8. In Dengue virus infection, patients often
(D) cleavage by chymotrypsin develop haemorrhagic fever due to internal
bleeding. This happens due to the reduction of
3. Patients who have undergone organ transplants (2012]
are given anti-rejection medications to- [2011] (A) platelets (8) RBCs
(A) minimize infection
(C) WBCs (D) lymphocytes
(B) stimulate B-macrophage cell interaction
(C) prevent T-lymphocyte proliferation
(D) adopt the HLA of donor
9. Which one of the following techniques i.s used
for the detection of proteins? (20131
4_ Saline drip is given to a Cholera patient (A) Northern blotting
because- [2011] (D) Western blotting
(A) NaCl kills Vibrio cholera (C) Southern blotting
(B) NaCl generates ATP (D) In-situ hybridization
(C) Na+ ions stops nerve impulse and hence
sensation of pain 10. 109 bacteria were spread on a agar plate
(D) Na~ ions help in retention of water in body containing penicillin. After incubation
tissue overnight at 37°C, 10 bacterial colonies were
observed on the plate. That the colonies arc
5. The disorders that arise when the immune
likely to be resistant to penicillin can be tested
system destroys self cells are called
autoimmune disorders. Which of the following by (20131
would be classified under this? [2012] (A) measuring their growth rate
(A) rheumatoid arthritis (B) observing the colour of the col onics
(B) asthma (C) checking their ability to grow on another
(C) rhinitis plate containing penicillin
(D) eczema (D) checking their ability to cause disease

www.puucho.com
CP Publication KVPY chapterwise with www.puucho.com

TOPIC-WISE KVPY rsTREAM SXJ Questions with Solutions


11. Eco RI and Rsa I restriction endonucleases 15. IU1inoviruses are the causative agents of {2014)
require 6 and 4 hp sequences respectively for (A) Diarrhoea (I3) AIDS
cleavage. In a 10 kb DNA fragment how many (C) Dengue (D) Common cold
probable cleavage sites arc present for these
enzymes {2013]
16. What is the genetic material of Ebola virus ?
(A) 0 Eco R1 and 10 Rsa I
(2014]
(B) 1 Eco RI and 29 Rsa I
(A) Single-stranded DNA
(C) 4 Eco RI and 69 Rsa I
(D) 2 Eco RI and 39 Rsa I (B) Double-stranded RNA
(C) Single-stranded K.NA
12. Presence of plastids in Plasmodium suggests (D) Double-stranded DNA
[20131
(A) it is a plant species 17. How many linear DNA fragments will be
(B) it is a parasite with a cynobacterium as an produced when a circular plasmid is digested
endosymbiont 'cf
with'a restriction enzyme having 3 sites?
(C) it is a parasite with an archebacterium as an [2014]
endosymbiont
(A) 4 (B) 5
(D) it is a plant species with an archebacterium
(C) 3 (D) 2
as an endosymbiont

13. A scientist has cloned an 8 Kb fragment of a 18. A circular plasmid of 10,000 base pairs (bp) is
mouse gene into the Eco R1 site of a vector of 6 digested with two restriction enzymes, A and
Kb size. The cloned DNA has no other Eco Rl B, to produce a 3000 bp and a 2000 bp bands
site within. Digestions of the cloned DA is when visuali;,;cd on an agarose gel. When
shown below. digested with one enzyme Ht a time, only one
Which one of the following sets of DNA band is visible at 5000 bp. If the first site for
fragments generated by digestion with both Eco enzyme A (A I) is present at the 100th base, the
RI and Barn HI as shown in (iii) is from the order in which the remaining sites {A2, Bl and
gene ? [2013]
D2) are present is - [20141
EcoRI only BamH1 only EcoRI and BamHI
(A) 3100, 5100, 8100 (B) 8100, 3100, 5100
8.0K (C) 5100, 3100, 8100 (D) 8100, 5100,. 3100

6.0K
5.5 Kb -- 19. Immunosuppressive drugs like cyclosporin
4.5 Kb
4,0 Kb - 4.0Kb
3.5 Kb
-- delay the . rejection of graft post organ
3.0 Kb - transplantation by (2015}
2.SKb - (A) inhibiting T cell infiltration
1.0 Kb - (8) killing 8 cells
(C) killing macrophages
(i) (ii) (Hi)
(D) killing dendrite cells
(A) 1 Kb and 4 Kb (D) 1 Kb and 2.5 Kb
(C) 1 Kb and 3 Kb (D) 1 Kb and 3.5 Kb 20. In biotcchnolo 6,y applications, a selectable
marker is incorporated in a plasmid [2015]
14. A bacterial colony is produced from (2014] (A) to increase its copy number
(A) a single bacterium by its repetitive division (B) to increase the transformation efficiency
(B) multiple bacterium without replication (C) to eliminate the non-transformants
(C) clumping of two to three bacteria (D) to increase the expression of the gene of
(D) a single bacterium without cell division interest

www.puucho.com
1
L.
CP Publication KVPY chapterwise with www.puucho.com

BIOLOGY IN HUMAN WELFARE AND BIOTECHNOLOGY


21. Following are the types of immunoglobulin and 16. Passive immunization is achieved by
their functions. Which one of the following is administcri ng 11016)
INCORRECTLY paired? [2015) (A) Heat killed vaccines
(A) JgD : viral pathogen
(B) Toxoids
(B) IgG : phagocytosis
(C) lgE: allergic reaction {C) Live attenuated vaccines
(D) IgM : complement fixation (D) Antibodies

22. How many bands are seen when 27. Match the human disorders shown in Group I
immunoglobulin G molecules are analysed on a
sodium dodecyl sulphate polyacrylamide gel
with the biochemical processes in Group IL
electrophoresis (SOS - PAGE) under reducing Choose the correct combination [2016]
conditions? [2015) Group I Group II
(A) 6 (B) 1 (C) 2 (D) 4 P. Phenylketonuria i. Melanin synthesis
Q. Albinism ii. Conversion
23. In a mixed culture of slow and fast growing
of Phenylalanine
bacteria, penicillin wiJI [2015]
(A) kill the fast growing bacteria more tltan the to Tyrosine
slow growing R. Homocystinuria iii.Tyrosine degradation
(B) kill slow growing bacteria more than the S. Argininemia iv. Methionine
fast growing metabolism
(C) kill both the fast and slow growing bacteria
v. Urea Synthesis
equally
(D) will not kill bacteria at all (A) P-ii, Q-i, R-iv, S-v
(B) P-i, Q-iv, R-ii, S-v
24. A mixture of equal numbers of fast and slow (C) P-ii, Q-i, R-v, S-iii
dividing cells is cultured m a medium (D) P-v, Q-iii, R-i, S-ii
containing a trace amount of radioactively
labeled thymidine for one hour. The cells are
then transferred to regular (unlabelled) 28. Interferons combat viral infection by
medium. After 24hrs of growth in regular (5 Nov, 2017]
media 120151 (A) inhibiting viral packaging directly.
(A) fast dividing cells will have maximum (B) increasing the binding of antibodies to
radioactivity viruses.
(B) slow dividing cells will have maximum (C) binding to the virus and agglutinating them.
radioactivity (D) restricting viral spread to the neighboring
(C) both will have same amount of cells.
radioactivity
(D) there will be no radioactivity in either types 29. Which ONE of the following is NOT essential
of cells for Polymerase Chain Reaction (PCR)?
I5 Nov, 20171
25. Consider the linear double stranded DNA {A) Restriction enzyme
shown below. The restriction enzyme sites and (B) Dcnaturation of D:-.JA
the lengths demarcated are shown. This DNA is (C) Primers
(D) DNA polymerase
completely digested with both EcoRl and
BamHI restriction enzymL-s. If the product is 30. Antigen-angtibody reactions (5 Nov, 2017]
analyzed by gel electrophoresis, how many (A) always result in precipitation of the
distinct bands would be observed? I2015) complex
(B) depend only on covalent interactions.
lkbl 3kb l 5kb I 3kb (C) arc irreversible.
Eco Rl Barn H1 Eco RI (D) depend on ionic and hydrophobic
interactions.
(A) 5 (B)2 (C)3 (D)4

www.puucho.com
CP Publication KVPY chapterwise with www.puucho.com

TOPIC· WISE KVPY rsTREAM SXl Questions with Solutions


31. Which ONE of the following combinations of 34. Skin·prick test on the forearm is conducted to
molecular masses of polypeptides are obtained identify the responsible allergen.. This is
from purified human IgM when analysed on because 119 Nov, 2017)
sodium dodccyl suplhate polyacrylamide gel
(A) of the presence of mast cells under the
skin.
electrophoresis (SOS-PAGE) under reducing
(B) lymphocytes migrate rapidly from the
conditions? (5 Nov, 2017) blood to the skin.
(A) 55 kDa, 15 kDa (C) hair follicles can enhance the reaction.
(B) 70 kDa, 25 kDa, 15 kOa (D) Neutrophils migrate rapidly from the blood
(C) 55 kDa, 25 kDa to the skin.
(D) 155 kDa
35. Which ONE of the following processes in
E coli does NOT directly involve RNA ?
32. Molecular weight of E. Coli DNA is 3.1 x 109
(19 Nov, 20171
glmol. A vcrage molecular weight of nucleotide (A) DNA replication (B) Transcription
pair is 660 g/mol and each nucleotide pair (C) Translation (D) DNA repair
contributes to 0.34 run to the length of DNA
The length of E. coli DNA molecule will be 36. Frederick Griffith performed an experiment
approximately (5 Nov, 2017) where mice were killed when injected with a
(A) 0.8 nm (B) 1.6 nm mixture of killed S-type Streptococcus (HKS)
(C) 1.6 µm (D) 1.6 mm and live R-type Streptococcus (LRS) but not
with HKS or LRS separately. Mice were killed
33. The schematic below describes the status of lac because [19Nov,2017J
operon in the absence of lactose. Which ONE (A) lipids from HKS made LRS virulent
of the following happens when lactose is (B) RNA from HKS transformed LRS and
present in the cell ? (S Nov, 2017] made it virulent
(C) proteins from HKS made LRS virulent
(D) DNA from HKS transformed LRS and
I
ma de it virulent
Repressor •
mRNA ,' '-tt.__ . f Repressor binds to o and
! prevents the transcription 37. In an experiment, bacteria were infected with

I ..
· ofz,yand J 32Plabelled virus in a ratio of 5 : 1. The culture
Reprcssor .............. --·· was rigorously shaken followed by
P' : i promoter .
Protein i : lac repressor gene ccntrifugation. Radioactivity was
r'0 '": lac promoter ~ (19 Nov, 20171
0 : lac operator (A) lost due to metaholic activity
z, y, a ; three lac genes
(B) detected in supernatant as inorganic
phosphate
(A) Lactose binds to P and stops the (C) detected in the supernatant in association
transcription of i. with viral capsid
(B) Lactose is· converted to allolactose, which (D) detected in bacterial cell pellet
binds to pac and results in the displacements
of the rcprcssor from 0. 38. As indicated in the gel image, lanes X and Y
(C) Lactose is converted to allolactosc, which represent samples obtained from a circular
bind<; to the repressor protein and prevents plasmid DNA after complete digestion using
its interaction with 0. restriction enzyme X or Y with different sites,
respectively. How many sites for X and Y are
(D) Lactose has no effect on the status of the
present in the plasmid (sizes of the bands in
lar; opcron. kilo base pairs (kb) is shown)? (19 Nov, 2017]

www.puucho.com

\
CP Publication KVPY chapterwise with www.puucho.com

BIOLOGY IN HUMAN WELFARE AND BIOTECHNOLOGY


40. Match the enzymes in column-I with their
X y
respective biochemical reactions in column-IL
6kb - Choose the CORRECT combination from
5kb - below 119 Nov, 20171

4kb - Column-I Column-II


(P )Transamina ses (i) removal of phosphoryl
(A) l for X, l for Y (B) 2 for X, 1 for Y add group from a specific
(C) l for X, 2 for Y (D) 2 for X, 2 for Y ammo
(Q) Protein (ii) removal of a-amino
39. Matthew Meselson and Franklin Stahl grew Kinases add group from a specific
E.coli (doubling time is 20 min) in medium amino
containing 15N!-iic1 for many generations. (R) Protein (iii) addition of phosphoryl
Then the E.coli was transferred to medium Phosphatascs group lo a specific
containing 1'NlL;Cl. After 40 minutes, the cells
acid amino
were harvested and DNA was extracted and
subjected to cesium chloride density gradient (S) Dehydrogcnascs (iv) interconversion of
centrifugation. The proportion of light and optical isomers
hybrid DNA densities will be fl9 Nov, 2017] (v) oxidation and reduction
(A) 50% light and 50% hybrid DNA of substrates
(B) 100% light DNA
(A) P-iv, Q-ii, R-iii, S-v
(C) 100% hybrid DNA
(B) P-ii, Q-i, R-ii, S-iv
(D) 25% light and 75% hybrid DNA
(C) P-ii, Q-iii, R-i, S-v
(D) P-v, Q-ii, R-iii, S-i

www.puucho.com
CP Publication KVPY chapterwise with www.puucho.com

TOPICw WJSE KVPY rSTREAM SXl Questions with Solutions

ANSWERS
1. (A) 2. (D) 3. (C) 4. (D) 5. (A) 6. (B) 7. (D)

8.(A) 9. (D) 10. (C) 11. (D) 12. (B) 13. (A&C) 14.A)

15. (D) 16. (C) 17. (C) 18. (C) 19. (A) 20. (C) 21. (A)

22. (C) 23. (A) 24. (A) 25. (C) 26. (D) 27. (A) 28. (D)

29. (A) 30. (D) 31. (D) 32. (C) 33.(C) 34. (A) 35. (D)

36. (D) 37. (D) 38. (D) 39. (A) 40,(C)

5. Rheumatoid arthritis is a chronic, systemic 11. Eco RI-,. 6 bp sequence


inflammatory disorder that may affect many So, probability of occurance of site
tissues and organs, but principally attacks flexible 4" =4 6 =4096
joints. RA is considered a systemic autoimmune R.sa I -,. 4 bp sequence
disease arises from an inappropriate immune
So, probability of occurrence of site
response of the body against substances and
411 =44 = 256
tissues normally present in the body.
Total DNA length = l 0 Kb = 10,000 bp
So, probable cleavage would be
6. Tue comph:ment system helps the ability of
antibodies and phagocytic cells to clear pathogens Eco RI' = 10000 ::: 2.44
from an organism. it si a part of innate immune 4096
system. it is operational via classical pathways. it Rsa I= IOOOO = 39.06
is triggered by antigen bound antibody molecule 256
preferably IgG or IgM. Although IgM is move
effective 12. Plasmodium is a parasite.
Presence of plasrid indicates endo-symbiotic
7. Diabetes incipidus (DI) is a condition presence (Plastid is believed to have originated
characterized by excessive thirst and excretion or from cyanobacteria)
large amounts of severely diluted urine. DI is
caused by a deficiency of vasoprcssin also known 13.
as antidiuretic hormone.

8. Dengue fever also known as break bone fever is


an infections tropical disease caused by dengue
Barn HI
virus. This results· in bleeding, low levels of blood
platelets and blood plams leakage. EcoRl

9. Western blotting technique is used to transfer


proteins and detection there after.

10. The resistant bacterial colonies will grow on


other agar plates containing penicillin.

www.puucho.com
CP Publication KVPY chapterwise with www.puucho.com

BIOLOGY IN HUMAN WELFARE AND BIOTECHNOLOGY

14. A bacterial colony on culture media is formed 23. Penicillin inhibit the cell wall formation in
on repetitive division of bacterium. bacteria thus it kill rapidly growing bacteria
more than the slow growing bacteria.
15. Rhinovirus are the primary cause of common
cold. 24. Radioactivity is mostly incorporated in rapidly
dividing cell during S phase. Thus after 24 hour
16. Ebola virus consist of ss RNA. it will be mainly present in few rapidly dividing
cell
17.
25.
~ restriclio11 enzyme site
: G dciphoresis
- fiagmc11tsof\incar DNA
2

Plasmid There are 4 fragment but 2 fragment of 3 Kb


will appear as one band so three band will
18. appear

~I~ 26. Administration of already prepared antibodies


2000 bp start JOOO bp
is called passive immunisation
/
B2
27. Phenylketonwia is due to non-conversion of
phenylalanine inco tyrosine. Albinism is non·
synthesis of melanin pigment. Homocystinuria
is associated with methionine metabolism.
Argininemia is associated with urea synthesis.

28, In a typical scenario, a virus defected cell will


release interferons causing near by cells to
heighten their anti-viral defense

29. RE is not required in PCR


Question asks the position in A2 -> BI ~ B2
so,5100,3100, 8100
30. Antibody- antigen interaction is essentially
Non-Covalent, Electrostatic interaction,
19. Immunosuppressive Drug inhibits T-cell
Hydrogen Bonds, Vander walls forces and
in:fil tra tion
Hydrophobic interactions are all known to be
involved depending on the interaction sites.
20. Selectable marker is used to eliminate the non
transformant from the transformants
32. 3,lxl09 X 0.34
21. · IgD activates, B- lymphocyte 660
33. Lac operon is inducible operon here.
22. On SDS -PAGE oflgG two principal Bands
arc visible

www.puucho.com
CP Publication KVPY chapterwise with www.puucho.com

TOPIC•WISE KVPY rsTREAM SXl Questions with Solutions.


34. Mast Cells release histamine. 38.

35. RNA primers are involved in DNA replication

36. Transformation occurs when DNA is taken up


by R·strain form dead $-strain.
.Y
37. Bacteriophage infected bacteria found at the 39. DNA rcplicalion i<i scmiconscrvative
bottom containing viral DNA i.e. radioaL-tivc.
40. Fact based

www.puucho.com
CP Publication KVPY chapterwise with www.puucho.com

,,

Ecology "
I
1. Tfyou compare adults of two herbivore species s. Species that are most effective at colonizing
of different sizes, but from the same new habitats show (2013]
geographical area, the amount of faeces (A) low reproductive ability
produced per kg body weight would be- (2010] (B) high dispersal ability
(A) More in the smaller one than the larger one
(C) slow growth and maturation
(B) More in the larger one than the smaller one
(D) high competitive ability
(C) Roughly the same amount in both
(D) Not possible to predict which would be
more 6. The figure below demonstrates the growth
curves of tv,·o organisms A and B growing in
2. In a food chain such as grass ~ deer~ lion, the same are.a. What kind of relation exists
the enerb,ry cost of respiration as a proportion of between A and B ? (20131
total assimilated eneTb,')' at each level would be
[2012] t
"1 _ organism A only
(A) 60%- 30 % - 20% (B) 20%- 30 %- 60% en
Cl ,. ,,
(C) 20% - 60 %- 30% (D) 30%- 30 %- 30% E ,, ,. organism B c,nly
.9
,!:l
,, . _ oq;nnis m B in presence of A
.9
3. Insects constitute the largest animal group on _......
earth. About 25-30% of the insect species are
<l.l
V)
ro
,...
<l,J
-· org,mism A in presence of B
(,)
known to be herbivores. In spite of such huge
..9~-------
herbivore pressure, globally, green plants have Time-).
persisted. one possible reason for this
persistence is~ {2012) (A) Competition (B) Symbiosis
(A) Food preference of insects has tended to (C) Comrnensalisms (D) Mutualism
change with time
(B) Herbivore insects have become inefficient 7. In some species, individuals forego
feeders of green plants reproduction and help bring up another
(C) Herbivore population has been kept in individual's offspring. Such altruistic behaviour
control by predators CANNOT be explained by which of the
(D) Decline in reproduction of herbivores following? [2013)
with time (A) An individual helps relatives only and gets
indirect genetic benefits.
4. Chlorofluorocarbons (CFCs) arc believed to be
(B) The individual benefits because is can later
associated with cancers because, (2013]
inherit the breeding position.
(A) CFCs react with DNA and cause mutations
(C) The individual benefits because it gets access
(B) CFCs react with proteins involved in DNA
repair to resources, such as food and security from
(C) CFCs destroy the ozone layer and permit predators, in return.
harmful UV rays to reach the earth (D) The species benefits from a reduction in
(D) CFCs react with DNA polymerase and competition among off.,;;pring.
reduce fidelity of DNA replication

www.puucho.com
CP Publication KVPY chapterwise with www.puucho.com

TOPIC-.WISE KVPY rsTREAM SXl Questions with Solutions


8. Lions in India are currently restricted to Gir, 12. Study the following graph of metabolic rate of
Gujarat. Efforts are being made to move them various terrestrial mammals as a function of
to other parts of the country. This is because their body mass and choose the correct option
they are MOST susceptible to extinction due to below. . . . . - - - - - - - - - - , (2016]
infectious diseases under the following '
''
conditions when present as- (2013] ''
''
(A) several srna!l, isolated populations ''
''
(B) one large population
(C) several large, connected populations ·- -.. -- . ___ _
(D) several large, isolated populations
Bodyma~s
(A) Animals are distributed throughout the
9. Which one of the following interactions doest curve with the smaller animals towards the
NOT promote coevolution? [2014] left and progressively bigger animals
(A) Commensalism towards the right
(B) Mutualism (B) The smaller animals below a certain critical
(C) Parasitism mass cluster at the left end of the curve and
the larger animals above the critical mass
(D) Interspecific competition
cluster on the right end
(C) Animals are distributed throughout the
10. Stratification is more common in which of the curve with the larger animals towards the
following? (2014] left and progressively smaller animals
(A) Deciduous forest (B) Tropical rain forest towards the right
(C) Temperate forest (D) Tropical savannah (D) The larger animals above a certain critical
mass cluster at the left end of the curve and
the smaller animals below the critical mass
11. Human height is a multigcnic character. If the
cluster on the right end
heights of all the individuals living in a
metropolis are measured and the percentages of 13. CO 2 acts as a greenhouse gas because
the population belonging to a specific heat are (5 Nov, 2017]
plotted as shown below, which of the plots (A) it is transparent to heat but traps sunlight.
would represent the most realistic distribution - (D) it is transparent to sunlight but traps heat.
(C) it is transparent to both sunlight and heat.
[2014]
(D) it traps both sunlight and heat.
p Q
,"
'' 14. A graph of species richness ~ area on log-log
I
'
I
' \
. ' .'
,'
axes 1s (5 Nov, 2017]
'' ' (A) linear (D) sigmoidal
'' ' (C) oscillatory (D) parabolic
,
I
'' ,,
'\ ,,
-~
~
'
,
'' ~- 15. Which ONE of the following options is TRUE
with respect to Emigration? (5 Nov, 2017]
:3
§' R s (A) It is the difference between the births and
P.
~ ,
,,
. ''
'' deaths in a population.
cd
,, ''
(D) It is the difference between individuals
~ , ,
, '' who have come to a habitat and who have
,, '' left the habitat.
,,
, ,, ''
', (C) It involves individuals of different species
,, coming to a habitat from elsewhere during
''
Height the period under consideration
(D) It involves individuals of a population
(A)P (B)Q (C) R (D) S leaving a habitat during the time period
under consideration.

www.puucho.com
CP Publication KVPY chapterwise with www.puucho.com

• ECOLOGY
16. In a population of families having three
children each, the percentage of population of
families having both boys and girls is
[19 Nov, 20171

(A) 10 (B)25
(C) 50 (D) 75

17. In a population interaction between the species


X and the species Y, which ONE of the
followjng statements is CORRECT ?
[19 Nov, 2017]
(A) When X benefits and Y is disadvantaged, it
is Competition
(B) When both X and Y benefit, it is
Mutualism
(C) When both X and Y arc disadvantaged, it is
Predation
(D) When both X and Y are disadvantaged, it is
Parasitism

www.puucho.com
CP Publication KVPY chapterwise with www.puucho.com

TOPIC- WISE KVPY fSTREAM SXl Questions with Solutions

ANSWERS
1. (A) 2. (B) 3. (C) 4. (C) 5. (D) 6. (A) 7. (D)

8.(C) 9.(D) 10. (B) 11.(A) 12. (A) 13. (B) 14. (A)

15. (D) 16. (D) 17. (B)

2. Actually around one half of the energy is lost 12. The metabolic theory of Ecology (MTE) is an
through respiration. extension of Kleiber's law and states that the
Hence best option is 20% - 30 % - 60% metabolic rate of organism is the fundamental
biological rate that governs most observed
4. Chlorine of CFCs destroys Ozone layer causing patterns in ecology
harmful UV rays (mainly UV-B) to reach to
earth surface which causes cancer. 13. Green house effect increases earth's
temperature by trapping heat.
5. Species which has the capacity to establish
itself at new area bas the power to struggle with 14. log S = log c + z log A
challenges such as availability of food & shelter
15. Emigration going out from one population
6, Graph represents competition as the organism
A and B individually shows increase in 16. Probability of 2 Boy'& 1 Girl = ~
biomass. 8

Probability of 1 Girl & 2 Boys= I


7. Altruistic behaviour helps the species in its 8
survival but the species showing it does not Total prob abi lily = ~+ ~ = ~ = 75%
benefit from reduction in competition among 8 8 8
offspring.
17, Both species are benefited mutualism
8. There are chances that the lions may get
infected in a large interconnected population.

9. Interspeci.fic competition doesn't lead to co-


evolution

10. Tropical rain forest shows vertical zonation i.e.


stratification.

11. P ~ plot
~ Height of human is mu!tigenic character
and shows bell shape curve as the oecurancc of
extreme height will he low but medium height
will be maximum.
~ Maximum % of population will have
average medium height

www.puucho.com
CP Publication KVPY chapterwise with www.puucho.com

Model Test Paper


[KVPY [STREAM-SX] 3 Practice Test]

www.puucho.com
CP Publication KVPY chapterwise with www.puucho.com

KVPY (Stream SX) Model Test Papers


Practice Test -1
I
PART- I Q.3 An air track glider of mass M is built,
One - Mark Questions consisting of two smaller connected gliders
with a small explosive charge located between
Questions 1 to 80 are Single correct answer type
them. The glider is traveling along a
questions. Each correct answer carries l mark. 0.25
frictionless rail at 2 mis to the right when the
marks will be deducted for each wrong answer,
charge is detonated, causing the smaller glider
PHYSICS with mass ..!. M, to move off to the right at 5
4
Q.1 An object travels along a path shown with
mis. What is the final velocity of the second
changing velocity as indicated by vectors A and
small glider?
B. Which vector best represents the net
5 mls, just after glider split
acceleration of the object from time tA to t 8 ?

(1) 4 mis to the left (4 mis)


(2) 2 mis to the left (2 mis)

(I) l (2) I Q.4


(3) 1 mis to the left (1 mis)
(4) 1 mis to the right (I mis)

A billiard ball hits the side of a pool table at an


angle 0 as shown in the top view above, and
bounces away at the same angle, and with the
(3) / (4) - - - -
same speed. Which vector indicates the
direction of the net change in momentum of the
Q.2 An object is dropped and accelerates billiard ball?
downwards. As it falls it is affected by air
fiiction, but never reaches terminal velocity . © . View looking
during the course of its fall. The graph that
could indicate the magnitude of the object's
}f- . -·-_ .·.--·a.,:-·,._.": ' \ :!own on pool
:able
~ .'
acceleration as a function of time is : J i :i-~. ~ . ·. ··1 ., ." ~·:·~.:· .. . ..~
,
~

(I)/ (2)-------

(3)\
www.puucho.com
CP Publication KVPY chapterwise with www.puucho.com

KVPY Model Test Papers


Q.5 A large cannon is mounted on a cart with U(x)

frictionless wheels that is initially at rest on a 4.01
horizontal surface, The cannon fires a large
cannonball tO the right with a Speed Vc"1monb:dl, 2.01
which is then caught by a trap fim1ly attached
to the cart \\lhat is the final speed v of the X
cannon-cart cannonball system?
-2.0J
cannon on
cannonball --4.0J
trap

(1) 6.0 J (2) 7.0 J


7 7 7 7 (3) 2.0 J (4)-2.0 J

(I) V > Vcamum ball, t O the Jeft ( V > Vca:monba!l, )


Q.8 A certain star, of mass m and radius r, is
(2) V > Y=u.onball, to the right ( V > Vcarurnnball) rotating with a rotational velocity ru. After the
(3) 0 star collapses, it has the same mass but with a
(4) V < Vcannonhali, to the left (v < Vrnnnonbo.Il) much smaller radius. Which statement below is
tn1e?
Q.6 The instantaneous velocity and net acceleration (I) The star's moment of inertia I has
for an object moving in a circular path arc decreased, aixl its angular momentum L
has increased
shown. At this moment in time, the object is:
(2) The star's moment of inertia I has
decreased, and its angular velocity (D has
decreased
Vinstant.:mC(] us. (3) The star's moment of inertia I remains
constant, and its angular momentum L has
increased
(4) The star's angular momentum L remains
constant, and its rotational kinetic energy
(I) speeding up in a clock\vise circle has increased
(2) slowing down in a cloch.'Wise circle
(3) speeding up in a cow1terclockwise circle Q.9 As shown below, Lily is using the rope through
(4) slowing down in a counterclockwise circle a fixed pulley to move a box with constant
speed v.
Q.7 The potential energy function U(x) 1s The kinetic friction coefficient between the box
associated with a conservative force F and and the ground is µ < I ; assume that the fixed
described by the graph given here. If a particle pulley is massless and there is no friction
being acted upon by this force has a kinetic between the rope and the fixed pulley. Then,
energy of 1.0 J at position Xo, what is the while the box is moving, which of the
particle's kinetic energy at position 14? following statements is correct?

www.puucho.com
CP Publication KVPY chapterwise with www.puucho.com

t PRACTICE TEST-1
Q.12 A vertical straight conductor carries a current
vertically upwards. A point Plies to the east of
it at a small distance and another point Q lies to
the west of it at the same distance. The
magnetic field at P is -
( 1) greater than at Q
(2) same as at Q
(1) The magnitude of the force on the rope is (3) less than at Q
constant (4) g;reater or less than at Q depending upon the
(2) The magnitude of friction between the strength of the current
ground and the box is decreasing Q.13 A wire of length "f, and canyjng a current i is
(3) The magnitude of the normal force of the placed along X-axis in a magnetic field given
ground on the box is increasing
by B-" B (i + J+ k) .
0 The magnitude of the
(4) The pressure of the box on the ground is
force acting on the wire is -
constant
(1) Bo i l (2) J2 Bo if
(3) 2 B0 i £ (4) Bo i £1.Jl
Q.10 A container of wat~r is sitting on a scale.
Originally, the scale reads Mi= 45 kg. A block Q.14 A charged particle projected perpendicularly
of wood is suspended from a second scale; into a magnetic field of (7 i - 3]) x 10-3 tesla
originally the scale read M2 = 12 kg. The acquires an acceleration of (xi+ 7 }) ms-2• The
density of wood is 0.60 f!)cm3 ; the density of
value ofx is -
the water is 1.00 g/cm3• The block of wood is (1) 2m (2) 3m (3)4m (4) 7m
lowered inlo the water until half of the block is
Q.15 Two circular, identical coaxial loops carry
beneath the surface. What is the resulting
same current i in the same direction. If the
reading on the scales? loops are brought close to each other, then -

~
(1) current will increase in each loop
(2) current will decrease in each loop
(3) current will remain same in each loop
(4) current will increase in one and will
decrease in the other
Q.16 The voltage and the current of an
a.c. circuit arc V = 1OD sin(lOO t) volt and
45.o kg I ??? kg i = 100 sin(l0O t + rc/3) mA respectively. The
power dissipated in the circuit is -
(1) M1 = 45 kg and M2 = 2kg (1) 10 4 W (2) 10 W (3) 2.5 W (4) 5.0 W
(2) M=45 kgandM2 = 6 kg
Q.17 Photoelectric effect experiments are performed
(3) M1 = 45 kg andM2 = JO kg
using three different metal plates p. q and r
(4) M1 = 55 kg and M2 = 2kg
having work functions 4'i, = 2.0 eV, ~ = 2.5 eV
Q.11 A helium nucleus makes a full revolution in a and <I>, = 3.0 eV, respectively. A light beam
circle of radius 0.8 min 2 s. The magnetic field containing wavelengths of 550 nm. 450 nm and
350 nm with equal intensities illuminates each
at the centre of the circle will be -
of the plates. The correct 1-V graph for the
(1) 10- 19/µo (2) 10- 19 µo
experiment is: [Take he= 1240 eV nm]
(3) 2 X 10-l?µQ (4) 2 X 10- 19/µo

www.puucho.com
CP Publication KVPY chapterwise with www.puucho.com

KVPY Model Test Papers -


I •
CHEMISTRY
(!)~: Q.21 Friedel-Crafts acylation is-
( 1) u-acylation of a carbonyl compound
V
I (2) acylation of phenols to generate esters
(3) acylation of aliphatic olctins
(4) acylation of aromatic ring
(3)~~
V Q.22 The order of acidity of compounds I-IV, is-

Q.18 Binding energy per nucleon verses mass O-cH20H Q-co2n


number curve for nuclei is shown in the figure.
W, X, Y and Z are four nuclei indicated on the (I) (ll)
curve. The process that would release energy
is- H3C-o-OH
~
:::E 8.5 - - - - - - y X (Ill) (IV)
.!:: 8.0 , , W
g ~5 ---- --,---~-- (l)I <III< II< IV (2) IV< I< lI < Ill
., I

g 5.0 (3) Ill <l < ll < IV (4) II< IV< TIJ < f
--E
'4
i:o 30 60 90 120 Q.23 The most stable isomer for 2,3-butane diol -
Mass number of nuclei
CH,
H--+OH H-+-OH
(1) Y .-+ 2Z (2) W .-+ X + Z
(l)HTOH 2
( )HO-t-H
(3) W .-+ 2Y (4) X 4 Y +Z
CH3 CH3
Q.19 Two radioactive nuclei P and Q, in a given (3) Both are same (4) None of these
sample decay into a stable nucleus R. At time
t = 0, number of P species are 4 No and that of Q.24 Identify Z in the following reaction sequence
Q are N 0• Half-life of P (for conversion to R) is
CH3l ~ X ~ Y ~ Z-
1 minute where as that of Q is 2 minutes. Etber ice Red P

Initially there arc no nuclei of R present in the (l)CH3COOH (2) CH3Mgl


sample. When number of nuclei of P and Q (3) CH3COCl (4) CICH 2COOH
are equal, the number of nuclei of R present in
the sample would be
Q.25 The rate of estcrfication of HCOOH (I),
(1) SNo (2) 2No CH3COOll (II), (CH3)2CHCOOH (Hl) and
2
(CH1)3CCOOH (IV) with ethanol follows in
(3) 3No (4) 9No the order-
2
(!) IV> ill> TI> I
Q.20 Which feedback is used in oscillator - (2) I > II > ill > IV
(1) Positive (2) Negative (3) II> I> lV > IU
(3) Both (4) None ofthes (4)1U>IV>l>Il

www.puucho.com
CP Publication KVPY chapterwise with www.puucho.com

, PRACTICE TEST•1

H•~,tf
0 Q.28 Select incorrect statement about the compound
N0[8F4]:
( l) It has 5cr and 2n: bond
RC0 1H NaOH
Q.26 )A B. (2) 1t is a diamagnetic species
(3) Nitrogen oxygen bond length is higher than
0 nitric oxide (NO)
II (4) B-F bond length in this compound is higher
,.,,-c, thaninBF3

(l) A is H~tJ Q.29 Which of the following equilibria would have


highest value of Kr at a common temperature?

(2) Bis
tJOH
H
( 1) BcC03 ~ BeO + CO2
(2) CaC03 ~ CaO + CO2
(3) SrC03 ~ SrO + CO2
(4) BaC03 ~ BaO + CO2

0 Q.30 Which of the following will show optical


I\ isomers?
(I) cis-[Co(NH3)i(cn)ir3
~ 0 /c, (II) trans-[IrCb(C204)2r 3
(3) A is (Ill) [Rh(en)3t3
(N) cis-[Ir(H20)3Cl3J
(1) I, Ht (2) U, IV (3) I, IH, IV (4) Ill
DH

(4)Bis H"""O Q.31 The magnetic moment of [Ni:X.ir2 ion is found


to be zero. Then the hyhridisation of Ni is :
(X = monodentate anionic ligand)
Q.27 Benzene on reaction with 'A' forms (I) sr3 (2) spd 2 (3) dsp2 (4) d2sp
0
Q.32 In which of the following pair of metals both
which on reaction with are commercially extracted from their
respective ores by carbon reduction method?
(1) Zn, Cu · (2) Fe, Cu
(3) Sn, Zn (4) Al, Ag

'A' and 'B' arc


Q.33 Formation of metallic copper from the sulphide
ore in the commercial metallurgical process
(1) Zn (Hg)+ cone. HCl, involves:
0
3
(I) Cu2S + 02 4 Cu20 + S02
2
~Cl
0 ; 0120 + C 4 2Cu + CO
(2) ~ Cl 'LiAlH4
3 .
(2) Cu2S + - 02 ~> Cu20 ,. S02
2
0 ; 2Cu20 + Cu2S 4 6Cu + S02
(3)~ ,NaBH4 (3) Cu2S + 202 4 CuS04
• Cl
0 ; CuS04 + Cu2S--), 3Cu + 2S02
(4) ~ , Zn (Hg) + cone. I-IC! (4) Cu2S + ½ 02 4 Cu20 + S02
Cl
; Cu20 + CO 4 2Cu + CO2

www.puucho.com
CP Publication KVPY chapterwise with www.puucho.com

KVPY Model Test Papers


Q.34 Heating of ammonium dichromate produces:
MATHEMATICS '
(1) NH3, Cr2O1 and H20
(2) N2, Cr2O1 and H2O
(3) NO, Cr03 and H2O
Q.41 Let ftan -I ( ta; x) dx = a then the value of
(4) N2O, Cr03 and H2O
J tan-1( tanx -3 2cotx dx; ( 0 < x <j f.J
Q.35 A compound is made up of atoms A and B.
equals to:
Atoms A are present at the corners of the cube
7tX x2 . 10{ x2
as well as at the centre of each face whlle (1) u+-+-+c (2) a--+-+c
atoms D are present at 50% of octahedral voids 2 2 2 2
and 25% tetrahedral voids. If the crystal cuts in ;rx x2 JtX x2
(3)-a--+-+c (4) -Ct+---+c
such a way that all face centred atoms from one 2 2 2 2
of the axis are removed the new formula of the
n
compound is :
(!) AB (2) A_iB3 (3) A3B4 (4) A2B3 Q.42 L r
,~1r4+r2+l
is equal to

1 ~

Q.36 For A -), B, AH = 4 kcal mor 1, (I) n +n (2 ) n- +Zn


h..S = 10 cal K-1 mor 1 reaction is spontaneous 2(n 2 +n+l) 2(n 2 +n+l)
2
when temperature can be : (3) 2~2+n (4) n +n
(I) 400 K (2) 300 K 2(11~ + n + I) (n 2 +n+l)
(3) 500 K (4) None of these
Q.43 In the expansion of (x + y + 2)9, which one of
Q.37 What mass of CuCh to be dissolved in I 00 g of the following is false :
water to raise the boiling point by O.16 degree? (1) every term is of the fonn 94 "Ck. x9-r. y-k. !-
[Consider complete ionisation. the molar mass of (2) coefficient of x 4y 7z 3 is I 0
CuC!i = 134.S g mor1 & Kb of water= 0.52 Km-l] (3) the number of terms is 55
(I) 1.345 g (2) 2.69 g (4) coefficient ofx 2 /z4
is 1260
(3) 4.035 g (4) 0.4035 g
Q.44 The function f (x) = 0 has eight distinct real
solutions and f also satisfy f (4 + x) = f {4 - x).
Q.38 When pressure is applied to the equilibrium
The sum of all the eight solutions off (x) = 0 is
system ice ~ water. Which of the following (1) 12 (2) 32
phenomenon will happen ? (3) 16 (4) 15
( I ) More ice will be formed
(2) Water will evaporate Q.45 Let the tangent to the curve .Ji + ../y = J;; (a > 0)
(3) More water will be formed
at any point on it cuts the coordinate axes at P
(4) Equilibrium will not be formed
and Q. Then OP + OQ, where O is origin is
equal to-
Q.39 The solubility product of AgCl is 4.0 x 10-rn at (1) a (2) 2a
298 K. The solubility of AgCl in 0.04 M CaClz
(3) a 2 (4) .Ji
will be:
(l) 5.0 x 10-9 M (2) 2.0 x 10-5 M
(3) 2.2 x 10--4 M (4) l.O x 10-'M Q.46 If [z 4 + 3ij .::;;: 1 and a and fl be the least and
greatest values of lzl and k be the least value of
Q.40 What is the reduction electrode potential (in x 4 +x 2 +4
on the interval (0, co), then k is
volts) of copper electrode when [Cu2+J in a X

solution is 0.01 M at 25°C? (E 0 of Cu2+/Cu equal to-


electrode is+ 0.34 V) (1) (l (2) ~
(I) 0.3991 (2) 0.2809 (3) (1 + ~ (4) None of these
(3) 0.3105 (4) 0.3695

www.puucho.com
CP Publication KVPY chapterwise with www.puucho.com

t PRACTICE TEST-1

a11-J an-I anti


Q.54 sin (¼cos- 1 x) = 1 has -
then an-6 an-3 8 11+3 is: ( 1) one solution (2) two solutions
an-14 an-7 an+7
(3) three solutions {4) none of these
(1) l (2) 2
(3) 0 (4) None of these Q.SS If the product of the intercepts made by the line
x tan n + y sec rx = 1 on the co-ordinate axes is
x2 ,; sin a then o. is equal to -
Q.48 If the ellipse -+- =1 meet the ellipse
(1) ,c/4 (2) ,c/2 (3) 1t (4) 0
4 I
x,. y2
1 + 32 = I in four distinct points and
Q.56 There are twenty bags each containing 10 bulbs
a= b2 -!Ob+ 25 then the value of b does not and it is lmown that no bag contains more than
satisfy- S defective bulbs and 3 bags have 5 defective
(!) (--o:,, 2) (2) (2, 3) bulbs. 4 bags have atleast 4 defective bulbs, 5
(3) (6, a:i) (4) (4, 6) bags have atleast 3 defective bulbs. 6 bags have
atleast 2 defective bulbs and 7 bags have atleast
Q.49 If loga 8 ... y, log~ n = -1 and log114 P=-1 then I defective bulb. Then the ratio of total
1 )Jog rs (~1-1-412) defective bulbs is to non defective bulbs is
(- +1 is equal to-
n (I) i (2) ~ (3) ~ (4) .!.
7 7 7 7
(1) Js (2) 5 (3) 25 (4) 625

If A, B and C are n x n matrices and det (A) =2,


Q.SO
det (B) = 3 and det (C) = 5, then the value of
Q.57 If f(x) = (-li~~~J, g (x) = lsin xi - lcos xj and
the det(A2 BC- 1) is equal to: cl,(x) = f(x) g (x) (where[.] denotes the greatest
integer function) then the respective
(1) ! (2) ~ (3) _!! (4) 24
5 5 5 5 fundamental periods off{x), g(x) and ifi(x) arc
(1) ,c, re, 'It (2) 1t, 21t, n
Q.51 The letters of word ZENITH are written in all re re
possible ways. If all these words are written in (3) re, n, - (4) 'It, - • 1t
2 2
the order of a dictionary, then the rank of the
word ZENITH is- Q.58 For every x e R the value of the expression
( I) 716 (2) 692 (3) 698 (4) 616
x2
y= - + x. cos x + cos 2x is never less than
Q.52 8
'It
(1)-1 (2) 0 (3) 1 (4) 2
(1) 0 (2) re (3) 2: (4)--
2 2

Q.SJ The perpendicular distance between the lines


Q.S9 If Re ( z - 2
z+l
i) = 1, then the Iocus of the point
represented by x2 -4xy + 4y2 + x- 2y- 6 = 0
is- representing z in the complex plane is -
( 1) circle (2) a straight line
(1) - 1 (2) J's (3) 2../s (4) 3./s
.rs (3) parabola (4) none of these

www.puucho.com
CP Publication KVPY chapterwise with www.puucho.com

KVPY Model Test Papers -


Q.60 If the pairs of lines x 2 + 2xy + ay2 =0 and Q.68 Crossing over occurs between :

ax.2 + 2xy + y2 = 0 have exactly one line in ( 1) two nonsister chromatids of same bivalent
(2) two sister chromatids of same chromosome.
common then the joint equation of the other
(3) two nonsister chromatids of different
two lines can be- bivalents
( 1) 3x2 + 8xy - 3y2 = 0 (4) none of the above
(2) 3x2 + lOxy + 3y2 = 0
Q.69 Mongolism is also known as :
(3) -/ + 2xy- 3x2 = 0
( l) Down syndrome
(4) x! + 2xy-3y2 = 0 (2) Turner syndrome
(3) Klinefelter syndrome
BIOLOGY (4) Hypothalamic syndrome

Q.61 Development of egg without fertilization 1s


Q.70 Which type of coiling is found in DNA?
called:
(1) Zig-zag (2) Opposite
(1) oogenesis (2) metagenesis (3) Left-handed (4) Right-handed
(3) gametogenesis (4) parthenogenesis
Q.71 Evolution of diversified species due to
Q.62 If the ovary is inferior, the outermost layer of environmental changes is called:
fruit produced by this ovary will be formed by: ( 1) divergent evolution
(1) Epicarp (2) Mcsocarp (2) evolutionary inertia
{3) Pericarp (4) Thalamus (3) convergent evolution
(4) none of these
Q.63 Ovulation occurs under the influence of:
Q.72 Which of the following sel<; are vestigial
( 1) LH (2) Estrogens
structures in man?
(3) FSH (4) Progesterone (1) Hair, olecrnnon . process, coccyx and
vermiform appendix
Q.64 Progesterone hormone is active during : (2) Wisdom tooth, mammary glands, coccyx
(1) follicular phase and patella
(2) secretory phase (3) Coccyx, nictitating membrane, appendix
(3) menstrual phase and ear muscles
( 4) proliferative phase (4) Hair, ear ossicles, patella and atlas

Q.73 DPT vaccine is given for:


Q.65 Saheli, a new oral contraceptive for the females
(1) tetanus, polio, plague
developed by Indian scientists is a :
(2) diphtheria, pneumonia, tetanus
(I) steroidal preparation
(3) diphtheria, whooping cough, tetanus
(2) honnonal preparation (4) diphtheria, whooping cough and leprosy
(3) non-steroidal preparation
(4} toxic preparation to kill sperms Q.74 Lysis of foreign cell is mediated through:
{l) IgM only (2) IgA only
Q.66 In a cross 45 tall and 14 dwarf plants were (3) IgM and IgG (4) IgD and lgE
obtained. Genotypes of parents are :
(1) Tt x tT (2) TT x tt Q.75 Green revolution means:
(3) Tr x Tt (4) IT x IT (1) increase in greenary everywhere
(2) increase in production of food crops
(3) increase in growth of green vegetation to
Q.67 Phenotypic ratio of dihybrid test cross is :
develop ecological balance
(1) 15 : I (2) 3 : 1 (4) none of the above
(3) 9 : 3 : 3 : 1 (4) 1 : 1 : I : 1

www.puucho.com
CP Publication KVPY chapterwise with www.puucho.com

• PRACTICE TEST-1
Q.76 In which country, the 'DOGS' were once
worshipped as 'GODS'?
(1) Italy ·
(3) Greece
(2) Egypt
(4) Mangolia
@>-4-----@
d

Q, 77 In genetic engineering, recombinant DNA
means: 2d
(1) DNA with a piece of RNA
(2) DNA with a piece of foreign DNA (2)4L1 (3) 6L1 (4) 8L1
(3) DNA which takes part in recombination
(4) DNA not associated with recombination Q.81 Consider the two orbits around the sun shown
below. Orbit P is circular with radius R, orbit Q
Q.78 Which of the following cut DNA at specific
sites? is elliptical such that the farthest point b is
(1) Ligase between 2R and 3R, and the nearest point a is
(2) Exonuclease between R/3 and R/2. Consider the magnitudes
(3) Alkaline phosphatase of the velocity of the circular orbit va, the
(4) Restriction endonuclease
velocity of the comet in the elliptical orbit at

Q.79 A lake with an inflow of domestic sewage rich the farthest point vb, and the velocity of the
in organic waste may result in : comet in the elliptical orbit at the nearest point
( 1) Death of fish due to lack of oxygen Va- Which of the following rankings is correct?
(2) Drying of the lake very soon due to algal
bloom
(3) An increased production of fish due to a lot
of nutrients
(4) Increased population of aquatic food web
organisms

Q.80 Effect of pollution is observed first on: (I) vb> v,> 2va (2) 2Vc> Vb> Va
(l) food crops (2) herbivores (3) lOvb > v11 > Ve (4) Ve> v11 > 4vb
(3) microorganisms (4) green vegetation
Q.83 In Young's double slit experiment, the
PART-II introduction of a thin: transparent mm reduces
Questions gh-'en below are single correct type
the intensity at centre of screen by 75%. Then
Questions. Each correct answer carries +2, 0.5
marks will he deducted for each wrong answers. (~1 = refractive index of film, t = thickness of
film and :,. = wavelength of light used)
PHYSICS
Q.81 A dumbbel1 consists of two masses m (1) µ =2.2 if)..= 2t
connected by a rigid rod of negligible mass and
length d. A physics student takes the dumbbell (2) µ = 2. in. = t
and rotates it about its center of mass with an 3
angular velocity w, giving it an angular
momentum L1. The student then takes a second (3) µ =~ if,.= 3t
dumbbell, with masses 2m and length 2d, and 3
rotates them with the same angular velocity ro. 4
What is the angular momentum ~ of this
(4) µ =3 for any value on.
second dumbbell?

www.puucho.com
CP Publication KVPY chapterwise with www.puucho.com

KVPY Model Test Papers


11 certain mass of gas undergoes a process Q.88 Two coils have self-inductances L 1 = 8 ml l and

L2 = 2 mH. In both of them currents are
given by dU = dW . If the molar heat capacity increased at the same constant rate. At a certain
2 instaut the power given to the two coils is the
of the gas for this process is ~ R, theri the gas same. If at that instant, i1 , V I J U1 and i2 , V2 ,
U2 be the currents, induced voltages and
2
energies stored in the two coils respectively,
is :
then
(l) monoalomic (2) polyatomic
(a)~-=_!_
(3) diatomic (4) data insufficient l1 4
(c) U2 =4 (d) V2 = .!_
Q.85 There is rectangular wire frame having a thin U1 v1 4
film of soap solution. A massless thin wire of (]) a, b, e (2)a,b,d
radius R and area of cross section A is placed (3) a, c, d (4)b,c,d
on the surface of film and inside portion of the
film is pricked. If surface tension of soap Q.89 Select the correct a!tcmative(s) : An electron-
solution is S and Young's modulus of wire is Y
beam is deflected into a circular path in a
then change in radius of the wire is :
uniform magnetic field which is perpendicular
to the beam at every point. Then :
(a) electrons do not acquire energy in the
magnetic field
(b) the momentum of electrons is proportional
to the radius of their circular path
(1) SR 2 (2) 2SR l (c) on increasing the magnetic field sufficiently,
AY AY the path may become parabolic
, (d) on applying an electric field parallel to the
(3) SR- (4) none of these
3AY magnetic field, the beam may again adopt
its original path
Q.86 In a photoemissive cell with exciting wavelength (I) a, c (2) a, b (3) a, d (4) None
X., the fastest electron bas speed v. If the
exciting wavelength is changed to 3,J4, the Q.90 A thin semi-circular conducting ring of radius
speed of the fastest emitted electron will be R is falling its plane vertical in a horizontal
(I) V (3/4)112
magnetic induction B . At the position MNQ
(2) V ( 4/3) 112
the speed of the ring is v and potential
(3) Less than v (4/3) 112
difference developed across the ring is -
(4) Greater than v ( 4/3) 112
X X X X X

B
Q.87 Select the correct alternative (s) : X X X X X
H +, He·.,_ and O·+ .
· havmg masses l amu, 4 amu
and 16 amu respectively, have same kinetic
X X XN X X

energy and pass through a uniform magnetic X X X

field perpendicular to their velocity. Then - X X


(a) H+ will be deflected most M Q
(b) o++ will be deflected most
(1) zero
(c) He+ and o* will be deflected equally
(2) B v 11:R 2/2 and Mis at higher potential
(d) all will be deflected equally
( l) a, c (2) a, b (3) RB v and Q is at higher potential
1t

(3) a, d (4) None of these ( 4) 2 R B v and Q is at higher potential

www.puucho.com
CP Publication KVPY chapterwise with www.puucho.com

t PRACTICETEST-1
N02 NOi

Q.91
withHBr:
CHEMISTRY
Two isomeric compounds I and II are heated (l)Y~& (2)2~&
~NH2 ~Ph
OH CI Br
A
&
N02

VacHJ (3)P~ (4)Noneoftbese


(II)
The products obtained arc - 0 2N ~ P h

(I)
0 0 CHiBr
I ""'
O
-:I

OH
Q.94 What is
Br

the quantity of
(in columobs) required to deposit all the silver
electricity

;t from 250 ml of a I M AgN03 solution?


(1) 2412.5 (2) 24125
(3) 4825.0 (4) 48250
~OCH3

0 Br
Q.95 Which of the following equation is incorrectly
written?
(1) P4 + 20 IINO, ----t 4I-l3P04 + 20NOz +41-lzO
(2) 12 + 10 HN01-), 2HI03 + IO N02 + 4Hz0
(3) S + 6 HN03-) H2S04 + 6N02 + 2H20
(4) None of these

Q.96 AgN03 _A_'> (W) + (X) + 02


(X) + H20 ~ HN03 + HN02
(W} + HN0_1--). (Y) +NO+ H20
~NH, H2SOs p K.!Cr201 Q
{Y) + Na2S203(excess)--> (Z) + NaN03
Q.92 O,NM H2SO.

C6H6
(1) W = Ag
Y = AgN01
X =N20
Z = Na2[Ag(S203)2}
{2} W =AgzO X=NO
NaOH---CaO R Sw'IICI - - - T---U
/1 HN02 NaOH Y = AgN03 Z = Na3[Ag(S203)2l
Uis (3) W=Ag X::,:N02
Y = AgN01 Z = Na3[Ag(S203)2]
(4}W = AgzO X=N2
Y = AgNO; Z = Na[Ag(S203)2]

Q,97 (X) + K2COJ + air ~ (Y)


(Y) + Clz ----), {Z)
pink
N02

Q.93
® Na1 S(aq)

NO;i
IIN02
[X]

C6116
BrCI
AIC\1

[Z]
[Y]
Which of the following is correct?
(1) X = black, Mn(h
Y = Blue , K2Cr04
{2) X = green , Cr203
Z = KMn04

Y = Yellow, K2Cr04 Z = K2Cr20,


(3) X = black, Mn02
NaOH
l HN03'H2S04
Y = green, K2Mn0 4 Z = KMn04
(4) X = black, Biz03
[P] Y = colourless, K.Hi0 2 Z = KBi03
The correct statement is -

www.puucho.com
CP Publication KVPY chapterwise with www.puucho.com

KVPY Model Test Papers


Q.98 In the reaction

CH3COCH3(g) -----+ C2H4(g) + H2(g) + CO(g)
the initial pressure was found to be 0.40 atm
Q.103 Jim
n---,..,,
[(~)u +sin 2-ln (a. e
n+l n
Q) is equal to

and after 10 min it was 0.58 atm. The rate ( I) e0 (2) -a. (3) c 1-« (4) e 1+u
constant for first order reaction is
[log 4 = 0.6021, log 3.5 =0.5441] Q.104 Coordinate of the vertices B and C of a triangle
(1) 0.0133 miri- 1 (2) 0.4 s- 1 ABC arc (2, 0) and (8, 0) respectively. The
(3) 10 s- 1 (4) 0.6 min- 1 vertex A is varying in such a way that

Q.99 Which of the following is non-ionic detergent? 4 tan ~ tan C = 1. The locus of A is-
2 2
(I) C15H 31 COONa
(1) (x-5)2 + i__= l (2) (x-5)2 + y2 =l
25 16 . 16 25
(2) c,H,--©--so;N,
(3) x-
( 5)2 2
+L=1 (4) cx-5)2 +L= 1
(3) [C11H3sN(CH3)3]Br 25 9 9 25

CH20H

( 4) C 1~
I
31 COOCH2-C--CH2OH
. )
Q.105 Let /(x) ~ ,!i~'.:/(x¼ -x• 1
"} x > 0, then

CH20H
fxf (x) dx equals to
Q.100 10.5 ml ofa mixture ofCH 4 and CO was mixed x2 x1 x2
(1) -£nx+c (2)--fn X +-+C
with 25 ml of oxygen and exploded. When the 2 4 2
resultant cooled gas was passed through KOH x2 x2 x2 x2
solution, all the CO2 got absorbed and the (3) - fn X +- + C (4) - £n X - -~ +C
2 4 2 4
volume of remaining gas was found to be 16
mL The volume of Cf-ti in the mLxture is : A A •

(l) 2.5 m1 (2) 3.5 ml Q.106 If vector i + 2 j + 2 k is rotated through an


(3) 7.0 ml (4) 8.0 ml angle of 90° so as to cross the positive direction
of y-axis then the vector in the new position is
2A r;:A 4A
lVIATHEMATICS (1)- ..js i+ ..;5 j- k -.JS
Q.101 1f the 1magmary part of the expression 2 r;: ; 4
-J5 .Js
A A

(2) i - -.; 5 J - k
e'° z-2
- - +-.0- is zero, then the locus of z is
z-2 c' (3) 4i - J- k
*
(where e is a constant such that 8 arg (z -2)) (4) None of these
(I) a straight line parallel to x-axis
(2) a parabola Q.107 1fa, b, care odd integers, then the equation
(3) a circle of radius l and centre (2, 0) ax 2 + bx + c = 0 can not have -
(4) a straight line parallel to y-axis (1) imaginary roots (2) real roots
(3) irrational roots (4) rational roots
Q.102 The range off(x) = cof\-x) - tan- 1 x + sec- 1 x
IS: Q.108 If m and n are positive integers satisfying

(!) l-2:. h]
2' 2
(2) [f, 1r)v (n, 3; ] 1 + cos28 + cos48 + cos60 + cos80 + cos 108
cos me sin n0 .
= . , then (m + n) 1s equal to-

(3) (-t' 3;) (4) (f,1r)v (n, 321t) (I) 9


sm0
(2) IO (3) 11 (4) 12

www.puucho.com
CP Publication KVPY chapterwise with www.puucho.com

f PRACTICE TEST•1
Q.109 A triangle has base 10 cm long and the base Q.115 Down syndrome and Twner syndrome occur in
angles of 50° and 70". If the perimeter of the human beings due to :_
triangle is x + y cos z0 , where z e (0, 90°), then (1) nullisomic and monosomie conditions
the value of x + y + z equals - respectively
(2) monosomic and nullisomic conditions
(1) 60 (2) 55 (3) 50 (4) 40
respectively
(3) trisomic and monosomic conditions
Q.110 When l! + 2! + 3! + 4! + ...... + 2009! is respectively
divided by 35, then remainder will be - (4) monosomic and trisomic conditions
(1)31 (2)32 (3) 33 {4) 36 respectively

Q.116 DNA fingerprinting refers to:


BIOLOGY {1) Molecular analysis of profiles of DNA
samples
Q.111 What would be the chromosome number of (2) Analysis of DNA samples using imprinting
endosperm if it is developed from Oenot/iera devices
type of embryo sac possessing chromosome (3) Techniques used for identification of
number equal to 16 (haploid)? fingerprints of individuals
(1) 32 (Diploid) (4) Techniques me for molecular analysis of
(2) 48 (Triploid) different specimens of DNA
(3) 16 (Haploid)
Q.117 Similarity in organisms with different
(4) None of these genotypes indicates :
(l) microevolution (2) divergent evolution
Q,112 If the vasa differentia of a man arc surgically (3) macroevolution (4) convergent evolution
cut or blocked :
Q.118 Agarose extracted from sea weeds finds use in :
(1) semen will be without sperms
(1) Tissue culture
(2) spermatogenesis will not take place
(2)PCR
(3) testosterone will disappear from blood (3) Gel electrophoresis
(4) spenns in the semen become nonmotile (4) Spectrophotometry

Q.113 A heterozygous round seeded (Rr) plant is Q.119 Which of the following is not used for
disinfection of drinking water?
crossed to recessive wrinkled (rr) seeded plant.
(1) Ozone (2) Phenyl
The progeny would be : (3) Chlorine (4) Chloramine
(1) 20 rounded : 99 wrinkled
(2) 99 rounded : 30 l wrinkled Q.120 Drinking of mineral water with very low levels
(3) 301 rounded : 100 wrinkled of pesticides (about 0.02 ppm) for long periods
may:
(4) 303 rounded : 301 wrinkled
(1) cause cancer of the intestine
(2) produce immunity against mosquito
Q.114 The length of DNA molecule greatly exceeds (3) cause leukaemia (blood cancer) in most
the dimensions of the nucleus in eukaryotic people
cells. How is this DNA accommodated ? (4) lead to accumulation of pesticide residues in
(1) DNase digestion body fat
(2) Super-coiling in nuclcosomes
(3) Deletion ofnonessential genes
(4) Through elimination of repetitive DNA

www.puucho.com
CP Publication KVPY chapterwise with www.puucho.com

KVPY Model Test Papers

ANSWERS
'

Qus. 1 2 3 4 5 6 7 8 9 10 11 12 13 14 15 16 17 18 19 20
Ans. 4 4 4 4 3 4 2 4 2 4 2 1 2 2 2 3 1 3 4 1
Qus. 21 22 23 24 25 26 27 28 29 30 31 32 33 34 35 36 37 38 39 40
Ans. 4 1 2 2 2 4 4 3 1 1 3 3 2 2 3 1 1 3 1 2
Qus. 41 42 43 44 45 46 47 48 49 50 51 52 53 54 55 56 57 58 59 60
Ans. 2 1 2 2 1 2 3 4 4 2 4 3 2 4 1 4 3 1 2 2
Qus. 61 62 63 54· 65 66 67 68 69 70 71 72 73 74 75 76 77 78 79 80
Ans. 4 4 1 2 3 1 4 1 1 4 1 3 3 3 2 2 2 4 1 3
Qt.is. 81 82 83 84 85 86 87 88 89 90 91 92 93 94 95 96 97 98 99 100
Ans. 4 3 2 3 2 3 1 3 2 4 1 2 2 2 4 3 3 1 4 1
Qus. 101 102 103 104 105 106 107 108 109 110 111 112 113 114 115 116 117 118 119 120

Ans. 3 2 3 1 4 4 4 3 4 3 1 1 4 2 3 1 4 3 2 4

- - · -::-:.....--. -------.:_-=--...:::. • So LUTI ONS e

PART-I (Tl1e acceleration is usually modeled as a


function of v or v2. depending on a number of
One Mark Questions factors.) The acceleration continues to decrease
PHYSICS until the acceleration of the object is 0, at which
point the velocity of the falling object remains

A
constant. The only graph consistent with this
analysis is d, where the acceleration curve can
1.
be seen to be approaching zero asymptotically.

11V = B-A 3. This is a conservation of momentum problem, in


which the total momentum of the glider at the
The direction of acceleration is the same as the beginning of the problem is equal to the sum of
direction of the changes in velocity, according to the momenta of the individual gliders at the end
of the problem
a= vf -vi . Because l1v = Yr - vi, we can Mv = m1v1' + m2v2 1
t
determine 11v graphically by adding Yr to the M V = -lM'
VJ + -3M'
Vz
negative of Yi, or B + (-A). Placing the B vector 4 4
"tip-to-tail" with the -A vector gives a 1 ! 3 I
v= -V1 + -V2
direction for tw (and therefore, a) to the left 4 4
::::::, V, I

= -43 ( V - -1 V1 ')
4
2. The falling object, when released from rest, has
an initial acceleration of 9.8 m/s 2 (if near the
surface of the earth). As its velocity increases, it
v2 1 = .'.: (2 - .!. 5') = 1 mis.
3 4
collides with air molecules at an increasing rate,
thus reducing the rate at which it accelerates.

www.puucho.com
CP Publication KVPY chapterwise with www.puucho.com

t PRACTICE TEST•1

4. The billiard ball has velocity vectors in both the 4.0 J + 1.0 J = -2.0 J + Kc
x and y directions, as shown here. TI1c Kr= 7.0 J
x-componcnt of the velocity remams
unchanged, but the y-componcnt has changed 8. According to conservation of angular
from the positive to the negative direction, a momentum. the angular momentum L of the star
change in velocity-and thus, momentum- in remains constant,
the negative y direction. so when its moment of inertia I increases ( due to
Vx-i11ilial v,._r,,,.1 the decreased radius), its angular velocity
-- --1j w goes up proportionally, according to :
I
I
I
L;.. ;,i.i = Lr..:,1
/
Vy-initial :
\:I y y-final l;w;=lrwi-
I I 1-
wr= ....!..w.
<9 e Ir i

The star's rotational kinetic energy, based on


5. The overall momentum of the system at the 1
beginning of the problem is 0, and with no Kro1ational = - Iw2 also goes up. Although I has
2
external forces applied, this will continue to be
decreased, K,o1atiimut increases with the square of
the total momentum of the system. During the
ball's motion to the right, the cart will have a w. leading to a net increase in energy.
motion to the left. However, once the
cannonball is caught by the trap, they have the /lol-- -
- - µoq - /lo2e
11. B- -- --
same velocity, which has to be zero if linear ZR 2TR 2TR
momentum is to be conserved
IIlc,,nVa,rt + ~ V""°"""ball = tllciid Vant' + tJbtVcart
(Olcan -1- lllcann,rnbalVYillitial = (111cmc + lI1c.nno11l,alJ)Vr,m1
(tlbt + ~ (0 m/s) = (111cut + m,___""1P0dhll~V!ina! 12.
Bwl-_--__ -- -@-- ______ _jBw iBr:
Vrrnal = 0 mis
BE
6. The instantaneous velocity of the object is
tangent to its circular path, and we know that 13. F=dlx B ~ = li{i) x Bo (i + j + k)
there's a radial (centripetal) aspect of the net
acceleration that points towards the center of the F= B0 li(k-}) ~ F= .J2B 0Il
circular path. Thus, we can conclude that the
object is traveling in a circular path that is 14. Fm=qvxB
located to its left, as shown here.
We can also see that the net acceleration must ma=qvxB => a.Lv so ii.v =O
include a tangential component of acceleration
that is in the opposite direction of the E I re
16. P= ...J!....9..cos-
instantaneous velocity, implying that the object is 2 3
slowing down as it trnvels along this circular
path. CHEMISTRY

Vmsr.:wt111eous
21.

22. Stability ofanion

7. This is a conservation of energy problem, and


the total mechanical energy of the system
remains constant :
Ui+K1=Ur+Kr

www.puucho.com
CP Publication KVPY chapterwise with www.puucho.com

KVPY Model Test Papers


23. When attraction force is present then optically 38. •
active isomer is more stable.
0
II
39. K~p= [Ac+J rcn
Cacti------) Ca+2 + 2Cr
,,,...c"' 0.04 0.08

Hi))-
0
4 x 10-lO"" KsP =S x (S + 0.08)

26. RCOiH )

-t3 A(product)
40.
Ksp"" 5 X 10-9 M

E
~eu•z ,eu =Eu,., ,eu
_ 0.059I 10 g_l~
n [Cu +2]

NaOH )
"tJOH ""'+o_34 _ 0.059 t 10 g_l_
2
"" 0.34 - 0·0591 x 2
0.01
= 0.2809 V
2
B
28. NO+ [BF4r MATHEMATICS
NU+- has bond order= 3
NO has bond order= 2.5
4 1. J~an-•Can x~2 cot x) dx
29. BcCO3 has least thermal stability
= Jtan- (tanx)dx- Jtan- (2cotx)dx
1 1

30. Cis-M(aa)2b2 and M(aa)3 show


optical isomerism = Jxdx- J;-coC (2cotx)dx 1

= f xdx- J;-tan_1(ta;x}tx
31. [Ni.X.ir2
Nt2 = [ArJ 4s 0 3d8
---~3_d_~- 4s 4p
1t-i1t-1.1ri1t1t1 D I I I I x 2 nx
=---+a.+c
magnetic moment is zero then ligand must be
2 2
strong field
~----Jd--~ 4s 4p
1r-1.1t-1.1 .i11 1i~ ID II 11
dsp2
34. (N"4)2Cr201 ~ N2+ Cr203 + H20

35. A=__!__ x 8+ .!. X4= 1 +2=3


8 2
1 I
B""- x4+- x&:=2+2=4
2 4
43, (x + y+ z)9 = [x + (y+ z:)]9
Tr,-1 = 9C,.x9-r (y + z)'
36. AH-TAS==O Now the general term of (y + zY = rcrcyr~kzk
4 X 103 -T X 10 = 0 Hence the general tcnn of given expansion
T=400K = 9Cr rc"x9-r 1-k,c
37. AT1,=iKbm the coefficient ofx4y7z3 is O and coefficient of
x2/z4 is 9~ 7C3 = 36 x 35 = 1260
0.16 = 3 x 0.52 x We x lOOO and number of tenns in the expansion= 9-l-2C2
134.5 100
We= 1.38 g = llxIO = 55
2

www.puucho.com
CP Publication KVPY chapterwise with www.puucho.com

44. Symmetric· about x = 4


2{[x1;x2]+[x3;·x4]+[xs;x6]+[x~;x8]}
:::::, fl-0 + 81X + a2X2 + .....+ a1i,X 2n
= a.ox2n + aix2n--l + .....+ a2n
On equating the coefficients of similar powers
ofx,

= 2{4 +4 +4 +4} = 16 X 2 = 32
we get ao= a2n, a1 = a2n-1, .••••, a,= a2n-;

45. •.. rx + rr =..1a => The value of given determinant is equal to 0


as 2nd and 3rd column are identical.

48, Clearly a> 1


b2 - 1Ob + 25 > l
b 2 - 10b + 24 > 0
b E (--co, 4) V (6, co)
X
p
Slope of tangent at (xi, Yi) is 49. log11413 = -1 :::::, P= 4
l 1
1 1 1 1 (dy) O
2..j;:;+2£ dx -
dy -Fi
:::::,dx- Fi
log~a=-1 ::::,a=-= -
p 4
3
y =lo8,:t8 = ; log1148 = - 2
. of tangent 1s
Equation . y- Y1 = -./y;
r. (x - X1 )
vx.1 1
Now- + 1 = 4 + l =5
·: point P on x-axis is (x, +Ni, 0) u

Point Q on y-axis is (0, Y1 + Ni) J3 2 +4'(= 16+4 ( -2 =25 3) 2

:.OP+OQ.=x1+y1+2~ =(,/i; +.Jri")2=a l )log_,; (jll +4y1)


:. ( - + 1 = 5108 Ji 25 = 625
a
46. Given.
1 ;?!jz-(4 _ 3i)I~ {lzl-14-3ij 50.
l4-3i 1-lzl
:::::, lz! S 6 and lzl ?. 4
:::::, 4 .<:: lzl ~ 6 :::::, a = 4, J3 = 6 51, The total number of words is 6! = 720. Let us
- x 4 +x 2 + 4 4 write the letters of word ZENITH
3
Lety= - - - - =x +x+- alphabetically, i.e. EHINTZ.
' x X
1
I 1 1
·
=x 3
+x+-+ -+ -+-
x X X X
52. Arg { (i4~13 i + {in + i.in ..:. in -Lin + 4)i}
Sincex E (0, oo), therefore
3 I .. Arg {-i + 4i} = Arg (3i) = ~
x ·, x , - are positive. 2
X .
. ~

53. Lmes are (x - 2yy + (x '--2y) - 6 = 0


Sum will be least when x3 ::: x = ..!_ Lctx-2y= t
X
:::::,x= 1 t2 + t-6 = 0-
.-. k=6 "(t + 3)(t-2) = 0
Hence, k = 13 :, t+3=0ort-2=0
x-2y+ 3 = 0 orx-2y-2 = 0
47. Given. distance = /5
(t + X + x2)" = 8o + a1x+ a2x2 + .....+ ainx.2n
.... (1)
54. , ( 1 cos -I x ) = l :::::, 5l cos-I x = 2n
Replacing x by..!.., we get
sm 5
X
(l + x + x2)° = aox2n+ a1x2n- 1+a.iX2n-2+ ....+a2<1 cos-1 x = -5,i; b
2 , ut cos. x. .e 0, 1t , no solution.
-1 [ ] .

www.puucho.com
CP Publication KVPY chapterwise with www.puucho.com

KVPY Model Test Papers

55, x= cot a and y = cos a. 64. In menstrual cycle after ovulation corpus luteum •
~ cot a. . cos a. = sin a. is develop in ovary which secrets the
tan a. = ± 1 ~ a = n/4 or 3n/4 progesteron hormon so this phase is called as
secretory phase.
56. Total number of bulbs= 200
Total number of defective bulbs 65. Saheli chemically (centchroman or onniloxifene) is
= 1(7-6) + 2(6-5) + 3(5-4) +4(4-3) + 5(3) a non steriodal female contraceptive.
=25.
66. 3 : I ratio is obtained on crossing heterozygous
Ans
wer1s -
. 25 = -.
1
fonns.
175 7
67. Test cross always shows only one type for each
57. Clearly ftx + n) = ftx), g (x + 11:) = g (x) and phenotype.

9 ( x+ ;J = {(-1) f{x)}{(-1) g (x)} = lji(x). 68. Crossing overy occurs between non sister
chromatid of homologous chromosomes.

x2 69. Down syndrome is caused due to trisomy 21s1


58. y=-+ x cosx + cos2x somatic chromosome.
8
x2
=-+ x cos x + 2 cos2 x-1 70. B-DNA (common form) is right handed.
8 [B-DNA]

=2(cosx+¾J-1. 71. Members of a same species diversify them self


& get settled in new environment, which causes
divergent evolution.
59. Re (z-2i+1-1)= 1
z+l 72. Coccyx, Niclitating membrane (3n1 eyelid) and
muscles of ear piona and vermiform appendix
:::>Re (-l- 2i +I)= 1 are vestigeal in human being.
z+l

::::::, Re (-1-2i) =O 73. DPT vaccine prevents against dipthcria,


z+l pertussis & tetanus.
~ x+ I +2y"" 0
75. Green revolution in fndia was brought by M.S.
Swami Nathan by increasing wheat production.
60. am2 + 2m + 1 = O
m2 +2m+a=O 76. Dogs are the oldest pets with man due to their
common mot m = 1 faithfulness, they were worshipped in Egypt.
other roots= I/a, a
m2 - (a+ 1/a) m + 1 = 0 77. Recombinant DNA means alignment of foreign
=:, x2 -(a + l/a)xy+ -I=
0 (also a< 1) DNA with host DNA.

BIOLOGY 78. Restriction endonuclease are highly specific and


identify palindromic sites.
61. Parthenogenesis is development of
79. Inflow of domestic sewage causes algal bloom
62. Thalamus form,; part of fruit where ovary is which decreases oxygen level in lake causing
inferior. death of fishes.

63. LH hormone stimulates the ovulation. 80. E.coli are good indicator of water pollution.

www.puucho.com
CP Publication KVPY chapterwise with www.puucho.com

PART-II Consider an element of wire forming angle 0 at


the centre
Two Marks Questions d0 . ··
2T sin - = 2 x [S x R x d8]
PHYSICS ' 2 .
81. Although the problem asks us to compare T=2SR
angular momenta of the two dumbbells, Y= _!_!_ ~ YAxAR =2SR.
L = Iro, and w is identical for both dumbbells. At.i.l R
Therefore, this is really a problem asking us to 2SR 2
compare the moments of inertia I AR=--
YA
1, = L> m 2

LdI
2
d 2m
1 d) m+ ( l d) m=-
2 88. e= - :::;,,eccL
11 = ( 2 2 2- dt

Now let's calculate 12,


Ii= Ir m
2

Ii= (d}22m + (dt2m 90 e = Bv(2R) and Q is at high potential


h= 2d22m=4d m

I 8(d;)
2 = = 81 1 CHEMISTRY

Because the moment of inertia Ii is 8 times I1,


the angular momentum L 2 is 8L1 as well.

83. As intensity becomes one-fourth


1 211' 41t Stable cation
:. cos (j)aa: - - ~ ~= - , - ............ .

and ~ =
21t
2
21t
3 3
,::-Ax= T
(!L- l)t +H&-6(glOH
84. dU= dW 92.
2 Me Mc @OH
by l St Jaw-} dQ = 3dU = 3nC,.dT
b'1..,t,SO, &NO, "::'.' 0 NO,
molar heat capacity C = dQ = 3Cv
ndT O,N~ N~ NO,

. C 15R
gtven =- -
2
5
Cv= -R ~~O,
2
degree of freedom = 5
so gas is diatomic.
N~

l
Sn+HCI

- ""'-_ &"·'' ~NM


Srd0

85.
T~T
rh
~
&""·
ClN~
' \ I II fn
\ tit!
.
'.;:,:: ./
,.,
,,
I

www.puucho.com
CP Publication KVPY chapterwise with www.puucho.com

KVPY Model Test Papers


94. moles of As+= M x V •
= l X 0.25 = 0.25 :::>r-!.=o
r
1
moles"" - Q =-r2=1
nF :::> (x-2)2 +y2= 1
1
Q = 0.25 x 1 x F = - x 96500
4 102. · f{x) = rr -(tan- 1x + cof 1x) + sec- 1~
= 24125 C
=2:+ sec-'x
2
96. AgNO] ~ Ag + N02 + 02
N02 -------4- HN03 + ~02 Range of f(x) ,={f, reJu ( x, 32rc]
Ag+ HN01 ~ AgNO; +NO+ 1120
AgN03 + Na2S203(c~cess) ~
lim n((....!!....Jc +sin.!.-1)
Na;[Ag(S201)2] +NaN01 . .
Ltn11t= o...., n+l n
103. C

97. Mn02 + K2C03 + air ~ K2Mn04 lim(sin(l/n) (l+(l/n)r-1. - ! J


= Co-.:{ l/n 1/n (!+(lln))Q
K2MnO4 + Cli ~ KMnO4
= e1-a
98. CH3COCH1 ~ C21-4 + H2 + CO
0.4-X X X X B
4tan-tan - = I
C
104.
2 2
0.4-x X X X
s-a
0.4+2x =0.5
X = 0.05 s 4
Solveitb+c= 10
2.3031
K -- - 0.4 -00133
- og--- . . -1
mm Focus F1 (2, 0) and F2(8, 0) and 2a = 10
10 0.35
(x-5)2 Y2
-'----'-+-=I
100. Cl-4 + 202 --> CO2 + 2H20 25 16
X
X 1/n _ X 1 /(n+I)
CO+ !020 CO2 . 105. ftx) = lim - - - - -
2 n-+"'
(10.5 -x) n2
l
2x + -(I0.5 x)=9
3
2
-x+5.25=9
=b xTl -xl-""'j"'] .[- n(:: 1)]
2 + 1)
n(n
x=2.5 mol.
= (-log.(x)) (-1) = + Iog,x
MATHEMATICS
J x.f(x)dx = Jx.(log 0 x)dx

101. z-2=re1a
x
= [ log x.--
e
2
2
J J x2
-x-dx
X 2
:. (x-2) + iy= r(cos a.+ i sin a.)
x2 x2
r2 = (x - 2)2 + ,J and tan a = .....:L_ = log.x. - - -+ C
. x-2 2 4
iO
, Z- 2
Th e expressmn e ;'a-0'
- -- + - - = re" , +-1 e-i{n·"'
·v, 106. Since i + 2 } + 2 k is rotated so as to cross
. e10 • z-2 r y-axis, the vector in new position would be
which is given as real parallel to the plane 2x- z = 0.
:. r sin (o.-8)-.!_ sin (a.- 8) = 0
r

www.puucho.com
CP Publication KVPY chapterwise with www.puucho.com

PRACTICE TEST•1
•107. Since a, b, c e Q, so both the roots should be BIOLOGY
rational or both irrational. If E.
is a root of it, 111. Oenothera possess diploid endosperm
q
p, q e I, as a, b, c e odd numbers 112. Vasa differentia transports sperms its absence
=;, ap 2 + bpq + cq2 = 0 is never possible. will cause lack of sperms in semen.

108. 1 + cos20 + cos40 + cos60 + coslOO 113. Round and wrinkled seed would be obtained in
= Ree +
[ iO + c i20 c i40 + .... + eii OB,J 1 : 1 ratio.
[cil29 -1] .
= Re . , c' 0 = (cos9 + isin9) 114. DNA undergoes various level of packaging.
[e'2e -1]
= cos20cos 120-cos 128-cos2O+ I+ si11 28sinl28 115. Down syndrome is due to trisomy of 21 st
2(1-cos 20) chromosome & hrrners syndrome due to
1 + cosl08- 2cos78cos 58 monosomy of sex chromosome (XO) condition.
4sin 2 0
117. Similarity in organism in spite of different
sin60cos50
genotype shows convergent evolution
sine
on comparing we get n = 6, m = 5 118. These arc basically obtained from brown and
(m + n) = 11. red algae and termed as phyco colloids

109. Perimeter=b +a+ 10 = x + y cosz


Using sine rule
10x2 a h
J3 = sin50° = sin 70°
20
a = ../3
.
SID
s"oo , b = ..Jj
20 . 7
Stn 0°

a+ b = ~ (sin 50" + sin 70")

= ~ x 2 cos 10° sin 60"


=2 cos 10°
Perim:ter 10 + a + b = 10 + 20 cos ID° = x + y cos z
On comparing we get
X + )' + Z = 10 + 20 + JO = 40

110. 7! onwards are divisible by 35. Hence remainder


of(l ! + 2l + 3! + 4! + 5! + 6!), when divided by
35 is 873 = 33
35

www.puucho.com
CP Publication KVPY chapterwise with www.puucho.com

KVPY (Stream SX) Model Test Papers


Practice Test -2
I
PART .. I Q.4 An electron moves along the line AB which
One - Mark Questions lies in !be same plane as a circular loop of
Questions I to 80 arc Single correct answer type conducting wire as shown in figure. What will
questions. Each correct answer carries 1 mark. 0,25
be the direction of th.e current induced if any in
marks will be deducted for each \\Tong answer.
the loop? ·

Q.1
PHYSICS
Two bar magnets of the same mass, same
length and breadth but having magnetic
moments M and 2M are joined together pole
-e
A V
0 B
for pole and suspended by a string. The time
period of assembly in a magnetic field of (l) No current will be induced
strength H is 3 seconds. If now the polarity of
(2) The current will be clockwise
one of the magnets is reversed and the
combination is again made to oscillate in the (3) The current will be anticlockwise
same field, the lime period of oseilla tion is : (4) 1be current will change direction as the
( 1) ./3 sec (2) 3.J) sec electron passes by
(3) 3 sec (4) 6 sec
Q.5 A solenoid has 2000 turns wound over a length
Q.2 Two magnets A and B arc identical and these are of 0.3m. The area of its cross-section is
arranged as shown. Their lengths are negligible in
1.2 x 10-3 m2 • Around its central portion a coll
comparison to separation between them. A
magnetic needle is placed between the magnets of 300 turns is wound. If an initial current of
at point P and it gets deflected through an angle 2 amp in the solenoid is reversed in 0.25 sec.,
9. The ratio of distances d 1 and d2 will be the emf induced in the coil is equal to
(l)6xl0 4 V (2)48mV

9--- - p·· ---[}


Q.6
(3)6x 10-2 y (4)48kV

In the circuit shown in the following figure


~d1--"""'""--<l2-I E = lOV, Ri =2 ohm, R2 = 3 ohm and R1 = 6 ohm
(I) (2 cot 9) 113 (2) (2 tan 8)11.l and L = 5 henry. The current Ii just after
(3) (2 cot 9) (4) c2 tan er!/J pressing the switch Sis
11
Q.3 The tangent galvanometers having coils of the
same radius are connected in series. A current
flowing in them produces deflections of 60"
and 45° respectively. The ratio of the number of
turns in the coil is :
(l) (4/3) (2) (../3 + 1)/1
(1) (10/4) amp (2) (to/5) amp
(3) [..[j.Ji-I
+lJ (4) .fj /I (3) (l 0/12) amp (4) (10/6}amp

www.puucho.com
CP Publication KVPY chapterwise with www.puucho.com

• PRACTICE TEST-2
Q.7 In the Q.6, the current 11 long after pressing the (1) Kinetic energy K is conserved
switch is (2) Total Energy E is conserved, but angular
(1) (10/4) amp (2) (10/5) amp momentwn L is not conserved
(3) (10/12) amp (4) (10/6) amp (3) Angular momentum L is conserved, but
linear momentum P is not conserved
(4) Angular momentum L is conserved, and
Q.8 When a ferromagnetic material is taken through
total energy Eis conserved
a hysteresis cycle, the magnetic susceptibility
cannot be: Q.12 Two ice skaters, of mass 30 kg and 80 kg, are
(I) Zero (2) Infinity skating across the surface of a frozen lake on a
(3) N~gative (4) Constant collision course, w:ith respective velocities of
2.0 m's in a general north direction, and 1.0 mis
Q.9 In the circuit of figure the bulb will become generally west, as shown above. After they
suddenly bright if: collide, the pair of skaters move off in a
direction north of west with a momentum of
B
approximately I 00 kg mis. How much kinetic
energy was lost in the collision?

B
+ ]>----~/'_---~
(I) Contact is made or broken
(2) Contact is made
2.011Vs 1
(3) Contact is broken
(4) Would not become bright at all 30kg'
(1) OJ (2) 110 J (3) 55 J (4) 70 J
Q.10 Which types of wire is suitable for flowing
A.C.: Q.13 An ideal monoatomic gas is heated so that it
( l) Single stramed wire expands at constant pressure. What percentage
(2) Multi strained wire of' the heat supplied to the gas is used to
(3) both increase the internal energy of the gas?
(4) None of these (1) 100% (2) 0%
(3) 60% (4) 50%
Q.11 An asteroid traveling through space collides
with one end of a long, cylindrical satellite as Q.14 When a tuning fork of frequency f is excited
and held near one end of a straight pipe of
shown and sticks to the satellite. Which of the
length L open at both ends, the air column in
following is true of the isolated asteroid-
the pipe vibrates in its fundamental mode and is
satellite system in this collision? in resonance with the tuning fork The pipe is
now kept vertical in a jar containing water so
that half the lenbith of the pipe is inside water.
What should be the frequency of the tuning
fork to be used to make the air column vibrate
in its fundamental mode in resonance with the
tuning fork now ?
(1) f (2) 2f
(3) fi'2 (4) f74

www.puucho.com
CP Publication KVPY chapterwise with www.puucho.com

KVPY Model Test Papers


Q.1 S A wheel of radius R is rolling along a Q.19 In figure shown, find the magnitude of •
horizontal surface with a speed u. A pebble acceleration of m, given that string is
trapped on the wheel gets separated from the inextensible and mass less and the acceleration
highest point of the wheel arrives at position P of M is 2 rrJs 2 towards left -
(figure). The horizontal range PQ of the pebble
is:
......... .
...
'' M
''
''
''
p '
Q
(1) 2..JJ m/s 2 (2) 3 ./2 m/s2
(I) uJR/g (2) 2uJR/g
(3) 4-/2 m/s2 (4) 2./5 m/s2
(3) 4u~R/g) (4) 1/·,./Rlg
Q.20 During the process A-B for an ideal gas -
Q.16 A particle starts from the origin at time t =0 p
with velocity 2 j and moves in the x-y plane
with a constant acceleration of 2i + 4 j where i
and ) are unit vectors along the x-direction and
A/B
,,
y direction respecfr-:1cly. What will be the ~-----T
x-coordinate of the particle when its (l) Work done on the gas is zero
y-coordinate becomes 12 m - (2) Density of the gas is constant
(!) 4 m (2) 6 m (3) 6.8 m (4) 8 m (3) Slope of line AR from the T-axis is directly
proportional to the number of moles of the
Q.17 Choose the incorrect statement - gas
(1) A dimensionally correct equation may be (4) All of the above
correct
(2) A dimensionally correct equation may be
CHEMISTRY
incorrect
(3) A dimensionally incorrect equation may be
correct Q.21 Among the oxides of nitrogen N 20, NO and
(4) A dimensionally incorrect equation may be N0 2, molecules with unpaired electrons are :
incorrect (1) N20 and NO
(2) NO and N02
Q,18 Two solid bodies rotate about stationary (3) N20 and N02
mutually perpendicular intersecting axes with (4) N02 and its dimer
constant angular velocities co 1 and CO?- What is
Q.22 Which of the following compounds have the
the magnitude of angular velocity of one with
same no. of lone pairs with their central atom ?
respect to the other ?
(I) XeFs-. (II) BrF3 (III) XeF2 (IV) H3S ...
(1) ro, - roi (2) ~(J); + COi (I) IV and I (2) I and Ill
(3) I and ll (4) 11 and IV
(3) ~m; - m~ (4) 001+ 002

www.puucho.com
CP Publication KVPY chapterwise with www.puucho.com

-
~,7

t PRACTICE TEST•2
Q.23 Coordination compounds have great Q.27 Pb4 does not exist because:
importance in biological systems. In this (1) iodine is not a reaetive
contcict which of the following statements is (2) Pb (IV) is oxidizing and r is strong
incorrect? reducing agent
(1) Carboxypeptidase-A is an enzyme and (3) Pb (IV) is less stable than Pb (II)
(4) Pb4+ is not easily formed
contains zinc
(2) Haemoglobin is the red pigment of blood
Q.28 1 gm sample of H2O2 solution containing x %
and contains iron
H202 by weight requires x ml of KMn04
(3) Cyanocobalamin is B 12 and contains cobalt solution for complete oxidation under acidic
(4) Chlorophylls are green pigments in plants conditions. Normality ofKMn04 solution is-
and contains calcium (1) 1.21 N {2) 0.122 N
(3) 0.588 N (4) 0.7S8 N
Q.24 An aqueous solution of titanium chloride, when
subjected to magnetic measurement, measured Q.29 Calculate the energy required to excite one litre
zero magnetic moment. Assuming the of hydrogen gas at 1 atm and 298 K to the first
octahedral complex in aqucou.~ solution, the excited state of atomic hydrogen. The energy
for the dissociation ofH-H bond is 436 kJ/mol.
formulae of the complex is:
(1) 112.5 kJ (2) 98.17 kJ
(1) [Ti (II2O)6] Ch
(3) 74.32 kJ (4) 55.21 kJ
(2) [Ti (H20)6] C4
(3) [TiC!J (H20}3] Q.30 The excluded volume of a gas will be larger,
(4) [TiCh (H20)4]
·r
1 -Tc .lS-
Pc
Q.25 Give the correct order of initials T or f for (2) large
( I) small
following statements. Use T if statement is true (3) equal to I (4) less than unity
and F if it is false.
(i} Every mineral is an ore but every ore is not a Q.31 One mole of an ideal monoatomic gas at 27°C
mineral is subjected to a reversible isoentropic
(ii) Slag is product formed during extraction of compression and until final temperature
metal by combinations of flux and reached to 327°C. If the initial pressure was
impurities LO atm then find the value ofln P2 :
(iii) Highly pure metals can be obtained by (Given: In 2 =0.7)
(1) 1.75 atm (2) 0.176 atm
zone refining
(3) 1.0395 atm (4) 2.0 atm
(iv) Camallite is an ore or magnesium and
sodium
Q.32 The energy of a photon of wavelength .A = 1
(l)TITF (2) FITF meter is {Planck's constant= 6.625 x I0-34 Js,
(3) FTIT (4) TF1F speed oflight = 3 x 108 mis)
(1) 1.988 )( 10-:!S J (2) 1.988 X rn·2B J
Q.26 Among the following statements, the incorrect (J) 1.988 X 10-JO J (4) 1.988 X 10-27 J
statements is :
(I) calamine and cerrusite are carbonate ores Q.33 In metallic solids, the number of atoms for the
(2) rutile and cuprite are oxide ores face-centered and the body-centered cubic unit
(3) zinc blends and pyrites arc sulphide ores cell, are, respectively-
(!) 2, 4 (2) 2, 2 (3) 4, 2 (4) 4, 4
(4) malachite and azurite are sulphate ores of Cu

www.puucho.com
CP Publication KVPY chapterwise with www.puucho.com

KVPY Model Test Papers



Among
enantiomers

~·tf,~
the structures given, select the Q.37

(l)H
H f
Which among the following is a rhreo isomer?

CH20H
OH
OH (Z) Ht=OH
Br
CH3

CH3
H

H H CH3 H H CH3
COOH
(I) (11) (III)
(4)H+NH2
(I) I and II
H-t-ou
(2) I and III
(3) II and Ill CH3
(4) 1, II and Ill
Q.38 Selective oxidation of
R 1C = C(CH2),.CH = CHR2 with CrO;i in acetic
Q.35 Which of the following reactions does not acid gives-

n
represent the major product given? ( l) R1 C = C(CH2)n C02H + R2CO
1H1 (2) R1C02H + H02C (CH2)11CH = CHRi
Br H1c- -o- 1 (3) R1CO---CO(CH2)nCH = CHR2
(1) CH1 1 (4) R 1C = C(CH2):iCO-CHR2
DMSO
Q.39
CHrCH=C--CHi-C-H (i) Aq2O I OH8
II
0
(ii) SOCl2 (iii) HrPd-BaS04

s
Final product is :
(1) CH3---CH=CH---CH2---C-H
(4) ~
II
0

Q.36 lfA+B....2!!'.'..... OO CH-CH3


what are A & B

(1)00 &00
(2) CI-1:,C-H & CHrC-H
II II

(3) a: 0

&
0

CH,-CH,
Q.40
'
+ 0 A 21\TBS
-A--1>B---c-n
Ale.KOH

The final product D in the above sequence of


Pd/C
-Hz

(4) OO & CIIrC-H


II
0
reactions is-
( I) Benzene
(3) Decalin
(2) Anthracene
(4) Naphthalene

www.puucho.com
CP Publication KVPY chapterwise with www.puucho.com

• PRACTICE TEST~2

MATHEMATICS (1) pq (2) er


Pll

6 r
Q.41 The value of Jmax(I 2-· 1x 11.4-1 x l,3)dx is (3) 0 (4) l
-6
(!) 40 (2) so (3) 60 (4)30 Q.48 If A, B and C are exhaustive events satisfying
- 1 1
411. 2-2)} . P((AvB)nC )= 5 ,P(BnC)-P(AnBnC)= 15
Q.42 If x = --~ and y = 2 wherp ). is a real
1+:i..- !+A
parameter and x 2 - xy + y2 lies between [a, b] and P(A n C) = J_, then P(C n (A u B)') is
10
then (a+ b) is equal to-
(1) 8 (2) 10 (3) 13 (4) 25
( 1) .!.?. (2) .!! (3) .!2. (4) 20
Q.43 Defore a race, the chances of three runners A, D 30 30 30 30
and C were estimated to be proportional to
5 : 3 : 2, but during the race; A meel<i with an Q.49 If A(l .L i), B(3 + 4i) and C(z) arc the vertices
accident which reduces his chance to l/3. The
respective chances ofB and C now winning arc ofa A.ABC in which LBAC =~ and AC= 2AB.
3
( I) 3 2 (Z) 2 4 (3 ) 7 2 (4 ) 3 2 Thcnz is-
s's .s'Ts Ts·is 10· 10 (l) 3 + 4i + i../3 (2 + 3i)
Q.44 Solution of the differential equation (2) (3 + 4i) + ~- i(2 + 3i)
dy y(x -yfny) .
- - - - - ' - - - lS
dx x(xtnx-y) (3) (2 + 3i) + i.fi (3 + 4i)
(l) xfnx+ yfny C (2 ) xtnx- yfny == C (4) (2 + 3i) + ~ (3 + 4i)
xy xy
(3 ) fox + tny ~ C (4) enx _ Cny =C
X y X y
Q.50 Given that

Q.45 lf
!~ tlog(n 2
+r~)-21ogn log 2 + f- 2,
j
then
t{n) !~ ((1+sin%X1 +sin 2x )..(1 +sin 2~)Y', 2
lim +kn2 + ,2r(n2 + 22r .... (2n 2 )m r'n
n-+GOn m
then lim f(n) =
n...;,ce is equal to -
(1) l (2) e (3) 0 (4) 00 ( 1) zmem(ro'2-2)
(3) e'*~'2-l)
Q.46 If parabola of latusrectum ,~, touches a fixed
equal parabola, the axes of the two curves
being parallel, then the locus of the vertex of Q,51 I--;::::::======-=
(x 3
-l)dx
equals:
x 4 ~6x 6 -4x.l+2
the moving curve is -
(1) a parabola oflatusrectum 2). (1) 'i/6 X - 4X .r.2
6 3
• +C
(2) a parabola oflatusrectum). 12x 3
(3) an ellipse whose major axis is 2).
(4) an ellipse whose minor axis is 2).

Q.47 If S be the sum of coefficients in the expansion


of (px + qy - rzt
(where p, q, r > 0), then the (3) ~6x 6 -4x3 +2+c
6x 3
value of lim l/aS , is :
n-;,,; (S +ly:1 (4) _!_An(6x6- 4x3 +2) + C
12

www.puucho.com
CP Publication KVPY chapterwise with www.puucho.com

KVPY Model Test Papers


From an external point P, pair of tangent lines Q.58 If the shaded portion represents the set of

i
are drawn to the parabola = 4x. If OI and 91 be complex numbers then which of the following
the inclinations of these tangents with the axis of set of complex numbers satisfy the inequality
tan- 1 (lug, 12z - 11) > lan- 1 (log3 12z + 11)
x such that 91 + 82 = .::. , then the locus of P is
4
(l) lm(z)h . () rllm(z).
(l)x-y+l=O (2) X +y- l = 0 2
(3) x-y- l =O (4)x+y+ I =O ~z) ~z)

Q.53 The three planes 4y + 6z = 5, 2x + 3y + Sz = 5


and 6x + 5y + 9z = 10
( l) meet in a point
(2) have a line in common
(3) form a triangular prism
(4) none of these 1t
Q.59 If O :;;; arg (z) ~- , then the least value of
Q.54 Consider the family of lines 4
(x + y-1) + 1(2x + 3y - 5) = 0 and ..fi [2z - 4il is
(3x + 2y-4) + µ (x + 2y- 6) = 0, the equation (1) 6 (2) 1 (3) 4 (4) 2
of a straight line that belongs to both the
families is Q.60 (a, b) and (c, d) are solutions of simultaneous
(l)x-2y-8=0 (2) X - 2y + 8 = 0 equation 5(logy x + log~ y) = 26, xy = 64 then
(3) 2x + y-8 = 0 (4) 2x-y-8 = 0 a+b+c+d=
(1) 51 (2) 17 (3) 68 (4) 34
Q.55 Let A, B, C be three events in a probability
space. Suppose that P(A) = 0.5, P(B) = OJ, BIOLOGY
P(C) = 0.2, P(A fi B)= 0.15, P(A n C) = 0.1
Q,61 Apogamy is:
and P(B fi C) = 0.06, the greatest possible
(I) reproduction of virus
value of P(Ac n B~ n Cj is
(2) development of bacteria
[Note: Ac denotes compliment of event AJ
(3) failure of fusion of gametes
(1) 0.31 (2) 0.25
(4) loss of function of reproduction
(3) 0 (4) 0.26
Q.62 In maize, a!euron layer develops in outermost
Q.56 If a. and p are roots of the equation region of:
( l) Pericarp (2) Epicarp
ax 2 + bx + c =Othen roots of the equation a
(3) Testa (4) Endosperm
(2x + 1) 2 - b{2x + l )(3 - x) + c(3 - x)2 = 0 are-
(1) 2a + 1, 2~ + 1 (2) 3a + 1, 3P + 1 Q.63 Major part of semen is secreted by :
a.-3 p-3 a.-2 p-2 (1) Seminal vesicle
(2) Prostate gland
(3) 2a. -1 ' 2p + I (4) None of tl1ese
a.-2 '3-2 (3) Cowper's gland
(4) Bartholin's gland
Q.57 The number of integral valuc(s) of 1a.' for which Which one of the following is not a phase of;
Q.64
expression log(log 113(1og,(sin x + a.))) is the menstrual cycle? ·
mcaningf4! x ER, is (1) Lutcal phase (2) EstrolL~ phase
(1) 1 (2) 2 (3) 3 (4) 5 (3) Follicular phase (4) Menstrual phase

www.puucho.com
CP Publication KVPY chapterwise with www.puucho.com

• PRACTICE TEST-2

Q.65 Copper-T : Q.73 Diphtheria is caused by:


(1) acts as a barrier (I) Bordetella (2) Clostridium
(2) prevents cleavage (3) Treponema (4) Coryncbactcria
(3) suppresses fertilization
(4) prevent~ implantation of blastocysts
Q.74 The most important long acting antibody
representing about 80% of the antibody that is
Q.66 When a tall pea plant (TD is crossed with a
dwarf plant (tt), what will be the F2 generation? able to pass across the placenta is:
( l) All tall plants (1) JgD (2) IgG
(2) All dwarf plants (3) lgM (4) lgA
(3) Both tall and dwarf plants in 3 : I ratio
(4) Both tall and dwarf plants in l : I ratio Q.75 Central Rice Research Institute is at:
( 1) New Delhi (2) Kolkata
Q.67 Test cross involves:
( 1) crossing the F 1 hybrid with a double
(3) Cuttack (4) Mumbai
recessive genotype
(2) crossing between two genotypes with Q.76 The closest pets of human beings are:
dominant trait ( l) cat and dog (2) dog and sheep
(3) crossing between nvo genotypes with (3) cat and cow (4) cattle and buffaloes
recessive trait
(4) crossing between two F 1 hybrids
Q.77 Introduction of foreign genes for improving
genotype is :
Q.68 Alleles of different genes found on same
chromosome may be separated by: (I) tissue culture (2) vernalization
(I) epistasis (3) biotechnology (4) genetic engineering
(2) pleiotropy
(3) crossing over Q.78 Which of the following pairs is correctly
(4) continuous variation matched?
(1) Central dogma - Codon
Q.69 Trisomy of chromosome number 21 in man
(2) Okazaki fragments - Splicing
causes:
(I) Thalassemia (2) Down syndrome (3) RNA polymerase - RNA primer
(3) Turner syndrome (4) Sickle-cell anaemia (4) Restriction enzymes - Genetic engineering

Q.70 Width of DNA molecule is: Q.79 Escherichia coli is used as an indicator
(I)I5A (2)34A organism to determine pollution of water with;
(3)25A (4)20A (1) heavy metals
Q.71 Which is incorrect? (2) faecal matter
( 1) Wings of insects and bats are analogous (3) industrial effluents
(2) Wings of bats and birds are homologous (4) pollen of aquatic plants
(3) Wings of insects and birds are analogous
(4) Wings of insects and birds are homologous Q.80 Common indicator organism of water pollution
Q.72 Which of the following is not a vestigial organ is :
in man? (I ) Escherichia coli
(1) Nail (2) Entamoeba histolytica
(2) Tail vertebra (3) Lemna pancicostata
(3) Wisdom tooth (4) Eicchornia crassipes
(4) Vermiform appendix

www.puucho.com
CP Publication KVPY chapterwise with www.puucho.com

KVPY Model Test Papers


Q.83 Match Column-I with Colunm-ll

PART-II
Column_-I_ _ _ ; Column-II
Questions given below arc single correct type
(A) Force on an electron (P) iGravitational
Questions. Each correct answer carries +2.
in an atom .fi_fi_or_ce_ _ _ _ !
0.5 marks will be deducted for each \'Vl'Ong answers.
(B) Force between a (Q) Strong force
PHYSICS proton and a neutron
inside nucleus
Q.81 Which statement is correct- (C) Force between a (R) Coulomb force
(A) Electric field of nonconducting charged proton and proton
sheet = a/2 E o inside nucleus
(B) Electric field of conducting charged (D) Conservative force (5t
jElectric for_c~ __ _
sheet= a/eo (1) A->P,R,S; B-+P,Q; C-+P,Q,R,S; D-+P,R,S
(C) Electric field inside charged conducting
(2) A-+P,R,S; R~P; C-+PtQ,R; D-+P,R,S
sphere is zero
(3) A-+R,S; B---+P,Q; C-+P,Q,R,S; D---tR,S
(l)A,B,C (2)B, C,D
(4) A-+P,R; B-+P,Q; C-+ Q,R,S; D-+P,R,S
(3) A, B, C, D (4) A, B, D

Q.84 Which transistor is best from following:


Q,82 Match the facts given in Colurrm-1 with the
(1) NPN (2) PNP
system given in column-Il
f

j
•• , ~ ~ - - -
(3) Both (4) None of these
I
I
I
F
I
Column-I i Column-II
(A) 1Electric field (P) ~ non conducting Q.85 What is value ofE in given circuit:
strength is zero in. solid sphere 2Q 2n. IQ

[I3;J
the volume 1charged uniformly
F

(B) i Electric field (Q) A conducting


strenb'1h is non-zero spherical shell
in the volume but charged
zero at the centre uniformly (1) 6V (2) 12V
. (R) A non conducting (3) 3V (4) 2V
(C) Electric field
~trength is hollow sphere Q.86 A piece of granite floats at the interface of
I •
maximum on t b e charged mercury and water contained in a beaker (Fig.).
surface uniformly If the densities of granite, water and mercury
(D) Electric field (S) A conducting are p, P1 and P2 respectively, the ratio of the
volume of granite in water to the volume in
strength outside the solid sphere
mercury is-
! system varies charged
!
inversely as the uniformly
square of the
distance from the Water
centre
!

(1) A---tQ,R; B-+P; C-+Q,R,S; D-+P,Q,R,S


l\{ercury
(2) A-+Q,R.S; B-+P; C-+P,Q,R,S; D--loP,Q,R,S
(3) A-+Q,R,S; B-+P,Q; C->P,Q,R,S; D-+P,Q,R
(4) A-+Q,S; B-+P; C--+P,Q,R,S; D-+Q,R;S (1) (P2- p)/(p- Pt) (2) (P2 + p)/(p + Pi)
(3) P1P:J/p (4) p1/p2

www.puucho.com
CP Publication KVPY chapterwise with www.puucho.com

• PRACTICE TEST-2
Q.87 A simple pendulum is constructed by attaching Q.90 A particle moves in the x-y plane with velocity
~ ~ ~
a mass m to a thin rod of length ;l.. The
v = ai + bxj , where a and b are constants.
pendulum is pulled back to some angle 9 > 30°
Initially, the particle was at the origin. The
from the vertical and released. Which of the
trajectory of the particle is
following techniques could be used to change
the frequency fofthis pendulum?
y 11,1
I. Changing the mass m on the end of the
pendulum
ll. Changing the length 'A. of the pendulum
(J) ~ < (2) - + - - - X

Ill. Changing the angte 0 from which the


pendulum is released
(I) I only (2) I and II only
(3) II only (4) IT and III only

Q.88 Tbe diagram sbows an iron wire AB mounted


in a brass frame and attached to the frame at
both ends A and B. The length of AB at 03 C is
300 cm and the diameter of the wire is 0.6 mm.
What extra tension will be set up in the
stretched wire when the temperature of the
system is raised to 40°C in powers of 106 dyne? CHEMISTRY
Given a.bmss = 8 x 10-6 /"C, O.uon = 12 x 1ff~?C
Q.91 Predict product (s) in the following reaction,
and Yiroo = 21 x 10 11 dyne/cm2
P4 + Off ----;Product (s)
(1) PH3 (2) P0/-
(3) H2P02- (4) Both (1) & (3)

Q.92 Which of the following ion will show


(I) 0.95 (2) 0.165 paramagnetism?
(1) yS+ (2) y4-1- (3) 0 2-
(3) 0.142 (4) 0.125

Q.89 Two identical trucks each of mass M Q.93 The product of :r with Mn0-4 in alkaline
(excluding sacks of rice) move on national medium is:
highways with _speeds v1 and v 2 towards each (I) h (2) I0-3 (3) 10-
other. When they meet each other, a sack of
rice of mass m is thrown from one track to the Q.94 For a complex reaction A ~ products
other and an identical sack of rice is thrown E al = 180kJ/mol l· E al. =80klmol·,
from the second to the first. Calculate their
E" l ~ 50 kJ/mol
velocities v 1' and v2' after the exchange of
sacks, given m = 50 kg, M = 200 kg, Overall rate constant k is related to individual

1~1 =50m/sand 1~1 =200nv's. rate constant by the equation k-(kt, Jn


Friction of the road may be neglected- Activation energy (kJ/mol) for the overall
(1) 0, -150 mis (2) 150 mis, - 50 mis reaction is-
(3) 100 m's, 150 mis (4) 100 mis, 50 mis (1) 100 (2) 43.44 (3) 150 (4) 140

www.puucho.com
CP Publication KVPY chapterwise with www.puucho.com

KVPY Model Test Papers

If the equilibrium constant for the reaction (i) (BHJ)i



Q.99 ?
H\aq) + OH-(aq) ~ 1120(/) is 10 13 at certain (ii) 11:;01,01 r

temperature then what is the E0 for the reaction,


2H2O(/) + 2e- ~ H2(g)

Given; 2 ·3 03 RT = 0.066
+ 2OI-r (aq). (2) CI HJ
OH

F
(1) 1.230 V (2)--0.858 V
(4)Q-<~n
(3)--0.80 V (4)-0.8274 V
Q.100 The TUPAC name (including the stereochemical
Q.96 Diborane is formed the elements as shown in notation for the chiral carbon) of
equation ( l) H 0
2B(s) + 3H2(g) ~ l½H6 (g) .... (1)
~
Given that
H2O(/) -4- H20 {g) lllii" = 44 kJ ,, H
ts -

Br H
2B(s) + 3/2O2(g) --)- B203(s)
lll-I2°=-1273 kJ ( l) (S)-2-Bromo-(Z)-hex-3-en-4-one
(2) (R)-2-Dromo-(E)-hex-3-en-4-one
Il2H6(g) '+ 3O2(g)-4- B203(s) + 3H2O (g)
(3) (R)-5-Bromo-(Z)-hex-3-cn-2-onc
illi3° =-2035 kJ
(4) (S)-5-Dromo-(E)-hcx-3-cn-4-one
H2(g) + 1/2 02(g)--)- l-IiO(/) Af-I4° = -286 kJ
the n.H0 for the reaction (1) is-
( l )36 kJ (2) 509 kJ MATHEMATICS
(3) 520 kJ (4)-3550 kJ
Q.101 Let f(x) be a twice differentiable function for
Q.97 A solution containing 8.0 g of nicotine in 92 g all real values of x and satisfies f( I) = l,
of water freezes 0.925 degrees below the f(2) = 4, f(3) = 9. Then which of the following
normal freezing point of water. If the molal is definitely true?
freezing point depression constant (J) f"(x) = 4 XE (I, 3)
1
K2 - l.85°C mor then the molar mass of (2) f "(x) = f'(x) = 5 for some x E (2, 3)
nicotine is- (3) f "(x) = ~ XE (2, 3)
(1) 16 (2) 80 (4) f"(x) = 2 for some x E (1, 3)
(3) 320 (4) 160
Q.102 If Si, S2 and S~ denote the sums upto IO tcnns
Q.98 Which reaction results in the formation of a
of three sequences in A.P., whose first terms
pair of enantiomcrs?
are unity and common difference are in ll.P.,
(1) l ..
/')( D~~ then the value of
2S 1S3 -S1S2 -S 2 S3
equals to--
S1 -2S 2 +S 3
l H
(1) I (2) 5 (3) 10 (4) 20
(2)~ H20

(3) H3C ~
l
1111 oH- Q.103 If !f(x)[ +.Jr+ cos 21tx = tan2 ( 1;) f(x) ;
l_J DMP
CH30H
then f(3) is equal to :
(4)~ (1) _1 (2) _I_ I 1
I H (3)- .,/z (4) - 2.fi.
fj, 2Jz

www.puucho.com
CP Publication KVPY chapterwise with www.puucho.com

• PRACTICE TEST•2

Q,104 If g(x) is a differential real valued function Q.110 The interval in which F-1,UlCtion
satisfying g"(x) - 3g'(x) > ~ x ;::>-: 0 and f(x) = sin- 1 (x2 -3x + ?) is increasing, is-
g'(O) =-1, then g(x) + x for x> O is
(1)[1, 2] (2)[¾,co)
(1) an increasing function
(2) a decreasing function
(3) a constant function (4) None of these
(4) Data insufficient

Q.105 The tangent at a point whose eccentric angle BIOLOGY


60" on the ellipse .;-+..;-
a
2

b
2
= 1 (a> b) meet the Q.111 In embryo sac of a typical angiosperm, there
are:
auxiliary circle at L and M. If LM subtends a (1) Egg cell, synergids and antipodals
right angle at the centre, then eccentricity of the (2) Egg cell, synergids and secondary nucleus
ellipse is- (3) Egg cell, synergids, central cell and polar
(1) _I (2) ~ (3) 2__ (4) _!_ nuclei
.fi .fi .,ff 2 (4) Egg cell, synergids, polar nuclei and
antipodals
Q.106 The direction cosines of two lines are related
by ,._ + m + n = 0 and al 2 + bm2 + cn2 = 0. The Q.112 Ovulation in the human female normally takes
lines are parallel if- place during the menstrual cycle :
(1) a+ b + c = O (2)a- 1 +b- 1 +c- 1 =O (I) at the mid secretory phase
(3) a=b = C (4) None of these (2) at the end of the proliferative phase
(3) just before the end of the secretory phase
(4) at the beginning of the proliferative phase
Q.107 The maximum value of sec-i ( 7 - S~x 2 + 3)
2(x + 2)
J
Q.113 A pure tall and a pure dwarf plant were crossed
is to produce offsprings. Offsprings were self
(I) 5n (2) 5Jt (3) 71t (4) 21t crossed, then find out the ratio between true
6 12 12 3 breeding tall to true breeding dwarf'?
( 1) l : 1 (2) 3 : 1

Q.108 If a = lim (-3- 1 - +


n.- n +I n +I
+ ...... + +
n3 + I
~J and
(3)2:1 (4)1:2

Q.114 If there are I 6 chromosomes in the roots of


rt
I-' = 1·1m -sin2x
.--
th en the qua dr atlc
. equation
. onion, then the number of chromosomes in
J<-l-0 sm8x
spore mother cell will be :
whose roots are a., ~ is (1)2 (2) 8
(I) 12x2 -7x +I= 0 (2) x2 + 19x-120 = 0 (3) 4 (4) 16
(3)x -17x+60=0 (4)x 2 -7x+l2=0
2

Q.115 The chromosomal pattern of Turner syndrome


Q.109 Let n 1 = sin 7 + cos 7, n2 = .Jsin 7 + ..fcos7, usually have:
n3 = ../I+sin14,11,4= l,then- (1)2A+XXY
(2) OneX
(1) n2 > n3 > !14 > n1 (2) n3 > Il.4 > n2 > n 1
(3)2A+ XXX
(3) n3 = n1 > n.i > n2 (4) n2 > n1 = n3 > Il.4
(4) Two X only

www.puucho.com
CP Publication KVPY chapterwise with www.puucho.com

KVPY Model Test Papers


Which of the. following is the best way to Q.118 Plasmids are: •
determine paternity? (I) cDNA
(I) Gene counting (2) viral RNA
(2) Protein analysis (3) mitochondrial DNA
(3) DNA fingerprinting (4) circular extra chromosomal DNA in bacteria
(4) Chromosome counting
Q.119 Fluoride pollution mainly affects:
( 1) teeth (2) heart
Q.117 Which of the fol lowing is the correct order of
(3) brain (4) kidney
evolutionary history of man 7
(1) Peking man, Homo sapiens, Neanderthal,
Q.120 "Mina1nata disease" is produced due to the
Cro-Magnon
pollution of water with :
(2) Peking man, Neanderthal, Homo sapiens,
( 1) Crude oil
Cro-Magnon
(2) Silver nitrate
(3) Heidelberg man, Peking man, Neanderthal,
(3) Mercuric chloride
Cro-Magnon
(4) Methyl isocyanate
(4) Peking man, Neanderthal, Homo sapiens,
Heidelberg man

www.puucho.com
CP Publication KVPY chapterwise with www.puucho.com

• PRACTICE TEST-2 ii
f ANSWERS
Qus. 1 .2 3 .4 :5.-. ·s 7 8 9: 10 11 12 .. ta. : .14 15 16 17: -1.8 19 20
Ans. 2 1 4 4 2 2 1 4 3 2 4 3 3 1 3 1 3 2 4 4

Qus. 21 22 23 24 25 .. _26 27 28 - 29. 30 31 32 33 34 35 36 37' 38 39 40

Ans. 2 3 4 2 2 4 2 3 2 2 1 1 3 3 2 4 2 1 4 4

aus. 41 42 43 44 45 46 47 48 49 50 51 52 53 54 55 56 57 ·- 58 59 60
Ans. 1 1 2 1 2 1 3 3 1 1 3 3 2 2 1 2 3 2 3 3
Quss 61 - 62 63 64 65 66 67 68 69 70 71 72 73 74 75 76 77 78 79 80

Ans. 3 4 1 2 3 3 1 3 2 4 4 1 4 2 3 1 4 4 2 1
Qus. 81 82 83 84 85 86. 87 88 89 90 91 92 93 ~4 95 96 97 98 99 100
Ans. 3 2 1 1 2 1 4 1 1 3 4 2 2 4 2 1 4 2 1 4
.··

Qus. 101 102 103 104 105 106 107 108 109 110 111 112 113 t14 115 116 117 t1B 119 120
Ans. 4 3 1 1 2 2 4 1 4 3 4 2 3 4 2 3 3 4 1 3

E
PART RI
-------··
SOLUTlONS

o;/;1',""""'"":·;;:.,,...,,"":,;:o;::,;: _ _ _--..,__,___ _ _ _ _ _ _ _ _ __J
3. · For tangent galvanometer
µ NI -B N _ tan0
One Mark Questions -0 - - utan8 ::::> ~1 - - -1
2R N2 tan0 2
PHYSICS
4. First c- come near then goes away.

~~~M
. 1. I
1: T1 oc
.JM+2M
s. e = L[I_-(-1)] = 2LI = 2 x µ0N2 Ax.!.
T2 a; I t t I. t
~2M
M N ..J2M-M
6. equivalent circuit

2.
-T1
T2
HE
= ~ => T2=
3M

kM
./3T1
r:n E~-

CT
B2= ---
d~
7. - 30

Bi= B2 tan8
2klv1 ldv1 2 8.
:::>-- = - tan9 - - JI
d3 d3 tane
! 2 '

= (~J
dz
3
::::> ~
d2
= (2cote)11 3
Slope of 1-H curve for ferro may be positive,
negative and zero so never be constant

www.puucho.com
CP Publication KVPY chapterwise with www.puucho.com

KVPY Model Test Papers

9. When contact is broken ~ net emf 14. In the fundamental mode the air column in the •
open pipe (pipe open at both ends) vibrates
11. For isolated systems, all three conservation with consecutive antinodes at its ends so that
laws are always in effect : total energy is the length L of the pipe is equal to ')J2 where A
conserved (although kinetic energy K is not is the wave length of sound in air. Therefore,
conserved in this perfectly inelastic collision), ;\.=2L.
linear momentum is conserved, and angular The air column in the closed pipe (pipe closed
momentum is conserved. at one end) on the other hand vibrates in its
fundamental mode, with a node at the closed
12. . The kinetic energy lost in the collision can be
end ( at the water surface inside the pipe) and
found by subtracting the skater's final K from
the neighbouring antinode at the open end so
their initial K :
that U2 "" J,.J4. Again we obtain A = 2L. The
1 2 1 2
Kirutla1= 2 m1v1 + 2 m2v2 frequencies in the two cases are same.
-~. - . . . .. . - .
~ ~ . . . . ..... -· -· . .
l . 2 Antiuode
= - (m, + m2)vr
K.11ne1
2 . / A
AK = K11na1 - Kini1ia1
r .
(½(m1+ m2)v:) - (½m1v~ +½m2v~) Nod~

.l . .,
aK = /./1 r:: ).l.S:
Nod~'
'
L-=V1.
AK = (.!_ (30 + 80)( 100kg • m / s )2)
2 110kg ............................ ,
...... -· --- ..........
- (½(30kg){2rn/s) 2 +½(80kg)(lm/s) 2 ) i ' --~~- -.............
~. ~~-~-·-~~ ". . " '
aK=45 J-(60 +40) =-551
i
; ' \
~ ....... Ill."' .... ·--"'-·
........ -- . ·---~--
L_____ ._..... __ ._--~ti~odr. . _ . . __ .. ___ . _
13. Since a monoatomic ga1- molecule bas 3
15. Considering the vertical motion of the
degrees of freedom. the molar heat capacity
(als'o called molar specific beat) of the projectile (pebble), the time taken to reach the
monatorn.ic gas at constant volume (Cv) is (3/2) ground after leaving the wheel is given by
R where R is universal gas constant. Its molar 2R = 0 + (l/2)gt2, from which t = 2-V(R/g)
heat capacity at constant pressure (Cp) is (5/2)
R. This follows from Meyer's relation, (We have used the equation of the one
Cp= Cv+ R. dimensional motion, X =XO+ Vot + (1/2) ar]
This means that when heat energy equal to The horizontal range of the pebble is
(5/2) R joule is supplied to one mole of a
monatomic gas to make it expand at constant PQ = horizontal velocity x time of flight
pressure, (3/2) R. The centre of mass oftbe wheel is moving with
This means that when heat energy equal to (5/2) speed u. The pebble at the topmost point of the
R joule is supplied to one mole of a monatomic wheel is moving horizontally with speed u with
gas to make it expand at constant pressure, (3/2) respect to the wheel so that the horizontal
R joule is used to increase the temperature by 1
velocity of the pebble with respect to the
K and the remaining R joule is used for doing
work against the forces which oppose the ground is u + u = 21L ,
expansion. 3/2 is 60% of 5/2. Therefore, horizontal range
PQ = 2u x 2../(R/g) = 4u-V(R/g)

www.puucho.com
CP Publication KVPY chapterwise with www.puucho.com

• PRACTICE TEST•2
16. The position vector r 1 of the particle at the 19. Let X be the leftward displacement of M and x
instant tis given by and y be the leftward and downward
r 1 = v0t + 1/2at2 where v0 is the initial velocity displacement of m.
and a is the constant acceleration LetAB=J!1; BC=r2: CD= £3 and Am= .€4
~ ~ ~

Wehavevo= 2j and a= 2i + 4j When M moves towards left, say by x, then


Therefore, ri = 2] t + (1/2) (2 i + 4] )r
= t2 i + (2t + 2t2 ) }

The above equation shows that the x-coordinate


of the particle at time t is t2 and the
y-coordinate is (2t + 2f)
The time t at which the y-coordinate becomes -c3_,.
12 metre is given by
AB=(J!i-x)
2t + 2t2 = 12
Or, 2t2 + 2t-12 = 0
BC =.f.2

This gives t = [~ 2 ± .,/(4 + 96)}/4 = 2 seconds, CD=f,-x


ignoring the negative time. Am=£4+y
Since the x-coordinate of the particle is t2, its .'. £1 - X + fi + t3- X+ £4 + Y = £1 + £2 + f3 + £4
value when the y-coordinatc becomes 12 m (at :. 2x =y
time 2 seconds) is 4m.
2x = y
17. With the help of dimcm;ional analysis, we Further x = x'
cannot arrive at constants and algebraic, :. x = x
exponential and trigonometric functions. Given x = 2mls 2
Though dimensionally equation is satisfied, but :. x "' 2 m/s2 and y = 4 m/s2
it may not be correct.
Acceleration ofm is I--;, I= ~a~ +a~
18. Let us consider any two particles of the body.
Let a particle of the first body rotate in the x-y = ~2 2 + 4 2 = ../20 = 2 Js m/s 2•
plane in a circle of radius r 1•
Simlla.rly, let a particle of body 2 move in the 20. P-T graph is a straight line passing through the
y-z plane in a circle of radius r2as shown in the origin.
figure. Therefore V = constant
:z
: . work done ort the gas is zero.

Further, density of the gas, p = ~


V
I
.-. pa: V

Since volume of the gas is constant, density of


the gas is constant.
PV=nRT
""? ~ ""?
Then ro 1 = ro1 k and ro 2 = eu:i i
-)- ---t -,. -"" A
~P= (~)T
W11 = ffi2 - <01 = W:z i - W1 k
i.e., slope of P-T line is proportional to n.

www.puucho.com
CP Publication KVPY chapterwise with www.puucho.com

KVPY Model Test Papers

CHEMISTRY 37. In compound (2), the two identical atoms (H,H)



lie on tbc opposite sites of the Fisher projection
formula and hence it represents the thrco-
21. isomer.

38, A double bond is more reactive than a triple


22. XeFs- and BrF3 have 2 lone pairs each. bond and hence it is more easily oxidized i.e,
( 1) is correct.
24. [Ti (HiO)t;] C4
40.

26.
In Ti+4 all electrons are paired.

Both are carbonate ores


r?'
~
+ 0 Diels._Alder
reaction
(X) 2NBS
-2HBrl

27, Pb4 ~ Pbh + I2 Br

28.
Pb+4 is oxidsing and r is reducing agent.

meq. oflhO2=---xl000= -
X lOx
~~~Ale.KOH
vV -2H2 V._) -2HBr
Naphthalene Tetralin
¢oBr
100xl7 17
IOx 10
-= xN2 => N2 = - = 0.588
17 17
MATHEMATICS

29. n= lxl =0.041


0.082x 298
E = 0.041 (436 + 1968) = 98.56 KJ
J, .J, 41.
BEfrnole excitation energy for 2mo1e·H atoms

Sa 6

30.
Tc 21Rb _ 8b
-=---- or Tc oc b
fmax(l 2-1 x ll,4.- Ix l,3)dx
Pc u R Pc -6
27b 2 = 2(o.ABCD + DDEFC + o.EFGF)

31. Ti= 3001{, T2 = 600K, P1 = 1 atm, P2 =? = 2[½(3 + 4)x 2+ 2x3 + ½(3+ 4)x 2]
Pi ) y-1 = -Ti => (1-l)log ( -~' ) =log ( -Ti )
(-
= 2[14 + 6] = 40.
~ ~ ~ ~
(1.66-1) [ln P1 - ln P2] ""-ln 2 42, Let A,= tan a
-ln2
lnP2= - - = 1.06 :. x = 2 sin 28 => y = 2 cos 20
-0.66 :. xz + /-xy=4-2 sin48
E= be= 6.62xl0-34 x3x10 8 x 2 + y2 - xy E [2, 6]
32.
"- l :. a+ b = 8

33. FCC -+ 6 x .!_ + .!_ x 4 2 3 2


2 8 43. P(B) = -.- = -
3 5 5
BCC -+ 1 + .!_ x 8
8 2 2 4
=> P(C) = -.- = -
3 5 15
34. Mirror images.

35. Elimination product is non major product

www.puucho.com
CP Publication KVPY chapterwise with www.puucho.com

• PRACTICE TEST-2

Given dy = y(x -y Rny) 47. S = (p + q - r)" {Putting x = y °" z = I}


44.
' dx x(xin x -y) . S . (p +q-r)1'
11m - - - - = 11m --=--__.c_----''------
=> x2£n x dy-xy dy= xy dx-y2 £n y dx " .."" (Sv" +lt Il-->"' {(p+q -r)+ W
1:'nx 1 l fny n
=> - d y - -dyc....c -dx- - d x . p+q -r )
y 2 xy xy x2 = 11m (
,,--><o (p+q-r+l)
(on dividing by x2y2)
1 fox I
=> -dx- -dy+-dy- -dx=O
lny =O {asr\.p+q-r+l
p+q-r )<1}
xy y 2 xy x2

48.

On integrating both sides, we get


x £nx +y .f.n y= C xy

45. f(n) = (I)"' apply the formula


C
f(n) = lim c«(1+s,nf)(i~sin;, }-.{lfs1n 2:}1J 1
- =x+v+y
x-tO . 5
(I+(5mf+smf,-+ . .Hinfo-Hs~f.sin f,-+-}) 1
-
l
=w=> - =u+t
= Jim e x
15 10

= Jim e
['inI +sin(;,)+. . + 2:
X X
sin
X
l=e
(l-1__!,.+... ,_l_)
2 2 2"
x+y+z+u+v+w+t= 1
z=?

x-tO
49.
.!..
1

(1-!1
sum of infinite G.P. = e 21 = e.

46. Let (h, k) be the coordinates of the vertex of the z - (1 + i) (2 + Ji) i;t/3
---=---e
moving parabola and its equation be AC AB
(y-k)2 = -4a(x-h)
Let the equation of fixed parabola be
•y.2 = 4ax
... (i)

.•. ("II")
=>z-I-i-2(2+3i) (½+ ~iJ
From Eqs. (i) and (ii) => z- l -i = 4-6./i +i(6+4.Ji)
2
(y-k) 2 =-4a 4a -h (y2 ) => z = 3 + 4i + i,J3 (2+ Ji)

=> 2/ - 2k y + k 1 - 4ah = 0
Since, two parabolas touch each other, therefore
D=O so.
. Llo{~2n~~)
Given - - - ' - - - ~
n
= log 2 + - - 2
=> 41<- 8(k2 -4ah) = 0 => k2 = Bah n 2
=> Locus of the vertex oft he moving parabola is Let
/=Sax A= lim ~[(n 2 + 12
n--J--con-m
r (n 2 + 2 2 )on .... (2n 2 r] ltn
whose latusrectum =Sa= 2(4a) = 2£

www.puucho.com
CP Publication KVPY chapterwise with www.puucho.com

KVPY Model Test Papers


55. P(Ac r-, Bc r-, Cc) •
= 1 - P(A) - P(B)- P(C) + P(AB) + P(BC)
+ P(CA)- P(ABC)
P(Ac n Ben Cc)m""
= 1 - P(A) - P(B)- P(C) + P(AB) + P(BC)
+ P(CA)
= 0.31

56. a{2x + 1) 2 + b(2x + l) + c = 0


(x-3)2 (x-3)
51. f (1- X-))X-4
~6-4x- 3 +2x- 6 ~ 2x + I = o. or 2x + l = p
x-3 x-3
Put x- 3 = t => -3x-4 dx = dt
~ 2x + l = a.x-3o. => x(a.-2) = 1 + 3a.
=_.!.I c1.- t) dt 1 +3a. 1+3p
=>x=--,~-
3 ~6-4t+2t 2 a.-2 p-2
Again put 6-4t + 2t2 = z
(-4 +4t)dt = dz 57. Let y = log (log113 (log7 (sin x +a.))),';/ x E R
1 will exist
(1 - t)dt = -- dz
4 if logw (log 7(sin x + a.)) > 0
but log7(sin x + o.) > 0 ... (!)
Take antilog log 7 (sin x + a) < 1 ... (2)
From (I) and (2)
0 < log 7 (sin x + o.) < 1
l<sinx+a.<7
1-sin x < a.< 7-sinx
:. max. value of 1 - sin x = 2
min. value of7 - sin x = 6
:. 2 < 0. < 6 & 0. E l
52. Since, y= mx + J_ or m2h-mk -1- 1 =O,
m :. a.= 3, 4, 5
k I
we have m1 + m2 = - ,and m1m2 = - 58. lo~ l2z - 11 > lo&:il2z + l I
h h
~ (2z-1)(2z-l) > (2z + 1) (2z +I)
Given, B1 + 82 = ~ => 4izf- 2(z + z) + 1 > 4 ]zj2 + 2(z + z) + 1
4
Im(z)
1t
:. tan (B1 + 82) = tan -
4

=> m1 +m2 = l => ~ = l - _!_


l-m 1m 2 b h ~ 4.2 Re(z) < 0 => Re(z) < 0
:::>y=x-1 =>x-y-1 =O
59. n represent t hc region
Since, 0 :S: arg (z) :S:-, · of
53. Solution of these three equations is infinite. 4
So planes contain straight line. complex. plane lying in the first quadrant and
bounded by x-axis and the line y = x.
l2z - 4il = 2Iz -2il
54. Ist family lines passes through (-2, 3) and u nd
Least value of !z -2ij is length of perpendicular
(-1, 7/2)
Required line x - 2y + 8 = 0
from (0, 2) toy= x, which is .fi..
So, the least value of ..Ji l2z - 4il is 4.

www.puucho.com
CP Publication KVPY chapterwise with www.puucho.com

• PRACTICE TEST-2

60. 5(logyx + log._ y) = 26, xy = 64 72. Nails are useful body parts which provides
strength to digits of helps in holding object.
=> 5 (logy X + - 1- ] = 26
logy X
73. Diptheria is caused by corynebacterium
Put logyx= t diptheriac, which produces diptheria toxin that
:. 5(t2 + 1) = 26t::::;:, 5t2 -26t ·I· 5 = 0 after protein function in the host by inactivating
elongation factor (EF-2) required for
=>t=5 .!_ transcription. It results is pharyngitis.
' 5
I 74. IgG is found in blood plasma & provides
:. logyx = 5, -
5 protection against antigen & provide protection
Sox =y5, y 115 to foetus.
Butxy = 64 => y5. y= 64or y 115• y = 64
:. y = 2 or 32 77. Foreign genes are introduced to provide
:. Solution set will be (32, 2) & (2, 32) resistance or eliminate disease.

:. a+ b + c+ d= 32 + 2 + 2 + 32 = 68
78. Restriction enzymes arc key tools in genetic
engineering.
BIOLOGY
79. E. coli commonly found in faecal material
61. Apogamy leads to formation of haploid
pollutes the drinking vmter in rivers, pond<;. It'.<:
sporophytc.
presence shows contamination.

62. Alcurone and endosperm are usually triploid.


80. E. coli number is decreased in poll utcd water.
63. Maximum part of semen is seminal fluid which
is secreted by seminal vesicle. PART-II
Two Marks Questions
64. Estrous phase is the part of estrous cycle which
is found in non-primate mammales. PHYSICS
86. The weight of a floating body is equal to the
65. Copper-T releases the copper ion which
weight of the displaced fluid. If V and v
inhibitcs the motility of sperms.
represent the total volume of the piece of
granite and volume of granite in water
66. Mendelian monohybrid cross gives 3 : 1 ratio.
respectively, we have
V pg= v pig+ (V-v) p2g
67. It is used to identify homozygous &
Or, v(p1 - P2) = V(p- p2)
heterozygous f 1 progeny.
Therefore, v/V = (p- P2) /(pi - P2)
The ratio required in the question is v/(V - v)
68. Crossing over leads to recombination of alleles
and is given by
by reparation between homologous
chromosome. v/(V - v) = (p - P2)/[(p1 - P2) - (p - P2)]
Or,
69. It is known as mangoloid idiocity which results v/(V - v) = (p- p2) /(p1 - p) =(p2- p) /(p- Pi)
from trisomy due to chromosomal non
disjunction during gamete formation. 87. For small angles of 8 (typically less than 15°),
the frequency of oscillation for a simple
71. Both have different origin & similar function, pendulum is approximately.
so they are analogous organs.

www.puucho.com
CP Publication KVPY chapterwise with www.puucho.com

KVPY Model Test Papers



f= ~ = 21,r ~ v2'=-[M:m V2 - M:m.v 1 ]

For increasingly large value of 0, however, the


=-· ( 200 x 200- 50 x so]
acc~lcration no longer varies linearly with 250 250
displacement. Thus, for larger angles, the =-150 mis.
frequency f will be a affected by the angle of
release 0, as well as by the length of the --> - -
pendulum. 90. v = ai + bxj
vK = a and Vy= bx

88. We have L= Lo(]+ a.0)


dx
-=a
For brass Ln - LoB = Ll 0.B.e dt
.
For ,ron r . - rL--1J_iron = L Oiron a;,..,~. 8 .'.. x=at+C
....,ron
Since x = 0 at t = 0, C = 0
Effective increase in length of the iron wire
:. x= at
M= (L;;;"Hairon -L~o.B)")( 8
dy =bx=abt ....... (1)
Extra tension= Y x Strain x A <lx
abt 2
=YxJ1r2 x -llf
.- [Lo B =Loiron] :. y= - - +c
[;~n 2
abt 2
=y x m2 x (u- -r_,}1 La.a J
mm 0. ,roo
x9 ..· y =2-
. . 0 as y = 0 at t = 0 ...... (2)
=21 X 10 11 X 3.14 X (0.3 X }0-l)2 X
. abt 2 b 2
(12 X 10--(i-8 X JO-ti) X 40 From equation (1) and (2), y = - - = - x
2 2a
= 95 x I 0 4 dyncs :::: 0.945 x l 0 6 dynes Hence the trajectory is a parabola symmetrical
about the y-axis.
89. Let the first truck move to the right and the
second to the left. Let us take the rightward CHEMISTRY
direction as positive. In the horizontal
direction, friction being absent, no external
91.
force is acting on the system. Hence
93.
momentum is conserved.
The momentum carried away by a sack thrown
2
from the first is mv 1. the momentum brought in 94. E =-(180 + 80-50) = 140 kJ/molc
3
by the second sack to the first ism (-v2) .
.'.. By the law of conservation of momentum of 95. 2H2O(l) ~ 2I-t (aq.) + 2OI-r (aq.) .1.G1°
(truck+ sacks of rice) 2H+(aq.) + 2e ~ H2(g) 8Gz 0

(M + m) v1 - mv 1 + m (-v2) = (M + m) vi'
2H2O(/) + 2~H2(g) + 2mr (aq.)
where v 1' is the new velocity of truck I.
6.G=AG/+AG;
:. Mv1 - mv2= (M + m) v1' ~-2FE =-RT ln(I0- 6) -2 X F X 0
0 2

, M m
-~Yr=--~---~ E0 = 0.066 tog(10-2 6) = - 0.858
M+m M+m
2
= 200 .50 - so .200 = 0
250 250
Similarly comidering corn,ervation of 96.
momentum of the second (truck .,. sacks of AfhO = -1273 kJ
rice), we get for the second truck.

www.puucho.com
CP Publication KVPY chapterwise with www.puucho.com

t PRACTICE TEST-2
102. Let the common difference of the three A.P's
be dr, d2 and d3
10 . S -10
3 =-286 X 3
ll.H4° X Then S 1 = -[2.1
1
+ (l0-l)d1]:::;, d1 = - -
2 5.9
B2O3 (s) + 3H20(g) ----l- B2H6(g) + 302(g)
ilHJ° = 2035 kJ
s~ -10 S3 -10
Similarly, we have ch= ---- and <l) = - -
5.9 5.9
3H20 (g)-) 3H20(g)
Since di, d2 and d3 are given to be in H.P.,
ilH1° = 44 X J therefore
1 1 1 1
3B(s) + 3H2(g) ~ B2H6(g) ilH = 36
---~---
d2 di d3 d2
8 I I l 1
:::::, - - - - - = - - - - -
S2 -10 S1 -10 S3 -10 S2 -10
97. 9x2 X 1.85 = 0.925
81 -S2 = S2 -S3
1000 :::::, (S2 - lO)(S 1 -10) (S 2 -10) (S 3 -10)
x= 173.9 S1 -S2 _ S2 -S3
:::::,---
S1 -10 S3 -10

98.
rXH Il20
SN' ) d, £ racemic mixture
=> 2S1S3 - s,s2 -S2SJ = 10
SI -2S:, +S3

103. lf(x)I + ~2cos 2 rcx = ( tan 2 i;-Jf(x)

At x = 3, lf(3)1 + .J2 = 3 f{3)


I
If f{3) :::: 0 => f\3) = .,fi

& iff(3) < O; fl3) = ~ not possible

100. The configuration about the double bond is (E) 104. ~ (g'(x)e-3x) > 3. e-lx
while that around the chiral centre, i.e., Cs is S. dx
H 0 => ~ (g'(x)e-3x + e-lx) > 0
dx
6 + g'(x)) is an increasing function.
:::;, e· lx (1

Br
CH3 5
·~ .,H
Now, c-3x(1 + g'(x)) > (g'(O) + 1)::::;, x > 0
=> g'(x) + 1 > 0
Hence the correct name is :::;, g(x) + x is an increasing function.
(S)-5-bromo-{E)-hex-3-en-2-one.
105. Point P will be (a cos 60", a sin 60°)
MATHEMATICS . b x cos 60° y sin 60°
Tangent w111 c - - - + ~ - - =1
a b
101. Let g(x) = f(x)-x 2• We have
g(l) = 0, g(2) =0, g(3) = 0 ~+ y-,J3 = I ... (1)
[ ·:f(l) = 1, fl2) = 4, f(3) = 9] 2a 2b
From Rolle's theorem on g(x), g'(x) = 0 for at
least x E (1, 2).
Similarly, g(x) = 0 for at lca.,;;t one x E (2, 3).
g"(x) = O ~ f"(x) = 2 for some x E (1, 3)

www.puucho.com
CP Publication KVPY chapterwise with www.puucho.com

KVPY Model Test Papers


Homogenise x2 + ,j =a2 with the help of eqn. ( 1) 109. ·: .Ji +sinl4 > 1

Wegetx2 +y2=a2 [~+ rh]
2a 2b
2 .JI+sinl4 = ~(sin7 +cos7) 2
""sin 7 + cos 7 & ./siD. 7 > sin 7

- +xy.fil ./cos7 > cos 7


2
Xl
x2+y2=a2 [ - + Jy --
4a2 4b 2 2ab :. n2 > n1 = n3 > 1'14

3x 2 + ,j
4
(i- 3a4b 2
2
)- a.fixy = 0
2b
110. -1 :::;;x2 -3x+3:::;; I
=> x2 - 3x + 4 2: 0 and x2 - 3x + 2 $: 0
=>XE [1, 2]
For 90° at centre
f'(x)= 2x- 3 ~O=>x~~
3 3a 2
-+1--=0 ~1-(x 2 -3x+3) 2 2
4 4b 2
3 • a2 7
=- BIOLOGY
4 a (1-e2 2) 4

l -e2= l 111. A typical embryo sac is 7 called and 8 nuclei


7 structure with egg, synergids, polar nuclei and
antipodal.
~ e
2
= -4 =>e=-
2
7 .fi 112. At the end of proliferative phase level of LH is
106. For n = - (e+ m), the second relation gives increased which stimulates the ovulation.
ae2 + bm2 + c(e + m)2 = 0
or (a+ c) £2 + 2ct'm + (b + c) m2 = 0. 115. Turners syndrome is due to monosomy (XO).
For parallel tines, the two roots must be equal result,; is sterile female.
~4c2 -4(b + c) (a+ c) = 0
=> ab+ be+ ca= 0
116. In DNA finger printing technique. DNA is
isolated & segmented images arc developed. It
2
107. [ sec _ 1(-8-5x
- - -)) _
-sec-i ( - 2) -_ -21t
is unique for each individual.
2x 2 +4 max
3
117. There are prebistonic man, whose fossils have
. 1+4+a+ .... +n 2 been traced at different places they are named
108. a= I1m - - - - - -
o-+00 n3 + I after tbe place they lived in & shows some
= Hm n(n + 1)(2n + 1) evolutionary evidence over previous one.
n-;m:o 6(n 3 + I)

~= nm( sin 2x!(~J )( [8x2x]


,<-+o 2x ) sm2x
119. Fluroide pollution is mainly affects the tooth
which is known as flurosis.
j ,,
I
P=- 120. Minarnata disease caused by1pollution of
4
murcury in marine water, Murcury gets in food
1
aP'""- chain with small animals & fishes, which arc
12 consumed by man.
7
a.+~=-
12
12x2 -7x+ 1 =O

www.puucho.com
CP Publication KVPY chapterwise with www.puucho.com

KVPY (Str~am $X) Model Test Paper~


Practice Test -3
I
PART-I (1) ~ (2) f
Fsin$ Fsincji+mg
One .. Mark Questions
(3) _f_ (4) mg
Questions l to 80 are Single correct answer type mg f
questions. Each correct answer carries 1 mark. 0.25
Q.3 The free-body diagram shows all forces acting
marfill will be deducted for each wrong answer.
on a box supported by a stationary horizontal
PHYSICS surface, where the length of each force vector
is proportional to its magnitude. Which
Q.t A square of mass Mand sides of length Lhasa statement below is correct?
moment of inertia Io when rotated about an axis
perpendicular to its surface and passing
through its center, as shown. Now a lump of
clay, also of mass M is attached to one corner
of the square as shown. What is the new
moment of inertia of the masses about the same
axis of rotation'!
/=7 i

.
;.
1#~7
l ;
/~JM (l) The box must be moving to the left, due to
the force of friction 11cting in that direction
(2) The box must be accelerating to the right,
- (l) Io+ ML2 ML2 ~ as indicated by the force of friction in the
(2) Io+-.-
4 2 opposite direction
(3) lo+ /iML2 (3) The box must be moving to the right, as
(4) Io+ 2ML2 indicated by the force of friction in the
2
opposite direction
(4) None of these statements is correct
Q.2 A block of mass m is pushed across a rough
surface by an applied force F, directed at an Q.4 A mass m in three-dimensional space is
angle ~relative to the horizontal as shown. ,The subjected to three forces : F1, r 2 and F3.F 1 and
block experiences a friction force f in the_ F2 have the same magnitude, with F I in the
opposite direction. What is the coefficient of positive-x direction, and F2 in the positive-y
friction between the block and the surface? direction. 1f the mass has an acceleration of 0,
which of the following statements is false?
0 (I) The magnitude of F 3 is the same as that of
F1
(2) The object is in equilibrium and could be
stationary
(3) F3 lies in the x-y plane
(4) The object is in equilibrium and could be
moving

www.puucho.com
CP Publication KVPY chapterwise with www.puucho.com

KVPY Model Test Papers


A cube of mass 30 g wettable by water floats Q.10 A piano tuner stretches a steel piano wire with •
on the surface of water. Each face of the cube a tension of800 N. The steel wire is 0.4 m long
is 4 cm long. Surface tension of water and has a mass 2 g. The number of the rughest
= 70 dyncs/cm. The distance of the lower face harmonic that could be heard by a person who
of the cube from the surface of water is is capable of hearing frequencies up to 104 Hz
(g = 980 cm s-2): is :
(1) 1.9 cm (2) 1.93 cm (1) lOO (2) 150 (3) 50 (4)20
(3) 1.95 cm (4) 1.98 cm
Q.11 The electric intensity due to a dipole of length
Q.6 An object is placed 21 cm in front ofa concave
10 cm and having a charge of 500 µC, at a
mirror of radius of curvature 1 o· cm. A glass
point on the axis 20 cm from one of the charges
slab of thickness 3 cm and refractive index 1.5
in air is=
is tlicn placed close to the mirror in the space
(1) 9.28 x 107 N/C (2) 20.5 x 107 N/C
between the object and the mirror. The distance
(3) 6.25 x 107 N/C (4) 13.1 x 10 11 N/C
of the near surface of the slab from the minor
is 1 cm. The final image from the mirror will
be formed at: Q.12 Four electric charges +q, +q, --q and -q are
(1) 4.56 cm (2) 5.65 cm placed at the corners of a square of side 2L
(3) 6.56 cm (4) 7.67 cm. (See fig.). The electric potential at point A,
midway between the two charges +q and +q, is
Q.7 A simple pendulum has a period T on the earth. +q..---------.-q
What is the period T' of this same pendulum on
the moon, where the acceleration due to gravity
is 1/6 that of the earth? A
T
(I) - (2) ! (3) .J6T (4) 36T
../6 6
+q--------q

Q.8 The only force acting on a block is along x-axis (1) _1_2q(l+f5)
4r.8 0 L
is given by F =-(~1 +2
)·N. When the block
(2) _·1_2q(l+-1)
X

moves from x = - 2 m to x = 4 m, the change in


41ts0 L l ..fs
kinetic energy of block is -
(3) _1_2q(1--l)
(1) Positive 4n:c 0 L Js
(2) Negative
(4) zero
(3) Zero
(4) May be positive or negative
Q.13 The effective capacity between A and B in
figure will be :
Q.9 An aeroplane is flying in a horizontal circle at a
2µF
speed of 540 km/h. Banked for this turn, the
wings of the plane are tilted at an angle 45° :-·----,F.,---lµ_F_~2µF
from the horizontal. Assume that a lift force
acting perpendicular to the wings holds the 12µF
'
aircraft in the sky. The radius of the circle in
which the plane is flying is (Take g = l 0 m/s2) (!) 0.5 µF (2) 1.5 µF
(1) 1000 m (2) 2250 m
(3) 2 µF (4) 2.5 µF
(3) 500 m (4) 4500 m

www.puucho.com
CP Publication KVPY chapterwise with www.puucho.com

• PRACTICE TEST-3

Q.14 Figure shows four plates each of area A and Q,18 Let E and B denote the electric and magnetic
separated from another by a distanced. What is fields in a certain region of space. A proton
the capacitance between P and Q? moving with a velocity v along a straight line
enters the region and is found to pass through it
undeflected. Indicate which of the following
_P- (_ _ )>-------'Q statements arc consistent with the_
observations?
(I) A
i;: 0 (2) 2 EoA (3) 3 P.oA (4) 4 soA
-
Ci) E = o (ii) E "= o (iii) E 1"0 (iv) E , v
-
d d d d (1) (iii, iv) (2) (i, ii, iv)
(3) (i, ii, iii) (4) (ii, iv)
Q.15 In the adjoining figure, the emf of the cell is
1.8 V and the internal resistance is (2/3) Q. The Q.19 If the ratio of tho concentration of electrons to
current in 3 .Q resistance is : that of holes in a semiconductor is J_ and the
5
3Q
ratio of currents is J_, then what is the ratio of
4
their drift velocities?

60 (I)~ (3) ~ (4) i


8 4 7

Q,20 If the lattice constant of this semiconductor is


E= 1.8V decreased; then which of the following is
r= (2/3) Q correct?
(I) 0.4 A (2) 0.8 A Conduction ....................................................,..................-
(3) 0.2 A (4) 0.1 A band width - E!Ee
band gap EJ
Q.16 The current through the circuit shown in figure
is 1 A. If each of the 4.Qresistor is replaced by
Valence
band width
#J1$fff/#J1///i;1$$;7 l&,
2Qresistor, the current in circuit will become (1) All Ee, Eg, & increase
nearly: (2) Ee and Ev increase, but Eg decreases
(3) Ee and E,, decrease, but E8 increases
(4) All E0, Es, & decrease

CHEMISTRY
A Q,21 Select correct statement about hydrolysis of
-----1;'1-----
BCh and NCI 3 :
(I) l.l l A (2) 1.25 A (I) NCb is hydrolysed and gives HOC! but
(3) 1.34 A (4) 1.68 A BCh is not hydrolysed.
(2) Both NCh and BCb on hydrolysis gives
Q.17 A particle of mass m1 charge Q and kinetic HCL
energy T enters a transverse uniform magnetic (3) NC'3 on hydrolysis gives HOC! but BCI.,
gives HCL
field of induction B. After 3 seconds the (4) Both NCb and BC'3 on hydrolysis gives
kinetic energy of the particle will be: HOCl.
(1) T (2)4T (3) 3T (4) 2T

www.puucho.com
CP Publication KVPY chapterwise with www.puucho.com

KVPY Model Test Papers


The geometry with respect to the central atom Q.28 The concentration of a substance undergoing a

of the following molecules are : chemical reaction becomes one-half of its
N(SiH3)J ; Me3N ; (SiH1)3P original value after time t regardless of the
(1) Pyramidal, Pyramidal, Planar initial concentration. The reaction 1s an
(2) Planar, Pyramidal, Planar example of a-
(3) Pyramidal, Pyramidal, Pyramidal ( I) zero order reaction
(4) Pyramid.al, Planar, Pyramidal
{2) first order reaction
(3) second order reaction
Q.23 A complex has a composition corresponding to
the formula CoBr2Cl. 4NH3 . What is the (4) third order reaction
structural formula if com.luc!ance
measuremcnL~ show two ions per formula unit? Q.29 In the nuclear reaction ~ 4 Th ~~Pa+ X. X
Silver nitrate solution givci;; an immediate is-
precipitate of AgCl but no AgBr.
(1) [CoBrCl(NH1)4] Br ( l) -~ e (2) fe
(2) [C0Cl(NH3)4] Br2 (3) H (4) ~H
(3) [C0Br2Cl (NH3)4]
(4) [C0Br2(NH1)4]Cl
Q.30 Zinc sulphate contains 22.65% of zinc and 43 .9
Q.24 In [Pt(NH3)2Ch], Pt-Cl bond length is 2A and % of water of crystallization. If the law of
Cl-Cl distance is 2.88 A then the compound is constant proportions is true, then the weight of
( 1) tettahedral (2) square pyramidal zinc required to produce 20 g of the crystal will
(3) cis-squarc planar (4) trans-square planar be-
(1) 45.3 g (2) 4.53 g
Q.25 When ZnS and PbS minerals arc present (3) 0.453 g (4) 453 g
together, then NaCN is added to separate them
in the froth floatation process as a depressant, Q.31 ln what ratio by mass carbon monoxide and
because : nitrogen should be mixed so that partial
(1) Pb(CN)i is precipitated while no effect on
pressure exerted by each gas is same?
ZnS
(I) l : I (2) I : 2
(2) ZnS forms soluble complex Na2[Zn(CN)4]
(3)2:l (4)3:4
(3) PbS forms soluble complex Na2[Pb(CN)4]
(4) They cannot be separated by adding NaCN
Q.32 Water and chlorobenzene are immiscible
Q.26 The electrolysis of pure alumina is not feasible liquids. Their mixture boils at 90° C under a
because: reauced pressure of9.58 >< 104 Pa. The vapour
(1) it is bad conductor of electricity and its pressure of pure water at 90°G is 7.03 x I 04 Pa
fusion temperature is high and molecular weight of chlorobenzene is
(2) it is volatile in nature 112.5. On what precent basis, chlorobenzene ·
in the distillate is equal to-
(3) it is decomposed when fused
( I) 50 (2) 60
(4) it is amphoteric
(3) 70 (4) 80
Q.27 Which of the following is correct statement?
(I) F2 has higher dissociation energy than Ch
Q.33 6.Cp for a reaction is given by 2.0 + 02 T cal/deg.
Its enthalpy of reaction at 10 K is -14.2 kcal.
(2) F has higher electron affinity than Cl
Its enthalpy of reaction at 100 Kin kcal will be
(3) HF is stronger acid than HCl
(l)-13.03 (2)-15.37
(4) Boiling point increase down the group in
(3) 16.02 (4) 07.08
halogens

www.puucho.com
CP Publication KVPY chapterwise with www.puucho.com

t PRACTICE TEST~3
Q.34 . Consider the following reactions :

0
Reaction I:
0 0 0
Q.37 Compound
m •
O=C....Cl 0

on reduction with
H3~ ~ C F3~Jc,:i,OH I H3 c.,Aol\ + F3C)\0II
0

R~action II:
0 0 0 0

HP/"'·~CF3 cHJ~~,12011 H:icAoH +F3C Ao/\


Which of the following is a correct comparison
of the rate ofreactions I and 1I?
(1) r1 = ru (2) r, > rn (3) ri < ru (4) None

Q.35 Consider the given reaction


11
I Q.38 Decreasing order of acidic strength of different
CH3-(CH2).s-T-OH ~~· 2-chlorooctane groups is-
CH3
(S)
Which is the following statement is true for the
CHO
(w)
above reaction?
(1) Configuration of the product is S NH2)
(z)
(2) Configuration of the product is R
(3) Product is 1 : 1 racemic mixture
(4) Product is 2 : l raccmate (1) w> X > y> Z (2) w>z>x >y
(3) Z > W> X > y (4) z > x> w>y
Q.36 Consider the following reactions involving
Williamson ether synthesis :
Q,39 Identify the major product:
~J NC2H5
(!) l.._) +;H,cH,B,-v
Br
H
I ee
~ CH3
(2) CH3-T-ONa + lSJ)~-?-CH,
CH3
CHl g CH3
CH3

I ee I
(3) CH:i-C-ONa + CHrBr-> CH:,-(,°:·-0 CH 3 (I) (2)
I I
(;H3 CH 3
Br
El EB !
( 4) CH3-CH2-CH20Na + CH3-CH2T-CH3

! CH
I .1
CH3
(3) (4)
CHr-CHr-CH2 ---0--C-CHJ
!
CH3
Which of the above reactions constitute good
synthesis?

www.puucho.com
CP Publication KVPY chapterwise with www.puucho.com

KVPY Model Test Papers


CH2-0H Q.43 If J (x) = 2e"' - c In x monotonically increases •
Et ~ for every x E (0, o:J), then the true set of values
of c is -
Q.40
N CH2-C-C) (!)(major)

I
(1) (-oo, -} ) (2) (-oo, -})
H
{3) [O, oo) (4) (-oo, OJ

Q.44 Two mutually perpendicular tangents of the


parabola / = 4ax meet the axis in P1 and P2• If S
(1)
is the focus of the parabola, then _l_ + _I_
N (SP1) (SP2 )
I is equal to -
H
~CI!~-OII
(1) .±a (2) 3.
a
(3) .!.
a
(4) _1
4a
(2) ~ .. ) -,~ Et
N Q.45 f
Range of y = cos ( 7t sin( cos( 7t sin x))) ,
I
C-Cl'3 where '.I' e R, is -
II (1) [-7t, 7t] (2) [-1, 1]
0 (3) [O, I] (4) [-I, O]
(YClh
Q.46 A circle C1 is drawn having any point P on
(3)~ )
x-axis as its centre and passing through the
N centre of circle C: x2 + y2 = 1. A common
I tangent to Ci and C touches circles at Q and R
H
respectively. Then Q (x, y) also satisfie.s -

a
0
(l)x 2 =1 (2)x2 +y2=I
II
C-CH 3 (3) y2- l= o (4) None of these

(4) Et

N
I
Q.47 The value of ,ti!;, {[x +;)e! Z- x} ~ equal to
H (where (.} denotes fractional part of function) :
(1) 0 (2) 1
MATHEMATICS (3) 1/2 (4) does not exist
Q.41 What is the function whose graph is the
reflection about the line x + y = 0 of the inverse Q.48 Six people, all of different weights, are trying
function C 1(x) ofa function f(x): to build a human pyramid, that is they get into
(1)-f(x) (2)-f(-x) the formation
(3)-r- 1(x) (4) -r- 1 (-x) A
B C
Q.42 If the chords of contact of tangents from two
points (-4, 2) and (2, 1) to the hyperbola D E F
x2 Y2 We say that some one not in the bottom row is
- - - = 1 arc at right angle, then the "supported by" each of the two closest people
al b2
beneath her or him. The number of possible
eccentricity of the hyperbola is -
different pyramids, if nobody can be supported
.fi
~3fI ~2fI ,fi
(I) (2) (3) (4) by anybody of lower weight, are-
2 (l) 24 (2) 18 (3) 16 (4) 14

www.puucho.com
CP Publication KVPY chapterwise with www.puucho.com

• PRACTICE TEST-3

(x+2)3, -3<x5-l Q.56 Suppmc that 'a' and 'b' are two roots of the
Q.49 Let f(x) = { -~ and • 2 a+2
equation x + 3x + 5 = 0. If t = - - and
x3, -1<x<2 b+2
X b+2 1
s = - - arc the two roots of x · - mx + 1 = 0,
g(x) = Jf(t)dt, - 3 < x. < 2. a+2
-3 then the value of'm' is -
Then the number of extremum points of g'(x) is
( l) -5/2 (2) --5/3 (3) 5/2 (4) 5/3
(1) 0 (2) l (3) 2 (4) 3

Q.57 Iff (x) = ae2~ + bex + ex satisfies the conditions


Q.50 If f(x) is continuous and increasing function
such that domain of g(x) = .Jr(x)- x be Rand f
fo4
J(0) = -1.f '(log 2) = 31 & (f (x)-cx) dx
0
39
=-=-,
2
h(x) = - 1-, then the domain of
1-x then-
cp(x) = .Jf(f (f(x)))- h(h(h(x))) is - (1) a=4 (2) b =-6
(3) C = 2 (4) 11 =3
(1) R (2) {O, l}
(3)R- {O, l} (4)R-(O, 1)
Q.58 If a, b, c are first three terms of a GP, the
Q.51 The probability that the length of a randomly harmonic mean of a and b is 12, and arithmetic
chosen chord of a circle lies between 2/3 and mean of band c is 3, then -
5/6 of its diameter is - ( l) no term of this GP is square of an integer
(1) 1/4 (2) 5/12 (3) l/16 (4) 5/16 (2) arithmetic mean of a, b, c is 7
(3) b =± 6
Q.52 The sum of all the roots of the equation (4) common ratio of this GP is 2
sin(1dog{;)) = 0 in (0, 21t), is - Q.59 Let a and fj be the solutions of the quadratic
(I) 3/2 (2) 4 (3) 9/2 (4) 13/3 equation x 2 - 1154x + I = 0, then the value of

Q.53 Solution set of the equation


Va + ~ is equal to -
+ 32(x+6) = 0 is _ (I) 4 (2) 5 (3) 6 (4) 8
3 2x 1 _ z.3x'+x+G

(I) {-3,2} (2){6,-1}


Q.60 Let u = (log2 x }2 - 6 log 2 x + 12 where x is a real
(3) {-2, 3} (4) {1, ---6}
number. Then the equation xu = 256 has
a+2c 4 (1) no solution for x
Q.54 If a, b, c, d R such tbat - - + - = O,
E
(2) exactly one solution for x
h+3d 3
then the equation ax 3 + bx 2 +ex+ d = 0 has - (3) exactly two distinct solutions for x
(1) at least one root in (-1, 0) (4) exactly three distinct solutions for x
(2) at least one root in (0, l)
(3) no root in (-1, I)
(4) no root in (0, 2) BIOLOGY

Q.55 If matrix A = [a;jh• 3, matrix B= [b,:.;]JxJ, where


Q.61 Sexual reproduction leads to:
aii + aj, = 0 and bij- bj; = 0, then A 4• s3 is - ( 1) euploidy
(1) skew-symmetric matrix (2) polyploidy
(2) singular matrix (3) aneuploidy
(3) symmetric matrix (4) genetic recombination
(4) zero matrix

www.puucho.com
CP Publication KVPY chapterwise with www.puucho.com

KVPY Model Test Papers


Formation of an embryo from syncrgid or any Q.71

The organs which look_ different but have tl1e

other haploid cell refers to: same basic structure and origin are )cnown as :
( l} Parthenocarpy· (1) homologous (2) vestigial
(2) Parthenogenesis (3) heterologou.<; (4) analogous
(3) Apogainy
( 4) Both (2) and (3) correct Q.72 Which one of the following is not a vestigial
structure in Homo sapiens?
Q.63 The function ·ofvas deferens is to:
(I) Epiglottis (2} Third molar
(I) store the sperms
(3) Plica semilunaris (4) Pyramidalis muscle
(2) mature. the sperms ·
(3) conduct the sperms
Q.73 Cholera, leprosy and diphtheria arc : ·
(4) none of these
(1) viral diseases (2) fungal diseases
Q.64 Labium majora of a female mammal is (3) bacterial diseases (4) functional dis~ases
homologous to :
(I) scrotal sac (2) prostate gland Q.74 Most abundant_immunoglobulin is:
(3) cpididymis ( 4) seminal vesicle (1) IgG (2) IgD
(3) IgA (4) IgE
Q.65 Contraceptive oral pills used by females help in
birth control by : . Q.75 Inbreeding is possible between two members of
(1} killing of ova a:
(2) .killing of sperms (I) species (2) genus
(3} preventing ovulation {3) family (4) none of these
(4) forming barrier between sperms and ova
Q.76 The most important livest~ks of India are:
Q.66 Which of the following is the monohybrid (I) cattle and cat (2) cattle and elephant
ratio? (3) cattle and dog (4} cattle and buffalo
(1)3: I (2) 9: 7
(3) 1 : 2 (4) 9 : 3 : 3 : 1 Q.77 The technique which involves addition or
deletion of genes is:
Q.67 Mendel's law of independent assortment can be (1) gene therapy
dcmonstrai~d by: (2) gene splicing
(I) test cross (2) dihybrid cross (3) genetic engineering
(3) back cross (4) monohybrid cross ( 4) artificial synthesis

Q.68 W_hen closely placed genes on the sanie


Q.78 Manipulation of DNA in genetic engineering
chromosome are inherited together the
became possible due to the discovery of:
phenomenon is -known as :
(I) primase
( 1} linkage (2) crossing over
(3} gene interaction (4) multiple allelism (2) DNA ligase
(3) transcriptase
Q.69 In man, chromosomes arc : (4) restriction eudonuclease
(I) metacentric (2) acrocentric
(3) submctacentric (4) all ofthesc Q.79 Water pollution on the surface of pond can be
minimised by:
Q.70 Double helical model of DNA molecule was (1) Lichen (2) Hydri!la
proposed by: (3) Phytoplankton (4) Bacteria and Algae
(I) Erwin Chargaff
(2) Watson and Crick Q.80 Which is a degradable pollutant?
(3} Heinz Fraenkel Conrat (1) DDT (2) Mercury salts
( 4) Singer and Nicolson (3) Domestic wastes (4) Aluminium oil

www.puucho.com
CP Publication KVPY chapterwise with www.puucho.com

t PRACTICE TEST-3

PART-II
Questions given below are single correct type
Questions. Each correct answer carries +2.
0.5 marks will be deducted for each wrong answers.

PHYSICS
Q.81 A particle moves constantly in a circle centered
at the origin with a period T. lf iL<i position at (I)-128i+96]cm/s (2) -28i+48Jcm/s
time t = 0 seconds is (A, 0) meters, which (3) 128 i- 48] cm/s (4) 100 i + 48] cm/s
graph represents vx, the x-component of the
particle's velocity, as a function of time? Q.83 A nut is screwed onto a bolt with 12 turns per
I cm and diameter more than 1 cm. The bolt is
X I

(1)]
->. 0 --------r I
lying in a horizontal position. The nut spins at
216 rpm. Time taken by the nut to cover 3 cm
I
~ along the bolt is :
- - - - - - - -I, • T'1mc (l)lOs (2)12s (3)14s (4)16s
0 T
Q.84 A motor bike accelerates from rest at a constant
ii: rate of 3 ms-2 for some time and then moves
0 with unifonn velocity for the same duration.
(2) 'lj 0
0
Then it retards at a constant rate of 6 ms- 2 and
~ comes to rest. The bike was in motion for a
0 T period of 5 seconds. The total distance
travelled by the bike is ;
(l) 21 m (2) 24 m (3) 27 m (4) 30 m
~

(3) ·ob0 0 Q.85 Figure shows a sinusoidal wave of period T


-
~
travelling to the right along a string at time
t = 0. Which of the following statement is
0 T incorrect?

9
~
1
I
0
(4) ·g 0
? ..___ _ _ _ _ _,_~Time
0 T 5
(1) The point 3 on the string is moving upward
Q.82 Two plane mirrors are placed as shown in with maximum speed
figure. A point object O is approaching the (2) The point 5 on the string has the greatest
intersection point A of mirrors with a speed of upward acceleration
100 c:m/s. The velocity of image of the object (3) The point 9 on the string has the greatest
formed by M 2 with respect to velocity of image downward acceleration
of object formed by M I is ; (4) The point 2 on the string has a downward
velocity and upward acceleration

www.puucho.com
CP Publication KVPY chapterwise with www.puucho.com

KVPY Model·Test Papers


What is the minimum number of geostationary Q.90 A non-planar loop of conducting wire carrying

satellites required for uninterrupted global a current I is placed as shown in the figure.
coverage? Each of the straight sections of the loop of
(1)3 (2) 5 (3)7 (4) 9
length 2a. The magnetic field due to this loop
at the point P(a, 0, a) point,; in the direction:
Q.87 For sky wave propagation of 10 MHz signal,
what should be the miniIJmm electron density
in ionosphere ?
(I) 1.2 x 10 12 m-3 (2) 10 6 m-3
14
(3) 10 m- 3 (4) 1021 m-3

Q.88 One conducting U-tube can slide inside another ,.


j
.,. .,.
as shown in figure, maintaining electrical ~---------
contacts between the tubes, The magnetic field
B is perpendicular to the plane of the figure. If 2a
each tube moves towards the other at a constant l ~ • 1 • • •
speed v, then the emf induced in the circuit in (I) ../2(-j+k) (2) v'J(-j+k+i)
term<; ofB, land v, where l is the width of each
1 • • • 1 A •

tube, will be : (3) -JJ(i+ j+k) (4) h(i+k)


X X X
" "

CHEMISTRY
Q.91 SbF5 reacts with XcF4 to form an adduct. The
( 1) Blv (2) -Blv (3) zero (4) 2B!v shapes of cation and anion in the adduct are
respectively:
Q.89 The current/ in an inductance coil varies with (l) square planar, trigonal bipyramidal
time t according to the graph shown in fil::,'1ll"e. (2) T-shaped, octahedral
Which one of the following plots shpws the (3) square pyramidal, octahedral
variation of voltage in the coil with time? (4) square planar, octahedral
I
Q.92 Which of the following statements are correct
about Zn, Cd and Hg ?
(I) they exhibit high enthalpies of atomization

(ll+b., (2):~
as the d-subshell is full
(II) Zn and Cd do not show variable oxidation
states while Hg shows + I and + II
(III) Compound of Zn, Cd and I-lg are

(J)p= (4)Fv
paramagnetic in nature
(IV) Zn, Cd and Hg are called soft metals
(1) I, II, III (2) I, ID
(3) II, IV (4) IV only

www.puucho.com
CP Publication KVPY chapterwise with www.puucho.com

t PRACTICE TEST-3
Q.93 Silicate having one monovalent corner oxygen OH
atom in each tetrahedron unit is : I
r"(CHNJI2
(]) sheet silicate (I) NaRH4(l Eq), ~
(2) cyclic silicate
(3) single chain silicate
(4) double chain silicate
(2) Ni-112, in ethanol (1 Eq), 0 CI-hOH

r"(CHOH
Q.94 The solubility product ofMg(OH)2 is 1.0 x 10- 12• (3) LiAlH 4 (1 Eq), l_,jJH2
Concentrated aqueous No.OH solution is added to
r"(CH2NI·h
a 0.01 M aqueous solution of MgCli. The pH
( 4) Pd-H2 (1 Eq), ~ DH
at which precipitation occur is-
(1) 7.2 (2) 7.8 (3) 8.0 (4) 9.0
Q.99 Analyse the given information:
2,4-dinitro Fehling's lodoform Tollen's
Q.95 A metal with an atomic radius of 141.4 pm
Phenyl solution test test
crystallizes in the face centred cubic structure.
hydrazine test
The volume of the unit cell in pm3 is- (A) turns red -vc +vc +ve
(1) 2.74 X 107 (2) 2.}9 X 107 (B) turns red +vc +vc +ve
(3) 6_40 X J0 7 (4) 9.20 X 107 (C) turns red -ve - ve +ve

o~o
and consider the following molecules ;
Q.96 A saturated solution of H2S in 0.1 M HCI at
25°C contains a s2- ion concentration of
~H
10-23 mol 1- 1. The solubility products of some
sulphides are : CuS = 10--44, FcS = 10- 14,
MnS = 10-15, CdS = 10-25• If 0.01 M solutions
of these salts in 0.1 M HCl are saturated with JI
H2S, which of these will be precipitated? CH=O
(1) All
_ (2) All except MnS

Q.97
(3) All except r..1nS and FcS
(4) Only CuS

Given standard electrode potentials


Fe 2++ 2e- ---+Fe; E° = -0.44 V
Om
on

(1) A-IV; B-lll; C-1


20 IV
(2) A-IV; B-III; C-ll
Fe3+ + 3e- ---+Fe; E0 = -0.036 V (3) A-III; B-lV; C-11 (4) A-III; B-IV; C-1
The standard electrode potential
J+ 2+-
(E") for Fe + e ---+ Fe ts-
(1) - 0.476 V (2) --- 0.404 V 0
II
(3) + 0.404 V (4) + 0.772 V Q.100 Ph-----C---Cl H, > Ph-CHO HCHO (A)+ (B).
Pd-ll.'.ISO• "'-.KOH

Product (A) & (B) arc -


Q.98
(1) Ph-C02H, Ph-OH
(2) Ph-CO 2 -, HC02-
HCN LAI!
Q, (3) Ph-CH 20H, II-C02-
I!"
(4) Ph-CO2-, CH 30H
P and Qare-

www.puucho.com
CP Publication KVPY chapterwise with www.puucho.com

KVPY Model Test Papers



MATHEMATICS Q.106 I,II c2012)2x
V l -(2012)ix
(2012)"iii
!
t2012i· dx =
Q.101 The numbers 1, 2, 3, ...... , n are arrange in
random order. The probability that the digits 1, ( 1) (I og2012 c) (2012) sin 1(20 l2)' +C
2, 3, .... , k (k < n) appear as neighbours in that
(2) (log2012 e/ (2012) x+sin-:(2012f +C
order is-
(1) 1/n! (2) k!/n! (3) (log2012 e)2 (2012)sin-1(2012)' + C
(3)(n-k)!/n! (4) (n -k + 1)!/n! (20 l2)'in-l (2012)'
(4) 2 +C
(log2012 e)
Q.102 ff l{x) satisfies x + j f(x) I = 2f(x) then r1(x)
satisfies
Q.107 Ifthe slope ofchord PQ of f(x) = x3 -2x 3 + 10 is
(I) 3x + I r\x) I= 2r1(x)
9, then relation between the AM(A) and
(2) x + I I 1(x) I= 2f 1(x) GM(G) of abscissae of points P and Q is-
(3) f" 1(x)- Ix I~ 2x (1) 9G 2 -(7A2 - G 2) (G 6 + 2) = 0
(4) Jx - I f\x) I= 2r 1(x) where f(x) 2': 0 (2) 6G 6 -(7A2 - G 2) (G 6 + 2) = O
(3) 9G6 -(4A2 - G2) (G 6 + 2) = 0
Q.103 A function y = f(x) satisfies the condition (4) 6G6 -(4A2·- G2) (Ci+ 2) = 0
f'(x) sin x + f(x.) cos x = I, f{x.) being bounded

whenx > O. ffl=


rr/2
Jf(x)dx, then
0
Q.108 Letf (x) = [ co{ t 2 + :t + )dt,
1 O < x < 2.

re n2 1t 1t2 Thcnf(x)
(l)-<I<- (2) -<l< - (]) increases monotonically
2 4 4 2
(2) decreases monotonically
1t
(3) 1 <I<- (4)0<1<1 (3) has one point oflocal maximum
2 (4) ha<; one point oflocal minimum

Q.104 If A, 11 & C arc matrices of order 2 such that Q.109 The value of the sum
1,11 = _!_, !Bl = 5, ICI = 3 then !(SAC) (2BT I I is -1- + -1- 1
·I· - - + -1- _,_ oo
8
32 +1 4 2 +2 52 +3 6 2 +4 ' .... ,
equal to:
is equal to -
(1) ~ c2) I4 (3) 2 (4) ~
16 8 32 (1) Q (2) 12_
36 36
Q.105 A lamp of negligible height is placed on the (3) 12. (4) .!!
ground A1 m away from a wall. A man ). 2 m tall 36 36

is walking at a speed of ~ m/s from the lamp Q.U0 lff (x) = Lt]+ [x + 1/3] + [x + 2/3], then([.]
IO
to the nearest point on the wall. When he is denotes the greatest integer function)
midway between the lamp and the wall, the rate (1)/(x) is continuous atx = I, 10, 15
of change in the length of this shadow on the (2) f (,) is continuous at x = n/3, where n is any
wall is: integer
2f2 213
(1)- 5£2 mis
2
(2)--m/s
5 (3) Jf(x)dx = 1/3
0
f.2
(3)- il_m/s (4)- -mis
2 5 (4) lim/(x)=2
., ....2/J

www.puucho.com
CP Publication KVPY chapterwise with www.puucho.com

• PRACTICE TEST•3
Q.116 In DNA fingerprinting which of the following
BIOLOGY is true?
Q.111 Allium type bisporic type fomale gametophyte (I) House keeping or luxury genes are uses as
is always: probes
(I) 8-nucleate and ?-celled (2) Specific metabolic genes are used as probes
(2) 16-nucleate and 13-called (3) VNTR is used as probes
(4) All of the above
(3) 8-nucleate and 8-celled
(4) 4-nucleate and 4-cellcd
Q.117 Among the human ancestors the brain size was
more than 1000 cc in :
Q.112 Withdrawal of which of ·the following- (1) Ramapithecus
hormones is tbe immediate cause of (2) Homo habHis
menstruation? (3) Homo ercctus
(1) FSH (2) FSH-RH (4) Homo neanderthalensis
(3) Estrogen (4) Progesterone
Q.118 What are true of plasmids ?
Q.113 In human being; brown eye (B) is dominant ( I ) They are found in viruses
over blue eye (b). A brown-eyed couple has a (2) They are main parts of chromosomes
blue-eyed child. What is the possible genotypes (3) They are widely used in gene transfer
of the couple? (4) They contain gene for vital activities
(I) Bb X bb (2) Bb x bB
(3)BB x bb (4) BB x bB Q.119 Which of the following toxic materials was
present in Mina mata bay of Japan?
Q.114 Radioactive thymine when added to the (1) Hg (2) Mg
medium surrounding living mammalian cell (3) Cd (4) Pb
gets incorporated into a newly synthesized
DNA Which of the following type of Q.120 "itai-itai" ("ouch-ouch") disease is caused by
chromatin is ex:pectcd to be radioactive if cells the poisoning of:
are exposed to radioactive thymidine as soon as (1) Lead (2) Mercury
they enter the S-phase? (3) Chromium (4) Cadmium
( 1) Euchromatin
(2) Heterochromatin
(3) Both euchromatin and heterochromatin
(4) Neither euchromatin or heterochrornatin

Q.115 Syndrome in human in which an individual


somatic eel! contain only one sex chromosome
XO is called :
(I) Patau syndrome
(2) Down syndrome
(3) Turner syndrome
(4) Klinefelter syndrome

www.puucho.com
CP Publication KVPY chapterwise with www.puucho.com

KVPY Model Test Papers



ANSWERS
..
Qus. 1 2 3 4- 5 ,6 7 8 ,9 10 11 12 13 14 15 16 17 18 19 -20
Ans. 2· 2 3 1 3 4 3 2 2 4 3 3 3 2 1 1 1 3 3 3
Qus. ·21 22 23 24 25 26 27 28 29 30 31 32 33 34 35 36 . 37 38 39 40

Ans. 3 2 4 3 2 1 4 2 1 2 1 3 1 3 1 3 4 1 1 2
Qus. 41 42 43 44 45' 46 47 48 49 50 51 52 53 54 55 56 57 58 59 60
Ans. 2 3 4 3 2 1 1 3 3 3 1 3 3 2 2 2 2 1 3 2
Qus. 61 62 63 64 65 66 67 68 69 70 71 72 73 74 75 76 77 78 79 80
Ans. 4 4 3 1 3 1 2 1 4 2 1 1 3 1 1 4 1 4 4 3
Qus. 81 82 83 84 85 86 87 88 89 90 91 92 93 94 95 96 97 98 99 100

Ans. 4 1 1 1 4 1 1 4 3 4 2 3 1 4 3 3 4 4 1 3
Qus. 101 102 103 104 105 106 ~107 108 109 110 111 112 113 114 115 116 117 118 119 120

Ans. 4 4 1 4 2 3 3 3 1 3 1 4 2 1 3 3 4 3 1 4

PART-I 2. The key to finding the coefficient of friction


One Mark Questions µ is in calculating the correct normal force
acting on the block
PHYSICS
:EF/= ma
1. The moment of inertia for the system can be
- Farptied-y - Fg + Fnormal = 0
calculated by adding the two individual
Fnnrma1 = F sin lj, + mg
moments of inertia as following :
µ =
Ffii·.
______f!!Q!!_ =- -f- -
Ffl<lml.al F sin 4> + mg

3. The diagram suggests that the box is currently


moving to the right, and in the process of
slowing down due to a force o.f friction that is

(-f5. 2L) ML2 -


2 2 causing it to accelerate in the opposite direction
lc1ay= M =-
(slowing down the box).

MT} 4. By definition, the object is in equilibrium,


1!OHIJ
..,=r.+--
J{I 2
either static (unmoving) or dynamic (moving
with a constant velocity).

www.puucho.com
CP Publication KVPY chapterwise with www.puucho.com

t PRACTICE TEST•3
If the object has acceleration a = 0, the net force
acting on the mass must be 0 as well :
F1
d ={ 1 -tJ =3(1- / 5) = 1 Cfl\
so that apparent distance of the object= 20 cm
(i.e.) u = 20 cm
By the mirror fonnula .!.y + .!.u = .!..f
20
v = -3 cm = 6.67 cm
Fnet = ma
Fne1 = m(O) = 0 :. distance of the final image from the mirror
with forces F1 and F2 in the x-y plane, the force = 6.67 + I ""7.67 cm.
that will counteract them must lie in the x-y
plane as well, as shown. The magnitude of that 7, The length of the pendulum hasn't changed and
force F3 is equal to the vector sum of F 1 and F 2 the only other factor that determines the period
and can be calculated as follows: of a pendulum is the acceleration due to gravity

LFx = O=F2 -~-x T 1=2rr./-f


2/r ""o =F, -F3_r
!F3 j = ~Ff_x + Fi-~ T'=2· I'6g;
= ~F12 +Fi
= F1-./2
T'= J6[ 2,Jfl
both graphically and analytically, we can see
that the magnitude of F3 is not the same as that
r=../6T
of Fi.
8, As w .. Af<.
Force is along negative x-axis and displacement
5. Mass of the cube= 30 g is along + x-axis
length of each face of the cube = 4 cm W = negative
kt y be the distance of the lower face of the Hence
cube from the surface of water. dK.=negative
When the cube floats freely in water
a2y p g = mg + 4a o Li ft force F =mg
9.
. _ mg+4acr cos45°
.. y- 2 The centripetal force is provided by the
a pg
component of weight
30x 980 +4 x 4x70
=-------
4x 4xl x 980
29400+1120
15680
= 30520 =l. 95 cm
15680
mv2
Fcos45°= - -
r
6. u=21cm· f=R=~ =5cm
' 2 2 2
, 2 5)
( 540x-
on introducing the glass slab, the object as well mv- v 18
as the image will be shifted from the mirror r =-- = - =-'-----==-- = 2250 m
mg g 10
through a distance

www.puucho.com
CP Publication KVPY chapterwise with www.puucho.com

KVPY Model Test Papers

18.

IO.

19. I= ncAvtl
104= E_ 800
Ii n1 vJ1 vtl, 11 n2
2 2 X 10- 3 X 0.4 -=-X-~-=-X-
12 n2 vdi vdi l2 n1
~p=20
The highest harmonic that could be heard CHEMISTRY
=20

21. NCl 3 + 3lhO ~ NH 3 + 3HOCl


2kpr
11. Ea.<is = (r 2 -a 2)112 BCh + 31IiO ~ 3I-1Cl + H38Q3

r=20+5 22.
a=5 cm SiH 3
"-·N =-=SiH3
12. z(Kq_ Kq) H3Si
,yspz

L ,./sL
prt <ln; No
back bonding back bonding
13.
spJ
I'
, H3s·/
I
I "'-
SiH3 SiH3

Nu
back bonding

14.
Cl~A
24. ./2 ~-4~ - Pt
Fz:
Cl
C =2C= 2Eo A
net d
28. First order
15. 0.693
t112= - -
3Q 2x4 4
- k
--=-
2+4 3

6Q =G/3 I=/\
1.8
-+-
3 3
29. Z=90-91 =-1
A=234-234
=O

=0.9
1.8, 2/3 30. To prepare 20 g of the crystals,
. diVI'dc d m
. ratio
. -I : -I I .
zmc require
. d=-
22.65
- x 20
0.9 1s
3 4 6 100
=4.53 g
17. K.E. is constant in magnetic field

www.puucho.com
CP Publication KVPY chapterwise with www.puucho.com

• PRACTICE Tesr-3
w = [2.0 X 100 + 0.1 X (I 00}2] - [2'.Q + 0.1 X 102]
31, CO = w 1g = - 1 mol
28 ' = 1170 cal
= l.17 kcal
.lli2 = Mi 1 + 1.17 kcal
=-14.2 + 1.17
= -13.03 kcal

•u.,;:,#&ts;nt•.
41.
Similarly PN = w2 y
2 w I +w 7.
As p=PN2,
Hence w1 = w2 i.e. wi : w2
I:1

32. In steam distillation


weight of liquid distilled = P"1 xM1
y=-x
weight of water distilled P0 wxMw
= (9.58 x 10-4 - 7.03 x I04 ) x l 12.5
42. Chord of contact w.r. t. (-4, 2) is - 4 ~ - 2 ; = I
7.03xl0 4 xl8 a b
2.27 -2b 2
=-- Slope m1 = - , - ... (1)
a~
i.e. in a total weight of 3.37 g, weight of
chlorobenzene = 2.27 g
Chord of contact w.r.t. (2, l) is 2~ -
a.
4b = I
2b2
:. % of chlorobenzcne = 2·27 x 100 Slope m,~ = -a2
3.37
=67.4 :. m1m2 =-1
=70% 4b 4
-=l
a4
33. By Kirchoffs equation, ==> 2b2 = a2
=> 2a!(e2-I) = a2
L\H2 - .rnl = LiCp
T2-T1 rf
T,
=)e=
v1-2
or L\H2 - L\H1 = JL\CPclT
T, 43. f(x) = 2e' -cfo x
IOOK
f'(x) = 2e' - ~~ 0
= f(2.0+ 0.2 T)dT X

!OK => 2xe' - c ~ 0


2 ]100K X
= [ 2.0T + 0.2.!.... ==>cs 2xe'
Z HlK
=> csO
. 2 }00K
= [2.0T +O.lT .hoK

www.puucho.com
CP Publication KVPY chapterwise with www.puucho.com

KVPY Model Test Papers

44. 'SP,= a(l + tf ); SP2 = a(l + t~) 48. Without any loss of generality assume the wts

to be 1, 2, 3, 4, 5, 6.
t1t2=-l
It is obvious that I should be at the top of
pyramid.
If2, 3 make second row then
1
2 3 --------~ ! ::::> 12 ways
4 5 6 --------'3 !
X
· if 2, 4 make second row
I
2 4 -------2 :::> 4 ways
3 - 5 6 ------2
Total number of ways= 16
1 I
. ,· SP1 a(1 + t2 ) ' SP2 49. g'(x) = f(x)
.,. 1 J l
:. - + - = - y
SP, SP2 a

45. - - '1E S 1t sinx .$ 1t


::::>- l s; cos (11: sin x) s; 1 X

- ::::>- _I f x))) s; 1
s; sin ( (cos{n:sin
so two poinl<; of extrema at x = -1 and x = O
,::::>-'lE s; xsin (~cos(1t~in x) Js; 1t
1
50. h(x) = - - , x :to I
::::>-1 sys 1 l-x
x-1
b(b(x))= -,xcJ:-0, I-
46. Clearly tangent xx.1 + YY1 - a. (x + xi) = 0 at x
Q(x1,Y1) on the ~ircle ::::> h{h{h(x))) = X, X ¢ 0, 1
(x - r,.)2 + y2 - a:1 = 0 is also tangent to
g(x) ~ 0 Y x E R ::::> f{x) ~ x
::::> f(f(x)) ~ f(x) ~ x (as f(x) is an increasing
i+y2.:,,.l=O
function)
:::} IUX1 I - ::::> f(f(f{x))) ~ f(fl:x)) ~f(x) ~ x
Jcx1-o/+yf
::::> f{f{f(x))) -x ~ 0 for x E R-{O, l}

51.
- 1 1· {( 1 \.. 1 l}
47. X= t I~ t+tr -t

. e 1 + 2te1 + t 2 e 1
l1m ----- 1 (usingL' Hospital rule)
1-+0 I
{l} = 0

www.puucho.com
CP Publication KVPY chapterwise with www.puucho.com

• PRACTICE TEST-3
a + b = - 3; ab = 5; s + t = m
J~r2 _.!.!_r2) 56.
Pr b = "l9 36
0 • 2 s=--·t=--·
b+2 a+2
,rr a+2' b·+ 2'
20-11 1 s+t= (b+2)2+(a+2)2
=-~=-
36 4 (a+2)(b+2)
= (a 2 +b 2 )+4(a+b)+8
ab+2(a+b)+4
52. 1t lo~ ( ~) = Jm, k E /
m= (a +. b ) + 4(a +b) + 8
2 2

ab+ 2(a +b)+4


log{~)= kc:::;,x= rt- 5
s+t=- - =m
3
Possible values of k are - l, 0, 1, 2, 3, .....

57. f (x) = ae2r +be"+ ex


Since,j(O) = a + b
=4 + (l/3) i.e., a + h = - 1 ....(i)
1-(1/3) f'(x) = 2ae2r +be"+ c
:. /'(log 2) = 2ae210&2 + be1ag1. + c
=4+ __I_= _2_ =Sa +ib+c
2 2
= Sa + 2b + c = 31 ....(ii)
ln4
53. (3x' _ 3(x-!·6))2 =0
f(ae 2~ +e" +cx-cx)dx
~x1·=x+6 0
(n4
6) = 0
::::;} (x2 -
~
X -

(x - 3) (x + 2) = 0
= J (ae2:r +be")dx .. ._.,
; ,

0 ..
,};11
-.t~
:::::>x= 3 or x=-2
aei:r
= [ -2-+be" o
)'"4
54. a+2c +i =O
ae 2/11 4 a
b+3d 3 = - - + bel114 - - - b
2 · 2
<=:> 3a + 4b + 6c + I2d= 0
a 15a
l b C
<=:> -a+ - + - +d=O
= Sa + 4b - - - b = - + 3b
4 3 2
2 2
39
Consider
2
ax 4 hx 3 cx 2
f(x)= - +- +- +d.t" i.e., 15a + 6h = 39 .... (iii)
4 3 2 from equation (i), (ii) & (iii)
Then,J(O) = 0 =/( 1) 9a=45
.. J (x) satisfies the conditions of Rolle's :. a = 5, b = - 6 and c = 3
theorem in [0, l].
Hcnce,f'(x) = 0 has at least one solution in (0, I). 58. a, b, c are in GP
2ab = l Z b + C = J
55. Since, matrix A is skew-symmetric. a+b ' 2
:. !Al= 0 2a.ar
- - = 12 ' ar+ar= 6
2

:. 1,'14.B31=0 a+ar

www.puucho.com
CP Publication KVPY chapterwise with www.puucho.com

KVPY Model Test Papers


ar = 6(1 + r), w-(1 + r) = 6 67. Independent assortment of character can be •
i.e., 6(1 + r}2 = 6 i.e., (1 + r/ = 1 shown only upon crossing 2 character i.e .
.-. l + r =± I dihybrid cross.
i.e., r= 0 or r=-2
.-. r = - 2 68. Linkage prevents crossing over and so
if r = - 2, then a = 3 separation of genes .
.-. The GP is 3, - 6, 12, -24, 48.
Clearly, no term of the series is square of an
71. Homologous organs arc the organ with above
integral.
feature.
AM of a, b, c is 3 - 6 +12 = 3.
3
72. Epiglottis covers glottis during eating food,
59. o. + ~ = l_ 154 and a.fl '""' 1 preventing its entry into respiratory tract, it is
not vestigial.
(J-;;:+.IP/=o.+13+2 N
= 1154 + 2 = 1156 = (34) 2
73. There arc caused by vibrio cholerae,
/;;., + .jp =34 mycobacterinm laprac & corynebacterium
Agaia, diptheriae respectively.
(0.114 + ~114)2 = .,rr;__ + Jp + 2(a.Pl4
= 34 + 2 = 36 74. lgG is the most abundcnt immunoglobulin in
0.114 + l=il/4 = 6 plasma.

76. Cattles & buffalo are most important sources of


60. ).:(log, -~l'. 61og 2 .,~ 12 = 256
diary products.
(l- 6t + 12)! = 8,
where t = log2 x 77. Gene therapy is used to replace the defective
::::> (t-2)3 =O gene by a normal gene.
::::> t = 2
78. Restriction endonuclease is key tool for genetic
::::> X =4
engineering.

BIOLOGY
61. Meiosis in vol vcs crossing over which forms
PART-.ll
basis of recombinations. Two Marks Questions
PHYSICS
62. Embryo formed from syncrgid or haploid cell
doesn't involve fusion of gametes and so 81. Based on the description of the particle's
formed sporophyte is haploid. position at time t = 0,
we know that the equation that describes the
63. Vas defercns transfer sperm~ from testis to the particle's x-coordinate as a function of time is
sits of storage i.e. seminal vesicle. X = A cos ( rot)
To determine the x-component of the velocity,
65. Contraceptive pills prevents ovulation, because
wc take the time derivative of this function :
they are synthetic derivative of estrogen &
dx" d
progesterone. Vx = -==-A cos (rot) =-roA sin (wt)
dt dt

www.puucho.com
CP Publication KVPY chapterwise with www.puucho.com

t PRACTICE TEST•3
Thls is essentially a negative sine function, LX1OR = 74" = 29
which matches the. graph in aru;wer 4. TilC Then V™ 2 =-vcos20l + vsin28]
problem may be solved conceptually, too, by
considering the x-component of the rotating
particle as a mass on a horizontally-stretched 83. Pitch of the screw= J_cm
spring. The mass is released from a position +A 12
at time t = 0, at which point it begins to number ofrevolutions made to cover
accelerate in the x-dircction, beginning a 3
3cm= --=36
simple harmonic oscillation. This motion is 1112
consistent with answer. spin frequency of the nut = 216 rpm
~ = 216Hz
82, The components of various velocities are as 60
shown in figure 216
angular speed <0 = 21tv = 21t x -
60
= 7.2 x rad.ls
0
ro= -
t
0 21t11 2xx36
::::}t= - = - = - - - = 10s
oo co 7.21t

84. (A) Inv= u + at or v = u- at, ifl a I is doubled,


the time tis halved
Hence, if t is the time for acceleration, then .!.
2
is the time taken for retarded motion/
:. total time duration for travel is
t 5t
YrM 2 is given by the vector sum of components t+t+ - = -
2 2
of velocity of image with respect to M2 along
the normal and perpendicular to the normal
.?.!2 =Ss (given)

VrM
2
=[lOOsin 2 37<=>f + 100sin37°cos 37°}] :. t= 2s
(i) Distance covered in accelerated. motion
· +[-l 00 cos 2 37°i + lOOsin 37°cos 37°]] I 2
S1 = ut + -at
= [-28 I+ 96J1 emfs 2
· vIMz,lMi =VIM2 -v!M1
'' =0 X X 3 X 22 = 6 ffi
t + _!_
2
= (-128i + 96j) cm/ s (ii) Distance covered in uniform motion
Aliter : For the reflection at mirror Mi the v= u+ at
incident ray OX 1s reflected along ""0+3x2
OR (ON - normal) = 6 ms- 1
.·. S2 = vt = 6 x 2 = 12 m
(iii) Distance travelled in retarded motion
V

x·-----
--+ t

www.puucho.com
CP Publication KVPY chapterwise with www.puucho.com

KVPY Model Test Papers



Vy and ay are positive when ~ < kx < 1t
2
=6 X } - .!.
X 6 X 12 = Jm Vy and ay are negative when 3n <; kx < 1t
2 2
:. total distance traversed (4) is incorrect.
S.,. S1 + S2 + S3 = 6 + 12 + 3 = 21 m
(N.B) this problem can also be solved by
drawing v-t graph
86.
85. Displacement y(x, t) = A cos (kx - rot)
At t = 0, point 1 on the !.1ring has maximum
displacement.
Velocity Vy (x} t) = roA sin (kx- rot) one satellite cover 120° angle for 360° 3
satellite

1 87.

88. Equivalent cell

At t = 0, vy is maximum at kx = ..::

==- x = -1tA- = -i. 1.c.,


2x21t
.

Hence ( 1) is1correct.
4
. 3
pomt
2

on the strmg.
. f
e =Blv+ Blv = 2Blv

Acceleration ay (x, t) = ro2A cos (kx- rot) 89. e=L~


At t = 0, ay i.~ maximum downward acceleration dt
e ::::} Rate of change of current
if cos kx = I ::::} kx = 21t ::::} x = )..
i.e., point 9 on the string has maximum 90. Magnetic field is perpendicular to plane of coil.

downward acceleration.
CHEMISTRY
Hence (3) is correct.
i,.. 91. ...
At t = 0 kx "-' -x x = - i.e., the string has
' ' 2
94. (0.01) X [OHl 2 = 1 X 10- 12 -.-
maximum upward acceleration. [OHl = 10- 10
Hence (2) is correct [OHl= 10-s
p0H=5
Velocity vy is positive and acceleration ay is
pH=9
negative when O< kx < ~
. 2
95. a= 4x 141.4 =400 pm
. negative
Vy 1s . and ay 1s
. pos1t1ve
.' 1t < kx < -31t ,,fi
2 V = (400)3 = 6.4 x 107 pm3

www.puucho.com
CP Publication KVPY chapterwise with www.puucho.com

• PRACTICE lEST•3
96. [S 2l= 10-23 mol L-1, [M2+J= 10-2 M.
Jonie product [M2J [S2-] = l 0-25_ Ionic product
If x = 0, y is finite
:. C=O
--
is greater than Ksp ofCuS and CdS. X
:. y = - -
sinx
7t2 7t
97. Fc2+ + 2e- 4 Fe, Now I< -and I> -
.6.G"1 = -nFE 0
' 4 2
1t 1t2
= -2F(---O .44) = 0.88 F ' ... (i) Hence, - <I< -
Fe3 + 3e- """'? Fe 2 4
ti.G"2 =-n FE0
= -3F(---0.036) = 0.108 F ... (ii) 104. I (SAC) (2 Bf'I
Eqn.(i) - Eqn.(ii) gives
= ISACI j2B-11
1 1
Fe+3 + e- 4 Fe..2, = 25 IAI ICI x ~ [Bl
AG = -nFE0 = -0.772 F
I I I
=25x-x3x-X-
or E0 = 0 ·772 =0.772 V 8 4 5
1 15
=-
MATHEMATICS 32

\
,
,.I
.
101. The number of ways of arranging n numbers is
n! 1n each order obtained, we must now arrange
the digits 1, 2, .... , k as group and the n - k
105.

'
remaining digits. This can be done in (n - k + I)!
,..I ways. Therefore, the probability for the
required event is (n - k + I)!/ n!

102. Let y = f(x)


+--ti~
given equation x + IY I= 2y
case (I) y :a:: 0 Let BP= x. From similar ti. 1s property, we get
X = y => f 1(x) =XV X ~ 0
AO=!_i
case (2) y < 0
l1 X
x=3y
:f 1(x) = 3x, x < 0 ==>AO= l1l2
... which satisfies in option (4). X
'
d(AO) _ -l 1t 2 dx h
103, ==>------ 2
wen
dt x dt '
y
rr./2 x = !.!_ d(AO) = _ 2l 2 mis.
2 ' dt 5

106. Let sin- 1(2012)" = t


--------" 1 x c2012y lo&:2012 c1x = dt
Since, sin x : + y cos x = l 231
~· - (2012)
dy + y cot x = cosec x
dx
;. J (2012) 231 • (Z01 2)5;n- 1(2()12)" dx
~l-(2012)2 31

IF= efcotx.ix= e!nfs:inx)=sinx


J
y sin x = cosccx. sin x dx
f
= (log2012c) (20l~f dt

y sin x = x + C

www.puucho.com
CP Publication KVPY chapterwise with www.puucho.com

KVPY Model Test Papers



(2012/
= lOS2012e x - - - + C
loge 2012
:tT = .!_ (.!__.!_)5 .!_ (.!. _.!_)
:. S =
q= 3 n 3 2
+
3 3 6
+

= (log2oi2e)2 x (2012)5in" +c
½(¼-¼)+½ (¼-½) ............... .
1
( 2012 )'

= .!_ [6 + 4+3] = _!l


Let y = ftx) = x3 - !2£3 + 10
107.
:, Yi -Y2 = (xi 3 - xl) + 2(x2_3: _ Xi-3) S= .!_
3 2
[.!.+ .!.+.!_]
3 4 3 12 36
=(xi3-xl)+2(xr-x~)
(X1X2) 2/3
{
2
110. JJ(x)dx }
=> Yi-Y 2 =(xi2+x22+x1x2)(1+ 3) 0
X1 -X2 (X1X2,)

_ [(x 1 + x 2) 2 -x 1x 2][(x 1x 2)3 + 2]


- (x1x2)3
T([x] +[ •½] +[x+ ¼])
x dx

213 2/3 213


= (4A 2 -G 2 )(G 6 +2)
Gs .
9 "" IOd.T fOdx Jtu
0
+
0
+
1/3
I

)
=> 9G6 -(4A2 - G2) (G6 + 2) = O 2 1 1
=---=-
.3 3 3

108, f'(x) = cos (x +


2
:x + 1) BIOLOGY

= cos ( (x ~ t}2 ) 111. Allium type is bispore type but 7 called and 8
nucleate.
Since,
112. Sudden fall in level of progesterone, due to
Q S: X S: 2, ~ S: (X -1- 1)2 S: 2. regression of corpus lutum results is breaking
5 5 5 of uterine wall, causes menstr~al flow.
(X + 1) 2
cos--'----'-- = 0 115. Turner syndrome results from monosomy of
5 ' I
sex chromosome which results from
(x + 1) 2 it chromosomal non disjunction during gamete
only when -'----"- = -
5 2 formation.
_)

f'(x) > 0, ifO <x< N- - l 117. Neanderthal man had cranial. capacity of about ..:
1400 cc.
f'(x)>O:ifN- -1<x<2
,. 119. Murcury poisioning in marine water caused
minamata disease in Japan.
:. x = N- - 1 is a po1nt of local maximum.

. I
109. T,,=-2---
n +(n-2)
1
= - - - - , n = 3, 4, 5, ....
(n + 2)(n -1)

- ½[-,1-~-l --11-!-2]

www.puucho.com
CP Publication KVPY chapterwise with www.puucho.com

Books Category Target Course Classes

• 15 Years Solved Papers JEE Main Class 11 , 12 & 12 Pass

• 25 Year Solved Papers NEET / Pre-Medical Class 11 , 12 & 12 Pass

• 40 Years Solved Papers JEE Advanced Class 11 , 12 & 12 Pass

• Revision Book NEET / Pre-Medical Class 11 , 12 & 12 Pass

• Daily Problem Practce Sheet (OPP) of PCMB JEE Main/JEE Advanced/NEET Class 11 , 12 & 12 Pass

• KVPY (Stream SA & SX) KCPY Exam Class 11 , 12 & 12 Pass

• Formula Handbook PCMB JEE Main/JEE Advanced/NEET Class 11, 12 & 12 Pass

• Comprehension Based Books JEE Advanced Class 11 , 12 & 12 Pass

• Problem Books (Exam Taiyari Series) JEE Advanced Class 11 , 12 & 12 Pass

• Class Notes Series PCMB JEE Main/JEE Advanced/NEET Class 11 , 12 & 12 Pass

• Study Material PAckage PCMB JEE Main/J EE Advanced/NEET Class 11 , 12 & 12 Pass

• Challenging Problem For Lakhshya Batch JEE Advanced Class 11, 12 & 12 Pass

• Ulnt Wise Practice Test Papers JEE Main/JEE Advanced/NEET Class 11 , 12 & 12 Pass

• 10 Full Syllabus Mock Tests JEE Main/JEE Advanced/NEET Class 11 , 12 & 12 Pass

• BITSAT Rank Booster BITSAT Class 11 , 12 & 12 Pass

• 10 Full Syllabus Mock Tests For A IIMS AIIMS Class 11 , 12 & 12 Pass

• Ranker PAckage JEE Main/JEE Advanced/NEET Class 11 , 12 & 12 Pass

• 10 Most Likely Question Papers of PCMB CBSE Pattern Questions Class 12

• Most Probable Que. Bank & Sol.(Math, EN, SCI, SSC) CBSE Pattern Questions Class 10

• 10 Sample Que . wiht Sol (Math, EN, SCI, SSC) CBSE Pattern Questions Class 10

• Foundation Study Package Foundation NTSE, KVPY, IJSO, etc. Class 9 & 10

• IJSO (11 Years Solved Papers ) IJSO Class 9

For Full List, Please visit the website https://books,cppublication.com

Bestseller

t520.00

~wt~
-
CAREER POINT AlsoAvailable at

~ info@cppublication.com
@) www.cppublication.com
a~~o,. MfAh &1,M•f•tl•
0 facebook.com/cpkota
www.puucho.com

You might also like